Sainik School Class IX Entrance Examination 2019 Question Paper With Answer Key

Sainik School Class IX Entrance Examination 2019
Sainik School Class IX Entrance Examination 2019 Question Paper With Answer Key

Sainik School Class-IX Entrance Examination-2019

Paper-I

Mathematics

1. With the help of ruler and compass it is not possible to construct an angle of:

(A)    22.5°

(B)    37.5°

(C)    67.5°

(D)    40°

Answer: (D)

2. What time period is taken when interest is calculated half yearly?

(A)    twice as much as the number of given years

(B)    half as much as the number of given years

(C)    same as the number of given years

(D)    None of these

Answer: (A)

3. If a number is doubled then which of the following is a correct statement?

(A)    Its cube is two times the cube of the given number.

(B)    Its cube is three times the cube of the given number.

(C)    Its cube is six times the cube of the given number

(D)    Its cube is eight times the cube of the given number

Answer: (D)

4. Which of the following is the cube root of −64/243?

(A)    7/4

(B)    −7/4

(C)    4/7

(D)    −4/7

Answer: (D)

5. is equal to:

(A)    81/484

(B)    81/169

(C)    169/81

(D)    16/81

Answer: (B)

6. Which of the following is equal to x3 – 225x?

(A)    x(1 – 15x) (1+ 15x)

(B)    x(x – 15) (x + 15)

(C)    x(1 – 15x) (1 – 15x)

(D)    x(1 + 15x) (1 – 15x)

Answer: (B)

7. The points (−3, 2) and (2, −3) represent:

(A)    different points

(B)    same point

(C)    the origin

(D)    None of these

Answer: (A)

8. If the dimensions of a room are l, b and h, (∵ l → length, b → breadth the h → height) them which of the following is the area of its four walls?

(A)    2h(l + b)

(B)    2h(l + h)

(C)    2l(h + h)

(D)    2h + l + b

Answer: (A)

9. If [1X 2Y 6Z] is a number divisible by 9, then the least value of X + Y + Z is:

(A)    0

(B)    1

(C)    6

(D)    9

Answer: (A)

10. Which of the following is the Multiplicative identity for rational numbers?

(A)    1

(B)    −1

(C)    0

(D)    None of these

Answer: (A)

11. The mid value of a class interval is 42. If the class size is 10, then the upper and lower limits of the class are:

(A)    37.5 and 47.5

(B)    47 and 37

(C)    37 and 47

(D)    47.5 and 37.5

Answer: (C)

12. The speed of a car is  The distance travelled by it in 7/2 hours 35/2 minutes is:

(A) 

(B)      

(C) 

(D)      

Answer: (C)

13. The ages of A and B are in the ratio of 5 : 7. Four years from now the ratio of their ages will be 3 : 4. The present age of B is:

(A)    20 years

(B)    28 years

(C)    15 years

(D)    21 years

Answer: (B)

14. 36 is divided into parts such that 5 times the first part added to 3 times the second part makes 142. The two parts are:

(A)    10 and 26

(B)    12 and 24

(C)    15 and 21

(D)    17 and 19

Answer: (D)

15. Divided Rs 1500 into two parts so that 10% of the larger part exceeds 8% of smaller part by Rs 60. The value of larger and smaller parts are:

(A)    Rs 1200 and Rs 300

(B)    Rs 850 and Rs 650

(C)    Rs 900 and Rs 600

(D)    Rs 1000 and Rs 500

Answer: (D)

16. The value of x + y + z in the adjoining figure is:

(A)    180°

(B)    270°

(C)    360°

(D)    720°

Answer: (C)

17. If an exterior angle of a regular polygon is of measure 12°, then the number of its sides is:

(A)    12

(B)    18

(C)    22

(D)    30

Answer: (D)

18. The smallest whole number by which 44 should be multiplied so as to make it a perfect square is:

(A)    4

(B)    11

(C)    6

(D)    5

Answer: (B)

19. There are 12321 workers in a factory. They stand in such a way that the number of workers in each row is equal to the number of rows. How many workers stand in each row?

(A)    111

(B)    121

(C)    131

(D)    141

Answer: (A)

20. Cubical boxes of volume 15625 cm3 each are put in a cubical store of side 2.5 m. How many such boxes can be put in the store?

(A)    100

(B)    250

(C)    500

(D)    1000

Answer: (D)

21. If  then the value of   is:

(A)    ±4

(B)    ±3

(C)    ±6

(D)    ±5

Answer: (D)

22. If the sum of the lengths of bases of a trapezium is 12 cm and area is 14.1 cm2, then its altitude will be:

(A)    2.35 cm

(B)    4.70 cm

(C)    9.40 cm

(D)    1 cm

Answer: (A)

23. If the length, width and height of a cuboid are 4.2 m, 3 m and 1.1 m, then its capacity in litres will be:

(A)    12860 litres

(B)    13860 litres

(C)    14860 litres

(D)    15860 litres

Answer: (B)

24. A road roller is 350 cm long and its diameter is 84 cm. It takes 500 complete revolutions to travel the road. The area covered by it in m2 will be:

(A)    4620

(B)    6420

(C)    2460

(D)    4260

Answer: (A)

25. A solid cuboidal piece of wood measures 3 m ×5 m × 8 cm. Find the weight of the piece if 1 cubic cm of wood weighs 9 grams.

(A)    4500 kg

(B)    5000 kg

(C)    5400 kg

(D)    5600 kg

Answer: (C)

26. A well was dug with 14 m inner diameter and was 8 m deep. The earth dug out of it was evenly spread out on a rectangular plot of size 10 m × 8 m. Find the raise in the height of the plot.

(A)    15.6 m

(B)    15.4 m

(C)    15 m

(D)    15.5 m

Answer: (B)

27. If 53x + 4 = 25 × 54x – 1 then the value of x is:

(A)    −3

(B)    −5

(C)    5

(D)    3

Answer: (D)

28. Sum of the digits of a two-digit number is 9. If 9 s subtracted from the number, the digits interchange their places. The original number is:

(A)    81

(B)    54

(C)    72

(D)    45

Answer: (B)

29. You are given the multiplication of two numbers as below:

The values of the letters A, B, C, D, E, F and G are:

(A)    A = 2, B = 4. C = 0, D = 0, E = 2, F = 9, G = 6

(B)    A = 7. B = 4, C = 1, D = 0, E = 2, F = 0, G = 6

(C)    A = 2, B = 4. C = 1, D = 0, E = 2, F = 0, G = 6

(D)    A = 7, B = 4, C = 0, D = 9, E = 2, F = 9, G = 6

Answer: (A)

30. From a pack of 52 playing cards, one card is drawn at random. The probability of the drawn card being a black ten or a king is:

(A)    5/26

(B)    3/26

(C)    3/13

(D)    2/13

Answer: (B)

31. A spider is climbing a wall. It climbs up 5 cm, falls back 3 cm, climbs up another 4 cm, falls back 6 cm and climbs up another 5 cm. How far the spider has climbed from its starting point?

(A)    5 cm

(B)    6 cm

(C)    4 cm

(D)    23 cm

Answer: (A)

32. What will be the amount and compound interest on Rs 5000 in 3 years if the rat of interest is 4% for the five year, 3% for the second year and 2% for the third year.

(A)    Rs 5436.12 and Rs 436.12

(B)    Rs 5563.12 and 563.12

(C)    Rs 5063.12 and Rs 63.12

(D)    Rs 5463.12 and 463.12

Answer: (D)

33. The smallest square number which is divisible by each one of the numbers 8, 9, 10 is:

(A)    2600

(B)    3600

(C)    2900

(D)    3900

Answer: (B)

34. If 25% of a number is less than 18% of 650 by 19, then find the number:

(A)    293

(B)    329

(C)    239

(D)    392

Answer: (D)

35. If the cost price is 25% of selling price then the profit percentage is:

(A)    300%

(B)    305%

(C)    350%

(D)    355%

Answer: (A)

36. Area of a rectangle whose length is 4ab and breadth is 6b2 is:

(A)    24ab

(B)    24ab2

(C)    24ab3

(D)    24ab4

Answer: (C)

37. The total surface area of a cone having its slant height 9 dm and diameter of its base as 24 dm is:

(A)    792 sq. dm

(B)    729 sq. dm

(C)    279 sq. dm

(D)    297 sq. dm

Answer: (A)

38. A Cube of side 4 cm contains a sphere touching its side. Then the Volume of the gap in between is:

(A)    30 cu.cm

(B)    30.48 cu.cm

(C)    30.84 cu.cm

(D)    31 cu.cm

Answer: (B)

39. A chord of a circle is of length 6 cm and it is at a distance of 4 cm from the centre. Find the radius of the circle:

(A)    3 cm

(B)    4 cm

(C)    5 cm

(D)    6 cm

Answer: (C)

40. The mean of 40 observations was 160. It was detected on rechecking that the value of 165 was wrongly copied as 125 for computation of mean. Find the correct mean:

(A)    161

(B)    159

(C)    166

(D)    111

Answer: (A)

41. If x + y = 12 and xy = 27 and the value of x3 + y3:

(A)    756

(B)    765

(C)    567

(D)    576

Answer: (A)

42. In what time will a sum of money double itself at  on simple interest.

(A)    13 years

(B)    14 years

(C)    15 years

(D)    16 years

Answer: (D)

43. In a triangle ABC, E is the midpoint of median AD. Then the area of ∆ BED is:

(A)    Area of ∆ ABC

(B)    0

(C)    ½(area of ∆ ABC)

(D)    ¼(area of ∆ ABC)

Answer: (D)

44. The radius of a spherical balloon increases from 7 cm to 14 cm as air is being pumped into it. Find the ratio of surface areas of the balloon in the two cases:

(A)    1 : 4

(B)    1 : 2

(C)    4 : 1

(D)    2 : 1

Answer: (A)

45. Two isosceles triangles have equal vertical angles and their areas in the ratio 25 : 36. Find the ratio of their corresponding heights.

(A)    4 : 5

(B)    5 : 6

(C)    6 : 7

(D)    5 : 7

Answer: (B)

46. The face value of each share is Rs 10. If dividend is 16% then what will be the income from 600 shares?

(A)    Rs 900

(B)    Rs 960

(C)    Rs 860

(D)    Rs 800

Answer: (B)

47. A solid gold ball of radius 7 cm was melted and then drawn into a wire of diameter 0.2 cm. Find the length of the wire.

(A)    457.33 m

(B)    475.33 m

(C)    547.33 m

(D)    745.33 m

Answer: (A)

48. A horse is tethered for grazing inside a rectangular field 70 m by 52 m and is tethered to one corner by a rope 21 m long. How much area can it graze?

(A)    346.5 sq. m

(B)    340 sq. m

(C)    349.5 sq. m

(D)    348.5 sq. m

Answer: (A)

49. A race boat covers a distance of 66 km downstream in 110 minutes. It covers the same distance upstream in 120 The speed of the boat in still water is 34.5 km/hr. The speed of the stream will be:

(A)    1.5 km/hr

(B)    2 km/hr

(C)    2.5 km/hr

(D)    3 km/hr

Answer: (A)

50. The value of 100012:

(A)    1002001

(B)    100201

(C)    100020001

(D)    1000201

Answer: (A)

English Language

51. The correctly punctuated sentence is:

(A)    Mina said, wheres’ Kishore?

(B)    Mina said, where’s Kishore!

(C)    Mina said, ‘Where’s Kishore?’

(D)    Mina said, ‘Wheres’ kishore!’

Answer: (C)

52. “TO LET THE CAT OUT OF THE BAG’ means:

(A)    To jump out of a problem

(B)    Have a pet animal

(C)    To feel extremely happy

(D)    Reveal something that was kept a secret before

Answer: (D)

53. Choose the correct order to make the sentence below meaningful:

Month (1)/ while (2)/ Mumbai (3) / Last (4) / it (5) / in (6) / happened (7) / living (8) / I was (9)

(A)    1 8 9 2 7 5 6 3 4

(B)    1 2 3 4 5 6 7 8 9

(C)    5 7 8 9 2 4 3 6 1

(D)    5 7 2 9 8 6 3 4 1

Answer: (D)

54. The adjective form of ‘ADVANTAGE’ is:

(A)    advantageous

(B)    advantagly

(C)    advantage

(D)    advantagement

Answer: (A)

55. The active voice of –“He had not been invited by us” is:

(A)    We had been invited by us.

(B)    We had invited by him.

(C)    We had not invited him.

(D)    We had not invited by us.

Answer: (C)

56. Identify the tense in the sentence given below:

When I arrived Ram had just left.

(A)    Present Perfect

(B)    Present Continuous

(C)    Past Perfect

(D)    Past Continuous

Answer: (C)

57. The synonym of the word ‘SPLENDID’ is:

(A)    Shortage

(B)    Insignificant

(C)    impressive

(D)    excess

Answer: (C)

58. The word ‘brittle’ means:

(A)    not easily breakable

(B)    easily breakable

(C)    easily understandable

(D)    easily portable

Answer: (B)

Directions (Qs. No. 59 and 60): Find out the part containing error in the given statement.

59. He is one of the talented student in our class.

(A)    He is one of the

(B)    talented student

(C)    in our class

(D)    no  error

Answer: (B)

60. The thief was taken to the nearby police station.

(A)    The thief was

(B)    taken to the

(C)    nearby police station

(D)    no error

Answer: (D)

Directions (Qs. No. 61 and 62): Fill in the blank by choosing the appropriate preposition from the given options.

61. They have gone _____ an excursion.

(A)    to

(B)    for

(C)    with

(D)    on

Answer: (D)

62. The team rebelled ______ the captain.

(A)    upon

(B)    over

(C)    with

(D)    against

Answer: (D)

Directions (Qs. No. 63 and 64): Choose the correct option if the given statements are changed into comparative degree.

63. Silver is one of the most useful metals.

(A)    Silver is useful to any other metal.

(B)    Silver is useful than any other metal.

(C)    Silver is more useful than all other metals.

(D)    Silver is more useful than most of the metals.

Answer: (D)

64. Learning Italian is not as difficult as learning Japanese.

(A)    Learning Japanese is more difficult to learning Italian.

(B)    Learning Italian is not difficult than learning Japanese.

(C)    Learning Japanese is more difficult than learning in Italian.

(D)    Learning Japanese is more difficult than learning Italian.

Answer: (D)

65. The adjective form of ‘apathy’ is:

(A)    apathetically

(B)    apathical

(C)    apathetic

(D)    pathetic

Answer: (C)

66. The noun form of ‘Proud’ is:

(A)    proudy

(B)    pride

(C)    proudly

(D)    proudliness

Answer: (B)

Directions (Qs. No. 67 and 68): Read the following passage and answer the questions that follow.

     Books are by far, the most lasting product of human effort. Temples crumble into ruins. Pictures and statues decay, but books survive. Time does not destroy the great thoughts which are as fresh today as when they first passed through the author’s mind ages ago. The only effect of time has been to throw out the bad products, for nothing in literature can survive long unless it is really good and of lasting value. Books introduce us to the best society; they bring us into the presence of the greatest minds that have ever lived, we hear what they said and did: we see them as if they were really alive, we sympathize with them, enjoy with them and grieve with them.

67. According to the passage, books live forever because:

(A)    They have productive value.

(B)    Time does not destroy great thoughts.

(C)    They are in printed form.

(D)    They have the power to influence people.

Answer: (B)

68. According to the passage, temples, pictures and statues belong to the same category because:

(A)    All of them are beautiful.

(B)    All of them are substantial.

(C)    All of them are likely to decay.

(D)    All of them are fashioned by men.

Answer: (C)

69. Books introduce us into the best society as:

(A)    They give us a glimpse of the greatest minds.

(B)    They take us to the world of imagination.

(C)    They instill in us the qualities of the greatest minds.

(D)    They introduce us to elite class of the society.

Answer: (A)

70. Radha, “I won’t buy a new car”. (Choose the correct word to fill in the blank.)

Radha said that she _______ buy a new car.

(A)    won’t

(B)    will

(C)    wouldn’t

(D)    would

Answer: (C)

71. They will ask you a lot of questions at the interview. The sentence, when converted into the passive voice, would read as:

(A)    You are asked a lot of questions at the interview.

(B)    You were being asked a lot of questions at the interview.

(C)    You will be asked a lot of questions at the interview.

(D)    You are being asked a lot of questions at the interview.

Answer: (C)

72. The phrase in cold blood means:

(A)    Indifferently

(B)    cruelly

(C)    thoughtlessly

(D)    deliberately

Answer: (B)

73. The workers went out of the factory ______ to hold a protest rally. (Choose the correct word to fill in the blank.)

(A)    en masse

(B)    en route

(C)    impasse

(D)    de facto

Answer: (A)

74. To put up with means:

(A)    to close

(B)    to prolong

(C)    to tolerate

(D)    to forget

Answer: (C)

75. Fill in the blank with a possessive pronoun chosen from those given below.

Mani declared that the book was not______.

(A)    our

(B)    her

(C)    your

(D)    hers

Answer: (D)

General Science

76. Rhizobium bacteria:

(A)    Help is digestion

(B)    Help in nitrogen fixation

(C)    Cause diseases

(D)    All of the above

Answer: (B)

77. The metal which is stored in kerosene?

(A)    Phosphorus

(B)    Magnesium

(C)    Sodium

(D)    Calcium

Answer: (C)

78. Poor conductors are:

(A)    Plastics

(B)    Clothes

(C)    Wood

(D)    All of these

Answer: (D)

79. There are following zones of the a flame:

(A)    Two

(B)    Three

(C)    Four

(D)    No any zone

Answer: (B)

80. Force of friction always acts on moving objects and its direction shall be:

(A)    On any direction

(B)    Along the direction of motion

(C)    Perpendicular to the direction of motion

(D)    Opposite to the direction of motion

Answer: (D)

81. The stage of the embryo in which all the body parts can be identified is:

(A)    Fetus

(B)    Zygote

(C)    Infant

(D)    None of these

Answer: (A)

82. Diabetes is due to malfunctioning of:

(A)    Adrenal gland

(B)    Pituitary gland

(C)    Heart

(D)    Pancreas

Answer: (D)

83. Naphthalene balls are obtained from:

(A)    Carbon

(B)    Coke

(C)    Coal tar

(D)    Coal gas

Answer: (C)

84. John accidentally placed his hand over flame and immediately pulled it back. He felt the sensation of heat and reacted due to the action of:

(A)    Nerve cells

(B)    Blood cells

(C)    Skin surface

(D)    Nucleus of cells

Answer: (A)

85. A purple coloured nonmetal forms a brown solution in alcohol which is applied on wounds as an antiseptic. Name of the non-metal is:

(A)    Phosphorous

(B)    Carbon

(C)    Sulphur

(D)    Iodine

Answer: (D)

86. Given below are the harmful effects of weeds on crop plants. Choose the correct combination of statements.

(i) They interfere in harvesting.

(ii) They help crop plants to grow healthy

(iii) The complete with crop plants for water, nutrients, space and light.

(iv) They affect plant growth.

(A)    (i), (iii), (iv)

(B)    (iii), (iv)

(C)    (iii) only

(D)    (i), (ii), (iii), (iv)

Answer: (A)

87. Which of the following groups contain all synthetic substances?

(A)    Nylon, Terylene, Wool

(B)    PVC, Polythene, Bakelite

(C)    Cotton, Polycot, Rayon

(D)    Acrylic, Silk, Wool

Answer: (B)

88. Which of the following statement is true about endemic species?

(A)    They are found exclusively in a specific habitat

(B)    Endemic species can never become endangered

(C)    They are only fond in zoos and botanical gardens

(D)    They are not affected by the destruction of their habitat

Answer: (A)

89. Identify the correct statement about cells

(A)    All the cells have nucleus

(B)    Cells of an organ have similar structure

(C)    Cells of a tissue have similar structure

(D)    Shape of all types of cells is round

Answer: (A)

90. Aquatic animals in which fertilization occurs in water are said to be:

(A)    Viviparous without fertilization

(B)    Oviparous with external fertilization

(C)    Viviparous with internal fertilization

(D)    Oviparous with internal fertilization

Answer: (B)

91. The light from sun takes 500 s to reach the earth. Assuming that the speed of light is 3,00,000 kms1, calculate the distance between the sun and the earth.

(A)    100 million km

(B)    150 million km

(C)    1500 million km

(D)    15 million km

Answer: (B)

92. Which of the following is not an application of chemical effect of electric current?

(A)    Electroplating of metals

(B)    Purification of metals

(C)    Decomposition of elements

(D)    Decomposition of compounds

Answer: (C)

93. An earthquake of magnitude ‘6’ on Richter scale has:

(A)    Ten times more destructive energy than an earthquake of magnitude ‘4’.           

(B)    Hundred times more destructive energy than an earthquake of magnitude ‘4’.

(C)    Thousand times more destructive energy than an earthquake of magnitude ‘4’.

(D)    One and half times more destructive  energy than an earthquake of magnitude ‘4’.

Answer: (B)

94. Which of the following statements is correct regarding rods and cones in the human eye?

(A)    Cones are sensitive to dim light

(B)    Cones are sensitive to bright light

(C)    Rods are sensitive to bright light

(D)    Rods can sense colour.

Answer: (B)

95. Suppose a new planet is discovered between Uranus and Neptune, its time period of revolution around the sun would be:

(A)    Less than that of Neptune

(B)    More than that of Neptune

(C)    Equal to that of Neptune of Uranus

(D)    Less than that of Uranus

Answer: (A)

96. Ramesh was cooking potato curry on a chulha. To his surprise he observed that the copper vessel was getting blackened from outside. It may be due to:

(A)    Proper combustion of fuel

(B)    Improper cooking of potato curry

(C)    Improper combustion of the fuel

(D)    Burning of copper vessel

Answer: (C)

97. When a Copper vessel is exposed to moist air for long, it acquires a dull green coating. The green material is a mixture of:

(A)    Copper oxide and Copper carbonate

(B)    Copper hydroxide and Copper carbonate

(C)    Copper oxide and Copper nitrate

(D)    Copper hydroxide and Copper nitrate

Answer: (B)

98. The places meant for conservation of biodiversity in their natural habitat are:

(i) Zoological garden

(ii) Botanical garden

(iii) Sanctuary

(iv) National park

(A)    (i) and (ii)

(B)    (ii) and (iii)

(C)    (iii) and (iv)

(D)    (i) and (iv)

Answer: (C)

99. The same force ‘F’ acts on four different objects having the areas given below, one by one. In which case the pressure exerted will be the maximum?

(A)    20 m2

(B)    50 m2

(C)    100 m2

(D)    10 m2

Answer: (D)

100. Before playing the orchestra in a musical concert, a sitarist tries to adjust the tension and pluck the strings suitably. By doing so he is adjusting.

(A)    Intensity of sound only

(B)    Amplitude of sound only

(C)    Frequency of the sitar string with the frequency of other musical instruments

(D)    Loudness of sound

Answer: (C)

Social Science

101. The Chairman of the Drafting Committee of Indian Constitution was:

(A)    Sardar Patel

(B)    Dr. B.R. Ambedkar

(C)    Dr. Rajendra Prasad

(D)    J.L. Nehru

Answer: (B)

102. By the late of 18th Century East India Company was trying to expand the cultivation of:

(A)    Opium

(B)    Indigo

(C)    Cotton

(D)    Tea

Answer: (A, B)

103. EVMs were used for the first time in the _______ general elections.

(A)    2001

(B)    2002

(C)    2003

(D)    2004

Answer: (D)

104. There are ________ elected members in Rajya Sabha.

(A)    543

(B)    272

(C)    233

(D)    260

Answer: (C)

105. Minerals that lie at shallow depths are taken out by removing the surface layer, this is known as:

(A)    Open-cast mining

(B)    Shaft mining

(C)    Drilling

(D)    All of the above

Answer: (A)

106. The complainant has a ______ right to get a free copy of the FIR from the Police.

(A)    Fundamental

(B)    Legal

(C)    Political

(D)    Economic

Answer: (B)

107. Which article of the constitution states that untouchability has been abolished?

(A)    Article 14

(B)    Article 15

(C)    Article 16

(D)    Article 17

Answer: (D)

108. The Marathi newspaper Kesari was edited by:

(A)    Bipin Chandrapal

(B)    Bal Gangadhar Tilak

(C)    Sarojini Naidu

(D)    Lala Lajpat Rai

Answer: (B)

109. The difference between the birth rate and the death rate is called the:

(A)    Natural growth rate

(B)    Normal growth rate

(C)    Actual growth rate

(D)    None of the above

Answer: (A)

110. Dandi March was against the:

(A)    Salt Tax

(B)    Purna Swaraj

(C)    Non co-operation movement

(D)    Simon Commission

Answer: (A)

111. The British East India Company got a charter from Queen Elizabeth-I in:

(A)    1600 AD

(B)    1599 AD

(C)    1601 AD

(D)    1700 AD

Answer: (A)

112. Delhi Renaissance refers to the period from:

(A)    1830 – 1857 AD

(B)    1857 – 1885 AD

(C)    1825 – 1857 AD

(D)    None of these

Answer: (A)

113. Woods Dispatch of 1854 refers to:

(A)    Educational Reform

(B)    Financial Reform

(C)    Forest Reform

(D)    None of these

Answer: (A)

114. The leader of the ruling party in Lok Sabha is:

(A)    The Prime Minister

(B)    The President

(C)    The Vice President

(D)    The Leader of Opposition

Answer: (A)

115. As per which article of the Indian Constitution every arrested person is guaranteed Fundamental Rights:

(A)    Article 20

(B)    Article 21

(C)    Article 23

(D)    Article 22

Answer: (D)

116. Which of the following gas was released in Bhopal Gas Tragedy?

(A)    Methyl Isocyanite

(B)    Ethyl Isocyanate

(C)    Methyl Alcohol

(D)    Ethyl Alcohol

Answer: (A)

117. Which one of the following is a leading producer of Copper in the world?

(A)    Bolivia

(B)    Chile

(C)    Ghana

(D)    Zimbabwe

Answer: (B)

118. Identify the state with “Lowest literacy” as per census 2011.

(A)    Bihar

(B)    Haryana

(C)    Rajasthan

(D)    Gujarat

Answer: (A)

119. The total number of Anglo Indians nominated to the Lok Sabha is:

(A)    8

(B)    6

(C)    4

(D)    2

Answer: (D)

120. Which one of the following countries has the highest percentage of forest land?

(A)    Australia

(B)    India

(C)    France

(D)    Japan

Answer: (A)

121. Who was the founder of the ‘Brahmo Sabha’?

(A)    Annie Besant

(B)    Bal Gangadhr Tilak

(C)    Raja Ram Mohan Roy

(D)    Ishwar Chandra Vidyasagar

Answer: (C)

122. Child marriage restraint Act was passed in the year______.

(A)    1929

(B)    1909

(C)    1919

(D)    1853

Answer: (A)

123. What is a population pyramid?

(A)    A graphical presentation of the age, sex composition of a population.

(B)    When the population density of an area is so high that people live in tall buildings.

(C)    Pattern of population distribution in large urban areas.

(D)    Pattern of population distribution in rural areas.

Answer: (A)

124. Which age group of India is guaranteed free education by the Constitution?

(A)    6-14 years

(B)    5-13 years

(C)    7-14 years

(D)    6-12 years

Answer: (A)

125. Who wrote the book ‘Poverty and Un-British Rule in India’?

(A)    Dadabhai Naoroji

(B)    Badruddin Tyabji

(C)    Pherojshah Mehta

(D)    Bipin Chandrapal

Answer: (A)

Paper-II

Intelligence Test

Directions (Qs. NO. 126-128) : Choose the letters group that best represents a relationship similar to the one expressed in the original pair of letters group.

126. MONKEY : XDJMNL : : TIGER : ?

(A)    QDFHS

(B)    SDFHS

(C)    SHFDQ

(D)    UJHFS

Answer: (A)

127. MAD is to JXA as RUN is to:

(A)    ORX

(B)    OSQ

(C)    PRJ

(D)    UXQ

Answer: (*)

128. Kilometer is to Distance as Poundal is to:

(A)    Density

(B)    Acceleration

(C)    Momentum

(D)    Force

Answer: (D)

Directions (Qs. No. 129-131): Which number complete the second pair in the same way as the first pair.

129. 20 : 11 : : 102 : ?

(A)    49

(B)    52

(C)    61

(D)    98

Answer: (B)

130. 13 : 25 : : 48 : ?

(A)    95

(B)    97

(C)    109

(D)    105

Answer: (A)

131. Tiff is to Battle as Frugal is to:

(A)    Sprint

(B)    Vague

(C)    Miserly

(D)    Vital

Answer: (C)

132. A is 40 m South-West of B. C is 40 m South-East of B. Then C is in which direction of A?

(A)    East

(B)    West

(C)    North-East

(D)    South

Answer: (A)

133. Choose the figure, which is different from others.

(A)    1

(B)    2

(C)    3

(D)    4

Answer: (B)

134. If ENGLAND is written as 1234526 and FRANCE is written as 785291. How is GREECE coded?

(A)    381191

(B)    831191

(C)    832252

(D)    835545

Answer: (A)

135. Which word can’t be formed by using the letters of the INTELLIGENCE word? Find that word.

(A)    TILLAGE

(B)    INCITE

(C)    GENTLE

(D)    NEGLECT

Answer: (A)

136. If + means divide, divide means −, − means ×, × means ÷ then 12 + 6/3 – 2 × 8 = ?

(A)    2

(B)    4

(C)    8

(D)    5

Answer: (B)

137. Select the figure from the answer set that would come in place of the question mark (?).

(A)    1

(B)    2

(C)    3

(D)    4

Answer: (B)

138. A Cube painted blue on all the faces is cut into 125 cubes of equal size. Then, how many cubes are not painted on any face?

(A)    8

(B)    16

(C)    27

(D)    54

Answer: (C)

139. If 1st October is Sunday, than 1st November will be:

(A)    Tuesday

(B)    Friday

(C)    Wednesday

(D)    Thursday

Answer: (C)

140. Find the missing number in the box.

(A)    64

(B)    63

(C)    66

(D)    84

Answer: (B)

141. Which one of letters when sequentially placed at the gaps in the given letter series shall complete it?

ac_cab_baca_aba_aca_

(A)    acbcc

(B)    aacbc

(C)    babbb

(D)    bcbba

Answer: (B)

Directions (Qs. No. 142-145): In each of the following questions. Find the word which cannot be made from the letters of the given word.

142. REPUBLICAN

(A)    CLIP

(B)    PURE

(C)    ANKLE

(D)    BANE

Answer: (C)

143. ESTRANGE

(A)    GENERATE

(B)    SERGEANT

(C)    REAGENTS

(D)    GREAT

Answer: (A)

144. ADMINISTRATOR

(A)    ADMIT

(B)    NEST

(C)    MANTA

(D)    ROAD

Answer: (B)

145. SOCIALISATION

(A)    SCOUT

(B)    CLASS

(C)    LIAISON

(D)    ASSOCIATION

Answer: (A)

Directions (Qs. No. 146 and 147): Choose the letters group that best represents a relationship similar to the one expressed in the original pair of letters groups.

146. If LONDON is coded as MPOEPO. What code is needed for DELHI?

(A)    DEHLI

(B)    EFIMJ

(C)    HLDEI

(D)    EFMIJ

Answer: (D)

147. PNS : OOT : : DBH : ?

(A)    PPI

(B)    BBI

(C)    CCI

(D)    DDB

Answer: (C)

Directions(Qs. No. 148-150): In the given series, find the next/missing term/number.

148. MNOABCPQRDEFST??

(A)    GK

(B)    UV

(C)    GH

(D)    UG

Answer: (D)

149. AZ, CX, EV, ?

(A)    HT

(B)    HU

(C)    GS

(D)    GT

Answer: (D)

150. YX, UTS, ONML, ?

(A)    FEDCB

(B)    GFEDC

(C)    IHGFE

(D)    HGFED

Answer: (B)

Sainik School Class IX Entrance Examination 2020 Question Paper With Answer Key

Sainik School Class IX Entrance Examination 2020
Sainik School Class IX Entrance Examination 2020 Question Paper With Answer Key

Sainik School Class-IX Entrance Examination-2020

Paper-I

Section-A

Mathematics

1. The value of 

(A) 

(B) 

(C) 

(D) 

Answer: (D)

2. The abscissa of a point is its distance from the

(A)    Origin

(B)    X-Axis

(C)    Y-Axis

(D)    None

Answer: (C)

3. What is the value of m, if 

(A)    m = 1

(B)    m = −2

(C)    m = −1

(D)    m = 2

Answer: (C)

4. “If a number when divided by 4 leaves remainders 2 or 3”, then which one is the correct statement?

(A)    The number is not a perfect square

(B)    The number is a perfect square

(C)    The number is a prime number

(D)    None of these

Answer: (A)

5. The value of 

(A)    15

(B)    16

(C)    5

(D)    150

Answer: (A)

6. Three numbers are in the ratio 2 : 3 : 4. The sum of their cubes is 33957. The numbers are

(A)    16, 24 and 32

(B)    12, 18 and 24

(C)    14, 21 and 28

(D)    18, 27 and 36

Answer: (C)

7. Find the least square number divisible by each one of 8, 9 and 10.

(A)  360

(B)  36

(C)  3600

(D)  3.6 × 102

Answer: (C)

8. If  is a multiple of 11, where a is a digit, the value of a is

(A)    0

(B)    4

(C)    1

(D)    2

Answer: (B)

9. Find the value of A and B in 

(A)  A = 5 and B = 2

(B)  A = 5 and B = 5

(C)  A = 2 and B = 2

(D)  A = 2 and B = 5

Answer: (A)

10. Find the value of Z for which the number 471Z8 is divisible by 9.

(A)  4

(B)  5

(C)  7

(D)  8

Answer: (C)

11. If the area of an equilateral triangle is 64√3 cm2, then the side of the triangle is _____.

(A)  18√3 cm

(B)  9 cm

(C)  16 cm

(D)  3√2 cm

Answer: (C)

12. The value of  is:

(A)  (2m – a + b)

(B)  (2m – a – b)

(C)  (2m + a + b)

(D)  (2m + a – b)

Answer: (A)

13. The ratio between the speeds of two trains A and B is 3 : 5. If train B runs 300 km in 4 hours, the speed of train A will be

(A)  40 km/h

(B)  60 km/h

(C)  30 km/h

(D)  45 km/h

Answer: (D)

14. Two years ago, Dillip was three times a old as his son and two years hence, twice his age will be equal to five times that of his son. The present age of son and Dillip are:

(A)  14, 38 years

(B)  16, 40 years

(C)  12, 36 years

(D)  None

Answer: (A)

15. The probability of getting a 7 in a single throw of a dice is:

(A)  1

(B)  0

(C)  1/6

(D)  1/2

Answer: (B)

16. A’s income is 60% more than that of B. By what per cent is B’s income less than A’s ?

(A)  37%

(B)  37.5%

(C)  36.5%

(D)  36%

Answer: (B)

17. By joining (−3, 2), (−3, −3) and (−3, 4), which of the following is obtained?

(A)  A triangle

(B)  A straight line parallel to x-axis

(C)  A straight line parallel to y-axis

(D)  A straight line passing through origin

Answer: (C)

18. The number of times a particular observation occurs in a given data is called as

(A)  Frequency

(B)  Range

(C)  Mean

(D)  None of these

Answer: (A)

19.  By selling 33 m of cloth, a draper loses an amount equal to the selling price of 3 m of cloth. Find his gain or loss per cent.

(A) 

(B) 

(C)  gain7%

(D)  loss 7%

Answer: (B)

20. Three cubes of metal whose edges are 6 cm, 8 cm and 10 cm respectively are melted to form a single cube. The edge of the new cube is _______.

(A)  24 cm

(B)  20 cm

(C)  18 cm

(D)  12 cm

Answer: (D)

21. Find the single discount equivalent to two successive discounts of 20% and 10%.

(A)  25%

(B)  30%

(C)  28%

(D)  10%

Answer: (C)

22. Which of the following is not a case of direct variation?

(A)  Number of sheets of some kind are increased when their total weight is increased

(B)  More quantity of petrol is required to travel more distance with a fixed speed

(C)  More fees would be collected if number of students increase in a class

(D)  Time taken will be less, if number of workers is increased to complete the same work

Answer: (D)

23. If a + b + c = 9 and ab + bc + ca = 23, then the value of a2 + b2 + c2 equals to

(A)  35

(B)  81

(C)  127

(D)  217

Answer: (A)

24. On dividing 200 into two parts, 1/3 of the first part and ½ of the second part are equal. The larger of the parts is _______.

(A)  80

(B)  120

(C)  40

(D)  150

Answer: (B)

25. On selling a fan for Rs 810, Sunil gains 8%. For how much did he purchase it?

(A)  Rs 700

(B)  Rs 675

(C)  Rs 650

(D)  Rs 750

Answer: (D)

26. Find the value of x: 32x × 3x+3 ×34x = (√3)10

(A)  −1

(B)  0

(C)  1

(D)  2

Answer: (A)

27. 

(A)  −bc3/a2

(B)  bc4/a2

(C)  −ab4/c2

(D)  −bc4/a2

Answer: (D)

28. If n is a perfect cube, then every prime factor of ‘n’ occurs_____

(A)  One time

(B)  Two times

(C)  Three times

(D)  Four times

Answer: (C)

29. Find the greatest number of four digits which is a perfect square?

(A)  9800

(B)  9864

(C)  9999

(D)  9801

Answer: (D)

30. If the ratio of the ages (in years) of x and y, 8 years ago is 7 : , then which of the following can be the sum of their ages 8 years from now?

(A)  82

(B)  97

(C)  75

(D)  94

Answer: (B)

31. The age of the boy is one-fifth of the age of his mother and sum of the ages of the son and the mother is equal to the age of the father. After 15 y ears, the sum of the ages of the son and his mother will be four-third of his father’s age. Find the ratio of the present ages of son, mother and father respectively.

(A)  1 : 5 : 7

(B)  2 : 10 : 10

(C)  1 : 5 : 6

(D)  2 : 8 :  9

Answer: (C)

32. The ratio of the income of P and Q is 5 : 4. The ratio of expenditure is 4 : 3. The saving of P is more than that of Q by  What % of his incomes does P spend?

(A) 

(B) 

(C) 

(D) 

Answer: (B)

33. Mohan invested a sum of Rs 12,500 at 12% per annum compound interest. He received an amount of Rs 15,680 after x year. Then the value of x is _______.

(A)  1

(B)  4

(C)  3

(D)  2

Answer: (D)

34. Find the compound interest on Rs 1,25,000 for 9 months at 8% per annum, compounded quarterly.

(A)  7551

(B)  7651

(C)  7650

(D)  7655

Answer: (B)

35. Pipe A can fill a tank in 12 hours and pipe B can empty the tank is 18 hours. Both pipes are opened at 6 AM and after some time, pipe B is closed and tank is full at 8 PM. At what time was the pipe B closed?

(A)  10 AM

(B)  8 AM

(C)  9 AM

(D)  11 AM

Answer: (C)

36. A car covers 300 kms at a constant speed. If its speed was 10 kmph more, it would have taken 1 hour less to travel the same distance. Find the speed of the car.

(A)  60 kmph

(B)  50 kmph

(C)  40 kmph

(D)  75 kmph

Answer: (B)

37. Two trains are travelling in opposite direction with speed of 25 m/s and 30 m/s respectively. If the length of one train is 300 m and that of the other train is 250 m, then find the time taken by the trains to cross each other.

(A)  8 s

(B)  10 s

(C)  12 s

(D)  14 s

Answer: (B)

38. The sum of all digits of a two digit number is 9. If 27 is subtracted from the number then the digits get reversed. Find the number.

(A)  81

(B)  72

(C)  36

(D)  63

Answer: (D)

39. There are some four-wheelers and six-wheelers in a garage. The total number of wheels of these vehicles is 120. The number of four-wheelers is 3/2 times the number of six-wheelers. Find the number of six-wheelers in the garage.

(A)  20

(B)  5

(C)  15

(D)  10

Answer: (D)

40. What is the minimum interior angle possible for a regular polygon?

(A)  60°

(B)  75°

(C)  90°

(D)  None

Answer: (A)

41. What is the number of diagonals in a hexagon?

(A)  4

(B)  6

(C)  9

(D)  10

Answer: (C)

42. In the adjacent figure, the bisectors of ∠A and ∠B meet at a point P. If ∠C = 100° and ∠D = 60° find the  measure of ∠APB.

(A)  60°

(B)  70°

(C)  90°

(D)  80°

Answer: (D)

43. A square and a rectangle each have a perimeter of 40 m. The difference between areas of the two figures is 9 m2. What are the possible dimensions of the rectangle?

(A)  13 m, 7 m

(B)  14 m, 6 m

(C)  108 m, 1 m

(D)  15 m, 5 m

Answer: (A)

44. In a parallelogram ABCD, AB = 6 cm, BC = 5 cm and AC = 7 cm. Find the perpendicular distance between 

(A)  6√6 cm

(B)  12√6 cm

(C)  5 cm

(D)  2√6 cm

Answer: (D)

45. Some cubic metres of earth is dug out to sink a well which is 16 m deep and which has a radius of 3.5 m. If that amount of earth when taken out is spread over a rectangular plot of dimensions 25 m × 16 m, what is the height of the platforms so formed?

(A)  1.54 m

(B)  1.50 m

(C)  1.52 m

(D)  1.53 m

Answer: (A)

46. What is the difference between the total surface area and curved surface area of a cylinder whose radius is equal to 10 cm?

(A)  200π cm2

(B)  300π cm2

(C)  100π cm2

(D)  10π cm2

Answer: (A)

47. The mean of six numbers is 15. If 2 is taken away from every number, the new mean would be

(A)  13

(B)  4

(C)  17

(D)  8

Answer: (A)

48. The sides of the triangle are 45 cm, 60 cm and 75 cm. Find the length drawn to the longest side from its opposite vertex.

(A)  27 cm

(B)  21 cm

(C)  39 cm

(D)  36 cm

Answer: (D)

49. In the following figure, find the value of Y.

(A)  50°

(B)  65°

(C)  60°

(D)  70°

Answer: (D)

50. A cylindrical tank has a capacity of 5632 m3. If the diameter of its base is 16 m, find its depth.

(A)  28 m

(B)  25 m

(C)  16 m

(D)  29 m

Answer: (A)

Section B : English

51. Candidates must _______ the general conditions for admission.

(A)  do

(B)  prepare

(C)  satisfy

(D)  create

Answer: (C)

52. He resorts _______ sharp practice in his dealings.

(A)  at

(B)  to

(C)  in

(D)  for

Answer: (B)

53. _______ English is an international language.

(A)  No article

(B)  The

(C)  A

(D)  An

Answer: (A)

54. The word ‘Industrious’ means.

(A)  Working in industry

(B)  Labour in factory

(C)  Hard working

(D)  Laid back

Answer: (C)

55. Choose the correct order to make the sentence below meaningful:

Wear (1) / these (2) / people (3) / almost (4) / a (5) / majority (6) / of (7) / days (8) / a watch (9)

(A)  4 5 6 7 3 1 9 2 8

(B)  4 5 7 6 1 3 8 2 9

(C)  4 5 6 7 9 2 3 1 8

(D)  4 5 6 7 3 1 2 8 9

Answer: (A)

56. Select the word closest in meaning to “Mortal”.

(A)  Recurrent

(B)  Trivial

(C)  Fatal

(D)  Eternal

Answer: (C)

57. Mark the word with the correct spelling.

(A)  Maintenance

(B)  Maintennance

(C)  Maintanance

(D)  Maintainance

Answer: (A)

58. The coach insisted that Ronaldo_____ the centre position, even though he’s too short for that position. (Use the correct verb)

(A)  play

(B)  played

(C)  plays

(D)  None of these

Answer: (A)

59. _____ King of Scotland saw ______ spider trying to climb up to ______ ceiling of the cave. (Use Articles)

(A)  The, a, the

(B)  A, a, the

(C)  No article, a, no article

(D)  The, a, no article

Answer: (C)

60. Raju relishes not only Chinese _______ continental food. (Use appropriate Concjunction)

(A)  also

(B)  but

(C)  but also

(D)  rather

Answer: (C)

61. You must learn _______ English every day to improve it. (Use appropriate determiner)

(A)  a few

(B)  a little

(C)  a lot of

(D)  None of these

Answer: (B)

62. Arrange the jumbled words to form a meaningful sentence

most/inventor/world/Thomas/Alva/Edison/the/in/is/remarkable/the

(A)  The remarkable inventor in the most world in Thomas Alva Edison.

(B)  Thomas Alva Edison is the most remarkable inventor in the world.

(C)  Is the most remarkable inventor in the world Thomas Alva Edison.

(D)  In the world most remarkable inventor is Thomas Alva Edison.

Answer: (B)

Directions (Qs. No. 63 to 67): Read the following passages and answer the questions that follow:

The moon’s role in causing tides is much more high and important than that of the Sun. The reason is that the moon enjoys more proximity to the earth than the sun. As such its force is greater than that of the sun in attracting the surface water. Tides are of immense importance. In trade, navigation and fishing, tides are very useful. During the high tide, the water depth near the coast goes up and helps big ships to reach the ports. Kandla port in Gujarat and Diamond Harbour in West Bengal owe their very existence to the tides only. The significance of both London and Kolkata also depends on the tides. Tides also keep the harbours clear of refuse and mud brought down by rivers and thus they do not allow the harbours to be silted. Commonly, the tidal rivers are navigable. For the purpose of generating electricity, tidal waves are harnessed. Tides do not allow the sea water to be frozen by keeping the sea water in motion. Tides are also made use by the fisherman for sailing into the sea and returning to the harbour. In countries like Canada, U.K., France and Japan, tidal power stations are set up.

63. Why does the moon play a greater role than the sun in causing tides?

(A)  The moon is closer to the earth as compared to the sun.

(B)  The moon has greater gravitational pull.

(C)  The moon shines in night.

(D)  None of the above.

Answer: (A)

64. How are tides useful for the economy of the country?

(A)  Tides bring treasure of sea with them.

(B)  During high tides, big ships can reach the ports thus opening new vistas for business.

(C)  Tides destroy enemies of the country.

(D)  None of the above

Answer: (B)

65. How are tides useful in cold countries?

(A)  They bring fish for eating.

(B)  They bring water for drinking.

(C)  The don’t allow sea water to be frozen.

(D)  They keep the port silted.

Answer: (C)

66. How can tides solve the power problem of the world?

(A)  Electricity is being produced through tidal waves.

(B)  Tidal waves keep the steamers in motion.

(C)  Tidal waves melt the ice and save power.

(D)  None of the above

Answer: (A)

67. Which word in the passage means ‘to bring under control’?

(A)  Proximity

(B)  Silted

(C)  Immense

(D)  Harness

Answer: (D)

68. She plays better than _______ do. (Choose appropriate pronoun)

(A)  I

(B)  my

(C)  may

(D)  myself

Answer: (A)

69. I ______ in a school in Delhi since 2017. (Choose appropriate from of tense)

(A)  has studied

(B)  has been studying

(C)  have been studying

(D)  will study

Answer: (C)

70. There isn’t any smog in the Highlands of Scotland, ——– ? (Choose appropriate question tag)

(A)  aren’t it

(B)  isn’t it

(C)  is there

(D)  there isn’t

Answer: (C)

71. Choose the best meaning of the Idiom ‘to play to the gallery”.

(A)  to watch the play with interest

(B)  to enact the play in the gallery

(C)  to endeavour to gain cheap popularity

(D)  to sit comfortably

Answer: (C)

72. The speckled tortoise walks steadfastly towards his goal? (Choose the adverb)

(A)  speckled

(B)  steadfastly

(C)  towards

(D)  goal

Answer: (B)

73. I wrote an article for the school magazine. (Change from active to passive voice)

(A)  An article for the school magazine I had written.

(B)  An article was written by me for the school magazine.

(C)  I have written an article for the school magazine.

(D)  The school magazine and an article I wrote for.

Answer: (B)

74. Empty vessels _______ much noise. (Select the correct form of verb)

(A)  make

(B)  are making

(C)  have made

(D)  makes

Answer: (A)

75. Gullible (Choose the word with opposite meaning)

(A)  trusting

(B)  cynical

(C)  clever

(D)  resourceful

Answer: (B)

Section C : General Science

76. The metal present in Chlorophyll is:

(A)  Iron

(B)  Calcium

(C)  Oxygen

(D)  Magnesium

Answer: (D)

77. Name the gas present in LPG:

(A)  Hydrogen

(B)  Oxygen

(C)  Methane

(D)  Butane

Answer: (D)

78. Which gas is used to replace CFC?

(A)  HCFC

(B)  RCFC

(C)  DHFC

(D)  HHFC

Answer: (A)

79. The first Menstrual flow at puberty is termed:

(A)  Menopause

(B)  Menstruation

(C)  Puberty flow

(D)  Menarche

Answer: (D)

80. Which among the following statement is incorrect about all organelles?

(A)  They are found in all Eukaryotic cells.

(B)  They are found in Multi cellular organisms only.

(C)  They coordinate to produce new cell.

(D)  They are small sized and mostly internal.

Answer: (B)

81. The boy is pulling a cart by a force of 100 N. The frictional force experienced by the cart is 20 N. The force causing the motion of the cart is:

(A)  100 N

(B)  120 N

(C)  80 N

(D)  5 N

Answer: (C)

82. The cans used for storing food are made by electroplating:

(A)  Silver onto iron

(B)  Chromium onto iron

(C)  Gold onto iron

(D)  Tin onto iron

Answer: (D)

83. The impression of an image does not vanish immediately from the retina. It persist for:

(A)  (1/60)th of a second

(B)  (1/12)th of a second

(C)  (1/6)th of a second

(D)  (1/16)th of a second

Answer: (D)

84. The Sun appears to rise in the East and set in the West because:

(A)  Earth rotates from East to West on its axis.

(B)  Earth rotates from West to East on its axis.

(C)  The Sun is at the centre of universe.

(D)  None of these

Answer: (B)

85. Pressure is equal to

(A)  Area/force on which it acts

(B)  Force/area on which it acts

(C)  Volume/force on which it acts

(D)  Force/volume on which it acts

Answer: (B)

86. An ultrasound equipment works at a frequency.

(A)  Higher than 20,000 Hz

(B)  Higher than 10,000 Hz

(C)  Lower than 20,000 Hz

(D)  Lower than 10,000 Hz

Answer: (A)

87. When electrodes are immersed in water and electricity is passed, the bubbles formed on the negative terminal is actually _______ gas.

(A)  Hydrogen

(B)  Carbon dioxide

(C)  Oxygen

(D)  Nitrogen

Answer: (A)

88. If light falls perpendicularly on a plane mirror, the angle in which light will be reflected is ________.

(A)  45 degrees

(B)  90 degrees

(C)  180 degrees

(D)  360 degrees

Answer: (C)

89. All non-living things are known as________

(A)  Biotic Resource

(B)  Exhaustible Resource

(C)  Abiotic Resource

(D)  Human Resource

Answer: (C)

90. Malarial parasite is carried by

(A)  Culex mosquito

(B)  Male anopheles mosquito

(C)  Female anopheles mosquito

(D)  Aedes mosquito

Answer: (C)

91. Internal fertilization does not occur in

(A)  Dog

(B)  Cow

(C)  Parrot

(D)  Frog

Answer: (D)

92. Regeneration is observed in

(A)  Planaria

(B)  Spyrogyra

(C)  Yeast

(D)  Amoeba

Answer: (A)

93. Rapid combustion is

(A)  When gas burns, it produces heat and light

(B)  When material suddenly burst into flames

(C)  When there is evolution of heat

(D)  None of these

Answer: (A)

94. A student is carrying out distillation process in a lab. Water is boiling in distillation flask. Water that is collected in the receiver flask is refrigerated and ice cubes are formed. Ice cubes are then kept outside the refrigerator and they start melting. Arrange following phases of water in ascending order of their total (PE + KE) energy considering that the mass of water remains the same.

(i) Water collected in the receiving flask

(ii) Water boiling in the distillation\

(iii) Steam passing through the delivery tube

(iv) Ice cubes formed in the refrigerator

(A)  (iv), (iii), (ii), (i)

(B)  (iv), (i), (ii), (iii)

(C)  (iii), (iv), (ii), (i)

(D)  (iv), (i), (iii), (ii)

Answer: (B)

95. The difference samples of CO2 were found to contain carbon and oxygen in the same ratio of their mass. This illustrates

(A)  Law of conservation of mass

(B)  Law of definite proportions

(C)  Law of multiple proportions

(D)  Law of reciprocal proportions

Answer: (B)

96. An atlas of India is drawn by taking scale 100 cm = 50,000 km. The actual distance between the city of Bhopal and Cochin is 1,500 km, the distance between the two places in the atlas will be ______ cm.

(A)  3

(B)  1

(C)  10

(D)  2

Answer: (A)

97. Both sound and light waves can be propagated through

(A)  Vacuum

(B)  Air

(C)  Both (A) and (B)

(D)  None of the above

Answer: (B)

98. A man stands in front of a mirror and finds that his image is larger than himself. The mirror is a ________ mirror.

(A)  Convex

(B)  Concave

(C)  Plane

(D)  Both (A) and (B)

Answer: (B)

99. Each lung is enclosed in a double membrane called as pleura. The membrane which covers the surface of each lung is

(A)  Visceral pleura

(B)  Lung pleura

(C)  Peritoneal pleura

(D)  Parietal pleura

Answer: (A)

100. Nitrogenous waste products are eliminated mainly as _______

(A)  Urea in tadpole and ammonia in adult frog

(B)  Ammonia in tadpole and urea in adult frog

(C)  Urea in tadpole and adult frog

(D)  Urea in tadpole and uric acid in adult frog

Answer: (B)

Section D : Social Science

101. Who granted East India Company the sole right to trade with the East?

(A)  Robert Clive

(B)  Queen Elizabeth I

(C)  John Richardson

(D)  Queen Elizabeth II

Answer: (B)

102. The first Indian woman to become President of the Indian National Congress was:

(A)  Sarojini Naidu

(B)  Kamla Nehru

(C)  Kasturba Gandhi

(D)  Begum Rokeya Shakhawat Hossain

Answer: (A)

103. Land covered with grass shrubs on which animals can graze freely is known as:

(A)  Fallow land

(B)  Overgrazing

(C)  Pasture

(D)  Agricultural land

Answer: (C)

104. Name of the first country in the world to develop hydroelectricity.

(A)  Norway

(B)  Pakistan

(C)  India

(D)  Switzerland

Answer: (A)

105. Breeding of fish in specially constructed tanks and ponds is known as:

(A)  Agriculture

(B)  Sericulture

(C)  Pisciculture

(D)  Viticulture

Answer: (C)

106. Ahmedabad is referred as the ________ of India.

(A)  Ruhr

(B)  Manchester

(C)  Boston

(D)  Chicago

Answer: (B)

107. How many members are nominated by the President to the Rajya Sabha?

(A)  233

(B)  12

(C)  22

(D)  250

Answer: (B)

108. The Industrial Revolution started in ________ around 1750.

(A)  France

(B)  Britain

(C)  Russia

(D)  None of these

Answer: (B)

109. The British conquest of Bengal began with the Battle of _______.

(A)  Buxar

(B)  Plassey

(C)  Seringapatnam

(D)  Saraighat

Answer: (B)

110. The architect of New Delhi was ______

(A)  Edwin Lutyens & H Baker

(B)  King George V

(C)  Lord Lytton

(D)  Queen Elizabeth

Answer: (A)

111. The existence of more than one level of government is known as

(A)  Federalism

(B)  Secularism

(C)  Marginalism

(D)  Communalism

Answer: (A)

112. Money Bill is introduced

(A)  Only in a Rajya Sabha

(B)  Only in  Lok Sabha

(C)  In both the Houses

(D)  By the Speaker of Rajya Sabha

Answer: (B)

113. In which state of India is the “Jim Corbett National Park” situated?

(A)  Uttarakhand

(B)  Gujarat

(C)  Assam

(D)  Uttar Pradesh

Answer: (A)

114. Which state of India is the highest producer of Jute?

(A)  Assam

(B)  West Bengal

(C)  Bihar

(D)  Odisha

Answer: (B)

115. A form of government where people enjoy equal political right, elect their ruler and hold them accountable is known as:

(A)  Secular

(B)  Democratic

(C)  Socialist

(D)  Republic

Answer: (B)

116. In order to prevent religion based exclusion and discrimination of ‘lower castes’, the Indian Constitution bans

(A)  Untouchability

(B)  Religious practices

(C)  Religion

(D)  Religious institutions

Answer: (A)

117. Sustainable development seeks to prevent _______.

(A)  Wastage of resources

(B)  Pollution

(C)  Loss of biodiversity

(D)  All of these

Answer: (D)

118. Of the earth’s total water resources, the fresh water easily accessible for our use is

(A)  More than 90%

(B)  50%

(C)  1%

(D)  Less than 1%

Answer: (C)

119. These are made to protect our natural vegetation and wild life

(A)  National park

(B)  Wildlife sanctuaries

(C)  Biosphere reserves

(D)  All of these

Answer: (D)

120. The largest producer and exporter of mica in the world is

(A)  Australia

(B)  India

(C)  USA

(D)  Russia

Answer: (B)

121. Resources which can be renewed or reproduced are known as________

(A)  Exhaustible resource

(B)  Non renewal source

(C)  Renewal source

(D)  Useful resource

Answer: (C)

122. The Act on the “Protection of Women from Domestic violence” finally became a law in the year:

(A)  2003

(B)  2004

(C)  2005

(D)  2006

Answer: (C)

123. Right to Information (RTI) act guarantees people’s right to

(A)  Know governmental proceedings

(B)  Get universal primary education

(C)  Speak out their discontent freely

(D)  Hold meetings and public gathering

Answer: (A)

124. At the village level, the judicial functions are performed by the

(A)  Nyaya Panchayat

(B)  Gram Panchayat

(C)  District Judge

(D)  Munsif

Answer: (A)

125. Many of India’s most important mining and industrial centre are located in

(A)  Jamshedpur

(B)  Rourkela

(C)  Bokaro

(D)  All of these

Answer: (D)

Paper-II

Section E : Intelligence Test

Directions (Qs. No. 126-129) Choose the letters group that best represents a relationship similar to the one expressed in the original pair of letters group in the following.

126. FILM : ADGH :: MILK : ?

(A)  ADGF

(B)  HDGE

(C)  HDGF

(D)  HEGF

Answer: (C)

127. EIGHTY : GIEYTH :: OUTPUT : ?

(A)  TUOTUP

(B)  TUOUTP

(C)  UTOPTU

(D)  UOTUPT

Answer: (A)

128. CAT is to DDY as BIG is to

(A)  CLL

(B)  CLM

(C)  CML

(D)  CEP

Answer: (A)

129. Bag is related to Laggag in the same way as Ship is related to ________?

(A)  Coal

(B)  Stock

(C)  Cargo

(D)  Weight

Answer: (C)

Directions (Qs. No. 130 & 131): Which number completes the second pair in the same way as the first pair.

130. 3 : 243 :: 5 : ____?

(A)  425

(B)  465

(C)  546

(D)  3125

Answer: (D)

131. 6 : 24 :: 5 : ?

(A)  23

(B)  20

(C)  26

(D)  22

Answer: (B)

132. Find the missing character from among the given alternatives.

(A)  J15

(B)  K8

(C)  K15

(D)  L10

Answer: (B)

133. Choose the figure, which is different from others

(A)  1

(B)  2

(C)  3

(D)  4

Answer: (D)

134. Arrange the following words as per order in the dictionary.

(1) Live    (2) Litter

(3) Little  (4) Literacy

(5) Living

(A)  3, 4, 2, 1, 5

(B)  3, 2, 4, 5, 1

(C)  4, 3, 5, 2, 1

(D)  4, 2, 3, 1, 5

Answer: (D)

135. In a certain code, TEACHER is written as VGCEJGT. How is CHILDREN written in that code?

(A)  EJKNEGTP

(B)  EGKNFITP

(C)  EJKNFGTO

(D)  EJKNFTGP

Answer: (D)

136. If 30th January, 2003 was Thursday, what was the day on 2nd March, 2003?

(A)  Sunday

(B)  Monday

(C)  Saturday

(D)  Tuesday

Answer: (A)

137. In the following series, how many KGN occur in such a way that ‘G’ is in the middle and ‘K’ and ‘N’ are adjacent to it on both sides?

A K G L M N D Q K G C S N G K T G k G N D Z P U X G K E

(A)  5

(B)  3

(C)  1

(D)  2

Answer: (*)

138. Which word can’t be formed by using the letters of the DISTRIBUTION word? Find that word.

(A)  TRUST

(B)  SITUATION

(C)  TUITION

(D)  DISTURB

Answer: (B)

139. Select the figure from the answer set that would come in place of the question mark (?).

(A)  1

(B)  4

(C)  3

(D)  2

Answer: (A)

140. If Maya is the only daughter of Richa’s grandmother’s brother, how is Maya’s daughter related to Richa?

(A)  Niece

(B)  Cousin

(C)  Aunt

(D)  Mother

Answer: (B)

141. It is 3 O’clock in a watch and it is rotated by 10 degrees in manner such that if the minute hand points towards the North-East, then hour hand will point towards which direction?

(A)  South

(B)  South-West

(C)  North-West

(D)  South-East

Answer: (D)

142. Which one does not belong to the group?

(A)  63

(B)  65

(C)  84

(D)  91

Answer: (B)

143. Which square would replace the question mark?

(A) 

(B) 

(C) 

(D) 

Answer: (D)

144. Village ‘W’ is 20 kms to the north to village ‘X’. Village ‘Y’ is 18 kms to the east of village ‘X’. Village ‘Z’ is 12 ms to the west of village ‘W’. If Manu starts from village ‘Y’ and goes to village ‘Z’ via village ‘W’, in which direction is he from his starting point?

(A)  North-West

(B)  South

(C)  North-East

(D)  East

Answer: (A)

145. If the 7th day of a month is 3 days earlier than Friday, what day will it be one the 19th day of the month?

(A)  Monday

(B)  Sunday

(C)  Wednesday

(D)  Friday

Answer: (B)

146. Gaurav said to Tarun, “That boy playing with the football is the younger of the two brothers of the daughter of my father’s wife”. How is the boy playing football related to Gaurav?

(A)  Son

(B)  Brother

(C)  Cousin

(D)  Brother-in-law

Answer: (B)

Directions (Qs. No. 147): Study the following arrangement of the English alphabet and answer the question given below:

F J M P O W R N B E Y C K A V L D C X U H Q I S Z T

147. Which of the following pairs of letters has as many letters between them in the above arrangement as there are between them in the English alphabet?

(A)  AI

(B)  EL

(C)  LS

(D)  MO

Answer: (D)

Directions (Qs. No. 148-150): A solid cube of each side 4 cm has been painted all faces. If it is then cut into cubical blocks each of side 2 cm, answer the question:

148. How many cubes are there in all of a edge 2 cm?

(A)  2

(B)  4

(C)  8

(D)  16

Answer: (C)

149. How many cubes have no face painted?

(A)  0

(B)  2

(C)  4

(D)  8

Answer: (A)

150. How many cubes have only one face painted?

(A)  0

(B)  2

(C)  4

(D)  8

Answer: (A)

Dr. YSR Aarogyasri Health Care Trust Recruitment 2020 Aarogya Mithra & Team Leader Jobs in Prakasam

Name of the Posts: Aarogya Mithra, Team Leader

Last Date for Apply: 03-11-2020

Total Vacancy: 60

Dr. YSR Aarogyasri Health Care Trust

Aarogya Mithra & Team Leader Jobs in Prakasam

Application Fee

Nil

Important Dates

Date of Advertisement: 30/10/2020

Start Date for Apply Online Application: 01/11/2020

Last Date for Apply Online Application: 03/11/2020

Educational Qualification

B.Sc Nursing, M.SC Nursing, B Pharmacy, Pharmacy D, B.Sc Medical Lab Technology

Vacancy Details

Sl No Name of the Position No. of posts
1 Aarogya Mithra 54
2 Team Leader 6
 

Total

60

Detailed Notification: Detailed Notification

Apply Online: To Apply Online Application

Official Website:  Prakasam District Website

Back to Main Page

Dr. YSR Aarogyasri Health Care Trust Recruitment 2020 Aarogya Mithra & Team Leader Jobs in Prakasam PDF Download Below

Bureau of Indian Standards (BIS) Recruitment 2020 Group A, B, C Posts

Name of the Posts: Senior Secretariat Assistant, Junior Secretariat Assistant, Assistant Section Officer

Last Date for Apply: 26/09/ 2020

Total Vacancy: 171

Bureau of Indian Standards (BIS)

Group A, B, C Posts

Application Fee

For Assistant Director Posts: Rs. 800/-

For Remaining Posts: Rs. 500/-

For SCs/STs/PWDs/ Women and BIS serving employees: Nil

Payment Mode: Online

Important Dates

Start Date of Online Registration: 05.09.2020

Last Date of Online Registration: 26.09.2020

Online Examination Date: 08/10/2020

Age Limit as 01.09.2020

Maximum Age – 35 years

Age relaxation is applicable as per rules.

Educational Qualification

Degree / Chartered Accountant / MBA

Vacancy Details

Sl No Post Name No of Posts
1 Assistant Director 04
2 Assistant Section Officer 17
3 Personal Assistant 16
4 Junior Translator 01
5 Library Assistant 01
6 Stenographer 17
7 Senior Secretariat Assistant 79
8 Junior Secretariat Assistant 36
Total            171

Detailed Notification: Detailed Notification

Apply Online: Online Apply

Official Website: BIS Website

Back to Main Page

Bureau of Indian Standards (BIS) Recruitment 2020 Group A, B, C Posts PDF Download Below

All Fourteen 8000+ Meters Mountain Peaks in World

Mountain

(other names)

Location Height First to summit (nationality) Date
Meters Feet
1. Mount Everest*

(Sagamartha, Chomolangma or Qomolangma)

Nepal/Tibet 8,850 29,035 Edmund Hillary (New Zealander, UK), Tenzing Norgay (Nepalese) May 29, 1953
2. K2

(Mount Godwin-Austen or Chogori or Dapsang)

Pakistan/China 8,611 28,250 A. Compagnoni, L. Lacedelli (Italian) July 31, 1954
3. Kangchenjunga

(Kanchanfanga)

Nepal/India 8,586 28,169 G. Band, J. Brown, N. Hardie, S. Streather (UK) May 25, 1955
4. Lhotse Nepal/Tibet 8,516 27,940 F. Luchsinger, E. Reiss (Swiss) May 18, 1956
5. Makalu

(Makalufeng)

Nepal/Tibet 8,463 27,766 J. Couzy, L. Terray, J. Franco, G. Magnone-Gialtsen, J. Bouier, S. Coupé, P. Leroux, A. Vialatte (French) May 15, 1955
6. Cho Oyu Nepal/Tibet 8,201 26,906 H. Tichy, S. Jöchler (Austrian), Pasang Dawa Lama (Nepalese) Oct. 19, 1954
7. Dhaulagiri

(Aulagiri)

Nepal 8,167 26,795 A. Schelbert, E. Forrer, K. Diemberger, P. Diener (Swiss), Nyima Dorji, Nawang Dorji (Nepalese) May 13, 1960
8. Manaslu

(Kutang)

Nepal 8,163 26,781 T. Imamishi, K. Kato, M. Higeta, (Japanese) G. Norbu (Nepalese) May 9, 1956
9. Nanga Parbat

(Diamir)

Pakistan 8,125 26,660 Hermann Buhl (Austrian) July 3, 1953
10. Annapurna

(Morshiadi)

Nepal 8,091 26,545 M. Herzog, L. Lachenal (French) June 3, 1950
11. Gasherbrum I

(Hidden Peak or K5)

Pakistan/China 8,068 26,470 P. K. Schoeing, A. J. Kauffman July 4, 1958
12. Broad Peak

(Falchen Kangri or K3)

Pakistan/China 8,047 26,400 M. Schmuck, F. Wintersteller, K. Diemberger, H. Buhl (Austrian) June 9, 1957
13. Gasherbrum II

(K4)

Pakistan/China 8,035 26,360 F. Moravec, S. Larch, H. Willenpart (Austrian) July 7, 1956
14. Shisha Pangma

(Gosainthan or Xixabangma)

Tibet 8,013 26,289 Hsu Ching and team of 9 (Chinese) May 2, 1964

* The 1955 elevation of Everest, 29,028 ft. (8,848 m), was revised on Nov. 11, 1999, and now stands at 29,035 ft. (8,850 m).

 

MP Test

Shri. T.P.R. Selvame
Smt. B. Kobiga
Shri. E. Theeppainthan
Shri. S.V. Sugumaran
Shri. A. Namassivayam
Shri. Mnr. Balan
Shri. N.S.J. Jayabal @ Ayyanar
Shri. N. Rangasamy
Shri.K. VENKATESAN
VACANT
Shri. V.P. Sivakolundhu
Shri. M.O.H.F. Shahjahan
Shri. Vaiyapuri Manikandan
Shri. K. Lakshminarayanan
Shri. A. Anbalagan
Shri. R. Siva
Shri. V. NARAYANASAMY
Shri. A. Baskar
Shri. T. Djeamourthy
Shri. R.K.R. Anantharaman
Shri. M. KANDASAMY
Smt. V. VIZEAVENY
Shri. N. DANAVELU
Smt. S. CHANDIRA PRIYANGA
Shri. R. Kamalakkannan
Shri. P.R.N. Thirumurugan
Shri. K.A.U. Asana
Smt. A.Geetha
Dr. V.Ramachandran
Shri. Malladi Krishna Rao

National Institute of Fashion & Technology (NIFT) Under Graduate 2018 Question Paper With Answer Key

NIFT (Under Graduate) Solved Paper-2018

1. The average weight of a group of 20 boys was calculated to be 89.4 kg and it was later discovered that one weight was misread as 78 kg instead of 87 kg. The correct average weight is

(a) 88.95 kg

(b) 89.25 kg

(c)  89.55 kg

(d) 89.85 kg

Answer: (d)

2. A can contains a mixture of two liquids A and B in the ratio 7 : 5. When 9L of mixture are drained off and the can is filled with B, the ratio of A and B becomes 7 : 9. How many litres of liquid A was contained by the can initially?

(a) 10

(b) 20

(c)  21

(d) 25

Answer: (c)

3. The sum of first 50 odd natural number is

(a) 1000

(b) 1250

(c)  5200

(d) 2500

Answer: (d)

4. The number (6x2 + 6x) for natural number x is always divisible by

(a) 6 and 12

(b) Only 12

(c)  Only 6

(d) Only 3

Answer: (a)

5. By selling a bicycle for Rs 2850, a shopkeeper gains 14%. If the profit is reduced to 8%, then the selling price will be

(a) Rs 2600

(b) Rs 2700

(c)  Rs 2800

(d) Rs 3000

Answer: (b)

6. The least number, which is to be added to the greatest number of 4 digits so that the sum may be divisible by 345, is

(a) 50

(b) 6

(c)  60

(d) 5

Answer: (b)

7. If a number is much greater than 31 as it is less than 75, then the number is

(a) 44

(b) 74

(c)  53

(d) 106

Answer: (c)

8. Find the area of ∆ ABC whose vertices are A(10, −6), B(2, 5) and C(−1, 3).

(a) 12 sq units

(b) 37/2 sq units

(c)  455 sq units

(d) 49/2 sq units

Answer: (d)

9. Thrice the square of a natural number decreased by four times the number is equal to 50 more than the number. The number is

(a) 4

(b) 5

(c)  10

(d) 6

Answer: (b)

10. If the distance between the points (x, 0) and (−7, 0) is 10 units, then the possible values of x are

(a) 3 and 17

(b) −3 and 17

(c)  3 and −17

(d) −3 and −17

Answer: (c)

11. The sum of two number is 8 and their production is 15. The sum of their reciprocals is

(a) 8/15

(b) 15/8

(c)  23

(d) 7

Answer: (a)

12. Equal sums of money are lent to X and Y at 7.5% per annum for period of 4 yr and 5 yr, respectively. If the difference in interest paid by them was Rs 150, the sum lent to each was

(a) Rs 500

(b) Rs 1000

(c)  Rs 2000

(d) Rs 3000

Answer: (c)

13. A man ate 100 grape in 5 days. Each day, he ate 6 more grapes than those he ate on the earlier day. How many grapes did he eat on the first day?

(a) 8

(b) 12

(c)  54

(d) 76

Answer: (a)

14. A number whose one-fifth part increased by 4 is equal to its one-fourth part diminished by 10, is

(a) 260

(b) 280

(c)  240

(d) 270

Answer: (b)

15. The sum of two numbers is 42 and their product is 437. Then, the absolute difference between the numbers is

(a) 3

(b) 4

(c)  5

(d) 7

Answer: (b)

16. The average salary of all the workers in a workshop is Rs 8000. The average salary of 7 technicians to Rs 12000 and the average salary of the rest is Rs 6000. The total number of workers in the workshop is

(a) 20

(b) 21

(c)  22

(d) 23

Answer: (b)

17. I paid 3/5 of a bill. If Rs 400 of the bill amount is still due, what was the total amount of the bill?

(a) Rs 1200

(b) Rs 1500

(c)  Rs 1800

(d) Rs 1000

Answer: (d)

18. A start a business with Rs 4000. B joins him after 3 months with Rs 8000. C puts a sum of Rs 20000 in the business for 2 months only. At the end of year, the business gives a profit of Rs 16800. How should the profit be divided amongst them?

(a) A = Rs 5040, B = 7560, C = Rs 4200

(b) A = Rs 7560, B = 5040, C = Rs 4200

(c)  A = Rs 4200, B = Rs 7560, C = Rs 5040

(d) A = Rs 4200, B = Rs 5040, C = Rs 7560

Answer: (a)

19. 2/3 is a rational number whereas √2/√3 is

(a) also a rational number

(b) an irrational number

(c)  not a number

(d) a natural periodic number

Answer: (b)

20. A rational number is expressed as ……, where p and q are integers and q ≠

(a) pq

(b) p + q

(c)  p – q

(d) p/q

Answer: (d)

21. For what value of p, the following pair of linear equations in two variables will have infinitely many solutions?

px + 3y – (p – 3) = 0

       12x + py – p= 0

(a) 6

(b) −6

(c)  0

(d) 2

Answer: (a)

22. A and B can do a job together in 12 days. A is 2 times as efficient as B. In how many days can B along complete the work ?

(a) 18

(b) 9

(c)  36

(d) 12

Answer: (c)

23. The cost of carpeting of a room is Rs 120. If the width had been 4 m less, the cost of the carpet would have been Rs 20 less. The width of the room is

(a) 24 m

(b) 20 m

(c)  25 m

(d) 18.5 m

Answer: (a)

24. Two pipes (A and B) can fill a cistern in 3 h and 4 h, respectively and a waste pipe C can empty it in 2h. If all the three pies are kept open, then the cistern will be filled in

(a) 5 h

(b) 8 h

(c)  10 h

(d) 12 h

Answer: ()

25. A path of uniform width runs round the inside of a rectangular field 38 m long and 32 m wide. If the path occupies 600 m2, then the width of the path is

(a) 30 m

(b) 5 m

(c)  18.75 m

(d) 10 m

Answer: (b)

26. A certain number when divided by 175 leaves a remainder 132. When the same number is divided by 25 the reaminder is

(a) 6

(b) 7

(c)  8

(d) 9

Answer: (b)

27. A and B invest Rs 3000 and Rs 4000, respectively in a business. A receives Rs 10 per month out of the profit as a remuneration for running the business and the rest of the profit is divided in proportion to the investments. If in a year A totally receives Rs 390, what does B receive?

(a) Rs 630

(b) Rs 360

(c)  Rs 480

(d) Rs 3800

Answer: (b)

28. A man losses 20% of his money. After spending 25% of the remainder, he has Rs 480 left. What is the amount of money he originally had?

(a) Rs 600

(b) Rs 720

(c)  Rs 800

(d) Rs 840

Answer: (c)

29. The area of the square park is Rs 25 sq km. The time taken to complete a round the field once, at a speed of 3 km/h

(a) 4 h 60 min

(b) 4 h 50 min

(c)  6 h 40 min

(d) 5 h 40 min

Answer: (c)

30. A bag contains Rs 1, 50 paise and 25 paise coins in the ratio of 8 : 9 : 11. If the total money in the bag is Rs 366, then find the number of 25 paise coins.

(a) 264

(b) 364

(c)  241

(d) 245

Answer: (a)

DIRECTIONS (Q. Nos. 31-33) In each of the following questions, an idiomatic expression/proverb has been given, followed by four alternatives. Choose the one which best expresses the meaning of the given idiom/proverb.

31. In black and white

(a) in short

(b) useless

(c)  in writing

(d) in full swing

Answer: (c)

32. Wet behind the cars

(a) stupid and slow-witted

(b) young and inexperienced

(c)  drenched in the rain

(d) unpunctual and lethargic

Answer: (b)

33. To rock the boat

(a) to conspire against

(b) to agitate against

(c)  to upset the balance

(d) to create difficulties

Answer: (d)

DIRECTIONS (Q. Nos. 34-35) In each of the following questions below, only one among the given alternatives is correctly spelt. Find out the word with correct spelling.

34.

(a) Explanation

(b) Explination

(c)  Explaination

(d) Explainsion

Answer: (a)

35.

(a) Posesive

(b) Possessive

(c)  Possesive

(d) Posessive

Answer: (b)

Directions (Q. Nos. 36-37) In each of the following questions, choose the wrongly spelt word.

36.

(a) Aptitude

(b) Audible  

(c)  Anonimus

(d) Resignation

Answer: (c)

37.

(a) Beautiful

(b) Carpenter

(c)  Brevity

(d) Futerstic

Answer: (d)

DIRECTIONS (Q. Nos. 38-39) From amongst the options given below each word, choose the appropriate singular form.

38. Phenomenon

(a) Phenomenos

(b) Phenomenas

(c)  Phenomena

(d) Phenomenons

Answer: (c)

39. Machineries

(a) Machinery

(b) Machinerye

(c)  Machine

(d) Machineri

Answer: (a)

DIRECTIONS (Q. Nos. 40-41) From amongst the options given below each word, choose the appropriate plural form.

40. Goose

(a) Gooses

(b) Geese

(c)  Gease

(d) Geeses

Answer: (b)

41. Formula

(a) Formula

(b) Formulas

(c)  Formulae

(d) Formulaes

Answer: (c)

DIRECTIONS (Q. Nos. 42-45) In each of these question, choose the alternative which can replace the underlined word without changing the meaning of sentence.

42. Don’t light matches near inflammable

(a) non-flammable

(b) combustible

(c)  fire

(d) lightening

Answer: (b)

43. The sighting of the flying saucer was inexplicable.

(a) mysterious

(b) amusing

(c)  amazement

(d) enjoyable

Answer: (a)

44. Not even experts could figure out the magician’s clever

(a) original

(b) unimaginative

(c)  ingenious

(d) no replacement

Answer: (c)

45. Many have tried to copy the painter’s unique

(a) simple

(b) no replacement

(c)  ordinary

(d) inimitable

Answer: (d)

46. Johnny is a keen player but unfortunately he has ……….. skills.

(a) few

(b) none

(c)  some

(d) little

Answer: (a)

47. I didn’t have ……… trouble getting the passports. I only had a problem with my photo because it was an old one.

(a) any

(b) much

(c)  no

(d) several

Answer: (b)

48. When we get ready for dinner, I have to take my books …….. t he table.

(a) off

(b) from

(c)  out

(d) of

Answer: (a)

49. The lecture will be held right …………. the tutorial.

(a) at

(b) before

(c)  over

(d) beyond

Answer: (b)

Directions (Q. Nos. 50-53) In each of the following questions, choose the alternative which is opposite in meaning of the underlined word.

50. He has been living in prosperity right since his childhood.

(a) sickness

(b) poverty

(c)  humility

(d) grandeur

Answer: (b)

51. He is of a very affable

(a) humble

(b) irritating

(c)  unfriendly

(d) None of these

Answer: (c)

52. The rich fellow saved his money because he has not earned it himself.

(a) hoard

(b) developed

(c)  collected

(d) dissipated

Answer: (d)

53. I asked to repeat the performance by the spectators.

(a) participants

(b) bystander

(c)  observers

(d) None of these

Answer: (a)

DIRECTIONS (Q. Nos. 54-55) In each of the following questions, choose the alternative that can be substituted for the given sentence.

54. A constructor of crossword puzzles; also an enthusiast of word games, especially cross word puzzles

(a) Lexicographer

(b) Dittograph

(c)  Cruciverbalist

(d) Ambigram

Answer: (c)

55. One filled with excessive and mistaken enthusiasm in cause.

(a) Pedant

(b) Fanatic

(c)  Patriot

(d) Martyr

Answer: (b)

PASSAGE I

The superiority of the West is due to its intellectual integrity, the sincerity of its pursuit of truth. From the times of Socrates, the seeker of truth, down till today, Western mind, with rare exceptions, has been remarkably free from self complacency. Intellectual laziness and blind faith in ancients wisdom. It has never lost the sense of wonder, the eager curiosity to know and find out for oneself by observation and experiment, the truth of the near and far. Science has made life easier for millions of men and women. Our educators tried to impart how to use the scientific mind. The possession of the scientific spirit devices. We may be able to use the telephone and the wireless, the railway and the aeroplane, and yet be lacking in the scientific mind which is something organic and structural and not merely outward and decorative. The scientific spirit must be applied not only to be ordinary arrangement of life and leisure, to be distribution of materials goods, to the improvement of industry and agriculture, but also to the thing which touch the mind and the morals of the community.

Our scientific conscience must b e shocked not only by the contrasts of fabulous wealth and gruelling poverty but also by those of intense holiness and diseased superstition. In our relations with the another, we have failed to apply scientific and social wisdom. The failure is written large on our society. Social abuses like untouchability are tolerated simply because the spirit in us in supposed by the force of custom. They are practised by otherwise kind persons who have ceased to feel and whose understanding is petrified by tradition. There are millions in our country today who use scientific devices and yet revere superstition as mystical revelation and adhere to absurd social customs in the name of tradition. We owe our reverence to the seekers of truth, who conquer our minds by the spirit of truth and not be conformists, who enslave our minds in the name of tradition. Tradition cannot ever supersede truth, conscience cannot be silenced by scripture. We must clear mind of the cobwebs which are found there.

56. To what does the West owe its superiority over the East?

(a) Belief in material values

(b) Dignity of the individual

(c)  Intellectual discipline and disinterested search for truth

(d) Belief in moral and spiritual values

Answer: (c)

57. What do you mean by self complacency?

(a) Vanity

(b) Self-reliance

(c)  False sense of superiority self assurance

(d) Tendency of giving importance to self

Answer: (c)

58. Why is our scientific conscience shocked not only by the contrasts of fabulous wealth and gruelling poverty but also by those of intense holiness and diseased superstition?

(a) Because reason does not justify the intellectuality.

(b) Because these things are immoral.

(c)  Because these things are out of accord with our spiritual values of life.

(d) Because superstition kills open enquiry and encourages the tendency of accepting things as they are.

Answer: (a)

59. What do you mean by the expression ‘ the failure is written large on our society’?

(a) The society is suffering the consequences of the lack of the scientific spirit on our part.

(b) Not individuals but society has failed in coming out of grooves of dead habits.

(c)  The failures are recorded in the history books.

(d) The account of failures of society is written in large letters.

Answer: (a)

60. ‘Whose understanding is petrified by tradition’? What do you mean by this expression?

(a) The weight of customs and traditions is so great that their reason does not work to get at the reality.

(b) In course of time their understanding has left them and they have become totally insensitive to reason.

(c)  They are too much given to the worship of old ways and customs.

(d) Their common sense has been converted into stone.

Answer: (a)

PASSAGE II

Science, is so far as it consists of knowledge, must be regarded as having value, but in so far as it consists a technique, the question whether it is to be praised or blamed depends upon the use that is made of technique. In itself, it is neutral, neither good nor bad. Any ultimate view that we may have about what gives vale of this or that must come from some other source than science.

But good and evil almost, everything that distinguishes our age from its predecessors is due to science. In daily life we have electricity, the radio and the cinema. Because of the increased productivity of labour, we are able to devote a far greater proportion of our energies to war and preparation for wars that was formerly possible and we are able to keep the young in school very much longer than we formerly could. Owing to science we are able to disseminate information through the press and the radio practically to everybody. We can make it more enormously difficult that it used to the people whom the government dislikes to escape.

It would not be surprising if in the present day; a powerful anti-scientific movement were to arise as a result of the dangers to human life that are resulting from atom moms and may result from bacteriological warfare. But whatever people may feel about these horrors, they dare not turn against the men of science so long as war is at all probable because of one side equipped with scientists and the other not, the scientific side would almost certainly win.

61. What determines whether science is good or bad?

(a) Its analytical study

(b) Its Philosophical-conception

(c)  The manners of its origin

(d) The application in life

Answer: (d)

62. Which conceptions of science has the author mentioned in the passage?

(a) Its being good or bad.

(b) Its being a purely empirical enquiry.

(c)  Its being both a philosophical and empirical study.

(d) Science as knowledge and science as technique

Answer: (d)

63. If anti-science movement were to arise in the preset day what factor or factors according to the author would he held responsible for it?

(a) The misapplication of science

(b) Man’s destructive propensities

(c)  Man’s pugnacious instinct and h is desire for domination.

(d) Dangers to human life resulting from atom bombs and the, likely result from the bacteriological warfare.

Answer: (d)

64. How long are people not likely to turn against scientist?

(a) Until science is not fully developed.

(b) So long as war is at all probable.

(c)  Until the lot of mankind is ameliorated.

(d) So long as we are dominated by fighting instinct.

Answer: (b)

65. How is it most difficult in the present day for men disliked by the government to escape its clutches?

(a) A network of information media operating throughout the world will enable the government to catch hold of such persons.

(b) A highly efficient system of secret service will trace out the whereabouts of such persons.

(c)  People in the present day have become so slothful that they would not like to involve danger to their security.

(d) Such persons will find it difficult to obtain asylum anywhere in the world of today dominated of selfishness.

Answer: (b)

PASSAGE III

Edmund Burke called ‘the Press- the Fourth Estate of the realm, ‘I think he did not use his title for the Press thoughtlessly as social ruling group or class. The three Estates or Realms is England are the Lord Spiritual i.e. the Bishops in the House of Lords, the Temporal i.e. other Lords and Commons i.e., the common people. The Press has been rightly called the Fourth Estate as it also constitutes a ruling group or class like the Lords and Common. It cannot be denied in a free country, that the Press exercises a good deal of influence in shaping public opinion and pointing out the weakness or defects of society or of government and, in general bringing to light all those good or bad things in society which would have otherwise remained unnoticed. The power is not limited or put under any check. The Press instead of being controlled by any one, controls life and thought of a nation. Hence, the Press constitutes an state by itself. Obviously the power which the press in any country wields depends upon the number of newspaper readers. The opinions and comments of newspaper can influence the life of a nation only when they are read by people reading, in turn, requires that the general mass of people should be educated. Thus, the spread, of education determines the extent of the newspapers. Where readers are few, newspapers must necessarily be few. Their influence in the case, can extend, only to a small minority of population. In a country like India the percentage of literacy is very low and standard of journalism is not very high. So, the Press has to play the role of a teacher here.

66. What title was given by Edmund Burke to the Press?

(a) Instrument of public opinion

(b) Distributor of news

(c)  The Fourth Estate

(d) Lords Temporal

Answer: (c)

67. How much power does a free Press possess?

(a) Only that much which is allowed by government of the country.

(b) Unlimited power without any check.

(c)  Unlimited power subject to maintenance of law and order and public morality.

(d) No power at all

Answer: (c)

68. What is the secret power of the Press?

(a) The money which the newspaper owners can wield

(b) The number of newspaper readers

(c)  The extent to which it support official policy

(d) The patronage of the government enjoyed by it

Answer: (b)

69. What determines the number of newspaper readers?

(a) The low price of newspaper

(b) The patronage extended to it by the moneyed people

(c)  Education of the general masses of people

(d) The availability of newsprint to the newspaper owners

Answer: (c)

70. How does the press exercise power?

(a) It enlists the support of the people.

(b) It keeps watch over the acts of the Government.

(c)  It controls life and thought of a nation.

(d) It is a great materialistic concern.

Answer: (c)

PASSAGE IV

The emotional appeal of imperialism never completely stilled the British conscience, However, liberal thinks throughout the nineteenth century argued that democracy was incompatible with the maintenance of authoritarian rule over foreign people. To think empirically was to think in terms of restrictive and protective measures, in defiance of the revealed truths of classical economics. Thus, when the British government took over responsibility for India from the East India Company in 1858, many politicians were conscious of saddling Britain with a heavy burden. In the first seventy years of the nineteenth century, enlightened British liberals looked forward to the day when India would stand on its own feet. Even in the days of colonialism, British radicals continued to protest. The self proclaimed imperialists, however honourable their motives, would place fait accompli before the country and commit blunder of incalculable consequence. The danger they felt was all the greater because British foreign policy still remained a stronghold of the aristocracy, while that related and persuasive lobby, the British officer class also had a vested interest in imperial expansion.

It took the humiliation of the Boer to teach the British government what it would cost to hold an empire by force. However, this fact did not escape Gandhi, the supreme tactician of the Indian liberation movement. He saw that some perceptive British thinkers had much earlier recognised that Britain could not continue long to rule India except with the co-operations of may sections of its population. Once that co-operation was withdrawn, the foundation of British authority in India would crumble. Furthermore, the Indian nationalist leaders were able to exploit the aversion of the British liberal conscience to method used by the local colonial rulers in combating Indian non-cooperation.

71. ‘The emotional appeal of imperialism never completely stilled the British conscience’ What does it mean?

(a) The glorification resulting from imperialism never completely blinded the British people to the unjustness of their policy.

(b) The glorification resulting from the establishment of sway overseas did not make the British people complacent.

(c)  The British people became so dull and emotionally so cold that the glorification of their country attained through imperialism never stirred them of feel pround of this phenomenon.

(d) All of the above

Answer: (a)

72. What does the term authoritarian rule mean?

(a) Rule of authority of law

(b) Dictatorial rule unaccompanied by the rule of law

(c)  Arbitrary exercise of power by officials

(d) Rule having stability

Answer: (b)

73. What according to the author did the maintenance of imperialism result in?

(a) Economic exploitation of the subject people

(b) Mental enslavement of the subject people

(c)  The extinction of the sense of initiative in the people ruled by the imperialists

(d) Erection of trade barriers in the foreign trade and other restrictive measures

Answer: (d)

74. What do you think were the revealed truths of classical economics?

(a) Laissez fair and free trade

(b) Allowing only subsistence wages to the workers.

(c)  Clamping of artificial restriction on foreign trade.

(d) Wholesale nationalisation of the means of production.

Answer: (a)

75. What was the opinion of the British liberal thinkers regarding Britain’s continuing to rule India?

(a) That it could hold on only by force of arms.

(b) That it could maintain its rule only by following a policy of persuasion.

(c)  That British rule would collapse when the Indian masses rose in revolt against her.

(d) Thus, Britain could continue to rule India only with co-operation of many sections of its population.

Answer: (d)

DIRECTIONS (Q. Nos. 76-79) Each of these questions has an Assertion (A) and a Reason (R).

(a) If both ‘A’ and ‘R’ are true and ‘R’ is the correct explanation of ‘A’

(b) If both ‘A’ and ‘R’ are true but ‘R’ is not the correct explanation of ‘A’

(c) If ‘A’ is true but ‘R’ is false

(d) If ‘A’ is false but ‘R’ is true

76. Assertion (A) In India, the Judiciary is independent of the Executive.

Reason (R) Judiciary favours the Government and helps in the implementation of its plans.

Answer: (c)

77. Assertion (A) Clothes are not washed properly in hard water.

Reason (R) Hard water contains many minerals.

Answer: (b)

78. Assertion (A) Carbon monoxide when inhaled caused death.

Reason (R) Carbon monoxide combines with haemoglobin.

Answer: (a)

79. Assertion (A) Himalayas once laid under the sea.

Reason (R) Fossils of marine creatures are traced on the Himalayas.

Answer: (a)

80. How many 5’s are there in the following number sequence which are immediately followed by 4 but not immediately proceded by 6?

8 9 5 4 2 5 4 8 5 5 7 8 6 4 4 5 6 6 5 4 7 5 4 4 6 3 8

(a) 1

(b) 2

(c)  4

(d) 3

Answer: (d)

81. In a certain code language ‘Sun is bright’ is written as ‘pa ma la’, ‘sky is blue’ is written as ‘da ma re’ and ‘Sun is hot’ is written as ‘pa ma go’. Find the code for Sun.

(a) pa

(b) ma

(c)  Either ‘pa’ or ‘ma’

(d) re

Answer: (a)

82. In a students queue, Kamal is 12th from left and Deepak is 18th from right. When Kamal and Deepak interchange their position, then Kamal is 25th from left. Find the total number of students in the queue.

(a) 40

(b) 41

(c)  42

(d) 43

Answer: (c)

DIRECTIONS (Q. Nos. 83-84) Answer the questions based on the following information.

 A class is to be taught five subjects – Hindi, Physics, Chemistry, Biology 2nd Mathematics by five different teachers – A, B, C, D and E in five periods (1 to 5). A teacher can teach in only one of the periods. The following details are available about the teaching.

  • A teachers Mathematics which is not taught in the first period.
  • Physics is taught by D in an even numbered period.
  • Chemistry is taught in an odd period, and it precedes Mathematics period.
  • E teaches in the first period.
  • C teaches Chemistry but not in the first or the last period.
  • Hindi is taught in the last period.

83. Which of the following statements is necessarily true?

(a) Third period is of Hindi taught by B

(b) Second period of Physics taught by C

(c)  Fourth period is Mathematics taught by A

(d) Fifth period is Biology taught by D

Answer: (c)

84. Which subject is taught by B?

(a) Physics

(b) Chemistry

(c)  Biology

(d) Hindi

Answer: (d)

DIRECTIONS (Q. Nos. 85-88) Complete the series by replacing the question mark.

85. 8, 16, 28, 44, ?

(a) 60

(b) 64

(c)  62

(d) 66

Answer: (b)

86. 230, 246, 271, 307, ?

(a) 412

(b) 356

(c)  518

(d) 612

Answer: (b)

87. 1, 3, 7, 15, 31, 63, 127, ?

(a) 250

(b) 275

(c)  350

(d) 255

Answer: (d)

88. AZY, EXW, IVU, ?

(a) MTS

(b) MQR

(c)  NRQ

(d) LST

Answer: (a)

DIRECTIONS (Q. Nos. 89-92) Each question below is followed by two arguments numbered I and II. You have to decide which of the arguments is a ‘strong’ argument and which is a ‘weak’ argument?

Give Answer

(a) if only Argument I is strong

(b) if only Argument II is strong

(c) if either I or II is strong

(d) if both I and II are strong

89. Statement Should school education be made free in India?

Arguments I. Yes this is the only way to improve the level of literacy.

(II) No, it will add to the already heavy burden on the exchequer.

Answer: (b)

90. Statement Should there be complete ban on manufacture of fire crackers in India?

Arguments I. No, this will render thousands of workers jobless.

(II) Yes, the fire cracker manufacturers use child labour to a large extent.

Answer: (d)

91. Statement Should young entrepreneurs be encouraged?

Arguments. I. Yes, they will help in the industrial development of the country.

(II) Yes, they will reduce the burden on the employment market.

Answer: (d)

92. Statement Should luxury hotels be banned in India?

Arguments I. Yes, these are places from where international criminals operate.

(II) No, affluent foreign tourists will have no place to stay.

Answer: (b)

93. Sheela is Ravi’s sister-in-law. Ram is Ravi’s brother. Ram’s wife is Sheela. Deepa is Ravis sister. Deepa’s mother is Shanthi. How is Sheela related to Shanthi?

(a) Mother-in-law

(b) Daughter-in-law

(c)  Grand daughter

(d) Daughter

Answer: (b)

94. A is the mother of D and sister of B. B has a daughter C, who is married of F. G is the husband of A. How is G related to D?

(a) Uncle

(b) Husband

(c)  Son

(d) Father

Answer: (d)

95. If the code of STEADY, is 931785, what will be the code of SEDATE?

(a) 814195

(b) 954185

(c)  614781

(d) 918731

Answer: (d)

96. A child is looking for his father. He went 90m in the East before turning to his right. He went 20m before turning to his right again to look for his father at his uncle’s place 30m from this point. His father was not there. From here he went 100m to his North before meeting his father in a street. How far did the son meet his father from the starting point?

(a) 80 m

(b) 100 m

(c)  260 m

(d) 140 m

Answer: (b)

97. Raju facing North and moves 20 km, then he turned to his right and moves 20 km and then he moves 10 km in North-East, then he turned to his right and moves 20 km and then he turned to his right and moves 20 km and again he turned to his left and moves 20 km. Now, in which direction Raju is facing?

(a) South-East

(b) North-East

(c)  South-West

(d) North-West

Answer: (a)

98. Which of the following diagrams best depicts the relation among liquids, Milk and River water?

Answer: (a)

99. Identify the diagram that best represents the relationship among the given classes. Professionals, Charted Accountant, Female

Answer: (c)

100. Which diagram represents the relationship among female, mother and doctors?

Answer: (c)

101. Which of the following companies has started a rural marketing network called e-Choupals?

(a) ITC

(b) Dabur

(c)  Hindustan

(d) Protector and Gamble

Answer: (a)

102. Which one of the following clothing brands was the sponsor of the Femina Miss India Beauty Pageant 2017?

(a) Lifestyle

(b) Fbb

(c)  Pantaloons

(d) Gucci

Answer: (b)

103. Who is the current CEO of an Indian fashion e-commerce company Myntra?

(a) Mukesh Bansal

(b) Ashutosh Lawania

(c)  Vineet Saxena

(d) Anath Narayanan

Answer: (d)

104. The tagline ‘Be You’ is associated with which of the following fashion e-commerce portal?

(a) Myntra

(b) Fernia’s pupshop

(c)  Jabong

(d) Limeroad

Answer: (c)

105. Which among the following brands is not an Indian brand?

(a) Allen Solly

(b) Da Milano

(c)  Monte Carlo

(d) Steve Madden

Answer: (d)

106. Who has won the best actor award (male) at the 2018 Dadasaheb Phalke Excellence Awards?

(a) Salman Khan

(b) Akshay Kumar

(c)  Shah Rukh Khan

(d) Shahid Kapoor

Answer: (d)

107. Which city will host the 15th Pravasi Bharatiya Divas (PBS) 2019?

(a) Bengaluru

(b) New Delhi

(c)  Gandhinagar

(d) Varanasi

Answer: (d)

108. Where is the headquarters of Asian Development Bank (ADB) located?

(a) Philippines

(b) Geneva

(c)  New York

(d) Beijing

Answer: (a)

109. Who has won the 2018 Italian Open Tennis tournament?

(a) Alexander Zverev

(b) Rafael Nadal

(c)  Juan Martin del Potro

(d) Grigor Dimitrov

Answer: (b)

110. Which country has launched the world’s first floating nuclear power station?

(a) United States

(b) Russia

(c)  China

(d) France

Answer: (b)

111. Which country’s hockey team has won the 5th Women’s Asian Champions Trophy 2018?

(a) South Korea

(b) India

(c)  China

(d) Myanmar

Answer: (a)

112. ‘CaratLane’, the biggest online jewellery company in India, is associated with which brand?

(a) Kalyan

(b) Malabar Gold

(c)  Tanishq

(d) Tribhovandas Bhimji Zaveri (TBZ)

Answer: (c)

113. Who sworn in as Nepal’s 41st Prime Minister in February 2018?

(a) KP Sharma Oil

(b) Bidya Devi Bhandari

(c)  Nasirul Mulk

(d) Pushpa Kamal Dahal

Answer: (a)

114. Pankaj Advani is well-known Indian player of

(a) Cricket

(b) Chess

(c)  Billiards

(d) Hockey

Answer: (c)

115. Which of following countries got observer status in SAARC?

(a) Korea and China

(b) Russian and China

(c)  Japan and China

(d) India and Pakistan

Answer: (c)

116. NITI Ayog was set up in the year

(a) 2014

(b) 2015

(c)  2016

(d) 2017

Answer: (b)

117. Who won the Most Valuable Player in the 2018 Indian Premier League (IPL-11)?

(a) Rishabh Pant

(b) Shane Watson

(c)  Sunil Narine

(d) Kiwi Trent Boult

Answer: (c)

118. Which among the following sarees got the GI (Geographical Indication) tag?

(a) Banarasi saree

(b) Kovai Kora saree

(c)  Chanderi saree

(d) All of these

Answer: (d)

119. The ‘Valley of flowers’ is located in which state?

(a) Uttarakhand

(b) Punjab

(c)  Tamil Nadu

(d) Jammu and Kashmir

Answer: (a)

120. Which city is known as ‘Venice of East’?

(a) Ajmer

(b) Udaipur

(c)  Jodhpur

(d) Jaipur

Answer: (b)

121. Which is the only country in the world to produce all five varities of silk?

(a) China

(b) Brazil

(c)  India

(d) Indonesia

Answer: (c)

122. Which Indian actress has launched her own fashion private label brand called ‘All About You’ on fashion app Myntra?

(a) Kareena Kapoor

(b) Deepika Padukone

(c)  Katrina Kaif

(d) Bipasha Basu

Answer: (b)

123. Teesta river dispute is common to Indian and

(a) Pakistan

(b) China

(c)  Bangladesh

(d) Nepal

Answer: (c)

124. The Mohiniyattam dance form was developed in

(a) Andhra Pradesh

(b) Karnataka

(c)  Kerala

(d) Tamil Nadu

Answer: (c)

125. ‘Mohey’ from the house of ‘Manyavar’ has signed which film star as their new brand ambassador?

(a) Virat Kohli

(b) Anushka Sharma

(c)  Madhuri Dixit

(d) Deepika Padukone

Answer: (b)

DIRECTIONS (Q. Nos. 126-136) Read the following case and answer the questions given at the end each case on the basis of information provided.

CASE 1

Tata Group brand Mia, part of its jewellery division Tanisq, is eyeing Rs 1,000 crore turnover in the next 5-6 years as it expands it retail footprint across the country. Tanishq, which launched the brand for working women in 2011, sells the Mia jewellery range from its existing stores, standalone Mia stores and select shop-in-shops at multi brand outlets such as Shoppers Stop. The company plans to open 25 new standalone Mia stores by the end of the current financial year as it looks to grow turnover from the brand. While Tanishq will continue to sell Mia brand jewellery, growth of business for Mia will come from standalone stores. “Growth of Mia brand has been consistent for the last few years……upwards of 20-25 per cent CAGR. Right now, Mia brand is strongly under Tanisq’s shadow but we want to parallelly open a fresh retail line. We want to make Mia a Rs 1,000 crore brand in 5-6 years,” Titan Company Senior Vice-President (Retail and Marketing) – Jewellery Division, Sandeep Kulhalli told.

126. Tata Group brand Mia’s target group is

(a) Lower income class

(b) Higher income class

(c)  Working women

(d) Homemakers

Answer: (c)

127. While Tanishq will continue to sell Mia brand jewellery, growth of business for Mia will come from

(a) Existing stores

(b) Standalone stores

(c)  Multi brand outlets

(d) None of these

Answer: (b)

128. Tanisq wants to set up ‘Mia’ as a separate brand because

(a) it wants to target more working class women

(b) it wants to popularise it

(c)  it want to increase its turn over

(d) All of the above

Answer: (c)

CASE 2

The Luxury market in India gathers tremendous attention in recent years, as a vast majority of the international brands has arrived, or are relied upon to enter. Luxury retailers have understood that India is one of the world’s most energetic, diverse, and testing markets for brands who want to set sights in India. India is still at an early phase of its advancement as a luxury retail market. Due to its market fluctuation, it is difficult to analyse it. While India is pictured as an upcoming business sector due to its colossal potential, it is the size of the market which makes it extremely exciting for brands to venture into for the long haul. It was not too long ago we witnessed the badge brand syndrome boom, where the focus was all about logos which people wanted to flaunt since the premium positioning of brands had a strong foothold on consumers. The influence of Indian culture globally has grown tremendously by brands, designers and celebrities. Bollywood and sports celebrities are also playing a significant role in the overall domestic brand strategy which is trickling outwards on a global platform. They have changed the power of branding.

129. Luxury retailers are now flooding in India because they have understood that India is one of the world’s

(a) most energetic market

(b) most diverse market

(c)  most testing market

(d) All of the above

Answer: (d)

130. Indian luxury retail market is in

(a) mature phase

(b) early phase

(c)  developed phase

(d) None of these

Answer: ()

131. It is difficult to analyse Indian luxury retail market due to

(a) its fluctuating nature

(b) its strong nature

(c)  its high growth potential

(d) None of the above

Answer: (a)

132. Badge brand syndrome boom is condition in which

(a) people became brand conscious

(b) people got attracted to new market

(c)  market become full of logos

(d) None of the above

Answer: (a)

133. The power of branding has been changed by

(a) designers

(b) bollywood celebrities

(c)  sports celebrities

(d) All of these

Answer: (d)

CASE 3

SWACH cooperative conducted the brand audit survey at three locations Bavdhan, Kothrud and Garware bridge from 16th May to 20th May, 2018. According to the survey, 87% of the waste was attributable to local/Indian brand owners such as Haldiram, Parrle, ITC, Amul and Britannia international, while 13% of the waste was attributable to international brand owners (or Indian subsidiaries) such as Unilever, Nestle, Mondelez (Cadbury) and Coca  Cola.

International brands whie having a smaller share, are being used throughout the world so would have a higher share in the total global waste. Supriya Bhadawad, a SWACH waste picker involved in the brand audit, said, “We sort and send all the hard plastics, shampoo bottles, carry bags, PET bottles and milk packets for recycling every day. But items like food product packaging, chips and biscuit covers, we cannot sell because they have no value. Companies using these materials should either change the packaging and make it recyclable or buy this sort of material from us at affordable rates. That way we can collect and sell it. It will increase our income and reduce the waste going into the landfill.” They also found 86% of the branded packaging was from food product packaging, followed by 8% personal care products and 6% household products.

134. In a survey conducted by SWACH cooperative, it was founded that 87% of the waste during garbage collection was attributable to

(a) Indian brands

(b) international brands

(c)  village industries

(d) None of these

Answer: (a)

135. Items like food product packaging, chips and biscuit covers cannot be sold because

(a) they are biodegradable

(b) the y have no value

(c)  they are no biodegradable

(d) they are reusable

Answer: (b)

136. What is the share of among branded packaging food product packaging?

(a) 8%

(b) 86%

(c)  8%

(d) 10%

Answer: (b)

DIRECTIONS (Q. Nos. 137-150) In each of these questions a passage is followed by several inferences. You have to examine each inference separately in the context of the passage and decide upon its degree of truth or falsity.

(a) if the inference is ‘definitely true’ i.e., it directly follows from the facts given in the passage;

(b) if the inference is ‘probably true’ i.e, though not definitely true in the light of facts given;

(c) if you think the inference is ‘probably false’ though out definitely false in the light of facts given;

(d) if you think the inference is ‘definitely false’ i.e., it contradicts the given facts.

CASELET-1

Ace designer Manish Malhotra, who has crossed the 4 million mark on his Instagram account, says the reach of digital space in fashion industry is phenomenal as people now have become more vocal about their opinions and craftsmanship, and this helps a brand to grow. He believes that the reach of the digital space in our industry is phenomenal and the impact it has on you brand is incomparable. People from across the globe can now access your brand and get a window inside y our world that was initially not a possibility. Currently, the label is reaching out to over 8 million people across platforms, said the designer, whose label registered a 300 per cent growth last year for him,, the digital world has allowed him to stay connected to the world at the click of a button enhancing accessibility and thus,, transcending the constraints of bricks-n-mortar, all at once.

137. The customers are better connected to a brand through social media than otherwise.

Answer: (b)

138. As people now have become more vocal about their opinions and craftsmanship, this helps brand to grow.

Answer: (a)

139. The digital word has enabled the fashion designers to stay connected to the world It promotes their brand and popularity as it enhances their accessibility.

Answer: (a)

CASELET-2

A report from the World Health Organisation on the occasion of World No Tobacco Day (May 31) suggests that India’s efforts to cut the prevalence of cigarette smoking are paying off. By 2025, the report projected, India will come in line to meet its 2025 target under a WHO global plan to tackle non-communicable diseases. Gutkha and other chewable tobacco items are equally, if not more, harmful compared to cigarettes. Surveys show that these products are sometimes mixed with carcinogenic compounds called nitrosamines. This is why, India banned their sale under the 2011 Food Safety and Standards Regulations. Why do they continue to be consumed, then? Experts blame their availability on loopholes in the law. The food safety rules target pre-mixed tobacco products, such as gutkha, which contains lime, sugar and other spices. This leaves  unflavoured items,, such as khaini or surthi, out of regulatory purview. Meanwhile, mislabelling of smokeless tobacco is common. Even when a product contains tobacco, it is passed off as being tobacco-free. Worse, one of the tactics of the tobacco industry is to use flavours such as cardamom and saffron to attract youngsters, triggering life-log addiction.

140. A report from the World Health Organization declared India free from cigarette smoking.

Answer: (d)

141. Surveys show that the product like gutkha and other chewable tobacco are sometimes mixed with carcinogenic compounds called nitrosamines.

Answer: (a)

142. Why these tobacco products continue to be consumed, then? Experts blame their availability solely on the loopholes in the law and there is no fault on the part of the public in general.

Answer: (b)

143. By 2025, the report projected, India will come in line to meet its 2025 target under a WHO global plan to tackle communicable diseases.

Answer: (d)

144. People consumes these chewable and non chewable tobacco products because of the use of flavours such as cardamom and saffron, which triggers life – long addiction.

Answer: (b)

145. Cigarette smoking has more harmful impacts than gutkha and other chewable tobacco.

Answer: (c)

CASELET-3

Three hundred people live on the small Mauritian archipelago of Agalega. They watch as their home is turned slowly into an Indian naval base. There is little that they can do. The government of Mauritius knows that there is far more to be gained from India than from the people of Agalega. Mauritius is one of the main routes for Foreign Direct Investment (FDI) into India. It earns Mauritius a considerable fortune in fees – money that is enough for Mauritius to renege on its pledge to its own citizens. In 2015, Prime Minister Narendra Modi and his Mauritian counterpart Anerood Jugnauth watched as Navtej Srna (Ministry of External Affairs) and Sateeaved Seebaluck (Cabinet Secretary, Mauritius) signed an agreement that allows India to ‘develop infrastructure’ on the islands. The phrase is a euphemism for the building of military bases, which India is doing is not only on Agalega but also on Assumption Island (Seychelles).

146. Mauritian archipelago of Agalega has come under the control of Indian Navy after the development of Indian naval base there.

Answer: (d)

147. The government of Mauritius knows that there is far more to be gained from India than from the people of Agalega. Therefore, they allowed the construction of Indian naval base there.

Answer: (a)

148. Mauritius earns a considerable fortune in fees from India as it is one of the main routes for Foreign Direct Investment (FDI) into India.

Answer: (a)

149. The deal between India and Mauritius will remain all time favourable to the people of Mauritius.

Answer: (d)

150. India wants to increase its influence in Indian Ocean by developing naval base in Mauritius.

Answer: (b)

National Institute of Fashion & Technology (NIFT) Post Graduate 2019 Question Paper With Answer Key

NIFT (Post Graduate) Solved Paper – 2019

1. In a class of 80 students and 5 teachers, each student got sweets that are 15% of the total number of students and each teacher got sweets that are 25% of the total number of students.

How many sweets were there?

(a) 1030

(b) 1040

(c)  1050

(d) 1060

Answer: (d)

2. A sum of Rs 9000 is to be distributed among A, B and C in the ratio 4 : 5 : 6. What will be the difference between A’s and C’s shares?

(a) Rs 600

(b) Rs 1000

(c)  Rs 900

(d) Rs 1200

Answer: (d)

3. If a quantity is divided in the ratio 3 : 5, then the larger part is 75. Find the quantity.

(a) 120

(b) 130

(c)  240

(d) 150

Answer: (a)

4. A sum of money becomes Rs 13380 after 3 yr and Rs 20070 after 6 yr on compound interest. The sum (in rupees) is

(a) 8800

(b) 8890

(c)  8920

(d) 9040

Answer: (c)

5. B and C together can complete a work in 8 days. A and B together can complete the same work in 12 days and A and C together can complete the same work in 16 days. In how many days can A, B and C together complete the same work?

(a)  

(b)  

(c)    

(d)  

Answer: (b)

6. The ages of Sulekha and Arunima are in the ratio of 9 : 8, respectively. After 5 yr, the ratio of their ages will be 10 : 9. What is the difference in years between their ages?

(a) 4 yr

(b) 5 yr

(c)  6 yr

(d) 7 yr

Answer: (b)

7. A sum of Rs 2200 is invested at two different rates of interest. The difference between the interests got after 4 yr is 202.40 What is the difference between the rates of interest?

(a) 3.3%

(b) 2.3%

(c)  3.5%

(d) 2.5%

Answer: (b)

8. A man and a boy received Rs 800 a wages for 5 days for the work they did together. The man’s efficiency in the work was three times that of the boy. What are the daily wages of the boy?

(a) Rs 76

(b) Rs 56

(c)  Rs 44

(d) Rs 40

Answer: (d)

9. Ninad, Vikas and Manav enter into a partnership. Ninad invests some amount at the beginning. Vikas invests double the amount after 6 months and Manav invests thrice the amount invested by Ninad after 8 months. They earn profit of Rs 45000 at the end of the year. What is the Manav’s share in the profit?

(a) Rs 25000

(b) Rs 15000

(c)  Rs 12000

(d) Rs 9000

Answer: (b)

10. The profit earned after selling an article for Rs 1516 is the same as loss incurred after selling the article for Rs 1112. What is the cost price of the article?

(a) Rs 1314

(b) Rs 1343

(c)  Rs 1414

(d) Rs 1434

Answer: (a)

11. Manoj sold an article for Rs 15000. Had he offered discount of 10% on the selling price, he would have earned a profit of 8%. What is the cost price?

(a) Rs 12500

(b) Rs 13500

(c)  Rs 12250

(d) Rs 13250

Answer: (a)

12. If x: y = 5 : 2, then (8x + 9y) : (8x + 2y) is

(a) 22 : 29

(b) 26 : 61

(c)  29 : 22

(d) 61 : 26

Answer: (c)

13. An equal amount of sum is invested in two schemes for 4 yr each, both offering simple interest. When invested in scheme A at 8% per annum the sum amounts to Rs 5280. In scheme B, invested at 12% per annum it amounts to Rs 5920. What is the total sum invested?

(a) Rs 4000

(b) Rs 3500

(c)  Rs 4200

(d) Rs 8000

Answer: (d)

14. A car covers its journey at the speed of 80 km/h in 10 h. If the same distance is to be covered in 4 h, by how much the speed of car will have to increase?

(a) 8 km/h

(b) 100 km/h

(c)  120 km/h

(d) 160 km/h

Answer: (c)

15. In two vessels A and B, there is mixture of milk and water. The ratio of milk and water in these vessels is 5 : 2 and 8 : 5, respectively. In what ratio these mixtures be mixed together, so that the ratio of milk and water in the new mixture becomes 9 : 4?

(a) 7 : 2

(b) 2 : 7

(c)  3 : 5

(d) 5 : 3

Answer: (a)

16. The average age of 80 boys in a class is 15 yr. The average age of a group of 15 boys in the class is 16 yr and the average age of another 25 boys in the class is 14 yr. What is the average age of the remaining boys in the class?

(a) 15.25 yr

(b) 14 yr

(c)  14.75 yr

(d) Cannot be determined

Answer: (a)

17. Twice the square of a number is six times the other number. What is the ratio of the first number to the second?

(a) 1 : 4

(b) 2 : 5

(c)  1 : 3

(d) Cannot be determined

Answer: (d)

18. When all the students in a school are made to stand in rows of 54, 30 such rows are formed. If the students are made to stand in the rows of 45, how many such rows can be formed?

(a) 25

(b) 42

(c)  36

(d) 32

Answer: (c)

19. A man buys a single apple for Rs 25. If he were to buy a dozen apples, he would have to pay a total amount of Rs 250. What would be the approximate per cent discount he would get on buying a dozen apples?

(a) 32%

(b) 20%

(c)  12%

(d) 17%

Answer: (d)

20. Vikas gets Rs 350 for every day that he works. If he earns Rs 9800 in a month of 31 days, for how many days did he work?

(a) 25 days

(b) 30 days

(c)  24 days

(d) 28 days

Answer: (d)

21. The mean value of 20 observations was found to be 75, but later on it was detected that 97 was misread as 79. Find the correct mean.

(a) 75.7

(b) 75.8

(c)  75.9

(d) 75.8

Answer: (c)

22. The average of 5 consecutive integers starting with m is n. What is the average of 6 consecutive integers starting with (m + 2)?

(a)  

(b) (n+2)

(c)  (n +3)

(d)  

Answer: (a)

23.

(a) −1/162

(b) 1/108

(c)  7696/106

(d) 1/109

Answer: (b)

24. A train passes a man standing on a platform in 8 s and also crosses the platform which is 264 m long in 20 s. The length (in metres) of the train is

(a) 188

(b) 176

(c)  175

(d) 96

Answer: (b)

25. A is thrice as good a workman as B and therefore is able to finish a job in 40 days less than B. Working together, they can do it in

(a) 14 days

(b) 13 days

(c)  20 days

(d) 15 days

Answer: (d)

26. A sum of money was lent at simple interest at a certain rate for 3 yr. Had it been lent at 2.5% per annum higher rate, it would have fetched Rs 540 more. The money lent was

(a) Rs 6400

(b) Rs 6472

(c)  Rs 6840

(d) Rs 7200

Answer: (d)

27. A DVD is listed at Rs 300 with a discount of 20%. The additional discount to be offered to bring the price to Rs 216 is

(a) 15%

(b) 10%

(c)  8%

(d)  

Answer: (b)

28. In a division sum, the divisor is 10 times the quotient and 5 times the remainder. If the remainder is 46, then the dividend is

(a) 4236

(b) 4306

(c)  4336

(d) 5336

Answer: (d)

29. A dealer marks his goods at 40% above the cost price and allows a discount of 20% on the marked price. The dealer has a

(a) loss of 20%

(b) gain of 25%

(c)  loss of 12%

(d) gain of 12%

Answer: (d)

30. A box has 100 blue balls, 50 red balls and 50 black balls, 25% of blue balls and 50% of red balls are taken away. Then, percentage of black balls at present is

(a) 25%

(b)  

(c)  40%

(d) 50%

Answer: (b)

Directions (Q. Nos. 31-34) In each sentence given below a word or phrase is underlined. From the given options, choose the word/phrase closest in meaning to the underlined part.

31. My impetuous cousin bought a car even before he got a job.

(a) judicious

(b) funny

(c)  impulsive

(d) jolly

Answer: (c)

32. The callow employee did many mistakes in his first assignment.

(a) inexperienced

(b) experienced

(c)  jovial

(d) comely

Answer: (a)

33. We had a sumptuous meal yesterday at the newly opened restaurant.

(a) expressive

(b) heavy

(c)  light

(d) expensive

Answer: (d)

34. People have become indifferent to the rampant corruption.

(a) concerned

(b) unconcerned

(c)  irritated

(d) pleased

Answer: (b)

Directions (Q. Nos. 35-38) In the given questions, a word is given with four alternatives. Choose the word that is opposite in meaning to the given word.

35. Corpulent

(a) Fat

(b) Thin

(c)  Even

(d) Corrupt

Answer: (b)

36. Hilarious

(a) Amusing

(b) Delightful

(c)  Serious

(d) Momentous

Answer: (c)

37. Erudition

(a) Intelligence

(b) Hollowness

(c)  Slowness

(d) Ignorance

Answer: (d)

38. Vindicate

(a) Reprieve

(b) Absolve

(c)  Indict

(d) Summon

Answer: (c)

Directions (Q. Nos. 39-42) In each of the following sentences there are blank spaces. Below each sentence there are four words denoted by numbers a, b, c and d. Find out which word can be filled up in the blank in the sentence to make it grammatically correct.

39. A company car and a big apartment are some of the ………. that some with the post of CEO.

(a) pricks

(b) blurbs

(c)  pastime

(d) perks

Answer: (d)

40. ……….. means that you express effectively and stand up for your point of view.

(a) passive

(b) sordid

(c)  aggressive

(d) assertive

Answer: (d)

41. The …… use of mobile phones can cause long-term problems.

(a) prudent

(b) severe

(c)  indiscriminate

(d) judicious

Answer: (c)

42. The officious clerk seldom looked at people and always sounded………. .

(a) rude

(b) rudely

(c)  soft

(d) polite

Answer: (a)

Directions (Q. Nos. 43-47) Choose the most logical order of sentences from among the four given choices to construct a coherent paragraph.

43. (A) The elephant is distinguished by its massive body, large earns and a long trunk.

(B) Asian elephants differ in several ways from their African relatives.

(C) The largest mammal on earth, the African elephant weighs up to eight tons.

(D) Only some Asian male elephants have tusks.

(E) They are much smaller in size and their ears are straight at the bottom, unlike the large fan shape ears of the African species.

(a) ABCDE

(b) CEDAB

(c)  CABED

(d) CABDE

Answer: (c)

44. (A) The reporter-cum adventure is always on the move.

(B) Wherever Tintin goes, so does Snowy.

(C) And his pet dog Snowy is his ever-faithful shadow.

(D) The pair is inseparable, only leaving each other’s side when they are forcefully separated.

(E) Tintin can always rely on Snowy to be courageous in the face of danger, often helping him out of tight corners.

(a) BACED

(b) BACDE

(c)  ABCED

(d) ABCDE

Answer: (a)

45. (A) Aesop’s Fables is a collection of fables credited to Aesop.

(B) Aesop was a slave and storyteller.

(C) It is also called as Aesopica.

(D) This is a story from Aesop, set in Rome.

(E) He is believed to have lived on ancient Greece between 620 and 564 BCE.

(a) ACBDE

(b) ACBED

(c)  DECBA

(d) CBADE

Answer: (b)

46. (A) He was born on 8th February, 1828 in Nantes, France.

(B) His parents were Piere Verne, an attorney and Sophie Allote de la Fuye.

(C) His teacher, Mme Sambin, was a widow of a Naval Captain who had disappeared 30 years ago

(D) Jules Gabriel Verne is known as the father of science fiction.

(E) When he was six, he was sent to a boarding school.

(a) DAEBC

(b) ABEDC

(c)  BACED

(d) DABEC

Answer: (d)

47. (A) Low-lying areas like marshes and ponds, edges around lakes or oceans, and the delta at the mouth of a river are wetlands.

(B) A wetland is a place where the land is covered by water, either fresh or salty.

(C) They are known to support fauna-including mammals, birds and fish.

(D) They also support he cultivation of rice, a staple in the diet of half the world’s population.

(E) 2nd February is celebrated as International Wetlands Day.

(a) EBADC

(b) EABCD

(c)  BCEAD

(d) EBACD

Answer: (d)

Directions (Q. Nos. 48-51) Find the correctly spelt words

48. 

(a) Prophesied

(b) Propheside

(c)  Prophesiedd

(d) Prophside

Answer: (a)

49.

(a) Garrulouss

(b) Garrulous

(c)  Garullous

(d) Garrullous

Answer: (b)

50.

(a) Aassasinnation

(b) Assassinationn

(c)  Assassination

(d) Assessination

Answer: (c)

51.

(a) Cemetery

(b) Cemetry

(c)  Cemettery

(d) Cemeteryy

Answer: (a)

Directions (Q. Nos.52-55) Choose the correct plural form the given words.

52. Sheep

(a) Sheeps

(b) Sheep

(c)  Sheepes

(d) Sheepes

Answer: (b)

53. Elf

(a) Elves

(b) Elfs

(c)  Elf

(d) Elfes

Answer: (a)

54. Nucleus

(a) Nucleus

(b) Nucleuses

(c)  Nucleuss

(d) Nuclei

Answer: (d)

55. Datum

(a) Datums

(b) Datumes

(c)  Data

(d) Datas

Answer: (c)

Directions (Q. Nos. 56-75) Read the following passages and answer the questions that follow.

PASSAGE 1

I was abruptly awakened by a noisy scuffle. The sun, a mere fringe over the horizon, immediately chased away the grey half-darkness. I was too sleepy to notice what was happening. Yuri was rolling over on the ground. I ran up to him but was struck dumb. With his right hand he was holding a cobra by the neck. Two sharp fangs showed from its jaws. The battle was over in a few minutes. A hollow hissing and convulsive jerks were then only reminders of a just-ended tussle. The catcher half-opened the lid of the box and calmly put the quarry in.

56.When the writer saw Yuri holding a cobra by the neck, he was ‘struck dumb’. This means that he was

(a) extremely delighted

(b) very much helpless

(c)  rather surprised

(d) absolutely shocked

Answer: (d)

57. From the passage, Yuri appears to be a man who is

(a) calm and courageous

(b) cunning and crafty

(c)  noisy and dangerous

(d) active and jumpy

Answer: (a)

58. With reference to the passage, the following assumptions have been made

(1) The incident took place early in the morning.

(2) Yuri threw the snake away.

Which of these assumptions is/are correct?

(a) 1 only

(b) 2 only

(c)  Both 1 and 2

(d) Neither 1 nor 2

Answer: (a)

59. Which word from the passage means ‘a short fight that is not very violent’.

(a) fringe

(b) quarry

(c)  strife

(d) scuffle

Answer: (d)

PASSAGE 2

I must say a word about the Eiffel Tower. I do not know what purpose it serves today. But I then heard it greatly disparaged as well as praised. I remember that Tolstoy was the chief among those who disparaged it. He said that the Eiffel Tower was a monument of man’s folly, not of his wisdom. Tobacco, he argued, was the worst of all intoxicants, inasmuch as a man addicted to it was tempted to commit crimes which a drunkard never dared to do; liquor made a man mad, but tobacco clouded his intellect and made him build castles in the air. The Eiffel Tower was one of the creations of a man under such influence. There is no art about the Eiffel Tower. In no way can it be said to have contributed to the real beauty of the Exhibition. Men flocked to see it and ascended it as it was a novelty and of unique dimensions. It was the toy of the Exhibition. Sol long as we are children we are attracted by toys, and the Tower was a good demonstration of the fact that we are children attracted by trinkets. That may be claimed to be the purpose served by the Eiffel Tower.

60. Why did Tolstoy disparage Eiffel Tower?

(1) Man was foolish to build it.

(2) Huge man-made structures did not appeal to him.

(3) Men flocked to see it.

Which of the statements given above is/are correct?

(a) 1 only

(b) 1 and 2 only

(c)  1 and 3 only

(d) 2 and 3 only

Answer: (c)

61. Why did Tolstoy believe that tobacco was the worst of all intoxicants?

(a) Man lost his intellectual abilities under the influence of tobacco.

(b) Tobacco kept man in a state of inebriation.

(c)  People who commit crimes are invariably addicted to tobacco.

(d) Statements (a) and (b) above are correct in this context.

Answer: (a)

62. Why did men flock to the Eiffel Tower?

(a) Men were attracted to the castle built in the air.

(b) Men lost their wisdom under the influence of intoxicants.

(c)  Men were attracted to childish things.

(d) Men were attracted to things of no value.

Answer: (c)

63. Which word from the passage is the opposite of ‘praised’?

(a) folly

(b) disparaged

(c)  criticised

(d) trinket

Answer: (b)

PASSAGE 3

During the past three generation s the diseases affecting western societies have undergone dramatic changes. Polio, diphtheria, tuberculosis, commonly known as TB, are vanishing; one injection of an antibiotic often cures deadly diseases such as pneumonia or syphilis; and so many mass killers have come under control that two-thirds of all death are now associated with the diseases of old age. Those who die young are more often than not victims of accidents, violence, or suicide.

These changes in health status are generally equated with the decrease in suffering and attributed to more or better medical care. Almost everyone believes that at least one of his friends would not be alive and well except for the skill of a doctor. But there is in fact no evidence of any direct relationship between this change in the pattern or nature of sicknesses on the one hand and the so-called progress of medicine on the other hand. These changes are the results of political technological changes. They are not related to the activities that require the preparation and status of doctors or the costly equipment in which doctors takes pride. In addition, an increase in the number of new disease in the last fifteen years are themselves the result for medical intervention. They are doctor-made or iatrogenic.

64. In the western societies, the occurrence of polio, diphtheria and tuberculosis has

(a) increased

(b) completely stopped

(c)  decreased

(d) continued without changes

Answer: (c)

65. More deaths are now associated with old age than in the past because

(a) iatrogenic diseases are spreading faster now

(b) deadly diseases affecting the young have been well controlled

(c)  accidents, violence and suicide that killed many youths in the past are now under control

(d) political and technological changes now take better care of the young than the old

Answer: (b)

66. The writer probably is arguing for

(a) stopping the practice of western medicines completely

(b) stopping the use of costly equipment and medicines

(c)  rethinking about the successes and failures of the western medicines

(d) giving greater attention to new, iatrogenic diseases than to the old diseases such as polio, diphtheria and pneumonia

Answer: (c)

PASSAGE 4

Much has been said of the common ground of religious unity. I am not going just now to venture my own theory. But if anyone here hopes that this unity will come by the triumph of any one of the religions and the destruction of others, to him I say, “Brother, yours is an impossible hope.” Do I wish that the Christian would become Hindu? God forbid. Do I wish that the Hindu or Buddhist would become Christian? God forbid.

The seed is put in the ground, and earth and air and water are p laced around it. Does the seed become the earth, or the air, or the water? No. It becomes a plant. It develops after the law of its own growth, assimilates the air, the earth, and the water converts them into plant substance, and grows into a plant.

Similar is the case with religion. The Christian is not become a Hindu or a Buddhist, nor a Hindu or a Buddhist to become a Christian. But each must assimilate the spirit of the others and yet preserve his individuality and grow according to his own law of growth.

If the Parliament of Religions has shown anything to the word, it is this : it has proved to the world that holiness, purity and charity are not the exclusive possessions of any church in the world, and that every system has produced men and women of the most exalted character. In the face of this evidence, if anybody dreams of the exclusive survival of his own religions and the destruction of the others, I pity him from the bottom of my heart, and point out to him that upon the banner of every religion will soon be written in spite of resistance. : “Help and not fight,” “Assimilation and not Destruction,” “Harmony and Peace and not Dissension.”

67. According to the author of the passage, people should

(a) change their religions

(b) follow their religions and persuade others to follow it

(c)  follow their own religions and respect other religions

(d) disrespect other religions

Answer: (c)

68. The Parliament of Religions is

(a) a Christian organisation

(b) a Buddhist organisation

(c)  a Hindu organisation

(d) a platform for discussion about every religion of the world

Answer: (d)

69. What does the author think about those who dream about the exclusive survival of their own religions and the destruction of the others?

(a) He hates them

(b) He desires to imprison them

(c)  He pities them

(d) He praises them

Answer: (c)

70. According to the passage, what is ‘impossible hope’?

(a) One day, all the people of the world will follow only one religion

(b) One day, there will be no religion

(c)  Purity and charity are the exclusive possessions

(d) Banner of every religion will soon be written

Answer: (a)

PASSAGE 5

“Now, ladies and gentlemen,” said the conjuror, “having shown you that the cloth is absolutely empty, I will proceed to take from it a bowl of goldfish. Presto!”

All around the hall people were saying, “Oh, how wonderful! How does he do it?”

But the Quick Man on the front seat said in a big whisper to the people near him, “He-had-it-up-his-sleeve.”

Then the people nodded brightly at the Quick Man and said,, “Oh, of course”; and everybody whispered round the hall, “He-had-it-up-his-sleeve.”

“My next trick,” said the conjuror, “is the famous Hindostanee rings. you will notice that the rings are apparently separate, at a now they all join (clang, clang, clag)- Presto!”

There was a general buzz of stupefaction till the Quick Man was heard to whisper, “He must have had another lot up the sleeve.”

Again everybody nodded and whispered, “The rings-were-up-his-sleeve.”

The brow of the conjuror was clouded with a gathering frown.

“I will now,” he continued, “show you a most amusing trick by which I am enabled to take any number of eggs from hat. Will some gentleman kindly lend me his hat? Ah, than you-Presto!”

He extracted seventeen eggs, and for thirty-five seconds the audience began to think that he was wonderful. Then the Quick man whispered along the front bench, “He has a hen up his sleeve.” and all the people whispered it on. “He has a lot of hens up his sleeve.”

The egg trick was ruined.

It went on like that all through. It transpired from the whispers of the Quick Man that the conjuror must have concealed up his sleeve, in addition to the rings, hens, and fish, several packs of cards, a loaf o bread, a doll’s cradle, a live guinea pig, a fifty-cent piece, and a rocking chair.

71.“The brow of the conjuror was clouded with a gathering frown.” The sentence means that the conjuror

(a) was very pleased

(b) was very sad

(c)  was rather angry

(d) was very afraid

Answer: (c)

72. ‘The egg trick was ruined.’ This means that

(a) eggs were all broken

(b) people were unconvinced

(c)  conjuror was disappointed

(d) the trick could not be performed

Answer: (b)

73. The author believes that the Quick Man was really

(a) foolish

(b) clever

(c)  wrong

(d) right

Answer: (b)

74. According to the Quick Man, the conjuror

(a) had everything bought for production

(b) produced things with the magic he know

(c)  had things in the large sleeves of his coat.

(d) created an illusion of things with his magic

Answer: (c)

75. The conjuror extracted seventeen eggs from the hat of

(a) the Quick Man

(b) his own

(c)  on gentleman from the audience

(d) None of the above

Answer: (c)

76. If two is subtracted from each odd digit and if two is added to each even digit in the number 9275436, then what will be the difference between the digits which are third from the right and second from the left of the new number thus formed?

(a) 6

(b) 8

(c)  2

(d) 1

Answer: (c)

77. In a certain code language, BANKER is written as LFSCBO, how will CONFER be written in that code?

(a) GFSDPO

(b) FGSDOP

(c)  GFSEPO

(d) FHSDPO

Answer: (a)

Directions (Q. Nos. 78-80) Study the following information carefully to answer the given questions.

S is the husband of Q. A is the brother of D. A is the only son of B. D is the sister of Q. R is married to D. M is the father of R. N is the daughter of Q.

78. If V is the grandfather of N, then how is B related to R?

(a) Uncle

(b) Mother-in-law

(c)  Grandmother

(d) Aunt

Answer: (b)

79. How is S related to A?

(a) Father

(b) Grandfather

(c)  Brother-in-law

(d) Uncle

Answer: (c)

80. How is D related to N?

(a) Mother

(b) Sister-in-law

(c)  Cousin

(d) Aunt

Answer: (d)

81. In a row of 30 boys, R is 4th from the right end and W is 10th from the left end. How many boys are there between R and W?

(a) 15

(b) 16

(c)  17

(d) Cannot be determined

Answer: (b)

Directions (Q. Nos. 82-86) Study the following information carefully and answer the given questions.

Seven people A, B, C, D, E, F and G have different hobbies, viz. Travelling, Reading, Dancing, Painting Sculpting, Singing and Pottery making, but not necessarily in the same order. Each of them belongs to a different state, viz. Punjab, Odisha, Kerala, Rajasthan, Maharashtra, Gujarat and Karnataka, but not necessarily in the same order. A belongs to Maharashtra. D likes Pottery making. The person who likes Sculpting is from the state of Odisha.

The person who likes Dancing is from the state of Gujarat. F does not belong to Gujarat, Odisha, Punjab or Rajasthan. F does not like Singing, Reading or Painting. B does not belong to Kerala, Odisha, Punjab or Rajasthan. B does not like Painting, Travelling, Reading or Singing. C does not like Sculpting and he is not from Rajasthan or Punjab. Neither D nor G belongs to Punjab. A does not like Reading. The person from Kerala likes Singing.

82. Who among the following likes Singing?

(a) A

(b) C

(c)  E

(d) G

Answer: (b)

83. Which of the following combinations is true according to the given information?

(a) A-Travelling-Maharashtra

(b) C-Dancing-Gujarat

(c)  E-Reading

(d) D-Pottery making-Rajasthan

Answer: (d)

84. Who among the following belongs to the state of Karnataka?

(a) B

(b) D

(c)  F

(d) E

Answer: (c)

85. Which of the following combinations is true about G?

(a) Sculpting-Odisha

(b) Pottery making-Karnataka

(c)  Dancing-Gujarat

(d) Singing-Kerala

Answer: (a)

86. Person who belongs to Punjab, likes?

(a) Travelling

(b) Sculpting

(c)  Painting

(d) Reading

Answer: (d)

87. A boat moves from a jetty towards East. After sailing for 9 nautical miles, she turns towards right and covers another 12 nautical miles. If she wants to go back to the jetty, what is the shortest distance now from her present position?

(a) 21 nautical miles

(b) 20 nautical miles

(c)  18 nautical miles

(d) 15 nautical miles

Answer: (d)

Directions (Q. 88-90) Read the given information carefully and answer the given questions.

Each of the six friends, I, J, K, L, M and N working in an office handles different number of projects in a month. I handled the second lowest number of projects. K handles more projects than L and M, but less than J. J did not handle the maximum number of projects. L did not handle the minimum number of projects. The one who handle the third highest number of projects handled 31 projects . L. handled 12 projects.

88. How many projects did J possible handle?

(a) 28

(b) 10

(c)  36

(d) 9

Answer: (c)

89. If the number of projects handled by I + M is only three more than number of projects handled by L, how many projects were handled by I?

(a) 22

(b) 14

(c)  19

(d) 9

Answer: (d)

90. Which of the following is true regarding the number of projects handled by N?

(a) No one handles more projects than N

(b) Only J handled more number of projects than N

(c)  N possible handled 24 projects

(d) N handled more number of projects than only there people

Answer: (a)

Directions (Q. Nos. 91 and 92) In each question below a statement is given followed by two Assumptions/Inferences numbered I and II.

Give Answer

(a) If only Assumption I is implicit

(b) If only Assumption II is implicit

(c) If neither Assumption I nor Assumption II is implicit

(d) If both Assumptions I and II are implicit

91. Statement Farmers must immediately switch over to organic fertilizers from chemical fertilizers for better yield.

Assumptions

(I) All the farmers use only chemical fertilizers.

(II) Organic fertilizers are readily available to the farmers.

Answer: (b)

92. Statement For any kind of problems with your mobile phone, contact our help desk immediately.

Assumptions

(I) Help desk has a solution to all kinds of problems related to mobile phones or will guide accordingly.

(II) Unless the problem is reported immediately it cannot be solved.

Answer: (a)

93. In a certain code, JEALOUS is written as BFKKTVP. How is HEARTEN written in that code?

(a) OFUQBFT

(b) BFIQOFU

(c)  BFIQUFO

(d) UFOQIBF

Answer: (b)

94. A person walks towards his house at 8 : 00 am and observes his shadow to his right. In which direction he is walking?

(a) North

(b) South

(c)  East

(d) West

Answer: (b)

Directions (Q. Nos. 95 and 96) What will come in the place of question mark (?) in the following series.

95. AZY, EXW, IVU?

(a) MTS

(b) MQR

(c)  NRQ

(d) LST

Answer: (a)

96. 8, 16, 28, 44,?

(a) 60

(b) 64

(c)  62

(d) 66

Answer: (b)

Directions (Q. Nos. 97 and 98) In each question below is given a statement followed by two Courses of action numbered I and II.

Give Answer

(a) If only Course of action I follows

(b) If only Course of action II follows

(c) If neither Course of action I nor Course of action II follows

(d) If both Courses of action I and II follow.

97. Statement A large part of the locality was flooded as the main pipe supplying drinking water burst while the workers of a utility company were laying cables in the area.

Course of Action

(I) The civic authority should immediately arrange to repair the damage and stop loss of water.

(II) The civic authority should seek an explanation and compensation from the utility company for the damage caused by them.

Answer: (d)

98. Statement Millions of pilgrims are expected to take a dip in the Ganges at the holy place during the next fortnight.

Course of Action

(I) The Government should restrict the number of pilgrims who can take dip each day during the fortnight.

(II) The Government should deploy and adequate number of security personnel to maintain law and order during the next fortnight at the holy place.

Answer: (b)

Directions (Q. Nos. 99 and 100) Each question below is followed by two Arguments numbered I and II. You have to decide which of the arguments is ‘strong’ argument and which is a ‘weak’ argument.

Give Answer

(a) If only Argument I is strong

(b) If only Argument II is strong

(c) If neither Argument I nor II is strong

(d) If both Arguments I and II are strong

99. Statement Should vacations of court judges be reduced?

Arguments

(I) Yes, it will speed up judicial process and many people are likely to get justice in reasonable time.

(II) Yes, it is a sign of British legacy, why should we carry it further?

Answer: (a)

100. Statement Should the practice of transfers of clerical cadres employees from one city to another Government offices be stopped?

Arguments

(I) No, transfer of employees is a routine administrative matter and we must continue it.

(II) Yes, it involves lot of Governmental expenditure and inconvenience to many compared to the benefits it yields.

Answer: (c)

101. Who among the following is not a fashion designer?

(a) Ritu Kumar

(b) Shiv Kumar Sharma

(c)  Tarun Tahiliani

(d) Rahul Khanna

Answer: (b)

102. National Gallery of Modern Art is located at

(a) Mumbai

(b) Kolkata

(c)  New Delhi

(d) Lucknow

Answer: (c)

103. Who is the brand Ambassador of Lakme Cosmetics?

(a) Aishwarya Rai Bachhan

(b) Deepika Padukone

(c)  Kareena Kapoor

(d) Aushika Sharma

Answer: (c)

104. ‘Ekaya’, the traditional handloom Banarasi sarees brand is associated with

(a) Palak Shah

(b) Manish Arora

(c)  Rohit Bal

(d) Sabyasachi Mukherjee

Answer: (a)

105. Pond’s men face wash is being promoted by popular Indian filmstar?

(a) Varun Dhawan

(b) Shahrukh Khan

(c)  Hrithik Roshan

(d) John Abaraham

Answer: (a)

106. ‘Cherokee’, the global fashion lifestyle brand belongs to

(a) The USA

(b) The  UK

(c)  Singapore

(d) India

Answer: (a)

107. ‘Alrosa’, one of the largest rough diamond supplying companies globally belongs to

(a) South Africa

(b) Brazil

(c)  Egypt

(d) Russia

Answer: (a)

108. Who among the following fashion designers has own the prestigious Padma Shri Award on India’s 65th Republic Day?

(a) Sabyasachi Mukherjee

(b) Wendell Rodrick

(c)  Tarun Tahiliani

(d) None of the above

Answer: (b)

109. Popular Indian film star Amitabh Bachhan is associated with which one of the following jewellery brands?

(a) Amrapali Jewellers

(b) Kalyan Jewellers

(c)  Tribhovandas Bhimji Zaveri

(d) PC Jewellers

Answer: (b)

110. ‘AJIO.COM’ the online clothing, accessories and footwear brand belongs to …………

(a) Reliance

(b) Lifestyle

(c)  Aditya Birla Group

(d) Bharti Retail

Answer: (a)

111. ‘Inayat’, the designer jewellery collection is from the house of ………. .

(a) Malabar Jewellers

(b) Tanisq

(c)  Nakshatra

(d) PC Jewellers

Answer: (d)

112. Which one of the following sport lifestyle clothing line is launched by cricketer Yuvraj Singh?

(a) True Blue

(b) SKULT

(c)  YWC Fashion

(d) SEVEN

Answer: (c)

113. Indian Cricketer Virat Kohli endorses which one of the following Indian clothing brand?

(a) Manyavar

(b) Fashion by Big Bazaar

(c)  Pantaloons

(d) None of the above

Answer: (a)

114. In which of the following places world’s first Bollywood themed park is opened?

(a) London

(b) Singapore

(c)  Hong Kong

(d) Dubai

Answer: (d)

115. ……… city in Tamil Nadu is also called the ‘Textile city’.

(a) Thiruppur

(b) Tanjore

(c)  Coimbatore

(d) Nagapattinam

Answer: (a)

116. Which of the following fibres is the most difficult to dye?

(a) Cotton

(b) Nylon

(c)  Polyester

(d) Viscose nylon

Answer: (c)

117. Silk is produced from which among the following?

(a) Egg of silkworm

(b) Pupa of silkworm

(c)  Larva of silkworm

(d) Insect itself

Answer: (c)

118. Who among the following was popularly known as the ‘Frontier Gandhi?

(a) Hasrat Mohani

(b) Maulana Abul Kalam Azad

(c)  Iqbal Khan

(d) Khan Abdul Ghaffar Khan

Answer: (d)

119. Article 123 of the Indian Constitution provides for which among the following?

(a) Ordinance Making power

(b) Supreme Court

(c)  Special Status of Jammu and Kashmir

(d) An Election Commission

Answer: (a)

120. Which of the following rivers does not flow into the Arabian Sea?

(a) Narmada

(b) Sabarmati

(c)  Mandovi

(d) Tungabhadra

Answer: (d)

121. Mahatma Gandhi Marine National Park is situated in the state of

(a) Kerala

(b) Andaman and Nicobar Islands

(c)  Uttarakhand

(d) Sikkim

Answer: (b)

122. Who has been designated as a brand ambassador for global sportswear giant PUMA India?

(a) Swapna Barman

(b) Nirmala Sheoran

(c)  Dutee Chand

(d) Mary Kom

Answer: (d)

123. Which of the following publishes the Red List of Threatened Species?

(a) UNEP

(b) WWF

(c)  IUCN

(d) WNO

Answer: (c)

124. Which African nation is not a member of the African Union?

(a) Algeria

(b) Morocco

(c)  Lesotho

(d) Comoros

Answer: (b)

125. Which of the following decorative techniques would most successfully achieve fabric layering on jeans?

(a) Applique

(b) Beading

(c)  Gold foiling

(d) Stencilling

Answer: (a)

Directions (Q. Nos. 126-139) Read each case very carefully and answer the questions that follow.

CASE I

According to the 2019Bloomberg Healthiest Country Index, six of the ten healthiest countries in the world are on the continent of Europe. In first place came Spain, which jumped from sixth in 2017. Spain is also according to the Institute for Health Metrics and Evaluation, set to have the highest global life expectancy in 2040-showing that the Iberian nation is leading the way in both quality of life and longevity. What put Spain on top? Life expectancy, yes, but also environmental factors like access to clean water and sanitation, as well as a lack of tobacco use and obesity. Their focus on a Mediterranean diet, which emphasises local fruits, vegetables, oils, and fish, is also touted as a reason. Following is Spain is Italy-as it turns out, living la dolce vita is good for your mind, body, and soul. The island of Sardinia in Italy, which has almost 10 times more centenarians per capita than the United States, also follows a Mediterranean diet and encourages an active, outdoor lifestyle with its mild climate. In third place is Iceland. yes, you guessed it, they also have a fish-heavy diet( another staple is skyr, a cultured dairy product that resembles yogurt). Despite their cold climate, Iceland residents also live a life full of routine physical activities. Japan came in fourth, the first Asian country to make the list. Rounding out the top ten were: Switzerland, Sweden, Australia, Singapore, Norway, and Israel. The United States ranked 35th.

126. According to the 2019 Bloomberg Healthiest Country Index, most healthiest countries lies in continent of

(a) Europe

(b) America

(c)  Asia

(d) Europe

Answer: (a)

127. Spain is on the top due to

(a) high life expectancy

(b) access to clean water

(c)  access to sanitation

(d) All of these

Answer: (d)

128. The island of Sardinia in Italy encourages

(a) resting

(b) meditation

(c)  active, outdoor lifestyle

(d) drinking

Answer: (c)

129. Which among the following is true in the context of above case?

(a) Those who live in cold regions have healthy life

(b) Physical activities are nil in cold areas

(c)  People who eat Mediterranean diet have relatively healthy life

(d) People should eat meat to stay healthy

Answer: (c)

CASE 2

Marie Curie was one of the most accomplished scientists in history. She discovered radius, with her husband Pierre, an element widely used for treating cancer, and studied uranium and other radioactive substances. Pierre and Marie’s amicable collaboration later helped to unlock the secrets of the atom. At any early age, Marie displayed a brilliant mind and a blithe personality. Her great exuberance for learning prompted her to continue with her studies after high school. She became disgruntled, however, when she learned that the university in Warsaw was closed to women Marie and Pierre were married in 1895 and spent may productive years working together in the physics laboratory. A short time after they discovered radius, Pierre was killed by a horse-drawn wagon in 1906. Marie was stunned by this horrible misfortune and endured heartbreaking anguish. Curie’s feeling of desolation finally began to fade when she was asked to succeed her husband as a physics professor at the Sorbonne University. She was the first woman to be given a professorship at the world-famous university. In 1911, she received the Nobel Prize in chemistry for isolating radium. Although Marie Curie eventually suffered a fatal illness from her long exposure to radius, she never became disillusioned about her work. Regardless of the consequences, she had dedicated herself to science and to revealing the mysteries of the physical world.

130. Marie discovered radium, with her husband Pierre, which is used widely

(a) in electric appliances

(b) to treat penumonia

(c)  to treat cancer

(d) in tyres

Answer: (c)

131. Pierre and Marie’s collaborated to unlock the

(a) secrets of the atom

(b) algebraic formulas

(c)  distance between Earth and moon

(d) None of the above

Answer: (a)

132. Curie’s feeling of desolation finally began to fade when

(a) she became professor at the Sorbonne University

(b) she discovered new element

(c)  her daughter became scientist       

(d) she left her home

Answer: (a)

133. In which field did Marie Curie received Nobel Prize in 1911?

(a) Physics

(b) Biology

(c)  Anthropology

(d) Chemistry

Answer: (d)

CASE 3

When social media became a key communication platform for people to network, brands immediately began to realise their value to showcase their products and reach out to their audience. Few have done it more effectively than fashion brands, which realize their primary marketing impact is through the visual medium. With the advent of the Internet and social media, the ability to share and convey brand imagery exploded, leading to even niche brands and upcoming designers getting much greater exposure and visibility. This was seen as a boon for the fashion industry. With the increasing use of web and social analytics, designers are also able to predict style trends better, leading to a faster cycle of creation, and giving birth to the concept of fast fashion. Another important aspect of social media was the advent of the fashion bloggers, who express their opinions on fashion bloggers, who express their opinions on fashion and industry trends on the Internet and social media. Brands are unanimous in saying that these bloggers are helping them cover a much wider audience, leading to creation of new sales opportunities. Social media platforms have also provided an opportunity for all to showcase their own sense of style, clothing and a fashion outlook.

However, the flip side of this has been the reduction in the number of times a garment is worn. Once it is clicked and posted on social media, people do not want to the seen in the same outfit, therefore reducing the likelihood that it will be worn again. This is turn has led to the explosive growth of fast fashion as customers scan clothes more frequently. While this may lead people to think the fashion industry is doing well, the picture is not so rosy. People are now opting for cheaper clothes that looks stylish and look well in photographs, but are not necessarily durable and can be discarded after a couple of uses. This in turn creates pressure to manufacture at lower costs, using materials that are cheaper and not necessarily environment-friendly, dramatically increasing the carbon footprint of the industry. The industry is witnessing the impact of this tremendous growth of fast fashion in form of increasing pollution caused by the manufacturers and frequently discarded clothes. While several brands are taking steps to reduce the impact of synthetics on the environment, such as the use of recycled PET fabric in denims and sportswear, this effort is miniscule compared to the size of the problem.

134. How social media has impacted fashion industry?

(a) Greater reach to the masses

(b) Less competition in the market

(c)  More investments in the fashion industry

(d) Less development of trends

Answer: (a)

135. What is fast fashion?

(a) Fashion that changes quickly

(b) Vintage fashion

(c)  Discarding old clothes

(d) None of the above

Answer: (a)

136. Social media has resulted in the advent of the

(a) models

(b) fashion bloggers

(c)  photographers

(d) designers

Answer: (b)

137. Fast fashion is resulting in

(a) pollution

(b) wastage of fabric

(c)  animal cruelty

(d) None of these

Answer: (a)

138. People are now opting for

(a) cheaper clothes

(b) stylish clothes

(c)  not necessarily durable

(d) All of the above

Answer: (d)

139. What is the purpose behind introducing recycled PET fabric in denims?

(a) to make them environment friendly

(b) to make them cheap

(c)  to make them stylish

(d) to make them durable

Answer: (a)

Directions (Q. Nos. 140-150) Each of the following questions has a statement based on the preceding caselet. Evaluate each statement and mark answer:

(a) if the statement is major objective in making the decision or one of the goals sought by the decision maker, then mark (a) as answer.

(b) if the statement is major factor in making the decision: an aspect of the problem specifically mentioned in the caselet, which fundamentally affects or determines the decision.

(c) if the statement is a minor factor in making the decision: a less important element bearing on or affecting a major factor rather than a major objective directly.

(d) if the statement is a major assumption in making the decision, a projection or supposition arrived at by the decision maker before considering the factor and alternative.

CASELET 1

“A clean India would be the best tribute India could pay to Mahatma Gandhi on his 150 birth anniversary in 2019,” said Shri Narendra Mod as he launched the Swachh Bharat Mission at Rajpath in New Delhi. On 2nd October 2014, Swachh Bharat Mission was launched throughout length and breadth of the country as a national movement. The campaign aims to achieve the vision of a ‘Clean India’ (100%) by 2nd October 2019. The Swachh Bharat Abhiyan is the most significant cleanliness campaign by the Government of India. Prime Minister was helped spread the message of Swach Bharat by urging people through his words & action. he carried out a cleanliness drive in Varanasi as well. He wielded a spade near River Ganga at Assi Ghat in Varanasi under the Clean India Mission. He was joined by a large group of local people who cooperated in the Swachhta Abhiyan.

The Prime Minister, since the launch of this movement, said that the scope of hygiene was increased from 40 per cent to 98 per cent after the launch of the Swachh Bharat Abhiyan. “Women safety and dignity was my prime motive behind announcing the Swachh Bharat Mission on 15th August 2014. It was because of the efforts made by the people of India, especially women, that the sanitation coverage in rural India, which stood at merely 39 per cent as of October 2014 even after 67 years of India’s Independence, has today crossed the 98 per cent mark in just over four years of the Swachh Bharat Mission,” PM Modi said.

140. The Swachh Bharat Abhiyaan has a target of achieving 100% cleanliness in India.

Answer: (a)

141. This programme considers cleanliness as one of the responsibility of government.

Answer: (d)

142. Women is one of the major section of Indian society who have contributed towards the aim of achieving cleanliness.

Answer: ()

143. The programme has been launched in all states of India.

Answer: (a)

CASELET 2

The goal of Universal Health Coverage is to ensure that all people obtain the health services they need without suffering financial hardship when paying for them. This requires a strong, efficient, well-run health system; a system for financing health services; access to essential medicines and technologies; and a sufficient capacity of well-trained, motivated health workers. The National Health Protection Mission, dubbed “Modicare” after the country’s Prime Minister Narendra Modi, would offer 100 million families, or about 500 million people living in poverty, up to 500,000 rupees ( $ 7,011) of coverage each year. If rolled out effectively, the NHPM would be a significant step toward achieving universal health coverage a key Sustainable Development Goal for 2030. While Modi has announced the program will be launched later in the month, experts wonder whether the program can truly the off the ground. Every year, 55 million Indians are propelled into poverty because of out-of pocket expenditure on health care, primarily medicine costs. The country currently spend just 1.4 per cent of its gross domestic product on health care – one of the lowest expenditures in the word. Last year, however, the country released it new National Health Policy, the first in 14 years, and pledged to boost health care spending up to 2.5 per cent by 2025. While care in government-run hospitals is technically free for everyone, the system is marred by long queues, poor quality of care, and a lack of human resources.

144. To ensure that all people obtain the health services they need without suffering financial hardship when paying for them.

Answer: (b)

145. Every year, 55 million Indians are propelled into poverty because of out-of-pocket expenditure on health care, primarily medicine costs.

Answer: (a)

146. New National Health Policy pledged to boost health care spending up to 2.5 percent by 2025.

Answer: (b)

147. The system in government run hospitals is marred by long queues, poor quality of care, and a lack human resources.

Answer: (b)

148. The country currently spend just 1.4 percent of its GDP on health care which is one of the lowest expenditure in the world, so by increasing the expenditure on health sector, health services can be improved.

Answer: (d)

149. The goal of Universal Health Coverage is to provide health services to only weaker section of society because they face economic hardships.

Answer: (c)

150. There is a need of a strong, efficient, well-run health system: a system for financing health services; access to essential medicines and technologies; and a sufficient capacity of well-trained, motivated health workers.

Answer: (b)

National Institute of Fashion & Technology (NIFT) UNDER GRADUATE 2019 Question Paper With Answer Key

NIFT (Undergraduate) SOLVED PAPER-2019

1. A sum of Rs 16800 is divided into two parts One part is lend at the simple interest of 6% per annum and the other at 8% per annum. After 2 yr total sum received is Rs 19000. The sum lend at 6% of simple interest is

(a)  Rs 12200

(b)  Rs 12000

(c)  Rs 11000

(d) Rs 10000

Answer: (a)

2. 2 men and 3 women together or 4 men together can complete a piece of work in 20 days. 3 men and 3 women will complete the same work in

(a)  12 days

(b)  16 days

(c)  18 days

(d) 19 days

Answer: (b)

3. A vessel has 30 L of wine and 10 L of water. 4L of mixture is taken out from vessel and 4 L of water is added. This process is repeated one more time. The ratio of quantity of wine is left and initial quantity of wine is

(a)  80 : 92

(b)  81 : 100

(c)  45 : 47

(d) 89 : 91

Answer: (b)

4. A man bought toffees at 6 for Rs. 1. How many toffees for Rs 1 must he sell to gain 50%?

(a)  5

(b)  2

(c)  4

(d) 6

Answer: (c)

5. A sum of Rs 221 is dived among X, Y and Z such that X gets Rs 52 more than Y. Y gets Rs 26 more than Z. The ratio of the shares of X, Y and Z respectively is

(a)  9 : 5: 3

(b)  9 : 3 : 5

(c)  5 : 9 : 3

(d) 10 : 6 : 5

Answer: (a)

6. 11 friends went to a hotel and decided to pay the bill amount equally. But 10 of them could pay Rs 60 each, as a result 11th has to pay Rs 50 extra than his share. Find the amount paid him.

(a)  Rs 105

(b)  Rs 110

(c)  Rs 115

(d) Rs 120

Answer: (c)

7. If on selling 12 notebooks any seller makes a profit equal to the selling price of 4 notebooks, what is his per cent profit?

(a)  50%

(b)  25%

(c)  

(d) Data inadequate

Answer: (a)

8. The respective ratio between the present ages of Parag and Sapna is 21 : 19. Six years ago, the respective ratio between their ages was 9 : 8. How old is Lina, if her present age is 12 yr less than Sapna’s present age?

(a)  38 yr

(b)  28 yr

(c)  26 yr

(d) 30 yr

Answer: (c)

9. Samir drove at the speed of 45 km/h from home to a resort. Returning over the same route, he got stuck in traffic and took on hour longer, also her could drive only at the speed of 40 km/h. How many kilometers did he drive each way?

(a)  250 km

(b)  360 km

(c)  310 km

(d) 275 km

Answer: (b)

10. X is 3 times as fast as Y and is able to complete the work in 40 days less than Y. Then, the time in which they can complete the work together, is

(a)  15 days

(b)  10 days

(c)   

(d) 5 days

Answer: (a)

11. Samira, Malura and Kaira rented a set of DVDs at a rent of Rs 578. If they used it for 8h. 12 h and 14 h. respectively. What is Kiara’s share of rent to be paid?

(a)  Rs 238

(b)  Rs 204

(c)  Rs 192

(d) Rs 215

Answer: (a)

12. In a division sum, the divisor is 10 times the quotient and 5 times the remainder. If the remainder is 46, then the dividend is

(a)  4236

(b)  4306

(c)  4336

(d) 5336

Answer: (d)

13. Rs 730 were divided among A, B, C in such a way that if A gets Rs 3, then B gets Rs 4 and if B gets Rs 3.50 then C gets Rs 3. The share of B exceeds that of C by

(a)  Rs 30

(b)  Rs 40

(c)  Rs 70

(d) Rs 210

Answer: (b)

14. The mean temperature of Monday to Wednesday was 37°C and of Tuesday to Thursday was 34° If the temperature on Thursday was 4/5th that of Monday, then what was the temperature of Thursday?

(a)  36.5°C

(b)  36°C

(c)  35.5°C

(d) 34°C

Answer: (b)

15. If x : y :: 2 : 3 and 2 : x :: 4 : 8 the value of y is

(a)  6

(b)  8

(c)  4

(d) 12

Answer: (a)

16. A copper rod of 1 cm diameter and 8 cm length is drawn into a wire of uniform diameter and 128 m length. The radius (in cm) of wire is

(a)  1/15

(b)  1/30

(c)  2/15

(d) 15

Answer: (b)

17. A box filled with paper bundles weight 36 kg. If the weight of the box and paper bundles respectively are in the ratio of 3 : 22, then the weight of papers (in g) is

(a)  30680

(b)  30710

(c)  31500

(d) 31680

Answer: (d)

18. A solid sphere of 6 cm diameter is melted and recast into 8 solid spheres of equal volume. The radius (in cm) of each small sphere is

(a)  1.5

(b)  3

(c)  2

(d) 2.5

Answer: (a)

19. Three taps A, B and C together can fill an empty cistern in 10 min. The tap A alone can fill it in 30 min and the tap B alone in 40 min. How long will the tap C alone take to fill it?

(a)  16 min

(b)  24 min

(c)  32 min

(d) 40 min

Answer: (b)

20. A man travelled a distance of 80 m in 7 h partly on foot at the rate of 8 km/h and partly on bicycle at 16 km/h. The distance travelled on the foot is

(a)  32 km

(b)  48 km

(c)  36 km

(d) 44 km

Answer: (a)

21. The average of six number is 32. If each of the first three numbers is increased by 2 and each of the remaining three numbers is decreased by 4, then the new average is

(a)  35

(b)  34

(c)  31

(d) 30

Answer: (c)

22. A is twice as good a workman as B and together they finish a piece of work in 14 days. The number of days taken by A alone to finish the work is

(a)  11 days

(b)  21 days

(c)  28 days

(d) 42 days

Answer: (b)

23. A shopkeeper sells an article at a loss of  . Had he sold an article for Rs 51.80 more, he would have earned a profit of 6%. The cost profit of an article is

(a)  280

(b)  580

(c)  370

(d) 450

Answer: (a)

24. A moving train passes a platform 50 m long in 14 s and a lamp-post in 10 s. The speed of the train is

(a)  24 km/h

(b)  36 km/h

(c)  40 km/h

(d) 45 km/h

Answer: (d)

25. Ramesh bought 10 cycles for Rs 500 each. He spent Rs 2000 on the repair of all cycles. He sold five of them for Rs 750 each and the remaining for Rs 550 each. Then, the total gain or loss % is

(a)   

(b)   

(c)   

(d)  

Answer: (d)

26. Ratio between the monthly incomes of A and B is 9 : 8 and the ratio between their expenditures is 8 : 7. If they save Rs 500 each. Find A’s monthly income.

(a)  Rs 3500

(b)  Rs 4000

(c)  Rs 4500

(d) Rs 5000

Answer: (c)

27. When n is divided by 5 then remainder is 2. What is the remainder when n2 is dived by 5?

(a)  2

(b)  3

(c)  1

(d) 4

Answer: (d)

28. In an election there were only two candidates. One of the candidates secured 40% of votes and is defeated by the other candidate by 298 votes. Then the total number of votes polled is

(a)  745

(b)  1460

(c)  1490

(d) 1500

Answer: (c)

29. At some rate of simple interest. A lent Rs 6000 to B for 2 yr and Rs 1500 to C for 4 yr and received Rs 900 as interest from both of them together. The rate of interest per annum was

(a)  5%

(b)  6%

(c)  8%

(d) 10%

Answer: (a)

30. If the annual income of A, B and C is in the ratio 1 : 3 : 7 and the total annual income of A and C is Rs 80000, then the monthly salary of B (In Rs) is

(a)  20000

(b)  25000

(c)  30000

(d) 15000

Answer: (b)

Directions (Q. Nos. 31-33) In each of the following questions, an idiomatic expression/proverb has been given followed by some alternatives. Choose the one which best expresses the meaning of the given idiom/proverb.

31. The boy turned a deaf ear to the pleading of all his well-wishers.

(a)  listened carefully

(b)  was deadly opposed

(c)  posed indifference

(d) did not pay any attention

Answer: (d)

32. Jones is in the habit of fishing in troubled waters.

(a)  taking advantage of troubled conditions for personal profits.

(b)  putting others in trouble

(c)  indulging in evil conspiracies

(d) aggravating the situation

Answer: (a)

33. He rides the high horse because of his high connection.

(a)  is famous

(b)  talks in a flattering way

(c)  puts on airs

(d) is prosperous

Answer: (c)

Directions (Q. Nos. 34 and 35) In each of the questions below, only one among the given alternatives is correctly spelt. Find out the word with correct spelling.

34. 

(a)  Insolance

(b)  Insolence

(c)  Insoolence

(d) Insolense

Answer: (b)

35. 

(a)  Parlamentary

(b)  Parliamentary

(c)  Parliamantry

(d) Parliamentery

Answer: (b)

Directions (Q. Nos. 36 and 37) In each of the following questions, choose the wrongly spelt words.

36.

(a)  Acknowledgment

(b)  Acrostic

(c)  Audible

(d) Accommodate

Answer: (a)

37.

(a)  Occasion

(b)  Commitment

(c)  Brevity

(d) Commitee

Answer: (d)

Directions (Q. Nos. 38 and 39) From among the options given below each word, choose the appropriate singular form.

38. Knives

(a)  Knive

(b)  Kniveses

(c)  Knife

(d) None of these

Answer: (c)

39. Teeth

(a)  Tooths

(b)  Teeths

(c)  Tooth

(d) Toth

Answer: (c)

40. Baby

(a)  Babys

(b)  Babies

(c)  Babes

(d) None of these

Answer: (b)

41. Sheep

(a)  Sheeps

(b)  Sheepes

(c)  Sheep

(d) Sheepesis

Answer: (c)

Directions (Q. Nos. 42-45) In each of these questions, choose the alternative which can replace the underlined word without changing the meaning of the sentence.

42. The bullet would proved to be fatal and the soldier died immediately.

(a)  serious

(b)  dangerous

(c)  deadly

(d) grievous

Answer: (c)

43. It is very difficult to retain all that you hear in the class.

(a)  ignore

(b)  recall

(c)  preserve

(d) conserve

Answer: (c)

44. In spite of their efforts, the team of scientists could not make much headway to solve the problem.

(a)  progress

(b)  thinking

(c)  efforts

(d) start

Answer: (a)

45. We didn’t believe in his statements but subsequent events proved that he was right.

(a)  later

(b)  many

(c)  few

(d) earlier

Answer: (a)

Directions (Q. Nos. 46-49) Fill in the blanks.

46. I commenced work ……. the 1st of January.

(a)  to

(b)  of

(c)  from

(d) on

Answer: (c)

47. An old feud existed ………… the two families.

(a)  among

(b)  within

(c)  in

(d) between

Answer: (d)

48. She ………. being questioned unnecessarily.

(a)  is disliking

(b)  has disliked

(c)  dislikes

(d) disliked

Answer: (c)

49. It was cloudy in the morning but the sun ….. now.

(a)  shines

(b)  is shining

(c)  has been shining

(d) shine

Answer: (b)

Directions (Q. Nos. 50-53) In each of the questions, choose the alternative which is closest to the opposite in meaning to the underlined word.

50. The king consolidated the empire into a powerful nation.

(a)  divided

(b)  isolated

(c)  fragmented

(d) weakened

Answer: (d)

51. Unsettled conditions in the land led to the exodus of hundreds of its citizens.

(a)  departure

(b)  evacuation

(c)  entrance

(d) retreat

Answer: (c)

52. It is obligatory for a common citizen to follow the rules.

(a)  optional

(b)  superfluous

(c)  necessary

(d) advisable

Answer: (a)

53. The actor is well-known both for his humility and courage.

(a)  gentleness

(b)  courage

(c)  honesty

(d) pride

Answer: (d)

Directions (Q. Nos. 54 ad 55) In each of the following questions, choose the alternative that can be substituted for the given word/sentence.

54. A child who stays away from school without any good reason.

(a)  vagabond

(b)  delinquent

(c)  truant

(d) schizophrenia

Answer: (c)

55. An office or post with no work but high pay.

(a)  Honorary

(b)  Sinecure

(c)  Gratis

(d) Ex-officio

Answer: (b)

Directions (Q. Nos. 56-75) Read the following passages given below and answer the questions that follow.

PASSAGE I

Both plants and animals of many sorts show remarkable changes in form, structure, growth habits and even mode of reproduction, in becoming adapted to different climate environment, types of food supply or mode of living. This divergence in response to evolution is commonly expressed by altering form and function of some parts of the organism, the original identification of which is clearly discernible e.g., The creeping foot of the snail is seen in related marine pteropods to be modified into flapping organ useful for swimming and is changed into prehensile arms that bear suctorial disks in the squids and other cephalopods.

The limbs of modes of life for swift running (cursorial) as in the horse and antelope for swinging in trees (arboreal) as in the monkeys, for digging (fossorial) as in the moles and gophers, for flying (Volant) as in the bats, for swimming (aquatic) as in the seals, whales and dolphins and for other adaptations. The structure or organs that show man change in connection with this adaptive divergence are commonly identified readily as homologous, in spite of great alternations. Thus the finger and wrist bones of a bat and whale, for instance, have virtually nothing in common except that they are definitely equivalent element of the mammalian limb.

56. Which is the most appropriate title for the passage, based on its content?

(a)  Evolution

(b)  Our Changing Bodies

(c)  Adaptive Divergence

(d) Changes in Organs

Answer: (c)

57. The author provides information that would answer which of the following questions?

(i) What factors cause change in organism?

(ii) What is the theory of evolution?

(iii) Could structurally different organs be similar in evolution?

Which of the statement given above is/are correct?

(a)  Only (i)

(b)  Only (ii)

(c)  (i) and (iii)

(d) All of these

Answer: (a)

58. The author organises the passage by

(a)  comparison and contrast

(b)  general statements followed by examples

(c)  hypotheses and proof

(d) definition of key terms

Answer: (b)

59. Which of the following words could best be substituted to ‘homologous’ without substantially changing the author’s meaning?

(a)  divergent

(b)  corresponding

(c)  altered

(d) tactile

Answer: (b)

60. The author’s style can best be described as

(a)  objective

(b)  humorous

(c)  esoteric

(d) patronising

Answer: (a)

PASSAGE 2

A great deal of discussion continues as to the real extent of global environmental degradation and its implications. What few people challenge however, is that the renewable natural resources of developing countries are today subject to stresses of unprecedented magnitude. These pressures are brought about, in part, by increased population and the quest for an ever expanding food supply. Because the health, nutrition and general well-being of the poor majority are directly dependent on the integrity and productivity of their natural resources, the capability of governments to manage them effectively over the long term becomes of paramount importance.

Developing countries are becoming more aware of the ways in which present and future economic development must build upon a sound and sustainable natural resource base. Some are looking at our long tradition in environmental protection and are receptive to US assistance which recognizes the uniqueness of the social and ecological systems in these tropical countries. Developing countries recognize the need to improve their capability to analyse issues and their own natural resource management. In February 1981, for example AID funded a National Academy of Sciences panel to advise Nepal on their severe natural resources degradation problems. Some countries such as Senegal. India. Indonesia and Thailand, are now including conservation concerns in their economic development planning process. Because so many governments of developing nations have recognize the importance of these issues, the need today is not merely one of raising additional consciousness, but for carefully designed and sharply focused activities aimed at the establishment of effective resource management regimes that are essential to the achievement of sustained development.

61. Some of the developing countries of Asia and Africa have

(a)  formulated very ambitious plans of protecting habitat in the region

(b)  laid a great stress on the conservation of natural resources in their educational endeavour

(c)  carefully dovetailed environmental conservation with the overall strategy of planned economic development.

(d) sought the help of US experts in solving the problem of environmental degradation.

Answer: (c)

62. Technical know-how developed in the USA

(a)  cannot be easily assimilated by the technocrats of the developing countries

(b)  can be properly utilized on the basis of developing countries being able to launch an in-depth study of their specific problems

(c)  can be easily borrowed by the developing countries to solve the problem of environmental degradation

(d) can be very effective in solving the problem of resource management in tropical countries.

Answer: (c)

63. There has been a pronounced deterioirationof habitat all over the globe because of

(a)  rigorous operation of the Malthusian principle

(b)  unprecedented urbanization and dislocation of self contained rural communities

(c)  optimum degree of industrialization in the developing countries

(d) large scale deforestation and desertification

Answer: (a)

64. The poor people of the developing world can lead a happy and contented life if

(a)  there is a North-South dialogue and aid flows freely to the developing world.

(b)  industries based on agriculture are widely developed.

(c)  economic development takes place within the ambit of conservation of natural resources.

(d) there is an assured supply of food and medical care.

Answer: (c)

65. How much environmental pollution has taken place in the developing and the developed world?

(a)  There has been a marginal pollution of environment in the developed world and extensive damage in the developing world.

(b)  There has been a considerable pollution of environment all over the globe.

(c)  There has been an extensive environmental degradation both in the developed and the developing world.

(d) The environmental pollution that has taken place all over the globe continues to be a matter of speculation and enquiry.

Answer: (d)

PASSAGE III

It is strange that, according to his position in life, an extravagant man is admired or despised. A successful business man does nothing to increase his popularity by being careful with his money. He is expected to display his success, to have a smart car, an expensive life, and to be lavish with his hospitality. If he is not so, he is considered mean, and his reputation in business may even suffer in consequence. The paradox remains that if he had not been careful with his money in the first place, he would never have achieved his present wealth.

Among the low income group, a different set of values exists. The young clerk, who makes his wife a present of a new dress when he hasn’t paid his house rent, is condemned as extravagant. Carefulness with money to the point of meanness is applauded as a virtue. Nothing in his life is considered more worthy than paying his bills. The ideal wife for such a man separates her housekeeping money into joyless little piles so much for rent, for food, for the children’s shoes; she is able to face the milkman with equanimity every month, satisfied with her economizing ways, and never knows the guilt of buying something she can’t really afford.

As for myself, I fall into neither of these categories. If I have money to spare, I can be extravagant, but when as is usually the case, I am hard up, then I am the meanest man imaginable.

66. We understand from the passage that

(a)  thrifty may lead to success.

(b)  wealthy people are invariably successful.

(c)  all mean people are wealthy.

(d) carefulness generally leads to failure.

Answer: (b)

67. As far as money is concerned, we get the impression that the writer

(a)  doesn’t often have any money to save

(b)  would like to be considered extravagant

(c)  is never inclined to be extravagant

(d) is incapable of saving anything

Answer: (a)

68. It seems that low paid people should

(a)  feel guilty if they overspend

(b)  borrow money to meet their essential needs

(c)  not keep their creditors waiting

(d) not pay their bills promptly

Answer: (a)

69. How does the housewife, described by the writer, feel when she saves money? She…..

(a)  wishes she could sometimes be extravagant

(b)  is still troubled by  a sense of guilt

(c)  wishes life were less burdensome

(d) is content to be so thrifty

Answer: (d)

70. The statement “she is able to face the milkman with equanimity” implies that

(a)  she is not upset as she has been paying the milkman his dues regularly.

(b)  she loses here nerve at the sight of the milkman who always demands his dues.

(c)  she manages to keep cool as she has to pay the milkman only a month’s dues.

(d) she remains composed and confidant as she knows that she can handle the milkman tactfully.

Answer: (a)

PASSAGE IV

The sole aim of journalism should be service. The newspaper press is a great power, but jus as an unchained torrent of water submerges whole countryside and devastates crops, even so an uncontrolled press serves but to destroy.

If the control is from without, it proves more poisonous than want of control. It can be profitable only when exercised from within. If this line of reasoning is correct, how many of the journals in the world would stand the test? But who would stop those that are useless? And who should be the judge? The useful and the useless must, like good and evil, go on together, and man must make his choice. The superficiality, the one-sidedness, the inaccuracy and often even dishonesty that have crept into modern journalism, continuously mislead honest men who want to see nothing but justice done.

I have before me extracts from journals containing some gruesome things. There is communal incitement, gross misrepresentation and incitement to political violence bordering on murder. It is of course easy enough for the government to launch out prosecutions or to pass repressive ordinances. These ‘fail’ to serve the purpose intended except very temporarily, and in no case do they convert the writers, who often take to secret propaganda, when the open forum of the press is denied to them.

The real remedy is healthy public opinion that will refuse to patronize poisonous Journals. We have our journalists associations. Why should it not create a department whose business would be to study the various journals and find objectionable articles and bring them to the notice of the respective editors?

The function of the department will be confided to the establishment of contact with the offending journals and public criticism of offending articles where the contact fails to bring about the desired reform. Freedom of the press is a precious privilege that no country can forego. But if there is, as there should be, no legislative check say that of the mildest character, an internal check, say as I have suggested should not be impossible and ought not to be resented?

I hold that it is wrong to conduct newspapers by the aid of immoral advertisements. I do believe that if advertisements should be taken at all there should be a rigid censorship instituted by newspaper proprietors and editors themselves and that only healthy advertisement should be taken.

The evil of immoral advertisements is overtaking even what are known as the most respectable newspapers and magazines.

The evil has to be combated by refining the conscience of the newspapers proprietors and editors. That refinement can come not through the influence of an amateur editor like myself but it will come when their own conscience is roused to recognition of the growing evil or when it is superimposed upon them by a government representing the people and caring for the people’s morals.

71. Which of the following is correct in terms of what is stated in the passage?

(i) External control of press is more dangerous than a press without any checks and control

(ii) Legislative checks are an important means for governing the conduct of press

(a)  Only (i)

(b)  Only (ii)

(c)  Both (i) and (ii)

(d) Neither (i) nor (ii)

Answer: (a)

72. What measure is suggested by the author for regularizing the standard of advertisement in newspapers?

(a)  Development of a healthy public opinion against them

(b)  Regulation by journalist association

(c)  Censorhip instituted from within by the newspapers proprietors and editors

(d) All of the above

Answer: (c)

73. According to the author the real remedy to improve quality of newspapers and journals lies in………

(a)  to put a total ban

(b)  to initiate a healthy public opinion

(c)  bring the objectionable matter to the notice of its proprietors and editors.

(d) Both (b) and (c)

Answer: (d)

74. The author laments that most of the journals and newspapers would fail the ‘test’. Which test is being referred to here?

(i) The test of accurate reporting

(ii) The test of control from within

(iii) The test meeting the standards laid by legislative provisions

(a)  (i), (ii) and (iii)

(b)  (i) and (ii)

(c)  Only (i)

(d) Only (ii)

Answer: (d)

75. In order to improve the quality of newspapers and journals; the government should do all of the following except …………. .

(a)  contact the editors of the offending journals

(b)  stop the freedom of press

(c)  to conduct newspapers by the aid of useful advertisements.

(d) develop a healthy opinion

Answer: (b)

Directions (Q. Nos. 76 and 77) In the following series, replace the questions mark(?) with suitable option.

76. 7, 12, 22, 42, 82, ?

(a)  143

(b)  162

(c)  172

(d) 187

Answer: (b)

77. FUGT, HSIR, JQKP, ?

(a)  KNLO

(b)  LNNM

(c)  LOMM

(d) LOMN

Answer: (d)

Directions (Q. Nos. 78-81) Study the given information carefully to answer the given questions.

Eight people A, B, C, D, M, N, O and P are sitting around a square table (but not necessarily in the same order) in such a way that four of them sit at four corners while four sit in the middle of each of the four sides. The ones sitting in the middle of the sides are facing the centre and the ones sitting at the corners are facing outside (i.e., opposite to the centre). A sits in the middle of one of the sides. Only one person sits between A and M. A sits third to the right of B. Only three people sit between B and N. C sits second to the right of N. O and C face the same direction. M is not an immediate neighbor of O. P sits second to the right of D.

78. Who is to the immediate left of M?

(a)  N

(b)  B

(c)  D

(d) C

Answer: (d)

79. What is the position of O with respect to P?

(a)  Fourth to the left

(b)  Third to the left

(c)  Third to the right

(d) Immediate right

Answer: (b)

80. Which of the following statements is true with respect to the given arrangement?

(a)  None of the given statements is true.

(b)  Only three people sit between O and D.

(c)  D sits third to the left of A.

(d) M sits at one of the corners of the table.

Answer: (a)

81. Who amongst the following sit exactly between B and the one who sits to the immediate right of N, when counted from the left of B?

(a)  O, D

(b)  M, P

(c)  C, P

(d) M, C

Answer: (c)

82. In a certain code, PARTICLE is written as USBQFMDJ, how is DOCUMENT written in that code?

(a)  VDEPUONF

(b)  VDPENFUQ

(c)  VDPENFOU

(d) VDPEUOFN

Answer: (d)

83. In a survey of a town, it was found that 65% of the people surveyed watch the news of TV, 40% read a newspaper and 25% read a newspaper and watch the news on TV. What per cent of the people surveyed neither watch the news on TV nor read a newspaper?

(a)  5%

(b)  10%

(c)  20%

(d) 15%

Answer: (c)

Directions (Q. Nos. 84 and 85) Study the following information and answer the given questions.

Each of the five friends A, B, C, D and E scored different marks in an examination. B scored more than both C and D. A secured more than B but less than E. C did not score the minimum marks. The one who scored second highest marks scored 91 and the one who scored lowest scored 73 marks.

84. Which of the following is true with respect to the given information?

(a)  Only one person scored less than D

(b)  E scored the maximum marks

(c)  There is a possibility that B scored 95 marks

(d) C’s score was definitely more than 91

Answer: (b)

85. If C scored 15 marks less than A’s marks, then which of the following can be B’s score?

(a)  76

(b)  83

(c)  94

(d) 91

Answer: (c)

86. Pointing to a boy in a photograph Sudhir said, “He is the son of the maternal grandfather’s only child. How is the boy related to Sudhir?

(a)  Self

(b)  Brother

(c)  Cousin brother

(d) None of these

Answer: (b)

87. Pointing to a boy in a photograph Sudhir said, “He is the son of the maternal grandfather’s only child. How is the boy related to Sudhir?

(a)  Self

(b)  Brother

(c)  Cousin brother

(d) None of these

Answer: (b)

88. Raj is standing in the middle of a square field. He starts walking diagonally to North-East. Then, he turns right and reaches the far end of the field. Then, he turns right and starts waling. In the mid-way, he again turns right and starts walking. In halfway, the turns to his left and reaches a new far end. In which direction is Raj now?

(a)  North

(b)  South

(c)  North-West

(d) South-West

Answer: (d)

Directions (Q. No. 89) The question below is followed by two arguments numbered I and II. You have to decide which of the arguments is a ‘strong’ argument and which is a ‘weak’ argument.

(a) If only Argument I is strong

(b) If only Argument II is strong

(c) If either I or II is strong

(d) If neither I nor II is strong

89. Should the tenure of the President be longer?

Argument

(I) No, the President of India is non-executive head. Therefore, there is not need to extend his tenure.

(II) Yes, it will provide an opportunity to President to guide the Government to fulfil the long term objectives.

Answer: (d)

Directions (Q. No. 90) In question below a statement is given followed b two assumptions numbered I and II.

Give Answer

(a) If only Assumption I is implicit

(b) If only Assumption II is implicit

(c) If either Assumption I or Assumption II is implicit

(d) If neither Assumption I nor Assumption II is implicit

90. Statement The Municipal Corporation has announced 50 per cent reduction in water supply till monsoon arrives in the city.

Assumptions

(I) People may protest against the unilateral decision of the Municipal Corporation may reduce its taxes from the residents as it failed to provide adequate water.

Answer: (b)

91. Which one of the areas marked I-VII represents the urban educated who are not hard working?

(a)  II

(b)  I

(c)  IV

(d) V

Answer: (c)

Directions (Q. Nos. 92 and 93) Read the following information and answer the given questions.

There are 3 females A, B and E and 4 males C, D, F and G standing in a straight line. No two females are together. B is to right of C, F and D are not together as A is placed between them G is not near B or E but E and F are together. D is not to the right of B. E is not sitting at the extreme end.

92. Who is in the extreme end?

(a)  G and B

(b)  C and F

(c)  B and D

(d) None of these

Answer: (a)

93. Who is exactly in the middle?

(a)  A

(b)  F

(c)  E

(d) None of these

Answer: (b)

Directions (Q. Nos. 94 and 95) In each of the following questions, a statement is followed by two Conclusions I and II. Consider the statement and the following conclusions and decide which of the following conclusions from the statement.

Given Answer

(a) If only Conclusion I follows

(b) If only Conclusion II follows

(c) If either Conclusion I or II follows

(d) If neither Conclusion I nor II follows

94. Statement No country is absolutely self dependent these days.

Conclusions

(I) It is impossible to grow and produce all that a country needs.

(II) Country’s men in general have become lazy.

 

Answer: (a)

95. Statement The nation X faced the increased international opposition due to its decision to go on with eight nuclear explosions.

Conclusions

(I) The citizens of the nation have favoured the decision.

(II) Some powerful nations don’t want others to become powerful.

Answer: (d)

Directions (Q. Nos. 96-100) Study the following information carefully and answer the questions given below.

P, Q, R, S, T, V and W are seven students of a school. Each them studies in different standard from standard IV to standard X not necessarily in the same order. Each of them has favourite subject from English, Science, History, Geography, Mathematics, Hindi and Sanskrit not necessarily in the same order.

Q studies in VII standard and does not like either Mathematics or Geography. R likes English and does not study either in V or in IX. T studies in VIII standards and likes Hindi. The one who likes Science studies in X standard. S studies in IV standard. W likes Sanskrit. P does not study in X standard. The one who likes Geography studies in V standard.

96. In which standard does W study?

(a)  VII

(b)  IX

(c)  X

(d) None of these

Answer: (b)

97. Which subject does P like?

(a)  Geography

(b)  Mathematics

(c)  English

(d) History

Answer: (a)

98. Which subject does S like?

(a)  History

(b)  Geography

(c)  Mathematics

(d) Data inadequate

Answer: (c)

99. In which standard does P study?

(a)  IV

(b)  VII

(c)  IX

(d) V

Answer: (d)

100. Which of the following combination of student-standard-subject is correct?

(a)  T-VIII-Mathematics

(b)  W-VII-Sanskrit

(c)  Q-VII-Geography

(d) V-X-Science

Answer: (d)

101. ‘YOUR STYLE YOUR STORE’ is the tagline of which of the following retail chains?

(a)  Lifestyle

(b)  Pantaloons

(c)  Landmark

(d) Shoppers Stop

Answer: (a)

102. The popular fabric and apparel brand J . Hampstead belongs to……..

(a)  Raymond Group

(b)  Aditya Birla Group

(c)  Dinesh Suiting

(d) Siyaram Silk Mills

Answer: (d)

103. Which one of the following personalities is the brand ambassador of popular personal computers brand HP?

(a)  Priyanka Chopra

(b)  Deepika Padukone

(c)  Anushka Sharma

(d) Kangana Ranaut

Answer: (b)

104. ‘Mia’, the workwear jewellery is from the house of………. .

(a)  Joyalukkas Jewellers

(b)  Malabar Jewellers

(c)  Kalyan Jewellers

(d) Tanishq

Answer: (d)

105. Which one of the following clothing brands was the title sponsor of the Mr. India 2016 held in December 2016?

(a)  Peter England

(b)  Van Heusen

(c)  Park Avenue

(d) Louis Philippe

Answer: (a)

106. ‘Ajile’, is the sports lifestyle brand from the house of ……… .

(a)  Shoppers Stop

(b)  Reliance Trend

(c)  Lifestyle

(d) Pantaloons

Answer: (d)

107. ‘Yepme’ a popular fashion and sportswear brand belongs to

(a)  China

(b)  USA

(c)  UK

(d) India

Answer: (d)

108. Which one of the following clothing brands is not owned or marketed by Raymond Group?

(a)  ColorPlus

(b)  Arrow

(c)  Park Avenue

(d) Parx

Answer: (b)

109. Who is the founder Lakme Cosmetics?

(a)  JRD Tata

(b)  Aditya birla

(c)  Ratan Tata

(d) Mukesh Ambani

Answer: (a)

110. Which of the following retail stores belongs to Aditya Birla Retail Group?

(a)  Big Bazaar

(b)  Croma

(c)  Hypercity

(d) MORE

Answer: (d)

111. Who is the CEO of Biotique herbal cosmetic company?

(a)  Chandra Kochhar

(b)  Vinita Jain

(c)  Indra Nooyi

(d) None of these

Answer: (b)

112. Daikin is considered world’s number one air conditioning company from

(a)  China

(b)  South Korea

(c)  Japan

(d) Indonesia

Answer: (c)

113. Which of the following is the bank established by BRICS group of countries?

(a)  New Strategic Bank

(b)  World Development Bank

(c)  Fortune Development Bank

(d) New Development Bank

Answer: (d)

114. Who among the following is the CEO of ‘Google’?

(a)  Sunder Pichai

(b)  Satya Nadella

(c)  Chuck Robbins

(d) Mark Zuckerberg

Answer: (a)

115. Anirban Lahiri is a well known Indian player of ……….. .

(a)  Chess

(b)  Golf

(c)  Cricket

(d) Billiards

Answer: (b)

116. Which among the following products have not got GI tag?

(a)  Salem Fabric

(b)  Chanderi Sarees

(c)  Kota Doria

(d) Bandhani

Answer: (d)

117. Where is the headquarters of ASEAN located?

(a)  Beijing, China

(b)  Jakarta, Indonesia

(c)  Brazil, South America

(d) Geneva, Switzerland

Answer: (b)

118. Which among the following is a national acquatic animal of India?

(a)  Dolphin

(b)  Whale

(c)  Ghariyal

(d) Rohu

Answer: (a)

119. Which is the only riverine island of India?

(a)  Andaman

(b)  Lakshadweep

(c)  Majuli

(d) Mannar

Answer: (c)

120. Name the Indian city which is also known as ‘Manchester of East’.

(a)  Ahmedabad

(b)  Mumbai

(c)  Chennai

(d) Coimbatore

Answer: (a)

121. In which state is the famous Gir National Park located?

(a)  Rajasthan

(b)  Madhya Pradesh

(c)  Gujarat

(d) Kerala

Answer: (c)

122. Gaganyaan, a space mission, is scheduled to be launched by which year?

(a)  2020

(b)  2021

(c)  2023

(d) 2022

Answer: (d)

123. The classical dance from ‘Sattriya’ belongs to which Indian state?

(a)  Meghalaya

(b)  Karnataka

(c)  Kerala

(d) Assam

Answer: (d)

124. Which Indian river is known as ‘Tasangpo’ in Tibet?

(a)  Godavari

(b)  Brahmaputra

(c)  Ganga

(d) Kaveri

Answer: (b)

125. Name the private label brand launched by Deepika Padukone on Myntra APP?

(a)  Be You

(b)  All Bout You

(c)  Your Fashion Your Style

(d) Today’s Fashion

Answer: (b)

Directions (Q. Nos. 126-138) Read each case very carefully and answer the questions that follow.

CASE 1

Sustainable fashion is today is highly debated and increasingly covered topic in media and at seminars worldwide.

More and more clothing companies are transforming their business models and improving their supply chains to reduce overall environmental impacts, improve social conditions in factories, etc. We also see a growing awareness among consumers, especially younger generations. When we learn about ‘sustainable fashion’, we soon realize that there are many forms of (more) sustainable fashion. Some individuals – usually younger-prefer to experiment with and renew their wardrobes often; using ‘Second hand & Vintage’, ‘Repair, Redesign & Upcycle’ and ‘Rent,, Loan & Swap’ would possibly be of most interest. For people who prefer newly manufactured clothes without’ history and who has a consistent style, ‘On demand’ and ‘High quality & Timeless design’ are likely to be most attractive. All strategies promoting more environmentally, socially and ethically conscious production and consumption are  important steps towards a more sustainable industry.

126. Sustainable fashion today is a

(a)  highly debated topic

(b)  easy to understand

(c)  local concern

(d) outdated

Answer: (a)

127. More and more clothing companies are transforming their business models into

(a)  Environment friendly

(b)  Socially acceptable

(c)  Technology oriented

(d) All of these

Answer: (a)

128. Younger designers prefer to experiment with

(a)  old clothes

(b)  renew clothes

(c)  new clothes

(d) both (a) and (b)

Answer: (d)

129. For people who prefer newly manufactured clothes would prefer those styles which are

(a)  On demand

(b)  Repaired

(c)  Vintage

(d) Second hand

Answer: (a)

130. Important steps towards a more sustainable industry includes ………… sound production and consumption.

(a)  environmentally

(b)  socially

(c)  ethically

(d) All of the above

Answer: (d)

131. Sustainable fashion encompasses

(a)  reusage

(b)  least wastage

(c)  redesign

(d) All of these

Answer: (d)

CASE 2

In a sea of glitz and shimmer, Sabyasachi brought a new idea of beauty – dark shades, dull gold embroidery, mismatched patchwork, tea-stained hues, and a snob’s knowledge of textiles. His subversive aesthetic made him a disruptor, a badge he has worn with great pride for the past two decades. Several have come close, but not sustained their advance. Twenty years later, Sabyasachi still stands strong and stands alone. Here’s why Sabyasachi was the first Indian designer to earn Rs 100 crore a year, in 2014, and his company now notches up a turnover of Rs 253 crore. About seven years ago, there were some companies who were interested, most famously, L Capital, the investment arm of the international luxury conglomerate LVMH, but Sabyasachi says he was not ready then. It isn’t just the numbers, Sabyasachi’s intangibles are his greatest clout. He remains the most plagiarized designer and brand in India. Almost every big label seems to be inspired by him – either the clothes, or the multiple jewels, or even the presentation of ad campaigns.

132. Sabyaschi brought a new knowledge of

(a)  textiles

(b)  architecture

(c)  designs

(d) brands

Answer: (a)

133. Sabyasachi is famous for his

(a)  patchwork

(b)  tea-stained hues

(c)  dark shades

(d) All of these

Answer: (d)

134. Few years back Sabyasachi did not accept the investment from luxury conglomerate LVMH because

(a)  his brand was not ready then

(b)  money was insufficient

(c)  company was small

(d) more time was required by him

Answer: (a)

CASE 3

Tanishq has taken one step further to be more accessible to its customers by launching into the Augmented reality experience at the Bangalore and Delhi airports. Tanishq is leaving no stone unturned to keep their customers happy by adopting innovative ways to display their product range. Tanishq is implementing MirrAR, an Augmented Reality software platform in collaboration with StyleDotMe, a startup focused in innovative application of Augmented Reality (AR) and Artificial Intelligence (AI) for providing the next generation experience to consumers who are interested in the Gems and Jewellery industry. Using the platform users can virtually try on the jewellery in real time, without actually having to wear them. With this technological advancement, customers will have the option of ‘Try and Buy’; trying out the jewellery virtually looking at the AR screen.

135. Tanishq has made its brand more accessible to its customers by launching

(a)  Augmented Reality

(b)  New Store

(c)  Branch

(d) Virtual Reality

Answer: (a)

136. Augmented reality will help costumers

(a)  to purchase more

(b)  to try and buy

(c)  to look at the store

(d) to get custom made design

Answer: (b)

137. Introducing such measures by Tanishq will

(a)  increase their products

(b)  increase their customers

(c)  increase their market

(d) keep customer happy

Answer: (c)

138. Tanishq is implementing MirrAR in collaboration with

(a)  StyleDotMe

(b)  Oneforty

(c)  Voicendo

(d) Hornet

Answer: (a)

Directions (Q. Nos. 139-150) In each of these questions a passage is followed by several inferences. You have to examine each interference separately in the context of the passage and secede upon its degree of truth or falsity.

(a) if the inference is definitely true i.e, it directly follows from the facts given in the passage;

(b) if the inference is probably true i.e, though not definitely true in the light of facts given;

(c) if you think the inference is probably false i.e., though not definitely false in the light of facts given;

(d) if you think the inference is definitely false i.e., contradicts the facts given.

CASELET 1

India ranks 153 out of 190 nations in the percentage of women in the lower house of world parliaments. According to a list compiled by the Inter-Parliamentary Union. Rwanda ranks first with 61% of its lower house representatives being women. Even Pakistan with 20% participation from women is ahead of India. India had 65 women out of 545 Members of Parliament (MPs) elected to the 16th Lok sabha in May 2014, for a 12% representation. Only the 15th and 16th Lok sabha changed a previously stagnant representation of under 9% recorded by Indian women MPs since Independence. While we allocate total seats to states by population, the resultant women’s representation at 12% is far below the actual population of women. So, on grounds of fairness, this is a anomaly/ A fully representative Parliament allows the different experiences of genders to craft priorities and shape the economic and social future of a democratic society.

139. India ranks 153 out of 190 nations in the percentage of women in the lower house of world parliaments which shows that India has poor record of women empowerment.

Answer: (c)

140. Pakistan with 20% participation from women has better representation of women in lower house of parliament than India.

Answer: (a)

141. A fully representative Parliament allows gender sensitive crafting of priorities and economic and social future of a democratic society.

Answer: (a)

142. Without proper representation of women in parliament, society cannot grow.

Answer: (d)

143. To empower woman, it is necessary to give them voice at political level also.

Answer: (a)

CASELET 2

Police personnel patrolling in the forest carrying rifles on their shoulder and looking for smugglers may appear to be a fancy idea. But for the forest department, an armed police patrol force is a much-needed last resort to ward off enemies of the mangroves. The mangrove ecosystem plays vital role in protecting the land against the ingress of sea water during a tidal surge. It’s an invaluable eco-wall and so, and armed police have had to be roped in. It’s been done, albeit rarely, in other parts of the country too. In Kendrapara, powerful prawn mafias have for decades illegally occupied vast stretches of area after clearing mangrove forests. Aquaculture is lucrative business: it gets very high returns, on minimal investment. This is why there is always a race to clear mangroves and convert it into fish ponds.

144. The mangrove ecosystem plays a vital role in protecting the land against the ingress of sea water during a tidal surge.

Answer: (a)

145. Mangroves in Kendrapara are facing threats from sand mafia.

Answer: (d)

146. People are clearing mangrove forests to gain benefits in aquaculture and converting them into fish pond.

Answer: (a)

147. Mangroves are protected because they give sufficient resources to the government.

Answer: (c)

CASELET 3

The creative director of Chanel since 1983, Lagerfeld was designing an average of 14 new collections a year, ranging from couture to the high street. In a show entitled Paris-Bombay Art Show, that he had organized in 2011, the designer took inspiration from India and incorporated heavy silks and traditional South Asian teekas on the women who walked the ramp. The show saw Indian influences being kept to the traditional, with the incorporation of churidar-style leggings as well as ornate teekas. In keeping the traditions, the show had even used the old moniker for Mumbai-Bombay. During his long career, Lagerfeld’s signature combinations attracted a host celebrities including Rihanna; Princess Caroline of Monaco; Christine Lagarde and Julianne Moore among others.

148. Lagerfeld took inspiration from India for his designs shows that Indian tradiational outfits are popular abroad

Answer: (c)

149. By incorporation of churidar-style leggings as well as ornate teekas in his western outfits, Lagerfeld tried to change the traditional concept of Indian fashion.

Answer: (b)

150. Lagerfeld’s signature combinations attracted a host of celebrities including Rihanna; Princess Caroline of Monaco; Christine Lagarde among others.

Answer: (a)

Medical PG Orthopaedics

MEDICAL PG ORTHOPAEDICS

1. Damage to the radial nerve in the spinal groove spares which muscle
(A) Lateral head of triceps
(B) Long head of triceps
(C) Medial head of triceps
(D) Anconeus


Answer: (B)

2. Meralgia paraesthetic is due to the involvement of the
(A) Ulnar nerve
(B) Median nerve
(C) Lateral cutaneous nerve of thigh
(D) Lateral peroneal nerve of thigh

Answer: (C)

3. Cubitus varus is commonly caused by
(A) Supracondylar fracture of humerus
(B) Fracture of the lateral epicondyle
(C) Chronic osteromyelitis
(D) Acute epiphysitis

Answer: (A)

4. Management of Smith’s fracture is
(A) Open reduction and fixation
(B) Plaster cast with forearm in pronation
(C) Closed reduction with below-elbow cast
(D) Above-elbow cast with forearm in supination

Answer: (D)

5. Ortolani’s test is done for
(A) Congenital dislocation of hip
(B) Traumatic dislocation of hip
(C) Rheumatoid arthritis
(D) Tubuerculous arthritis

Answer: (A)

6. Treatment of choice for one week old fracture neck femur at 65 years age is
(A) Hemi-replacement arthroplasty
(B) Closed reduction and internal fixation by cannulated cancellous screws
(C) Closed reduction and internal fixation by Austin more pins
(D) Total hip replacement

Answer: (A)

7. All the following are benign tumors except
(A) Chondroma
(B) Chordoma
(C) Teratoma
(D) Meningioma

Answer: (B)

8. Solitary bone cyst is most common in the
(A) Upper end of humerus
(B) Lower end of humerus
(C) Lower end of humerus
(D) Lower end of femur

Answer: (A)

9. Erosion of bone is seen with all of the following except
(A) Gout
(B) SLE
(C) Psoriasis
(D) Rheumatoid arthritis

Answer: (B)

10. Which of the following is true regarding a phantom limb
(A) Occurs in leprosy
(B) Follows amputation
(C) Follows a psychiatric illness
(D) After filariasis

Answer: (B)

11. The commonest complication of Colle’s fracture is
(A) Malunion
(B) Non-union
(C) Sudeck’s osteodystrophy
(D) Stiffness of fingers

Answer: (D)

12. Injury to the Ulnar nerve at the wrist causes paralysis of
(A) Apposition of the thumb
(B) Abduction of the carpo-metacarpal joint of the thumb
(C) Adduction of the thumb
(D) Flexion of the MCP joint of the middle finger

Answer: (C)

13. All the following are causes of a painful limb except
(A) Slipped femoral epiphysis
(B) TB of the hip
(C) Perthes disease
(D) Infantile Coxa Vara

Answer: (D)

14. Which joint is commonly involved in Osterochondritis Dissecans
(A) Ankle joint
(B) Knee joint
(C) Wrist joint
(D) Elbow joint

Answer: (B)

15. Painless effusion in joints Congential Syphilis are called
(A) Clutton’s joint
(B) Charcot’s joint
(C) Barton’s joint
(D) Chronic osteromyelitis

Answer: (A)

16. In rheumatoid arthritis, pathology starts in the
(A) Articular cartilage
(B) Capsule
(C) Synovium
(D) Muscles

Answer: (C)

17. Treatment of choice for Paget’s disease of the bone is
(A) Vitamin D
(B) Immobilization of the limb
(C) Surgical treatment
(D) Calcitonin

Answer: (D)

18. Commonest site of TB spine is
(A) Dorsolumbar
(B) Lumbar
(C) Lumbosacral
(D) Cervical

Answer: (A)

19. All the following are true about musculocutaneous nerve injury at shoulder except
(A) Loss of flexion at shoulder
(B) Loss of flexion of forearm
(C) Loss of supination of forearm
(D) Loss of sensation on radial side of forearm

Answer: (A)

20. The nerve involved in anterior dislocation of the shoulder is
(A) Radial nerve
(B) Axillary nerve
(C) Ulnar nerve
(D) Musculocutaneous nerve

Answer: (B)

21. The most common type of Supracondylar fracture is
(A) Flexion type
(B) Extension type
(C) Abduction type
(D) Adduction type

Answer: (B)

22. Complications of Colles’ fracture all except
(A) Malunion
(B) Non union
(C) Sudeck’s dystrophy
(D) Ruputre of extensor policis longus

Answer: (B)

23. The most common site of fracture neck of femur that causes avascular necrosis is
(A) Sub-capital
(B) Intertrochanteric
(C) Trans-cervical
(D) Basal

Answer: (A)

24. Treatment of choice in fracture neck of femur in a 40-year old male presenting after 2 days is
(A) Hemiarthroplasty
(B) Closed reduction and Internal fixation by cancellous screws
(C) Closed reduction and Internal fixation by Austin Moore pins
(D) Plaster and rest

Answer: (B)

25. Action of intramedullary ‘K’ nail is
(A) Two-point fixation
(B) Three-point fixation
(C) Compression
(D) Weight concentration

Answer: (B)

26. Which type of injury causes more damage to the semi-lunar cartilage in the Knee
(A) Flexion and extension at the ankle
(B) Rotation on a flexed knee
(C) Rotation on an extended knee
(D) Squatting position

Answer: (B)

27. Which of the following is not a benign bone tumor
(A) Osteoid osteoma
(B) Chondroma
(C) Enchondroma
(D) Chordoma

Answer: (B)

28. Most common nerve to be damaged in dislocation of shoulder is :
(A) Axillary nerve
(B) Radial nerve
(C) Median nerve
(D) Musculocutaneous nerve

Answer: (A)

29. Most common joint to undergo recurrent dislocation is :
(A) Shoulder joint
(B) Patella
(C) Knee joint
(D) Hip joint

Answer: (A)

30. Most common complication of colle’s fracture is :
(A) Stiffness of fingers
(B) Sudeck’s dystrophy
(C) Nonunion
(D) Tendon rupture

Answer: (A)

31. “Tennis elbow’, is characterized by :
(A) Tenderness over the medial epicondyle
(B) Tendinitis of common extensor origin
(C) Tendinitis of common flexor origin
(D) Painful flexion and extension

Answer: (B)

32. Open reduction & internal fixation is done for all of the following #s except :
(A) Patella #
(B) Olecranon #
(C) Von barton’s #
(D) # Lateral condyle of humerus

Answer: (C)

33. All of the following factors fascilitate non union except
(A) Haematoma formation
(B) Periosteal injuries
(C) Absence of nerve supply
(D) Chronic infection

Answer: (A)

34. Most common site of scaphoid fracture is :
(A) Waist
(B) Proximal fragment
(C) Distal fragment
(D) Tilting of the lunate

Answer: (A)

35. First symptom in tuberculous cord compression is :
(A) Sensory change
(B) Decrease tendon reflex
(C) Spasticity
(D) Lower limb weakness

Answer: (C)

36. Most common charcot’s joints involved in diabetes mellitus are those of:
(A) Shoulder
(B) Ankle
(C) Knee
(D) Foot

Answer: (D)

37. Characteristic crystals in pseudogout are :
(A) Calcium pyrophosphate
(B) Sodium monourate
(C) Potassium urate
(D) Sodium pyrophasphate

Answer: (A)

38. Complications of Colle’s fracture include all of the following except :
(A) Malunion
(B) Non Union
(C) Sudeck’s osteodystrophy
(D) Rupture of EPL tendon

Answer: (B)

39. Distal interphalangeal joint is not involved in :
(A) Rheumatoid arthritis
(B) Psoriatic arthritis
(C) Multicentric histeocytosis
(D) Neurophatic arthropathy

Answer: (A)

40. Ankle sprain is due to :
(A) Rupture of anterior talo-fibular ligament
(B) Rupture of posterior talo-fibular ligament
(C) Rupture of deltoid ligament
(D) Rupture of Calcaneo-fibular ligament

Answer: (A)

41. Most common complication of intertrochanteric fracture femur is :
(A) Malunion
(B) Nonunion
(C) Osteoarthritis
(D) Nerve injury

Answer: (A)

42. Commonest site for tuberculous spondylitis is :
(A) T12/L1
(B) C6-7
(C) S1-2
(D) None

Answer: (A)

43. All of the following tumor are benign tumour except :
(A) Chondroma
(B) Chordoma
(C) Osteochondroma
(D) Enchondroma

Answer: (B)

44. Which of the following is persistant biochemical marker of rickets :
(A) S.Ca⁺⁺
(B) S. Alkphosphate
(C) S. Acid phosphate
(D) S. phosphate

Answer: (B)

45. Biochemical marker in early gouty arthritis is :
(A) S. uric acid
(B) Increased urinary uric crystal
(C) Synovial fluid urate crystal
(D) Alkaline phosphate

Answer: (C)

46. Most common site for pseudotumour like growth in haemophilic arthropathy is :
(A) Quadriceps femoris
(B) Hamstring muscle
(C) Gastrocnemius
(D) Iliopsoas

Answer: (D)

47. Pseudoarthrosis may be seen in all of the following conditions except :
(A) Fracture
(B) Idiopathic
(C) Neurofibromatosis
(D) Osteomyelitis

Answer: (D)

48. All of the following are associated with chronic osteomyelitis except :
(A) Sequestrum
(B) Amyloidosis
(C) Myositis-ossificans
(D) Metastatic abscess

Answer: (C)

49. Most common site of Actinomycosis amongst the following is :
(A) Tibia
(B) Rib
(C) Mandible
(D) Femur

Answer: (C)

50. Defective mineralization of proximal zone of cartilar and epiphysis is a feature of :
(A) Rickerts
(B) Scurvy
(C) Osteomalacia
(D) Syphilis

Answer: (A)

51. Most common site of avascular necrosis amongst following is :
(A) Medial candyle of femur #
(B) Talus #
(C) Olecranon #
(D) Head of the radius #

Answer: (B)

52. Commonest site of fracture leading to fat-embolism are :
(A) Tibia #
(B) Femur #
(C) Humerus #
(D) Ulna #

Answer: (B)

53. Jefferson’s # is :
(A) Cl
(B) C₂
(C) C₂C₁
(D) C₂C₃

Answer: (C)

54. Osteoarthritis is associated with all of the following except :
(A) Decrease joint space
(B) Subchondral sclerosis
(C) Osteophyte formation
(D) Ca⁺⁺ deposition in joint space

Answer: (D)

55. According to a newer hypothesis Ewings sarcoma arises from :
(A) Epiphysis
(B) Diaphysis
(C) Medullary cavity
(D) Cortex

Answer: (C)

56. Most common nerve involvement in volkmann’s ischaemic contracture is :
(A) Median nerve
(B) Radial nerve
(C) Ulner nerve
(D) Posterior interosseous nerve

Answer: (A)

57. In posterior dislocation of hip, there is:
(A) Flexion, abduction, int. rotation
(B) Flexion, abduction, ext. rotation
(C) Flexion, adduction, int. rotation
(D) Flexion, adduction, ext. rotation

Answer: (A)

58. Bunion is:
(A) Exostosis
(B) Adventitious bursitis lateral to 1st metatarsal
(C) Base of 1st metatarsal
(D) Head of 1st metatarsal

Answer: (B)

59. Paronychia is defined as infection of:
(A) Pulp space
(B) Web
(C) Subcuticular space
(D) Folds of finger nail

Answer: (D)

60. Clergymen’s knee is due to involvement of:
(A) Prepatellar bursa
(B) Intrapatellar bursa
(C) Supraptellar bursa
(D) Infrapatellar bursa

Answer: (D)

61. Crutch palsy in axilla, nerve most often involved is
(A) Radial
(B) Ulnar
(C) Musculocutaneous
(D) Axillary

Answer: (A)

62. Most common site of osteoblastoma is:
(A) Epiphysis
(B) Metaphysis
(C) Diaphysis
(D) B or C

Answer: (C)

63. Anterior dislocation of shoulder is most commonly complicated by:
(A) Recurrent dislocation
(B) Circumflex nerve injury
(C) Axillary nerve injury
(D) Axillary artery injury

Answer: (C)

64. Syme’s amputation involves joint:
(A) Ankle
(B) Mid tarsal
(C) Tarsometatarsal
(D) All of the above

Answer: (A)

65. Bony ankylosis is caused by:
(A) Pyogenic arthritis
(B) TB
(C) Osteoarthritis
(D) Rheumatoid arthritis

Answer: (A)

66. In talipes equnovarus the foot is in position of:
(A) Equinus, eversion, abduction
(B) Equinus, inversion, abduction
(C) Equinus, inversion, adduction
(D) Equinus, eversion, abduction

Answer: (C)

67. Inversion and eversion occurs at which joint
(A) Subtalar/midtarsal
(B) Metatarsophalangeal
(C) Calcaneocuboid only
(D) Tibiofibular

Answer: (A)

68. Most common muscle involved in lower leg in polio is:
(A) Ant. tibialis
(B) Post. tibialis
(C) Extensor pollicis longus
(D) Flexor pollicis longus

Answer: (A)

69. In cervical rib following are seen except
(A) Cervical rib palpable
(B) Ischaemic pain of muscles
(C) Atrophy of muscles
(D) Radial pulse not palpable

Answer: (A)

70. Most common complication of supracondylar #:
(A) Malunion with gun stock deformity
(B) Volkman’s ischaemicc contracture
(C) Blood vessel injury
(D) Genu valgum

Answer: (A)

71. Complication of acetabular fracture is:
(A) Avascular necrosis of head of femur
(B) Sciatic N injury
(C) Secondary osteoarthritis of hip joint
(D) Fixed deformity of hip joint

Answer: (C)

72. Least correction in remodeling of bone in children is seen in:
(A) Fracture shaft of femur
(B) Fracture shaft of humerus
(C) Fracture above lateral condyle
(D) Subtrochanteric fracture

Answer: (B)

73. Galeazzi fracture is:
(A) Lower 1/3rd # radius with radioulnar dislocation
(B) Upper 1/3rd # ulna with inf. radioulnar dislocation
(C) Lower 1/3rd # ulna without radioulnar dislocation
(D) Upper 1/3rd # ulna with sup. radioulnar dislocation

Answer: (C)

74. Von rosen splint is used in:
(A) CDH
(B) CTEV
(C) Fracture shaft of femur
(D) Fracture tibia

Answer: (A)

75. Burlow’s sign is seen in:
(A) CDH
(B) Fracture neck femur
(C) Perthes disease
(D) Polio

Answer: (A)

76. Psoas spasm is seen in following except:
(A) TB hip
(B) Ac. appednicitis
(C) TB spine
(D) Mesentric iliac lymphadenitis

Answer: (A)

77. Most common organism causing osteomyelitis in drug abusers:
(A) Staph. aureus
(B) Klebsiella
(C) E.coli
(D) Pseudomonas

Answer: (A)

78. Rheumatoid arthritis most commonly causes:
(A) Pericarditis
(B) Endocarditis
(C) Myocarditis
(D) Pancarditis

Answer: (A)

79. Enthesopathy is characteristic of:
(A) OA
(B) TB arthritis
(C) RA
(D) Ankylosing spondylitis

Answer: (D)

80. True about giant cell tumour is
(A) Never recurs after excision
(B) Periarticular epiphyseal
(C) Malignant potential is not predictable
(D) All of the above

Answer: (B)

81. Vitamin required for collagen synthesis is:
(A) Vitamin C
(B) Vitamin E
(C) Vitamin A
(D) Vitamin D

Answer: (A)

82. Trigger finger occurs in :
(A) Rheumatoid arthritis
(B) Trauma
(C) Osteoarthritis
(D) Contracture

Answer: (A)

83. Not attached to pisiform bone :
(A) Flexor retinaculum
(B) Extensor carpi ulnris
(C) Flexor carpi ulnaris
(D) Pisohamate ligament

Answer: (B)

84. Definitive diagnosis for blow out # of orbit is by :
(A) CSF rhinorrhoea
(B) Loss of vision
(C) Tear drop sign
(D) Fracture of sphenoid sinus

Answer: (C)

85. Albert Schonberg disease is also called as :
(A) Osteoporosis
(B) Osteodystrophy
(C) Osteopetrosis
(D) Osteitis punctata

Answer: (C)

86. Caffey’s disease is :
(A) Chronic osteomyelitis is children
(B) Osteomyelitis of jaw in children
(C) Infantile cortical hyperostosis
(D) Renal osteodystrophy

Answer: (C)

87. Chaissagne tubercle is related to :
(A) Cervical vertebra
(B) Digastric tendon
(C) Carotid sinus
(D) Stellate ganglion

Answer: (A)

88. X-ray appearance of malignancy are following except :
(A) Sclerotic margins
(B) Soft tissue mass
(C) Periosteal elelvation
(D) Cortical destruction

Answer: (B)

89. Dupuytren’s contracture is seen in :
(A) Supracondylar # humerus
(B) Colle’s fracture
(C) Thickening of palmar fascia
(D) Radial nerve palsy

Answer: (C)

90. Increase in Limb length is not found in :
(A) Neurofibromatosis
(B) Salter harris #
(C) Haemophilia
(D) Rh. arthritis of juvenile type

Answer: (B)

91. Fasciectomy is indicated when intracompartmental pressure rises above mm Hg :
(A) 10
(B) 60
(C) 45
(D) 100

Answer: (C)

92. Difficult to diagnose salter harris type by X-ray :
(A) V
(B) I
(C) IV
(D) III

Answer: (A)

93. Median nerve injury at wrist causes :
(A) Loss of opposition of thumb
(B) Claw hand
(C) Saturday night palsy
(D) Policemans tip deformity

Answer: (A)

94. About sequestrum all are true except :
(A) Dead piece of bone
(B) Lighter than live bone
(C) Heavier than live bone and trabeculated
(D) Surround an infected nidus

Answer: (C)

95. Backahe, radiating to groin, along lateral part of thigh, front of let and dorsum of foot is mostly due to disc prolapsed at :
(A) L₄ L₅
(B) S₁- S₂
(C) L₅, S₁
(D) L₂ L₃

Answer: (A)

96. Not sensitive to pain :
(A) Synovial membrane
(B) Atricular cartilage
(C) Muscle
(D) Periosteum

Answer: (B)

97. Osteogenesis imperfect is defect in :
(A) Calcification
(B) Bone
(C) Collagen
(D) Cartilage

Answer: (C)

98. Crystals found in pseudogout are of :
(A) Sodium urate
(B) Pyrophosphate
(C) Potassium urate
(D) Oxalate

Answer: (B)

99. Commonest cervical vertebral fracture of :
(A) C₃
(B) C₄
(C) C₂
(D) C₅

Answer: (D)

100. Fatigue # does not occur in :
(A) Metatarsal
(B) Metacarpal
(C) Calcaneum
(D) Tibia

Answer: (B)

101. X-ray in atlanto-axial dislocation should be taken in which position :
(A) Flexion
(B) Extension
(C) Neutral
(D) Open mouth view

Answer: (D)

102. Crystals in synovial fluid are seen in :
(A) Pseudogout
(B) Diabetes mellitus
(C) Syphilis
(D) Osteoarthritis

Answer: (A)

103. Pain along hip, back of thigh and loss of sensation along the lateral b order of foot and decreased ankle jerk, site of lesion is :
(A) L₄ -5
(B) L₅-S₁
(C) L₃
(D) L₁-T₁₂

Answer: (B)

104. Stress # is most often seen in :
(A) 2nd/3rd metalarsal bone
(B) Talus
(C) Calcaneum
(D) Metacarpals

Answer: (A)

105. Trigger finger is caused by:
(A) OA
(B) Rheumatoid arthritis
(C) Tenosynovitis
(D) Injury to tendons

Answer: (B)

106. Treatment for CTEV is started at:
(A) 6 months
(B) Immediately after birth
(C) 1 year
(D) At puberty

Answer: (B)

107. Commonest dislocation of shoulder is:
(A) Subcoracoid
(B) Subacromial
(C) Supraacromial
(D) Posterior

Answer: (A)

108. Gun stock deformity is seen in fracture of:
(A) Lateral condyle
(B) Humerus, lower end
(C) Radius, lower end
(D) Ulna, upper end

Answer: (B)

109. Onion peel appearance on X-ray is seen in:
(A) Giant cell tumour
(B) Ewing’s sarcoma
(C) Osteosarcoma
(D) Fibrosarcoma

Answer: (B)

110. Carpal tunnel syndrome, all are seen except :
(A) Loss of sensation over middle finger
(B) Opponens pollicis atrophy
(C) Flexor pollicis Brevis atrophy
(D) Adductor policies atrophy

Answer: (B)

111. Von-Rosen’s splint is used in :
(A) CDH
(B) CTEV
(C) # shaft of femur
(D) Scoliosis

Answer: (A)

112. Blue sclera and multiple # are seen in :
(A) Osteoporosis
(B) Osteopetrosis
(C) Osteogenesis imperfecta
(D) Osteosclerosis

Answer: (C)

113. Trident hand is seen in:
(A) Mucopolysaccharidosis
(B) Achondroplasia
(C) Diaphyseal achlasia
(D) Chondrodysplasia

Answer: (B)

114. In young man cause of backache and asymmetrical lower limb weakness :
(A) Seronegative spondyloarthropathy
(B) RA
(C) OA
(D) Gout

Answer: (A)

115. Weakest part of mandible is :
(A) Subcondylar part
(B) Coronoid process
(C) Angle
(D) Neck of condyle

Answer: (D)

116. Blount’s disease :
(A) Tibia vara
(B) Renal osteodystrophy
(C) Genu valgum
(D) Genu vara

Answer: (D)

117. Pulled elbow is :
(A) Subluxation of radius head
(B) Subluxation of wrist
(C) Subluxation of humerus
(D) Fracture ulna

Answer: (A)

118. Barlow’s test is used in :
(A) CDH
(B) Coxa vara
(C) Bow legs
(D) Knock knee

Answer: (A)

119. The normal angle of the acetabulum is :
(A) 5-7
(B) 30-32
(C) 35-37
(D) 50-52

Answer: (B)

120. Tom-Smith arthritis spreads to the hip joint because :
(A) Metaphysis is inside the joint
(B) Epiphysis is absent
(C) Periosteum lacks a cambium layer
(D) All of the above

Answer: (A)

121. The most radiosensitive part of the bone is :
(A) Epiphysis
(B) Osteoblastic layer
(C) Growing cartilage cells
(D) Fibroblasts

Answer: (A)

122. Froments sign is used to test :
(A) Adductor pollicis
(B) Flexor pollicis longus
(C) Flexor pollicis brevis
(D) Flexor digitorum profundus

Answer: (A)

123. The classical position adopted in posterior dislocation of the hip is :
(A) Flexion, adduction and internal rotation
(B) Flexion, abduction and external rotation
(C) Flexion, abduction and internal rotation
(D) Extension, adduction and internal rotation

Answer: (A)

124. Patella is at a higher level in :
(A) Recurrent dislocation
(B) Nail-patella syndrome
(C) Rheumatoid arthritis
(D) Plica syndrome

Answer: (A)

125. Progressive stiffening of a joint is seen in :
(A) Peri-arthritis of shoulder
(B) Osteochondritis
(C) Gout
(D) Ankylosis

Answer: (A)

126. Which among the following should not be given by acute gout :
(A) Acetyl salicylic acid
(B) Indomethacin
(C) Piroxicam
(D) Allopurinol

Answer: (D)

127. In a child with congential hip dislocation, the legs are abducted with maintained traction till a click is heard. This test is known is :
(A) Barlow’s test
(B) Ortolani’s lest
(C) Telescopy
(D) Trendelenburg’s sign

Answer: (B)

128. Spina ventosa is caused by :
(A) Tuberculosis
(B) Leprosy
(C) Metastasis
(D) Spine deformity

Answer: (A)

129. The muscle most commonly affected in polio is :
(A) Tensor fasica lata
(B) Tibialis anterior
(C) Tibialis posterior
(D) Quadriceps

Answer: (D)

130. The etiology of tom-smith arthritis is :
(A) Pyogenic
(B) Rheumatoid
(C) Fungal
(D) Syphilis

Answer: (A)

131. Trendelenburg sign is seen in palsy of :
(A) Superior gluteal nerve
(B) IIiohypogastric nerve
(C) Obturator nerve
(D) None of the above

Answer: (A)

132. True regarding mandibular fracture is :
(A) Some infection is harmless
(B) Aim is to get good occlusion
(C) Open reduction is better
(D) Always do wiring

Answer: (B)

133. Ring sign is seen in :
(A) Osteosarcoma
(B) Osteoclastoma
(C) Scurvy
(D) Barlow’s disease

Answer: (C)

134. Equinus deformity in poliomyelitis is due to weakness of :
(A) Dorsiflexors
(B) Plantar flexors
(C) Evertors
(D) Invertors

Answer: (A)

135. True about dupuytren’s contracture is :
(A) More common in females
(B) Palmar nodule is the earliest sign
(C) M.C. in orientals
(D) Contracture of dermal tissue

Answer: (B)

136. Bony ankylosis of hip is seen in :
(A) Pyogenic arthritis
(B) Tuberculosis
(C) Osteoarthritis
(D) Fracture neck femur

Answer: (A)

137. MC cause of genu valgum is :
(A) Trauma
(B) Rickets
(C) Polio
(D) Scurvy

Answer: (A)

138. Trophic ulcer is seen in A/E:
(A) Polio
(B) CNSTB
(C) Spinal dysraphism
(D) Leprosy

Answer: (A)

139. Osteomalacia is characterized by:
(A) Compression vertebral
(B) Cod fish vertebra
(C) Looser’s zone
(D) Bamboo spine

Answer: (C)

140. Treatment of fracture patella in 24 year old young male is :
(A) Patellectomy if undisplaced #
(B) No treatment required
(C) Internal fixation if communited fracture
(D) POP in full extension

Answer: (D)

141. Pointing finger is seen in :
(A) Ulnar N palsy
(B) Median N palsy
(C) Ulnar and median N palsy
(D) Radial N pasly

Answer: (B)

142. All are common with elbow dislocation except :
(A) Myositis ossificans progressive
(B) Median N palsy
(C) Brachial artery injury
(D) Volkmann’s contracture

Answer: (A)

143. Club foot in a 2 year old child is best treated by:
(A) Soft tissue release
(B) Tripple arthodesis
(C) Dennis brown splint
(D) Manipulation by mother and cast

Answer: (A)

144. Non union in long bones is treated by:
(A) Intramedullary nailing
(B) POP cast
(C) Open reduction and internal fixation with bones grafting
(D) Conservation Tt

Answer: (C)

145. Onion peel appearance is seen in :
(A) Osteosarcoma
(B) Ewings sarcoma
(C) Osteoclastoma
(D) Osteochondroma

Answer: (B)

146. Commonest tumour of mandible is :
(A) Ameloblastoma
(B) Osteosarcoma
(C) Squamous cell CA
(D) All

Answer: (A)

147. Soap bubble appearance is the radiological feature of :
(A) Osteoid osteoma
(B) Osteosarcoma
(C) Osteoclastoma
(D) Chrodoma

Answer: (C)

148. True about Osteoclast is all except :
(A) Derived from monocytes
(B) Stimulated by PTH
(C) Phagocytosis of foreign bodies
(D) Resorption of bone

Answer: (C)

149. Albers Schonberg disease is :
(A) Osteopetrosis
(B) Osteoporosis
(C) Osteochondritis
(D) Osteomalacia

Answer: (A)

150. In traumatic myositis ossificans, following are true except :
(A) Hydroxyapatite deposition
(B) Common in elbow injury
(C) Periosteal hematoma & leakage
(D) Injury of tendon over muscle

Answer: (D)

151. Commonest site of march # is :
(A) Involves shaft of 2 & 3rd metatarsals
(B) Avulsion # of 5th metatarsals
(C) Calcaneus involved
(D) Olecranon involved

Answer: (A)

152. Blount’s disease is :
(A) Genu valgus
(B) Tibia vera
(C) Flat foot
(D) Genu recurvatum

Answer: (B)

153. Pathological # is seen in following except :
(A) Radiation
(B) Anaemia
(C) Osteoporosis
(D) Osteomalacia

Answer: (B)

154. True about TB spine is all except :
(A) Early paraplegia is good prognosis
(B) Insidious onset paraplegia is good prognosis
(C) Dorsolumbar spine is commonest site
(D) It is commonest site for TB of bone in the body

Answer: (B)

155. Most common soft tissue tumour in a child :
(A) Rhabdomyosarcoma
(B) Histiocytoma
(C) Fibrosarcoma
(D) Liposarcoma

Answer: (A)

156. How to differentiate gout with pseudogout :
(A) Large joint involvement
(B) Birefringent (Particles) crystals
(C) Serum uric acid normal
(D) Associated with hyperparathyroidism

Answer: (B)

157. All are relevant in compartment syndrome except :
(A) Fasciotomy
(B) Splitting of tight pop cast
(C) Reexploration
(D) Exercise

Answer: (D)

158. Shephared crooke’s deformity is seen is :
(A) Achondroplasia
(B) Gaucher’s disease
(C) Hypothyroidism
(D) Fibrous dysplasia

Answer: (D)

159. Giant cell tumor is treated with :
(A) Resection of tumor part
(B) Curetage & bone graft
(C) Radiotherapy
(D) Melphalan

Answer: (B)

160. Which is/are not true tumor :
(A) Fibrosarcoma
(B) Osteochondroma
(C) Osteoclastoma
(D) Chondrosarcoma

Answer: (B)

161. Iliac horn’ is seen in :
(A) Achondroplasia
(B) Muco-polysaccharodosis
(C) Nail patell syndrome
(D) Tuberous sclerosis

Answer: (C)

162. Components of Gardner’s synd. a/e :
(A) Intestinal polyps
(B) Osteomas
(C) Sebaceous cyst
(D) Neuromas

Answer: (D)

163. Treatment of # with intraarticular extension is/are A/E :
(A) Excision of the joint
(B) Arthrodesis
(C) Traction
(D) Intramedullary nailing

Answer: (D)

164. Mallet finger is due to avulsion of extensor tendon of :
(A) Proximal phalanx
(B) Middle phalanx
(C) Distal phalanx
(D) None of these

Answer: (C)

165. Osgood Schlatters disease affects :
(A) Upper tibia
(B) Lower tibia
(C) Distal femur
(D) Proximal femur

Answer: (A)

166. CTEV is caused by all except :
(A) Neurological disorder
(B) Idiopathic
(C) Spina bifida
(D) Cubitus varus

Answer: (D)

167. Osteogenic sarcoma arise from :
(A) Epiphysis
(B) Metaphysis
(C) Growth plate
(D) Epiphyseal cortex

Answer: (B)

168. In traumatic injury of the hip :
(A) Anterior dislocation is common
(B) Posterior dislocation is common
(C) Avascular fracture is common
(D) Visceral injury usually associated with fracture femur

Answer: (B)

169. Absent lateral 1/3rd of clavicle is seen in :
(A) Hyperparathyroidism
(B) Turner’s syndrome
(C) Fibrous
(D) Cleidocranial dysostosis

Answer: (D)

170. Tubercular bursitis involve :
(A) Prepatellar bursa
(B) Pretibial bursa
(C) Olecranon bursa
(D) Subdeltoid bursa

Answer: (A)

171. True about Osteoclast is all except:
(A) Derived from monocytes
(B) Stimulated by PTH
(C) Phagocytosis of foreign bodies
(D) Resorption of bone

Answer: (B, C)

172. True about Osteoclast is all except:
(A) Derived from monocytes
(B) Stimulated by PTH
(C) Phagocytosis of foreign bodies
(D) Resorption of bone

Answer: (C, B)

173. Major mineral of the bone is:
(A) Calcium chloride
(B) Hydroxyapatite
(C) Calcium oxide
(D) Calcium carbonate

Answer: (B)

174. One of the features given below is essential in the diagnosis of fracture of a bone
(A) Deformity
(B) A crepitus
(C) A partial or complete loss of continuity of the bone
(D) None of the above

Answer: (C)

175. The one most consistent sign of fresh fracture is
(A) Crepitus
(B) Bony tenderness
(C) Deformity
(D) Abnormal mobility
(E) Shortening of bone

Answer: (A)

176. Pathognomonic sign of traumatic fracture is
(A) Redness
(B) Swelling
(C) Crepitus
(D) Tenderness

Answer: (C)

177. Direct impact on the bone will produce a:
(A) Transvers fracture
(B) Oblique fracture
(C) Spiral fracture
(D) Communited fracture

Answer: (A)

178. Pathologic fracture can occur in all except:
(A) Metabolic bone disease
(B) Osteosarcoma
(C) Osteochondroma
(D) Bone cyst
(E) Fluorosis

Answer: (C)

179. Pathological # is seen in following except:
(A) Radiation
(B) Anaemia
(C) Osteoporosis
(D) Osteomalacia

Answer: (B)

180. The commnonest cause of pathological fracture is generalized affection is:
(A) Carcinoma
(B) Osteoporosis
(C) Cyst
(D) All of the above

Answer: (B)

181. The most common cause of pathological fracture is
(A) Delayed union
(B) Mal union
(C) Non union
(D) Secondary deposits
(E) Any of thee

Answer: (D)

182. The treatment of choice in pathological fractures is
(A) Internal fixation
(B) Plaster of Paris casts
(C) Skin traction
(D) External skeletal fixation

Answer: (A)

183. Mirel’s criteria is developed for the evaluation of
(A) Risk of fatigue fracture
(B) Severity of osteoporosis
(C) Risk of pathological fracture
(D) Severity of neurological defecit

Answer: (C)

184. An army recruit, smoker and 6 months into training started complaining of pain at postero medial aspect of both legs. There was acute point tenderness and the pain was aggravated on physical activity. the most likely diagnosis is
(A) Bearger’s disease
(B) Gout
(C) Lumbar canal stenosis
(D) Stress fracture

Answer: (D)

185. Commonest site f march # is:
(A) Involves shaft of 2% 3rd metatarsals
(B) Avulsion # of 5th metatarsals
(C) Calcaneus involved
(D) Olecranon involved

Answer: (A)

186. Stress fracture is treated by:
(A) Rest
(B) Cast immobilization
(C) Closed reduction
(D) Internal fixation

Answer: (B)

187. What is March fracture?
(A) Fracture of 2nd metatarsal
(B) Fracture of 4th metatarsal
(C) Fracture of cuboids
(D) Fracture of tibia

Answer: (A)

188. The usual site of stress fracture includes
(A) Tibia
(B) First metacarpal bone
(C) Second metacarpal bone
(D) Second metatarsal bone

Answer: (D)

189. Stress fracture of involves:
(A) Metatarsals
(B) Metacarpals
(C) Tibia
(D) Calcanium

Answer: (B)

190. Fatigue fractures (Stress fractures) are most commonly seen in:
(A) Metatarsals
(B) Tibia
(C) Fibula
(D) Neck of femur

Answer: (A)

191. All of the following factors facilitate non-union except:
(A) Haematoma formation
(B) Periosteal injuries
(C) Absence of nerve supply
(D) Chronic infection

Answer: (A)

192. Last step in fracture healing is:
(A) Haematoma
(B) Callus formation
(C) Remodeling
(D) Consolidation

Answer: (C)

193. Initial stage of clinical union of bone is equivalent to
(A) Callus formation
(B) Woven bone
(C) Haematoma formation
(D) Calcification only
(E) None of the above

Answer: (B)

194. The time necessary for healing of fracture depends on the following factors:
(A) Age of the patient
(B) Location of the fracture
(C) Type of the fracture
(D) Degree of damage to soft tissues
(E) All of the above

Answer: (E)

195. The most important factor in fracture healing is:
(A) Good alignment
(B) Organization of blood clot
(C) Accurate reduction and 100% apposition of fractured fragments
(D) Immobilization
(E) Adequate calcium intake

Answer: (D)

196. The most common cause of non union is
(A) Infection
(B) Inadequate immobilization
(C) Ischaemia
(D) Soft tissue interposition

Answer: (B)

197. Delayed union of fracture of a bone following a surgical treatment may be due to
(A) Infection
(B) Inadequate circulation
(C) Inadequate mobilization
(D) Soft tissue interposition

Answer: (D)

198. Bone apposition is best in
(A) Osteoblastic activity at the area of stress
(B) Endochondral ossification
(C) Subperiosteal cambian layer
(D) Osteobalstic activity in howship’s lancunae

Answer: (D)

199. Regarding bone remodeling, all are true EXCEPT:
(A) Osteoclastic activity at the compression site
(B) Osteoclastic activity at the tension site
(C) Osteoclastic activity and osteoblastic activity are both need for bone remodeling in cortical and cancellous bones.
(D) Osteoblasts transforms into osteocytes

Answer: (A)

200. Which is not a marker of new bone formation?
(A) Alkaline phosphatase
(B) Osteocalcin
(C) Urine hydroxyproline
(D) Pro collagen

Answer: (C)

201. Bone resorption markers are:
(A) Serum propeptide of type I procollagen
(B) Osteocalcin
(C) Urine total free deoxypyridinoline
(D) Free glutamic acid cross linkage.

Answer: (C)

202. Bone formation markers:
(A) Procollagen-I
(B) Alkaline phosphatase
(C) Hydroxy appetite
(D) Calcitonin
(E) TRAP

Answer: (A, B)

203. Marker for bone formation is:
(A) Tartrate resistant acid phosphate
(B) Osteocalcin
(C) Urinary calcium
(D) Serum nucleotidase

Answer: (B)

204. Indicators of bone formation includes all of following except-
(A) Osteocalcin
(B) Alkaline phosphatase
(C) Hydroxyprolin
(D) Bone scan

Answer: (C)

205. For growth of bone, which term is most appropriate:
(A) Enlargement
(B) Apposition
(C) Hyper plasia
(D) Hypertrophy

Answer: (B)

206. Rate of mineralization of newly formed osteoid can be estimated by the following:
(A) Von Kossa staining for calcium
(B) Alzarin red stain
(C) Labeled tetracycline
(D) Immunoflurescence Calcein Stain

Answer: (C)

207. Callus induction is hampered in:
(A) Hypoxemia
(B) Fractured fragments of bone
(C) Micromovements
(D) Muscle interposition in between fractured fragments
(E) Early mobilization

Answer: (A, B, D)

208. Healing of # of bone is affected by:
(A) Micromovements
(B) Muscle interposition
(C) Hypoxia
(D) Bone fragments

Answer: (ALL)

209. Factors that promotes callus formation:
(A) Micro movements between the fracture fragments
(B) Appropriate approximation of the fragments
(C) Muscle interposed in between the fracture fragment
(D) Early initiation of mobilization
(E) Ischemia

Answer: (A, B)

210. Non – union is a complication of :
(A) Scaphoid #
(B) Colle’s #
(C) Inter – trochanteric # of hip
(D) Supra condylar # of the humerus

Answer: (A)

211. In some old fractures, cartilaginous tissue forms over the fractured bone ends with a cavity in between containing clear fluid. This condition is called as:
(A) Delayed union
(B) Slow union
(C) Non union
(D) Pseudoarthrosis

Answer: (D)

212. Pseudoarthrosis may be seen in all of the following conditions except:
(A) Fracture
(B) Idiopathic
(C) Neurofirbomatosis
(D) Osteomyelitis

Answer: (D)

213. The correct order of priorities in the initial management of head injury is
(A) Airway, Breathing, Circulation treatment of extra cranial injuries
(B) Treatment of extra cranial injuries Airway, Breathing, Circulation
(C) Circulation airway, Breathing treatment of extra cranial injuries
(D) Airway, circulation, breathing treatment of extra cranial injuries

Answer: (A)

214. In an injury with multiple fractures, most important is
(A) Airway maintenance
(B) Blood transfusion
(C) Intravenous fluids
(D) Open reduction of fractures

Answer: (A)

215. Patients comes with fracture femur in an acute accident, the first things to do is
(A) Secure airway and treat the shock
(B) Splinting
(C) Physical examination
(D) X-rays

Answer: (A)

216. Consider the following sign(s)
(1) Increasing pallor
(2) Restlessness
(3) Air hunger
(4) Water-hammer pulse
Haemorrhagic shock due to acute blood loss includes
(A) 1 & 4
(B) 1& 2
(C) 1, 2 & 3
(D) 2, 3 & 4

Answer: (C)

217. Polytrauma patient with open bleeding wound of right thigh should be managed first of all by
(A) Tourniquet application
(B) Tight bandage application
(C) Airway maintenance
(D) Blood transfusion

Answer: (C)

218. Severely injured patient with spinal fracture and unconsciousness first thing to be done is
(A) GCS scoring
(B) Spinal stabilization by cervical collar
(C) Mannitol drip to decrease ICT
(D) Airway maintenance

Answer: (D)

219. Which of the following is not a component of the crust syndrome.
(A) Myohemoglobinuria
(B) Massive crushing of muscles
(C) Acute tubular necrosis
(D) Bleeding diathesis

Answer: (D)

220. Crush syndrome is managed by
(A) 20% Dextrose
(B) Hydrocortisone
(C) Maintaining high urine output
(D) Acidification of urine

Answer: (C)

221. Compound fracture is
(A) Fracture with artery involvement
(B) Fracture with nerve involvement
(C) Fracture with muscle involvement
(D) Fracture with skin involvement

Answer: (D)

222. A patient presents with compound fracture of Tibia with 1 cm opening in skin. Which grade it belongs?
(A) Grade I
(B) Grade II
(C) Grade IIIA
(D) Grade IIIB

Answer: (A)

223. Tibial Fracture with > 1 cm wound, slight comminution and moderate crushing is
(A) Grade I
(B) Grade II
(C) Grade IIIA
(D) Grade IIIB

Answer: (B)

224. Following are principles in the treatment of compound fractures except
(A) Wound debridement
(B) Immediate wound closure
(C) Tendon repair
(D) Aggressive antibiotic therapy

Answer: (B)

225. Immediate treatment of compound fracture of tibia is
(A) Intravenous antibiotics
(B) Thorough debridement
(C) Internal fixation of fracture
(D) Amputation of limb

Answer: (B)

226. Open fracture is treated by
(A) Tourniquet
(B) Internal fixation
(C) Debridement
(D) External fixation

Answer: (C)

227. Internal splints (fixation devices) are used in all except
(A) Compound fractures
(B) Multiple fractures
(C) Fractures in elderly patients
(D) Fracture neck of femur

Answer: (A)

228. A compound fracture is initially treated by antibiotics, wound toilet and
(A) Skin over
(B) External splintage
(C) Prosthesis
(D) Internal fixation

Answer: (B)

229. Which of the following is the most appropriate hospital treatment of a patient with compound fracture?
(A) Under anesthesia, thorough scrubbing and cleaning of the area getting the fracture end inside, suturing the wound and applying continuous skeletal traction with adequate antibiotic cover
(B) Cleaning and suturing the wound, applying plaster Spica under traction on a Harly’s table and administering antibiotics round the clock
(C) Scrubbing and cleaning the area, resecting the protruding one inch of the bone, suturing the wound, bringing the fractured ends into alignment and applying plaster Spica with continuous antibiotic cover
(D) Thorough cleaning of the area, extending the wound, bringing the fragments into alignment under vision, fixing them with intra-medullary nail and giving antibiotics to the patient.

Answer: (A)

230. In shotgun injuries
(A) Each and every shot should be removed
(B) All the shots within accessible limits may be removed and thorough debridement of the tissue done
(C) Shots lodged in joints must be removed
(D) All the above are true

Answer: (B)

231. Tetanus is noticed usually in
(A) Burn cases
(B) Wounds contaminated with faecal matter
(C) Open fractures
(D) Gunshot wounds
(E) All of the above

Answer: (B)

232. Which of the following score evaluates chances of amputation in a traumatized limb:
(A) Revised trauma score
(B) Injury severity score
(C) Abbreviated injury score
(D) MES score

Answer: (D)

233. All of the following factors evaluate the chances of amputation in a limb, except
(A) Age
(B) B.P
(C) Velocity of trauma
(D) Presence of infection

Answer: (D)

234. Tarsometatarsal amputation is also known as
(A) Chopart’s amputation
(B) Lisfranc amputation
(C) Pirogoff amputation
(D) Symes amputation

Answer: (B)

235. Amputation is often not required in:
(A) Gas gangrene
(B) Buerger’s
(C) Chronic osteomyelitis
(D) Diabetic gangrene

Answer: (C)

236. In below elbow amputation the length of stump should be
(A) 10-15 cm
(B) 15-20 cm
(C) 20-25 cm
(D) 5-10 cm

Answer: (B)

237. Distance from Olecranon in amputation should be
(A) 5-10 cm
(B) 10-25 cm
(C) 15-20 cm
(D) 20-30 cm

Answer: (C)

238. In flap method of amputation which structure is kept shorter than the level of amputation:
(A) Bone
(B) Muscles
(C) Nerves
(D) Skin
(E) Vessels

Answer: (A)

239. Ring sequestrum is seen in
(A) Typhoid osteomyelitis
(B) Chronic osteomyelitis
(C) Amputation stump
(D) Tuberculosis osteomyelitis

Answer: (C)

240. Which of the following is true regarding a phantom limb:
(A) Occurs in leprosy
(B) Follows amputation
(C) Follows a psychiatric illness
(D) After filariasis

Answer: (B)

241. Myodesis is employed in amputations for all of the following indications except:
(A) Trauma
(B) Tumor
(C) Children
(D) Ischemia

Answer: (D)

242. Pain due to post-amputation neuroma is best treated by:
(A) Infrared therapy
(B) Interference therapy
(C) Ultrasound therapy
(D) Surgical Excision

Answer: (D)

243. All of the following statements about SACH feet are true, except:
(A) ‘SACH’ stands for ‘Solid Ankle Cushioned Heal’
(B) Forms the base of a lower limb prosthesis
(C) May wear out with time
(D) Wooden keel absorbs the impact of heel strike

Answer: (D)

244. Which of the following is true of shoulder joint?
(A) Composed of only 2 joints
(B) Anterior posterior gliding of a scapula of never occurs
(C) Acromioclavicular joint is more important
(D) Allows flexion, rotation and abduction

Answer: (D)

245. The Rotator cuff is composed of four of the following muscles except:
(A) Teres minor
(B) Supraspinatus
(C) Infraspinatus
(D) Teres major
(E) Subscapularis

Answer: (D)

246. Weakest portion of shoulder joint capsule is:
(A) Anterior
(B) Posterior
(C) Inferior
(D) Superior

Answer: (C)

247. Muscle crossing through the shoulder joint is:
(A) Biceps short head
(B) Biceps long head
(C) Triceps long head
(D) Coracobrachialis

Answer: (B)

248. Rotator interval is between:
(A) Supraspinatus &teresmonor
(B) Teres major &subscapularis
(C) Supraspinatus &subscapularis
(D) Subscapularis&infraspinatus

Answer: (C)

249. Luxatioerecta
(A) Tear of the glenoidal labium
(B) Inferior dislocation of shoulder
(C) Anterior dislocation of shoulder
(D) Defect in the humeral head

Answer: (B)

250. Recurrent dislocation are common in:
(A) Shoulder
(B) Patella
(C) Hip joint
(D) Elbow joint

Answer: (A)

251. Most common joint to undergo recurrent dislocation is:
(A) Shoulder joint
(B) Patella
(C) Knee joint
(D) Hip joint

Answer: (A)

252. Commonest type of shoulder dislocation:
(A) Subcoracoid
(B) Subglenoid
(C) Posterior
(D) Subclavicular

Answer: (A)

253. Recurrent dislocation is least common in
(A) Shoulder
(B) Knee
(C) Patella
(D) None

Answer: (B)

254. Recurrent dislocations are least commonly seen in:
(A) Ankle
(B) Hip
(C) Shoulder
(D) Patella

Answer: (A)

255. Attitude in subcoracoid dislocation of shoulder includes :
(A) Adduction
(B) Limb on side of body
(C) Abduction
(D) Elevation

Answer: (B)

256. The position of arm in anterior dislocation of shoulder is:
(A) By the side
(B) In abduction
(C) In adduction
(D) In external rotation

Answer: (A)

257. Bankart’s lesion is seen at
(A) Post surface of glenoid labrum
(B) Ant surface of glenoid labrum
(C) Ant part of head of humerus
(D) Post part of head of humerus

Answer: (B)

258. Bankart’s lesion involves the ___ of the glenoid labrum.
(A) Anterior lip
(B) Superior lip
(C) Antero-superior lip
(D) Antero-inferior lip

Answer: (D)

259. Recurrent dislocation of shoulder occurs, because of
(A) Incomplete labrum
(B) Superadded secondary infection
(C) Crushed glenoidal labrum
(D) Weak posterior capsule

Answer: (C)

260. Bankart’s lesion involves
(A) Anterior aspect of the head of humerus
(B) Anterior aspect of glenoid labrum
(C) Posterior aspect of glenoid labrum
(D) Posterior aspect of head of humerus

Answer: (B)

261. Hill-sachs lesion is associated with
(A) Recurrent dislocation of shoulder
(B) Recurrent dislocation of hip
(C) Perthe’s disease
(D) Fracture neck of femur

Answer: (A)

262. Hill-sachs lesion in recurrent shoulder dislocation is
(A) Injury to humeral head
(B) Rupture of tendon of supraspinatus muscle
(C) Avulsion of glenoid labrum
(D) None of the above

Answer: (A)

263. Bankart’s lesion is seen in
(A) Anterior border of head of humerus
(B) Posterior border of head of humerus
(C) Anterior glonoid cavity
(D) Posterior glenoid cavity

Answer: (C)

264. All are related to recurrent shoulder dislocation except:
(A) Hill sachs defect
(B) Bankart lesion
(C) Lax capsule
(D) Rotator cuff injury

Answer: (D)

265. The lesion associated with recurrent dislocation of shoulder include all, except:
(A) Hill-Sach’s lesion
(B) Bankart’s lesion
(C) Capsular laxity
(D) Supraspinatous tear

Answer: (D)

266. Recurrent dislocation is most common in the shoulder joint. Which one of the following is not an important cause for the same?
(A) Tear of the anterior capsule of the shoulder
(B) Associated fracture neck of the humerus
(C) Tear of the glenoid labrum
(D) Freedom of mobility in the shoulder

Answer: (B)

267. Which nerve is damaged in ant dislocation of shoulder :
(A) Axillary
(B) Median
(C) Radial
(D) Musculocutaneous

Answer: (A)

268. The nerve involved in anterior dislocation of the shoulder is:
(A) Radial nerve
(B) Axillary nerve
(C) Ulnar nerve
(D) Musculocutaneous nerve

Answer: (B)

269. Most common nerve to be damaged in dislocation of shoulder is:
(A) Axillary nerve
(B) Radial nerve
(C) Median nerve
(D) Musculocutaneous nerve

Answer: (A)

270. Anterior dislocation of shoulder is most commonly complicated by:
(A) Axillary artery injury
(B) Circumflex nerve injury
(C) Recurrent dislocation
(D) Axillary nerve injury

Answer: (B)

271. The most common complication of dislocation of shoulder joint is:
(A) Injury to brachial plexus
(B) Injury to circumflex nerve
(C) Rupture of supraspinatous muscle
(D) Rupture of deltoid muscle

Answer: (B)

272. In Anterior dislocation of the shoulder the nerve involved is:
(A) Radial nerve
(B) Circumflex nerve
(C) Ulnar nerve
(D) Median nerve

Answer: (B)

273. A patient with anterior dislocation of shoulder will most likely give a history of
(A) Abduction and internal rotation
(B) Adduction and internal rotation
(C) Abduction and external rotation
(D) Adduction and external rotation

Answer: (C)

274. In Recurrent Anterior dislocation of shoulder, the movements that causes dislocation is
(A) Flexion and internal rotation
(B) Abduction and external rotation
(C) Abduction and internal rotation
(D) Extension

Answer: (B)

275. All of the following statements about dislocation of the shoulder are true, except
(A) The injury is produced by a fall with the arm fully abducted
(B) The commonest position for the head of the humerus to move into is the subspinous
(C) The auxillary (circumflex humeral) nerve is likely to be injured
(D) The easiest way to reduce it is by simple pressure with the patient under general anesthesia with muscle relaxation

Answer: (B, D)

276. Duga’s test is helpful in
(A) Dislocation of hip
(B) Scaphoid fracture
(C) Fracture neck of femur
(D) Anterior dislocation of shoulder

Answer: (D)

277. Traumatic anterior dislocation of shoulder with sensory loss in lateral side of forearm and weakness of flexion of elbow joint, most likely injured nerve is:
(A) Radial nerve
(B) Ulnar nerve
(C) Axillary nerve
(D) Musculocutaneous nerve

Answer: (D)

278. Following anterior dislocation of the shoulder, a pt develops weakness of flexion at elbow and lack of sensation over the lateral aspect forearm; nerve injured is:
(A) Radial nerve
(B) Musculocutaneous nerve
(C) Axillary nerve
(D) Ulnar nerve

Answer: (B)

279. Traumatic glenohumeral instability on one direction with Bankarts lesion are treated by
(A) Conservative methods
(B) Surgery
(C) Rehabilitation
(D) Observation f/b inferior capsule shift

Answer: (B)

280. A patient with history of recurrent posterior dislocation of humerus is evaluated for a ‘Hill-Sach’s lesion. Which aspect of the head of humerus is likely to show this lesion in the above patient?
(A) Anteromedial
(B) Anterior
(C) Posterolateral
(D) Posterior

Answer: (A)

281. Following statement regarding dislocation of shoulder are true except:
(A) Head of humerus usually dislocates forward from shoulder joint
(B) Injury is produced by forced extension & external rotation of abducted arm
(C) In posterior dislocation, appearance of shoulder is not normal
(D) None of the above

Answer: (C)

282. Regarding Recurrent dislocation of the shoulder, which of the following is false?
(A) All traumatic dislocations will be recurrent
(B) Recurrent dislocation results when the capsule is stripped, not torn
(C) The humeral head is always within the capsule
(D) All of these

Answer: (A)

283. Which is true regarding shoulder dislocation?
(A) Posterior dislocation is often over-looked
(B) Pain is severe in anterior dislocation
(C) Radiography may misleading in posterior dislocation
(D) All of the above

Answer: (D)

284. Which is true about shoulder dislocation?
(A) Anterior dislocation is common than posterior
(B) Fixed medial rotation in posterior dislocation
(C) Kocher’s manoeuvre is effective in anterior dislocation
(D) All of the above

Answer: (D)

285. Which of the following is test of posterior glenohumeral instability:
(A) Fulcrum test
(B) Sulcus test
(C) Jerk’s test
(D) Crank test

Answer: (C)

286. A 6 year old boy has a history of recurrent dislocation of the right shoulder. On examination, the orthopedician puts the patient in the supine position and abducts his arm to 90 degrees with the bed as the fulcrum and then externally rotates it but the boy does not allow the test to be performed. The test done by the orthopedician is
(A) Apprehension test
(B) Sulcus test
(C) Dugas test
(D) MC Murray’s test

Answer: (A)

287. Patient is able to abduct, internally rotate and take his arm upto lumbosacral spine but not able to lift off. What is the probable diagnosis
(A) Subscapularis tear
(B) Teres major tear
(C) Long head of biceps tear
(D) Acromioclavicular joint dislocation

Answer: (A)

288. Lift off test is done to assess the function of:
(A) Supraspinatus
(B) Infraspinatus
(C) Teres Minor
(D) Subscapularis

Answer: (D)

289. Velpeau bandage and Sling and Swathe splint are used in?
(A) Shoulder dislocation
(B) Fracture scapula
(C) Acromioclavicular dislocation
(D) Fracture clavicle

Answer: (C)

290. All are TRUE about clavicle, EXCEPT
(A) No treatment required for fracture but rest
(B) Breaks at midpoint
(C) First bone to ossify
(D) Ossifies in membrane

Answer: (A)

291. True statement regarding Fracture of clavicle is:
(A) Most common Complication is Malunion
(B) Occurs at the jn.of medial 1/3rd& lateral 2/3rd
(C) Usually occurs due to fall on elbow
(D) Communitted fracture is common

Answer: (A)

292. Common injury to baby is:
(A) Fracture humerus
(B) Fracture clavicle
(C) Fracture
(D) Fracture femur

Answer: (B)

293. The most common bone fractured during birth
(A) Clavicle
(B) Scapula
(C) Radius
(D) Humerus

Answer: (A)

294. The most common complication of clavicle fracture is
(A) Injury to brachial plexus
(B) Malunion
(C) Stiffness of shoulder
(D) Non union

Answer: (B)

295. Clavicular fracture is usually treated by
(A) Traction
(B) Open Reduction & Internal fixation
(C) Figure of eight bandage
(D) Plate & Screw fixation

Answer: (C)

296. In treating a fractured clavicle in a 14 month old child, the best procedure is:
(A) Open reduction
(B) Shoulder cast
(C) Figure – of – eight bandage
(D) Kirshner pin

Answer: (C)

297. Fracture of clavicle is commonest at:
(A) Junction of medial 1/3rd and lateral 2/3rd
(B) Junction of medial 2/3rd and lateral 1/3rd
(C) Midpoint
(D) Scapular end

Answer: (B)

298. “Figure of Eight” bandage used commonly in the fracture of
(A) Scapula
(B) Humerus
(C) Clavicle
(D) Metacarpals

Answer: (C)

299. True about fracture clavicle is
(A) Malunion
(B) Most common site is medial 1/3rd& 2/3rd
(C) Comminuted fracture
(D) Due to fall on outstretched hand

Answer: (A, D)

300. Commonest fractures in childhood is:
(A) Femur
(B) Distal humerus
(C) Clavicle
(D) Radius

Answer: (C)

301. Treatment of fracture clavicle in an infant is best treated by:
(A) Cuff and sling
(B) Figure of 8 bandage
(C) Open reduction
(D) Shoulder cast

Answer: (B)

302. Fracture of the clavicle are very common injuries. The most frequent complication of this fracture is:
(A) Malunion
(B) Delayed union
(C) Non union
(D) Nerve injury

Answer: (A)

303. All are true regarding Clavicular fracture except:
(A) May be caused by a fall on to the outstretched arm
(B) Commonly occurs between the insertions of the caraco-Clavicular and the costoClavicular ligaments
(C) May jeopardize blood supply to the overlying skin
(D) Usually requires useful reduction

Answer: (D)

304. Fracture neck Humerus is common in:
(A) Elderly woman
(B) Young lady
(C) Elderly man
(D) All of these

Answer: (A)

305. In fracture surgical neck of humerus, the following nerve injury is common:
(A) Axillary
(B) Radial
(C) Ulnar
(D) Median
(E) Musculocutaneous

Answer: (A)

306. Treatment of choice for fracture neck of humerus is a 70 year old male:
(A) Analgesic with arm sling
(B) U-slab
(C) Arthroplasty
(D) Open reduction – Internal fixation

Answer: (A)

307. Treatment with fracture neck of humerus in a lady will be:
(A) Triangular sling
(B) Hemiarthroplasty
(C) Chest arm bandage
(D) Internal fixation

Answer: (A)

308. A 65 year female presents with impacted fracture surgical neck of humerus. Treatment of choice is:
(A) Arthroplasty
(B) Arm-Chest strapping
(C) Triangular sling
(D) Wait and watch

Answer: (C)

309. Treatment of choice in 65 year old female with impacted # neck of humerus is:
(A) Triangular sling
(B) Arm chest strapping
(C) Arthroplasty
(D) Observation

Answer: (A)

310. Most common nerve involvement in fracture surgical neck humerus :
(A) Axillary nerve
(B) Radial nerve
(C) Ulnar nerve
(D) Median nerve

Answer: (A)

311. A boy fell down from a tree and has fracture of neck humerus. He cannot raise his arm because of the involvement of:
(A) Axillary nerve
(B) Supraspinatus nerve
(C) Musculocutaneous nerve
(D) Radial nerve

Answer: (A)

312. Which of the following movements will be affected if the greater tubercle of the humerus is lost:
(A) Abduction and lateral rotation
(B) Adduction and flexion
(C) Adduction and medial rotation
(D) Flexion and medial rotation

Answer: (A)

313. Hanging cast is sued in:
(A) # Femur
(B) # Radius
(C) # Tibia
(D) # humerus

Answer: (D)

314. In a fracture shaft humerus, which of the following complication requires immediate surgery?
(A) Compound fracture
(B) Nerve injury
(C) Brachial artery occlusion
(D) Comminuted fracture

Answer: (C)

315. The most important cause of Nonunion of fracture of humeral shaft is
(A) Comminuted fracture
(B) Compound (Open) fracture
(C) Overriding of fracture ends
(D) Distraction at fracture site
(E) Operative reduction

Answer: (D)

316. First to appear amongst the ossification centres about the elbow is:
(A) Radial Head
(B) Olecranon
(C) Lateral epicondyle
(D) Capitellum

Answer: (D)

317. Three point relationship is reversed in all, except
(A) # medial epicondyle
(B) # lateral epicondyle
(C) Supracondylar #
(D) Posterior elbow dislocation

Answer: (C)

318. Three bony point relationship is maintained in:
(A) Supracondylar # humerus
(B) Dislocation of elbow
(C) # Lateral condyle
(D) Intercondylar #

Answer: (A)

319. Three point relationship is lost in all, EXCEPT
(A) Fracture of lateral epicondyle
(B) Fracture of medial epicondyle
(C) Posterior dislocation of elbow
(D) Supracondylar fracture of humerus

Answer: (D)

320. Most common nerve involved in supracondylar fracture of humerus is:
(A) Radial nerve
(B) Ulnar nerve
(C) Median nerve
(D) Anterior interosseus nerve

Answer: (D)

321. Triangular relation of Elbow is maintained in
(A) Fracture ulna
(B) Anterior dislocation of Elbow
(C) Posterior dislocation of Elbow
(D) Supracondylar fracture

Answer: (A)

322. Increased intercondylar distance is seen in fracture of all except:
(A) Olecranon
(B) Medial epicondyle
(C) Lateral epicondyle
(D) Lateral condyle

Answer: (B)

323. 3 point symmetry is NOT disturbed in which fracture
(A) Fracture ulna only
(B) Fracture radius only
(C) Fracture of radius & ulna both bones of forearm
(D) Weak posterior capsule

Answer: (B)

324. Postero lateral anconeus triangle is formed by
(A) Head of radius, lateral epicondyle, medial epicondyle
(B) Head of radius, lateral epicondyle, olecranon
(C) Olecranon, medial epicondyle, neck of radius
(D) Neck of radius, head of radius, lateral epicondyle

Answer: (B)

325. All true regarding fracture lateral condyle humerus except:
(A) Salter Harris type IV injury
(B) Most common complication of surgically treated cases is cubitus valgus deformity
(C) Tardy ulnar nerve palsy occurs
(D) Cubitusvarus occur more commonly than valgus
(E) Open reduction & internal fixation

Answer: (C)

326. Fracture of lateral condyle of humerus seen in age group of
(A) 2- 3 years
(B) 3 – 5 years
(C) 5 – 15 years
(D) 15 – 25 years
(E) 35 – 45 years

Answer: (A)

327. “Cubitus – valgus” deformity is complication of:
(A) # Lateral condyle of humerus
(B) # Interconduylar of humerus
(C) # Of the olercranon
(D) # Head of the radius

Answer: (B)

328. Tardy ulnar nerve paly seen in
(A) Medial condyle # humerus
(B) Lateral condyle # humerus
(C) Supracondylar condyle # humerus
(D) Fracture shaft humerus

Answer: (B)

329. Tardy ulnar neuritis may be due to
(A) Advanced osteo arthritis of elbow
(B) Cubitus valgus deformity
(C) Both of the above
(D) None of the above

Answer: (A)

330. A 12-year-old child presents with tingling sensation and numbness in the little finger and gives history of fracture in the elbow region 4 years back. The probable fracture is
(A) Lateral condyle fracture humerus
(B) Injury to ulnar nerve
(C) Supracondylar fracture humerus
(D) Dislocation of elbow

Answer: (C)

331. Tardy ulnar nerve palsy occur as a delayed sequel of
(A) Supracondylar fracture of humerus
(B) Posterior dislocation of elbow
(C) Fracture of lateral condyle of humerus in children
(D) Fracture of Olecranon

Answer: (A)

332. Tardy Ulnar nerve palsy is caused by
(A) Fracture lateral epicondyle of humerus
(B) Fracture medial epicondyle of humerus
(C) Elbow dislocation
(D) Supra condylar fracture of humerus

Answer: (D)

333. Which fracture requires open reduction in children?
(A) Fracture of both bones of forearm
(B) Epiphyseal separation of tibia
(C) Intercondylar fracture of femur
(D) Lateral condyle fracture of humerus

Answer: (C)

334. Open reduction in children is required for
(A) Fracture both bones of forearm
(B) Femoral condyles
(C) Lateral humeral condyle
(D) Distal tibial epiphysis

Answer: (B)

335. Which of the following is known for Non union in children, if left untreated?
(A) Inercondylar fracture of humerous
(B) Fracture shaft of humerus
(C) Fracture shaft of femur
(D) Fracture distal 1/3rd of tibia
(E) Fracture lateral condyle of humerus

Answer: (B)

336. A 6 year old child has an accident and had # elbow, after 4 years presented with tingling and numbness in the ulnar side of finger, fracture is
(A) supra condylar # humerus
(B) lateral condylar # humerus
(C) olecranon
(D) dislocation of elbow

Answer: (A)

337. “Cubitus-valgus” deformity is complication of:
(A) # Lateral condyle of humerus
(B) # Intercondylar of humerus
(C) # Of the olecranon
(D) # Head of the radius

Answer: (B)

338. A patient sustained injury to the upper limb 3 yrs back; he now presents with valgus deformity in the elbow and paresthesias over the medial border of the hand. The injury is likely to have seen:
(A) Supracondylar # humerus
(B) Lateral condyle # humerus
(C) Medial condyle # humerus
(D) Post dislocation of the humerus

Answer: (C, D)

339. Fracture lateral condyle of the humerus is a common injury in children. Which one of the following is the most ideal treatment for a displaced fracture lateral condyle of the humerus in a 7-year-old child?
(A) Open reduction and plaster immobilization
(B) Closed reduction and plaster immobilization
(C) Open reduction and internal fixation
(D) Excision of the fractured fragment

Answer: (C)

340. Growth disturbance, nonunion, elbow instability & late ulnar nerve palsy is commonly seen in
(A) Fracture supracondylar humerus
(B) Fracture medial condyle
(C) Fracture lateral condyle
(D) Fracture head radius

Answer: (C)

341. Medial epicondyle fracture results in injury to – nerve.
(A) Radial
(B) Median
(C) Ulnar
(D) Axillary

Answer: (A)

342. Most common associated injury of the fracture medial epicondyle:
(A) Elbow dislocation
(B) Monteggia fracture dislocation
(C) Fracture supracondylar humerus
(D) Vascular defecit

Answer: (A)

343. Suspected medial epicondylar fracture of humerus in a 4 year old child requires:
(A) X-ray both arms with elbow for comparison
(B) X-ray same limb only
(C) Examination under general anaesthesia
(D) POP in full flexed position

Answer: (D)

344. The most common injury in a 7 years old child due to fall on outstretched hand is
(A) Dislocation of shoulder
(B) Colle’s fracture
(C) Fracture of clavicle
(D) Supracondylar fracture of humerus

Answer: (C)

345. After falling from a height, a child lands on his fully out stretched hands. ON examination there is pain and swelling over his right elbow. Give your probable diagnosis:
(A) Fracture olecranon
(B) Post dislocation of elbow
(C) Supra condylar fracture of humerus
(D) Fracture both bones forearm

Answer: (C)

346. Most common elbow injury in adolescents is:
(A) Dislocation
(B) Physeal injury
(C) Supracondylar fracture
(D) Olecranon fracture

Answer: (A)

347. The most common elbow injury in children is
(A) Extension type of supracondylar fracture of humerus
(B) Dislocation of elbow
(C) Fracture lateral condyle of humerus
(D) Fracture medial epicondyle of humerus

Answer: (C)

348. In supracondylar fracture of humerus, the distal segment is often displaced to;
(A) Anteriorly
(B) Laterally
(C) Posterioly
(D) Medially

Answer: (C)

349. The most common type of supracondylar fracture is
(A) Neutral
(B) Flexion
(C) Extension
(D) Lateral

Answer: (D)

350. Fracture supracondylar fracture is usually caused by :
(A) Hyper flexion injury
(B) Axial rotation
(C) Extension injury
(D) Hyper extension injury

Answer: (A)

351. In flexion injuries causing supracondylar fractures, the distal fragment is often displaced to
(A) Anterior
(B) Posterior
(C) Medial
(D) Lateral

Answer: (D)

352. A 10-year-old boy presenting with a cubitusvarus deformity and a history of trauma 3 months back on clinical examination, has the preserved 3 bony point relationship of the elbow. The most probable diagnosis is
(A) Old unreduced dislocation of elbow
(B) Non-union lateral condylar fracture of humerus
(C) Malunitedintercondylar fracture of humerus
(D) Malunited supracondylar fracture of humerus

Answer: (B)

353. The malunion of supracondylar fracture of the humerus most commonly leads to:
(A) Flexion deformity
(B) Cubitusvarus
(C) Cubitus valgus
(D) Extension deformity

Answer: (D)

354. Cabitusvarus is most commonly seen in
(A) Rickets
(B) Post inflammatory epiphyseal damage
(C) Fracture lateral condyle humerus
(D) Malunited supracondylar fracture

Answer: (B)

355. The most common deformity seen in supracondylar fracture of humerus is
(A) Inability to supinate and pronate
(B) Varus
(C) Valgus
(D) None
(E) Malunion with gunstock deformity

Answer: (E)

356. The most common complication of supracondylar fracture is
(A) Osteosarcoma
(B) Genu valgum
(C) Blood vessel injury
(D) Volkman’sischaemic contracture
(E) Malunion with gun stock deformity

Answer: (C)

357. Gunstock deformity is due to
(A) Fracture of 1st metacarpal bone
(B) Fracture of lower end of radius
(C) Supracondylar fracture of humerus
(D) Lateral condylar fracture of humerus

Answer: (A)

358. Osteotomy done for mal united supracondylar fracture is:
(A) French
(B) Shanz’s
(C) McMurry’s
(D) McAlister

Answer: (E)

359. The following fractures are known for Non-union except:
(A) Fracture of lower half of tibia
(B) Fracture of neck of femur
(C) Fracture of scaphoid
(D) Fracture of patella
(E) Supracondylar fracture of humerus

Answer: (D)

360. All of the following are complications of supracondylar fracture of humerus in children, except:
(A) Compartment syndrome
(B) Myositis ossificans
(C) Malunion
(D) Non Union

Answer: (C)

361. Least common complication of closed supracondylar fracture in a 8 year old child is:
(A) Cubitus valgus
(B) Cubitusvarus
(C) Nonunion
(D) Vessel injury

Answer: (D)

362. Which complication may arise after supra-condylar fracture?
(A) Median nerve injury
(B) Damage to brainchild artery
(C) Cubitusvarus
(D) All of the above

Answer: (C)

363. Complications of supracondylar fracture of humerus are all except:
(A) Elbow stiffness
(B) Mal union
(C) Non union
(D) Myositis ossification
(E) Gun-Stock deformity

Answer: (D)

364. In supra condylar fracture of humerus, the nerve most commonly injured is
(A) Radial nerve
(B) Ulnar nerve
(C) Median nerve
(D) Auxillary nerve

Answer: (C)

365. Tardy unlar nerve palsy caused by:
(A) Supracondylar #
(B) Lateral condylar #
(C) Olecranon #
(D) Intercondylar
(E) Distal radio-ulnar dislocation

Answer: (B, A)

366. All of the following are associated with supracondylar fracture of humerus, EXCEPT?
(A) It is uncommon after 15 yrs of age
(B) Extension type fracture is more common than the flexion type
(C) Cubitusvarus deform commonly results following malunion
(D) Ulnar nerve is most commonly involved.

Answer: (D)

367. The true statement regarding supracondylar fracture of the Humerus in children
(A) Admission to hospital is essential following reduction
(B) It is due to a fall on the point of the elbow
(C) It require open reduction
(D) It is usually compound

Answer: (A)

368. A10-year-old boy presenting with a cubitusvarus deformity and a history of trauma 3 months back on clinical examination, has the preserved 3 bony pint relationship of the elbow. The most probable diagnosis is
(A) Old unreduced dislocation of elbow
(B) Non-union lateral condylar fracture of humerus
(C) Malunitedintercondylar fracture of humerus
(D) Malunited supracondylar fracture of humerus

Answer: (D)

369. The most common cause of Volkmann’s ischaemic contracture (V.I.C) in a child is:
(A) Intercondylar fracture of humerus
(B) Fracture both bone of forearm
(C) Fracture lateral condyle of humerus
(D) Supracondylar fracture of humerus

Answer: (D)

370. Vascular injury during childhood is common in fracture of:
(A) Lower end of humerus
(B) Lower end radius
(C) Upper end of femur
(D) Upper end of radius

Answer: (A)

371. The first again of Volkman’s ischemia is:
(A) Paresthesia
(B) Pain on passive extension of fingers
(C) Pain on active extension of fingers
(D) Swelling of fingers

Answer: (B)

372. The most important sign in Volkmann’s ischaemic contracture is:
(A) Pain
(B) Pallor
(C) Numbness
(D) Obliteration of radial pulse

Answer: (A)

373. Volkman’sischaemic contracture mostly involves
(A) Flexor digitorumsuperficialis
(B) Pronator teres
(C) Flexor digitorumprofundus
(D) Flexor carpi radialislongus

Answer: (C)

374. Most common muscle involved in volkmann’s ischemic contracture is
(A) Flexor pollicislongus
(B) Flexor-digitorumprofunds
(C) Flexor-indicis
(D) Abductor policis

Answer: (B)

375. EarliestIschaemic feature after reduction of Supracondylar fracture is
(A) Coldness
(B) Pain
(C) Swelling
(D) Tingling

Answer: (B)

376. The most common cause of anterior compartment syndrome is
(A) Fractures
(B) Post ischaemic swelling
(C) Superficial injury to muscles
(D) Operative trauma

Answer: (A)

377. Earliest symptom of Volkmann’s Ischaemic is
(A) Pain in flexor muscles
(B) Absence of pulse
(C) Pain on passive extension
(D) Cyanosis of limb

Answer: (C)

378. One of the following is associated with Volkmann’s ischaemic contracture
(A) Supracondylar fracture of humerus
(B) Fracture shaft humerus
(C) Intercondylar fracture of humerus
(D) Dislocation of elbow

Answer: (A)

379. In Volkman’s ischemia, surgery should be done;
(A) Immediately
(B) After 6 hours
(C) 24 hours
(D) 72 hours

Answer: (A)

380. A patient presenting with Volkmann’s Ischaemia all of the following are done except
(A) Split open the plaster of Paris cst and bandage
(B) Decompression by fasciotomy
(C) Exploration
(D) Sumpathetic ganglion blockade

Answer: (D)

381. Volkmann’s contracture:
(A) Is localized thickening of palmar fascia
(B) Develops at the ankle in a case of chronic venous ulcer
(C) Follows Ischemia of the forearm
(D) Is due to excessive scarring of the skin of the arm following a burn.

Answer: (C)

382. Volkmann’s Ischaemic Contracture is due to
(A) Injury to ulnar and median nerve
(B) Injury to median nerve alone
(C) Contracture of the palmar fascia
(D) Ischaemic vascular injury to the muscle
(E) All of the above

Answer: (D)

383. In Volkmann’s ischemia, surgery should be done within:
(A) 24 hours
(B) 36 hours
(C) 1 hour
(D) 6 hours

Answer: (C)

384. Volkmann’s ischameic contracture is commonly due to
(A) Tight
(B) palster
(C) Both
(D) None

Answer: (C)

385. What is true about compartment syndrome:
(A) Loss of pulses is reliable sign
(B) Pain on passive stretch is reliable sign
(C) Interstitial stretch is reliable sign
(D) Fasciotomy is earliest management

Answer: (B, D)

386. Volkmann’s Ischaemic contracture is due to
(A) Arterial injury
(B) Venous injury
(C) Nerve injury
(D) Increase of compartment pressure in the limb

Answer: (D)

387. All are relevant in compartment syndrome except:
(A) Fasciotomy
(B) Splitting of tight pop cast
(C) Reexploration
(D) Exercise

Answer: (D)

388. Which of the following does not involve nerve damage
(A) GuillianBarre syndrome
(B) Erb’s paralysis
(C) Volkmann’s paralysis
(D) Neurotmesis

Answer: (C)

389. The most common nerve involved in Volkamann’sischaemic contracture of forearm includes
(A) Radial
(B) Ulnar
(C) Median
(D) Posterior interosseous

Answer: (C)

390. All are correct regarding compartment syndrome
(A) Pulse is reliable indicator
(B) Pain on passive stretching
(C) Interstitial pressure > capillary pressure
(D) Hyperesthesia
(E) Fasciotomy is the earliest treatment

Answer: (A)

391. Myositis ossificans is most common around the ____ joint
(A) Knee
(B) Elbow
(C) Wrist
(D) Hip

Answer: (B)

392. Treatment of Acute Myositis Ossificans is
(A) Active mobilization
(B) Passive mobilization
(C) Infra Red Therapy
(D) Immobilization

Answer: (D)

393. Myositis ossificans is due to:
(A) Ossification
(B) New bone formation
(C) Ossification of subperiostealhaemoatoma
(D) Migration osteoblasts to haematoma
(E) All of the above

Answer: (E)

394. Treatment of choice in Acute myositis ossificans is:
(A) Immobilization of elbow
(B) Short wave daithermy
(C) Passive movements of arm
(D) Active exercises

Answer: (A)

395. Following are recognized F/O Myositis ossificans EXCEPT:
(A) It is a post traumatic ossification
(B) It follows either a posterior dislocation or a supracondylar fracture of elbow joint
(C) The complication less likely in children
(D) Diagnosis is made with certainly by skiagraphy

Answer: (C)

396. In traumatic myositis ossificans, following are true except:
(A) Hydroxyapatite deposition
(B) Common in elbow injury
(C) Periosteal hematoma & leakage
(D) Injury of tendon over muscle

Answer: (A, D)

397. In myositis ossificans mature bone is seen:
(A) At periphery
(B) In center
(C) Whole muscle mass
(D) In the joint capsule

Answer: (A)

398. Radiological feature differentiating myositis ossificans from bone tumor is:
(A) Peripheral ossificans
(B) Central lucency
(C) Discontinuity with the bone
(D) None

Answer: (C)

399. Which of the following is not true about Myositis ossificans?
(A) Associated with muscle tendon rupture
(B) Inflammation around the ruptured muscle deposition of hydroxyapatite crystals with
(C) Common in supracondylar fracture
(D) Ossification of musculo-periosteal haematoma

Answer: (A)

400. False about myositis ossificans progressive (child with heterotopic ossifications) is
(A) Pneumonia is common
(B) Life longevity is normal
(C) Most common site involved is the spine
(D) Onset is before 6 year

Answer: (B)

401. The basic pathology in Myositis Ossificans Progressive is in
(A) Muscle fibers
(B) Serum chemistry
(C) Body collagen
(D) None of the above

Answer: (A)

402. A person of 60 years age is suffering from myositis ossificans progressive. The usual cause of death would be:
(A) Nutritional deficiency
(B) Bed sore
(C) Lung disease
(D) Septicemia

Answer: (C)

403. All are true regarding myositis ossificans progressive except
(A) Usually involve childs
(B) Progressive form of normal myositis
(C) Respiratory problems
(D) Ankylosis
(E) Form normal bone

Answer: (B)

404. A patient presents with heterotopic ossification around the knee joint. The laboratory investigation of choice in him is:
(A) Alkaline phosphatase
(B) Serum calcium
(C) Serum potassium
(D) Serum acid phosphates

Answer: (A)

405. A child is spinned around by holding his hand by his father. While doing this the child started crying and does not allow his father to touch his elbow. The diagnosis is:
(A) Pulled elbow
(B) Radial head dislocation
(C) Annular ligament tear
(D) Fracture olecranon process

Answer: (A)

406. Pulled Elbow is:
(A) Disarticulation of elbow
(B) Subluxation of distal radio-ulnar joint
(C) Subluxation of proximal radio ulnar joint
(D) None of the above

Answer: (C)

407. Pulled elbow is
(A) A sprain of extensor tendons
(B) Dislocation of head of radius
(C) Fracture of lateral condyle of humerus
(D) Dislocation of elbow

Answer: (B)

408. A 30 y ears old male comes to ortho emergency with his 3 years old daughter who is crying. The father gives the history of child being swung by forearm. The most probable diagnosis is:
(A) Supracondylar humerus fracture
(B) Elbow dislocation
(C) Stress fracture
(D) Pulled elbow

Answer: (D)

409. A one and a half year old child holding her father’s hand slipped and fell but did not let go of her father’s hand. After that she continued to cry and hold the forearm in pronated position and refused to move the affected extremity. Which of the following management at this stage is most appropriate?
(A) Supinate the forearm
(B) Examine the child under GA
(C) Elevate the lib and observe
(D) Investigate for osteomyelitis

Answer: (A)

410. A 3 year old girl has developed painful elbow after being jerked by the forearms. The next thing to be done is:
(A) Cuff & collar sling immobilization.
(B) Raduction setting and AEPOP application
(C) X-ray and elevation of limb in posterior slab
(D) Fully supinate the forearm.

Answer: (D)

411. Excision of head of radius in a child should not be done because
(A) It produces instability of elbow joint
(B) It leads to secondary Osteo arthritis of elbow
(C) It causes subluxation of inferior radio-ulnar joint
(D) It cause myositis ossificans.

Answer: (C)

412. If head of the radius is removed, it will result in
(A) Lengthening of limb
(B) Valgus deformity
(C) Varus deformity
(D) No deformity

Answer: (B)

413. OR is not required in which fracture:-
(A) Patella
(B) Outer 1/3 of radius
(C) Condyle of humerus
(D) Olecranon displaced #

Answer: (B)

414. An oblique # of olecranon. If displaced proximally. The treatment is
(A) Excision &resuturing
(B) Tension band wiring
(C) Elbow is imbolised by cast
(D) Open reduction & external fixation

Answer: (B)

415. In fracture of the olecranon, excision of the proximal fragment is indicated in all of the following situations except:
(A) Old ununited fractures
(B) Non-articular fractures
(C) Fracture extending to coronoid process
(D) Elderly patient

Answer: (C)

416. Moneteggia fracture is fracture of
(A) Lower 1/3rd of radius
(B) Upper 1/3rd of radius
(C) Lower 1/3rd ulna
(D) Upper 1/3rd of Ulna

Answer: (D)

417. In Monteggia fracture, which is true about ulnar fracture and head of radius
(A) Both ulnar fracture and head of radius is displaced posteriorly
(B) Both ulnar fracture and head of radius is displaced anteriorly
(C) Ulnar fractures is posteriorly and head of radius is displaced anteriorly
(D) Ulnar fracture is anteriorly and head of radius is displaced posteriorly

Answer: (B)

418. Posterior interosseous nerve is injured in
(A) Posterior dislocation of elbow
(B) Monteggia fracture dislocation
(C) Reversed monteggia fracture dislocation
(D) Supracondylar fracture of humerus

Answer: (B)

419. Galeazzi fracture is fracture of:
(A) Upper end of ulna
(B) Lower end of ulna
(C) Upper end of radius
(D) Lower end of radius

Answer: (D)

420. Galeazzi fracture is
(A) Supracondylar fracture of the humerus
(B) Fracture of the distal radius with inferior radio ulnar joint dislocation
(C) Fracture of radius in the proximal site and dislocation of the elbow
(D) Fracture of the radial head

Answer: (B)

421. The basic principle in the treatment of fractures of both bones of the forearm is to
(A) Reduce angulation of radius and ulna
(B) Restore the normal relationship of radius and ulna
(C) Immobilize the elbow only
(D) Prevent over riding of fragments
(E) All of the above

Answer: (A, B)

422. The treatment of choice of fracture of radius and ulna in a an adult is:
(A) Plaster for 4 weeks
(B) Closed reduction and calipers
(C) Only plates
(D) Kuntscher nails

Answer: (C)

423. Fracture of both bones forearm at same level, position of the arm in plaster is
(A) Full supination
(B) 10 degree supination
(C) Full pronation
(D) Mid-prone

Answer: (D)

424. A colles fracture is
(A) Common in adolescence
(B) A fracture about the ankle joint
(C) Common in elderly women
(D) A fracture of head of the radius

Answer: (C)

425. Deformities present in colles fracture is/are:
(A) Ventral tilt
(B) Dorsalttilt
(C) Ventral displacement
(D) Dorsal displacement
(E) Shortening

Answer: (B, D, E)

426. Following displacement seen in Colle’s fracture EXCEPT:
(A) Dorsal tilt
(B) Ventral tilt
(C) Dorsal displacement
(D) Lateral displacement

Answer: (B)

427. Seen in Colles # A/E:
(A) Proximal shift
(B) Dorsal tilt
(C) Lateral tilt
(D) Pronation

Answer: (D)

428. About displacement of distal fragment in colles fracture, true is:
(A) Anteriorly and medially
(B) Posteriorly and laterally
(C) Anteriorly and laterally
(D) Posteriorly and medially

Answer: (B)

429. ‘Dinner fork deformity’ is present in case of
(A) Smith’s fracture
(B) Student’s elbow
(C) Colles’ fracture
(D) All of these

Answer: (C)

430. In colles fracture not seen is:
(A) Proximal impaction
(B) Lateral rotation
(C) Dorsal angulation
(D) Medial rotation

Answer: (D)

431. Position of wrist in cast of colle’s fracture is:
(A) Palmar deviation & pronation
(B) Palmar deviation & supination
(C) Dorsal deviation & pronation
(D) Dorsal deviation & supination

Answer: (A)

432. Most common complication of Colles # :
(A) Malunion
(B) Avascular necorsis
(C) Finger stiffness
(D) Rupture of EPL tendon

Answer: (C)

433. Most common complication of colle’s fracture is:
(A) Stiffness of fingers
(B) Sudeck’s dystrophy
(C) Nonunion
(D) Tendon rupture

Answer: (A)

434. The commonest complication of Colle’s fracture is:
(A) Malunion
(B) Non-union
(C) Sudeck’sosteodystrophy
(D) Stiffness of fingers

Answer: (D)

435. Commonest complication of Colles’ fracture is:
(A) Nonunion
(B) Malunion
(C) Vascular injury
(D) Sudeck’sosteodystrophy

Answer: (B)

436. All of the following can be complications of a malunitedColles fracture except:
(A) Rupture of flexor policies longus tendon
(B) Reflex sympathetic dystrophy (RSD)
(C) Carpal tunnel syndrome
(D) Carpal instability

Answer: (A)

437. Rupture of extensor pollicislongus occurs four weeks after:
(A) Colle’s fracture
(B) Radial styloid fracture
(C) Smith’s fracture
(D) Scaphoid fracture
(E) All of these

Answer: (A)

438. Which tendon gets involved in Colle’s fracture?
(A) Abductor pollicislongus
(B) Extensor pllicisbrevis
(C) Extensor pollicislongus
(D) All the above

Answer: (C)

439. Complications of Colle’s fracture include all of the following except:
(A) Malunion
(B) Nonunion
(C) Sudeck’sosteodystrophy
(D) Rupture of EPL tendon

Answer: (B)

440. Complications of Colles’ fracture are all except
(A) Malunion
(B) Non union
(C) Sudeck’s dystrophy
(D) Rupture of extensor policislongus

Answer: (B)

441. Not a complication of Colles’ fracture
(A) Stiffness of wrist
(B) Stiffness of shoulder
(C) Carpal tunnel syndrome
(D) Wrist drop

Answer: (D)

442. The complication of Colles’ fracture is
(A) Radial nerve palsy
(B) Stiffness of wrist joint
(C) Ulnar nerve palsy
(D) None of the above

Answer: (B)

443. Complications of fracture (#) radius are:
(A) Volmann ischemic contracture
(B) Myositis ossificans
(C) Infection
(D) Tendon rupture
(E) Angiodysplasia

Answer: (A, B, C, D)

444. A 50 year old lady sprained her ankle2 months back from which she made a steady recover. 2 months after the injury she gradually developed severe pain in her right ankle with significant limitation of ankle movement. Clinical examination reveals edema and shiny skin. What is the likely diagnosis?
(A) Fibromyalgia
(B) Complex Regional pain syndrome Type I (CRPS I)
(C) Complex Regional pain syndrome Type II (CRPS II)
(D) Peripheral Neuropathy

Answer: (B)

445. A lady presents with swelling of hand with shiny skin. She has a history of fracture radius and kept on POP cast for 4 weeks after which she develops this. Give the most likely diagnosis
(A) Myositis ossification
(B) Rupture of external pollicslongus tendon
(C) Reflex sympathetic dystrophy
(D) Malunion

Answer: (C)

446. Sudeck’s atrophy is associated with:
(A) Osteoporosis
(B) Osteophyte formation
(C) Osteopenia
(D) Osteochondritis

Answer: (C)

447. Sudeck’s atrophy is more common in:
(A) MalunitedColles’ fracture
(B) Malunited fracture femur
(C) Pott’s fracture
(D) Carries spine

Answer: (A)

448. Stellate ganglion block is useful in:
(A) Sudeckosteodystrophy
(B) Compound palmar ganglion
(C) Tenosynovitis
(D) Osteoarthritis of first CMC joint

Answer: (A)

449. Regarding sudeck’sosteosdystrophy all are true except
(A) Burning pain
(B) Stiffness & swelling
(C) Erythematous & cyanotic discolouration
(D) Self limiting& good prognosis

Answer: (D)

450. Management of Smith’s fracture is
(A) Open reduction and fixation
(B) Plaster cast with forearm in pronation
(C) Closed reduction with below-elbow cast
(D) Above-elbow cast with forearm in supination

Answer: (D)

451. Barton’s fracture of the wrist
(A) Involves radio carpal subluxation
(B) Is a severe form of a colles’ fracture
(C) Is often treated by open reduction and internal fixation
(D) All of the above

Answer: (A)

452. All are injuries of lower end of radius except
(A) Smith’s fracture
(B) Colle’s fracture
(C) Night stick fracture
(D) Barton’s fracture

Answer: (C)

453. Which one of the following statements is not correct regarding fracture of the scaphoid
(A) It is the most commonly fractured carpal bone
(B) Persistent tenderness in the anatomical snuffbox is highly suggestive of fracture
(C) Immediate X-ray of hand may not reveal fracture line
(D) Mal union is a frequent complication

Answer: (D)

454. Most common site of scaphoid fracture is
(A) Waist
(B) Proximal fragment
(C) Distal fragment
(D) Tilting of the lunate

Answer: (A)

455. Main risk in fracture Scaphoid is
(A) Non union
(B) Malunion
(C) Delayed union
(D) Avascular necrosis
(E) Carpal tunnel syndrome

Answer: (D)

456. Complication of fracture scaphoid is
(A) Avascular necrosis of distal part
(B) Injury to radial artery
(C) Injury to radial nerve
(D) Avascular necrosis of proximal part

Answer: (D)

457. The proximal fragment of scaphoid after fracture is predisposed for Avascular Necrosis because:
(A) Retrograde blood flow to the proximal fragment
(B) Difficulty in immobilizing the proximal fragment
(C) Fracture configuration of the proximal fragment is usually comminuted
(D) Proximal Fragment articulates with the radius

Answer: (A)

458. Avascular necrosis of bone is most commonly seen in
(A) Calcaneus
(B) Cervical spine
(C) Scaphoid
(D) Scapula

Answer: (C)

459. The best radiological view for fracture scaphoid is:
(A) AP
(B) PA
(C) Lateral
(D) Oblique

Answer: (D)

460. Avascular necrosis of bone is most common in
(A) Scapula
(B) Scaphoid
(C) Calcaneus
(D) Cervical spine

Answer: (B)

461. Oblique view is required to diagnose fracture of
(A) Capitate
(B) Scaphoid
(C) Navicular
(D) Hamate

Answer: (B)

462. Carpal bone which fracture commonly:
(A) Scaphoid
(B) Lunate
(C) Hammate
(D) Pisciform

Answer: (A)

463. A patient reported with a history of fall on an outstretched hand, complains of pain in the anatomical snuffbox and clinically no deformities visible. The diagnosis is:
(A) Colles’ fracture
(B) Lunate dislocation
(C) Barton’s fracture
(D) Scaphoid fracture

Answer: (D)

464. If an adolescent boy falls on a out-stretched hand, the most common bone to be injured is
(A) Fracture of lower end of radius
(B) Fracture of both bones of forearm
(C) Scaphoid fracture
(D) Supracondylar fracture of humerus

Answer: (C)

465. In Scaphoid fracture, important views are all except:
(A) AP
(B) Lateral
(C) Oblique
(D) Cone

Answer: (D)

466. Which of the following Scaphoid fracture is most prone to develop Avanscular necrosis?
(A) Fracture of waist of scaphoid
(B) Fracture of tubercle
(C) Fracture of distal pole
(D) All of the above

Answer: (A)

467. Fracture scaphoid is usually seen in:
(A) Elderly male
(B) Elderly postmenopausal female
(C) Young active adult
(D) Children

Answer: (C)

468. In children fracture scaphoid is through rare but usually involves:
(A) Waist
(B) Proximal pole
(C) Neck
(D) Distal pole

Answer: (D)

469. 19 year old boy had a history of fall on out stretched hand while playing. He developed slight radial side pain and tenderness. On examination pressure along the axis of thumb is painful and x rays are normal.
(A) Fracture lower end radius
(B) Wrist sprain
(C) Scaphoid fracture
(D) Perilunate dislocation

Answer: (C)

470. In nonunion of scaphoid vascularized muscle pedicle graft is taken from.
(A) Pronator teris
(B) Brachioradialis
(C) Pronator quadratus
(D) Extensor pollicis longus

Answer: (C)

471. Glass holding cast is also known as:
(A) Coll’s cast
(B) Smit’s cast
(C) Sarmento cast
(D) Scaphoid cast

Answer: (D)

472. In classical scaphoid cast position of wrist is:
(A) Dorsal & ulnar flexion
(B) Dorsal & radial flexion
(C) Ventral & ulnar flexion
(D) Ventral & radial flexion

Answer: (B)

473. Following are the common sites of Avascular necrosis, EXCEPT:
(A) Proximal half of scaphoid
(B) The body of talus
(C) Patella
(D) Head of the femur

Answer: (C)

474. Which of the following statement(s) is/are true?
(A) Oedema & tenderness over the anatomical snuff box is the pathognomonic features of Fracture of the scaphoid
(B) Normally the radial styloid is 1/2” lower than the ulnar
(C) Dinner fork deformity is characteristic of Colle’s fracture
(D) All of the above

Answer: (D)

475. Internal fixation is probably needed in all of the following except
(A) Fracture condyle of humerus
(B) Fracture neck of femur
(C) Fracture of Olecranon
(D) Fracture of scaphoid

Answer: (D)

476. The most common nerve involvement is dislocation of Lunate is
(A) Median nerve
(B) Anterior interosseus
(C) Posterior interosseus
(D) Median nerve

Answer: (A)

477. Which carpal bone fracture causes median nerve involvement?
(A) Scaphoid
(B) Lunate
(C) Trapezium
(D) Trapezoid

Answer: (B)

478. The commonly inured carpal bone next to scaphoid is
(A) Trapeium
(B) Trapezoid
(C) Lunate
(D) Capitate
(E) Hamate

Answer: (A)

479. Bennett’s fracture is dislocation of base of _____ metacarpal:
(A) 4th
(B) 3rd
(C) 2nd
(D) 1st

Answer: (D)

480. The term Bennett’s fracture is used to describe
(A) Fracture-dislocation of metacarpophalangeal joint of thumb
(B) Interphalangeal fracture dislocation of thumb
(C) Anterior marginal fracture of distal end of radius
(D) Fracture dislocation of trapezometacarpal joint

Answer: (D)

481. A Bennett’s fracture is difficult to maintain in a reduced position mainly because of the pull of the
(A) Flexor pollicislongus
(B) Flexor pollicisbrevis
(C) Extensor pollicisbrevis
(D) Abductor pollicislongus

Answer: (D)

482. A Bennet’s fracture is difficult to maintain in reduced position because of the pull of
(A) Extensor pollicislongus
(B) Extensor pollicisbrevis
(C) Abductor pollicislongus
(D) Abductor pollicisbrevis

Answer: (C)

483. During fixation of Bennett’s fracture, which muscle hinder it
(A) Extensor pollicisbrevis
(B) Flexor pollicisbrevis
(C) Flexor pollicislongus
(D) Abductor pollicislongus

Answer: (D)

484. A cricketer got injured while holding a catch, following which he complained of pain over the base of thumb. Which structure is most likely to be injured.
(A) Volar plate
(B) Extensor pollicislongus
(C) Abductor pillicislongus
(D) Ulnar collateral ligament

Answer: (D)

485. Game Keeper’s thumb is
(A) Ulnar collateral
(B) Radial collateral ligament injury of MCP joint
(C) Radial collateral ligament injury of CMC joint
(D) Ulnar collateral ligament injury of CMC joint

Answer: (A)

486. Mallet finger is
(A) Avulsion fracture of extensor tendon of distal phalanx
(B) Fracture of distal phalanx
(C) Fracture of middle phalanx
(D) Fracture of proximal phalanx

Answer: (A)

487. Avulsion of extensor tendon gives rise to
(A) Mallet finger
(B) Dupuytren’s contracture
(C) Trigger finger
(D) Swan neck deformity

Answer: (A)

488. In hand surgery which area is called no man’s land
(A) Proximal phalanx
(B) Distal phalanx
(C) Between distal phalanx crease and proximal phalanx
(D) Wrist

Answer: (C)

489. Middle palmar space ends distally
(A) Along the digital sheaths
(B) Into the flexor tendon sheaths
(C) Into the web space
(D) By mixing with the superficial palmer space

Answer: (C)

490. In pelvis fracture, the amount of blood loss is around
(A) 1-4 units
(B) 2-4 units
(C) 2-6 units
(D) 4-8 units

Answer: (D)

491. If a patient with a suspected fracture of the pelvis has some bleeding from the urethra and is unable to pass urine
(A) He should be encouraged to pass urine after being given antibiotics and analgesics
(B) He should be immediately catheterized in the ward
(C) A hot water bottle should be given followed by injection of carbachol
(D) He should be prepared for surgery and catheterization attempted in the O.T.

Answer: (D)

492. Jumper’s fracture is seen in:
(A) Calcaneum
(B) Tibia
(C) Pelvis
(D) Neck femur

Answer: (C)

493. Open book and bucket handle injuries are seen in:
(A) Spine
(B) Pelvis
(C) Femur
(D) Humerus

Answer: (B)

494. True about Crescent fracture is:
(A) Anteroposterior instability with rotational stability
(B) Diastasis of pubis with pubic rami fracture
(C) Antero-posterior compression is the mechanism of injury
(D) Fracture of the iliac bone with sacroiliac disruption

Answer: (D)

495. Late complication of Acetabular fracture
(A) Avascular necrosis of head of femur
(B) Avascular necrosis of lilac crest
(C) Fixed deformity of the hip joint
(D) Secondary osteoarthritis of hip joint

Answer: (D)

496. All of the following areas are commonly involved sites in pelvic fracture except:
(A) Pubic rami.
(B) Alae of ileum.
(C) Acetabula
(D) Ischialtuberosities.

Answer: (D)

497. Kocher Langenbeck approach for emergency acetabular fixation is done in all except:
(A) Open fracture
(B) Progressive sciatic nerve injury
(C) Recurrent dislocation inspite of closed reduction and traction
(D) Morel-Lavallee lesion

Answer: (D)

498. Emergency treatment of acetabular # area ll except:
(A) Recurrent dislocations despite fixation with traction
(B) Open acetabular fracture
(C) Progressive aciatic nerve involvement
(D) Morel-Lavallee lesion

Answer: (D)

499. Which is not true about Langenbeckkocher operation?
(A) Adequate exposal of posterior segment
(B) Anterior segment is not Visualized adequately
(C) Superior exposure is very adequate
(D) Sciatic nerve injury in 10 percent in the cases

Answer: (C)

500. Main blood supply to the head and neck of femur comes from
(A) Lateral circumflex femoral Artery
(B) Medial circumflex femoral Artery
(C) Artery of LigamentumTeres
(D) Popliteal Artery

Answer: (B)

501. An elderly woman was admitted with a fracture of the neck of right femur which failed to unite. On examination an avascular necrosis of the head of femur was noted. The condition would have resulted most probably from the damage to:
(A) Superior gluteal artery
(B) Inferior gluteal artery
(C) Acetabular branch of obturator
(D) Retinacular branches of circumflex femoral arteries

Answer: (D)

502. Pauvel’s angle is
(A) Neck shaft angle of femur
(B) The difference between neck shaft angle between two femurs of a patient
(C) Formed by joining a line extended from fracture line of femur neck to an arbitrary line depiciting the horizontal plane
(D) None of the above

Answer: (C)

503. Garden’s classification is applicable to
(A) Intertrochanteric fracture
(B) Fracture neck of femur
(C) Epiphyseal separation
(D) Posterior dislocation of hip

Answer: (B)

504. Garden – I fractures are also known as
(A) Complete fracture with out displacement
(B) Complete fracture with minimal (partial) displacement
(C) Complete fracture with full displacement
(D) Valgus impaction fractures

Answer: (D)

505. In fracture neck femur all the trabeculae of pelvis and femur are in alignment in which stage:
(A) Stage I
(B) Stage II
(C) Stage III
(D) Stage IV

Answer: (B)

506. Increase in Pauwel’s angle indicate
(A) Good prognosis
(B) Impaction
(C) More chances of displacement
(D) Trabecular alignment disrupted

Answer: (C)

507. A 60 year old female lands up in emergency with history of fall trauma, the attitude of limb is extension and external rotation, the most probable diagnosis is
(A) Intra capsular # neck of femur
(B) Posterior dislocation of hip
(C) Intutrochanteric #
(D) Acetabulam #

Answer: (A)

508. A woman aged 60 years suffers a fall; her lower limb is extended and externally rotated; likely diagnosis is:
(A) Neck of femur #
(B) Intertrochanteric femur #
(C) Postr dislocation of hip
(D) Anterior dislocation of hip

Answer: (A)

509. 65-year-old lady falls from height. On examination the leg is extended and externally rotated. Diagnosis is
(A) Fracture of acetabulum
(B) Intertrochanteric fracture
(C) Neck femur fracture
(D) Posterior dislocation of hip

Answer: (C)

510. The commonest hip injury in the elderly patients is:
(A) Stress #
(B) Extracapsular #
(C) Impacted # neck of femur
(D) Sub capital capsular fracture neck of femur

Answer: (B)

511. Concerning fractures of the neck of the femur which statement is considered now to be incorrect
(A) It is common in elderly women
(B) It can occur in young adults due to fatigue
(C) It can occur in young adults following severe violence applied in the long axis of the femur
(D) The bone fractures in an elderly woman because the falls.

Answer: (C)

512. 80 years old female after fall developed inability to walk with external rotation deformity on examination SLR is not possible and broadening of trochanter is present. The possible diagnosis is:
(A) # Neck femur
(B) # Inter trochanteric femur
(C) # Subtrochanteric femur
(D) # Greater trochanter

Answer: (B)

513. A 60-year-old man fell in bathroom and was unable to stand on right buttock region ecchymosis with external rotation of the led and lateral border of foot touching the bed. The most probable diagnosis is:
(A) Extra capsular fracture neck of femur
(B) Anterior dislocation of hip
(C) Intra capsular fracture neck of femur
(D) Posterior dislocation of hip

Answer: (A)

514. Features of fracture neck of femur includes
(A) Flexion at hip and lateral rotation
(B) Flexion at hip abduction
(C) Shortening and lateral rotation
(D) Shortening and flexion

Answer: (C)

515. Occult fracture of neck femur are best diagnosed by:
(A) Bone scan
(B) MRI
(C) X-Ray
(D) CT scan

Answer: (B)

516. An old lady had a history of fall in bathroom once and couldn’t move. Afterwards, she had led in externally rotated position. There was tenderness in Scarp’s triangle and limb movement could not seen done due to pain. No hip fracture was seen on X-ray. Next step
(A) MRI
(B) Repeat X-ray after one week
(C) Joint aspiration
(D) Give analgesic and manipulate

Answer: (A)

517. Nonunion is a very common complication of intracapsular fractures of the neck of femur. Which of the following is not a very important cause for the same?
(A) Inadequate immobilization
(B) Inadequate blood supply
(C) Inhibitory effect of synovial fluid
(D) Stress at fracture site due to muscle spasm

Answer: (D)

518. All of the following can occure as complication of fracture of the neck of femur except:
(A) Shortening
(B) Non-union
(C) Mal-union
(D) Avascular necrosis

Answer: (C)

519. Commonest complication of extra capsular intertrochanteric fracture of neck of femur is:
(A) Non union
(B) Ischemic necrosis
(C) Malunion
(D) Pulmonary complications

Answer: (C)

520. In Which one of the following femoral fractures is Avascular necrosis common?
(A) Pertrochanteric
(B) Transcervical
(C) Sub-Trochanteric
(D) Shaft of femur

Answer: (B)

521. Avascular necrosis of head of femur occurs commonly at:
(A) Transcervical region
(B) Trochanteric region
(C) Subcapital region
(D) Subchondral region

Answer: (C)

522. The most common complication of intracapsular fracture neck of femur is
(A) Mal union
(B) Osteoarthritis
(C) Non-Union
(D) Shortening

Answer: (C)

523. Commonest complication of Trans-cervical fracture of femur is
(A) Non union
(B) Malunion
(C) Avascular necrosis
(D) All of the above

Answer: (C)

524. AVN is seen in which type of # of femur
(A) Interochantric #
(B) Subcapital #
(C) Trans cervical #
(D) Basal #

Answer: (B, C, D)

525. The most common site of fracture neck of femur that causes avascular necrosis is
(A) Sub-capital
(B) Intertrochanteric
(C) Trans-cervical
(D) Basal

Answer: (A)

526. Best treatment for fracture neck femur in a 65 year old lady is
(A) POP cast
(B) Gleotomy
(C) Bone grafting and compression
(D) Hemireplacementarthroplasty

Answer: (D)

527. 65 year old man presented with fracture neck femur 3 days after injury, treatment of choice is:
(A) Multiple screw fixation
(B) Mc-Murray osteotomy
(C) Hemi-arthroplasty
(D) Total hip replacement

Answer: (C)

528. A 50 years male with fracture neck of femur comes after 3 days, treatment of choice is
(A) hemiarthroplasty
(B) total hip replacement
(C) hipspika
(D) CR & IF

Answer: (D)

529. Treatment of choice in fracture neck of femur in a 40-year old male presenting after 2 days is
(A) Hemiarthroplasty
(B) Closed reduction and Internal fixation by cancellous screws
(C) Closed reduction and Internal fixation by Austin Moore pins
(D) Plaster and rest

Answer: (B)

530. Treatment of choice for one week old fracture neck femur at 65 years age is:
(A) Hemi-replacement arthroplasty
(B) Closed reduction and internal fixation by cannulatedcancellous screws
(C) Closed reduction and internal fixation by Austin more pins
(D) Total hip replacement

Answer: (B)

531. In the case of 65 year old person with fracture neck of femur the treatment of choice is
(A) Closed reduction
(B) Closed reduction with internal fixation
(C) Open reduction
(D) Replacement of head and neck of the femur with a prosthesis

Answer: (B)

532. In 65 year old male with history of fracture neck of femur 6 weeks old, treatment of choice
(A) SP nailing
(B) Mc. Murray’s osteotomy
(C) Hemiarthroplasty
(D) None

Answer: (C)

533. The treatment of choice of a 4 weeks old Femoral neck fracture in a 55 years old man is
(A) Open reduction and internal fixation
(B) Mc Murray’s Osteotomy
(C) Hemi replacement arthroplasty
(D) Total hip replacement

Answer: (C)

534. In the case of a 70 year old lady with intra capsular fracture of the neck of femur, the ideal treatment would be
(A) Closed traction
(B) Hemiarthropalsty
(C) Internal fixation with nail
(D) Internal fixation with nail and plate

Answer: (B)

535. 3 days old intracapsular fracture neck of femur in a 50 year old patient is treated by
(A) POP hip spika
(B) Total hip replacement
(C) Cortical screw fixation
(D) Hemi replacement arthroplasty

Answer: (D)

536. Femoral neck fracture of three weeks old in an young adult should be best treated one of the following:
(A) Total hip replacement
(B) Reduction of fracture and femoral osteotomy with head
(C) Prosthetic replacement of femoral head
(D) Reduction of fracture and multiple pin or screw fixation
(E) Upper femoral displacement osteotomy

Answer: (D)

537. A 65 year old patient who presents 10 days after fracture neck femur is best managed by
(A) Internal fixation
(B) Replacement arthroplasty
(C) Mc Murray’s procedure
(D) Traction for 6 weeks

Answer: (A)

538. Prosthetic replacement of femoral head is usually indicated for
(A) Fresh intracapsular fracture head of femur in old patients
(B) Fresh intracapsular fracture of femoral neck in a young adult
(C) Unreduced posterior dislocation of hip
(D) Untreated femoral neck fracture in a patient over 65 years
(E) Pathological femoral neck fracture due to secondaries

Answer: (D)

539. Treatment of choice for a 4 week old femoral neck fracture in a 55yer old male is
(A) Open reduction and internal fixation
(B) Mac Murray’s osteotomy
(C) Hemi replacement arthroplasty
(D) Total hip replacement

Answer: (C)

540. Prosthesis at head of femur applied in:
(A) 40 years young male with # head of femur
(B) 40 year young male with # neck of femur
(C) 40 year young male with posterior dislocation of hip
(D) 65 year old male with non united fracture neck of femur

Answer: (D)

541. Fractured neck of femur is associated with all except
(A) Causes shortening of the leg
(B) Causes internal rotation of the leg
(C) May be pathological
(D) May be treated with hemiarthroplasty

Answer: (B)

542. Which of the following fractures would best be treated by Open reduction?
(A) Fracture of the femoral shaft of the child
(B) Collies’ fracture
(C) Displaced fracture of the femoral neck
(D) Fracture of humeral shaft

Answer: (C)

543. Trochanteric fracture of femur is best treated by
(A) Dynamic hip screw
(B) Inlay plates
(C) Plaster in abduction
(D) Plaster in abduction and internal rotation

Answer: (A)

544. Prosthetic replacement of femoral head is indicated for one of the following sites of fractures
(A) Inter-trochanteric fracture of femoral neck
(B) Subcapital fracture neck femur
(C) Transtrochanteric fracture femur
(D) Basal fracture of femoral neck

Answer: (B)

545. Mc Murray’s osteotomy is based on the following principle
(A) Biological
(B) Bio mechanical
(C) Bio technical
(D) Mechanical

Answer: (B)

546. Meyer’s procedure is a method for treatment of
(A) Recurrent shoulder dislocation
(B) Habitual dislocation of patella
(C) Congenital dislocation of hip
(D) Fracture neck of femur

Answer: (D)

547. A fracture neck femur in a child is best treated by
(A) Spica in abduction
(B) Spica in abduction + internal rotation
(C) Masterly inactivity
(D) Open reduction and internal fixation

Answer: (B)

548. In a 10 year old male transcervical fracture neck femur is best treated by:
(A) Spica
(B) Austin Moore pins
(C) K-Wires
(D) C.C.screw

Answer: (D)

549. Commonest dislocation of the hip is
(A) Posterior
(B) Anterior
(C) Central
(D) None

Answer: (A)

550. Which is true about dislocation of hip joint?
(A) Posterior dislocation is commoner
(B) In posterior dislocation whole lower limb is rotated medially
(C) In anterior dislocation whole lower limb is rotated laterally
(D) All of the above

Answer: (D)

551. Flexion, adduction and internal rotation is characteristic posture in
(A) Anterior dislocation of hip joint
(B) Posterior dislocation of hip joint
(C) Fracture of femral head
(D) Fracture shaft of femur

Answer: (B)

552. The attitude of limb in traumatic dislocation of hip joint is
(A) Flexion, adduction, external rotation
(B) Flexion, adduction, internal rotation
(C) Flexion, adduction, and external rotation
(D) Flexion and adduction only

Answer: (B)

553. A patient with hip in adduction and medial rotation and is unable to move probable diagnosis is
(A) Posterior dislocation head of femur
(B) Fracture shaft of femur
(C) Fracture neck of femur
(D) Sciatica

Answer: (A)

554. The pain around the hip with flexion, adduction & internal rotation of lower limb in a young adult after road traffic accident is suggestive of:
(A) Intracapsular fracture of the femoral neck
(B) Extra capsular fracture of the femoral neck
(C) Posterior dislocation of hip
(D) Anterior dislocation of hip

Answer: (C)

555. Traumatic dislocation of hip is characterized by
(A) Adduction internal rotation deformity
(B) Abduction external rotation deformity
(C) Adduction external rotation deformity
(D) Abduction internal rotation deformity

Answer: (A)

556. Dashboard injury results in
(A) Anterior dislocation of hip
(B) Posterior dislocation of hip
(C) Central dislocation of hip
(D) Fracture neck femur

Answer: (B)

557. Vascular sign of Narath is noticed in
(A) Fracture neck of femur
(B) Perthes disease
(C) Posterior dislocation of hip
(D) All of the above

Answer: (C)

558. A 20 year old male, following a road traffic accident was brought to the casualty. His right leg is shortened, internally roated and adducted. The diagnosis is
(A) Fracture neck of femur
(B) Anterior dislocation of the hip
(C) Posterior dislocation of the hip
(D) Trochanteric fracture of the femur

Answer: (C)

559. Post dislocation of hip leas to which attitude of the lower leg:
(A) Adduction, external rotation and flexion
(B) Adduction, internal rotation and flexion
(C) Abduction, external rotation and flexion
(D) Abduction, internal rotation and flexion

Answer: (B)

560. Deformity of posterior dislocation of hip
(A) Flexion, adduction, Internal rotation
(B) Flexion, abduction, external rotation
(C) Extension abduction, Internal rotation
(D) Extension adduction, external rotation

Answer: (A)

561. Maximum shortening of limbs occur in:
(A) Trochantric # femur
(B) Post dislocation of hip
(C) # neck femur
(D) Anterior dislocation of hip

Answer: (B)

562. A 30-year old male was brought to the casually following a road traffic accident. His physical examination revealed that his right lower limb was short, internally rotated, and fixed and adducted at the hip. The most likely diagnosis is:
(A) Fracture neck of femur
(B) Trochanteric femur
(C) Central fracture dislocation of hip
(D) Posterior dislocation of hip

Answer: (D)

563. Sciatic nerve palsy may occur in the following injury
(A) Posterior dislocation of hip joint
(B) Fracture neck of femur
(C) Trochanteric fracture
(D) Anterior dislocation of hip

Answer: (A)

564. Kumar, a 31 yrs old motorcyclist sustained injury over his Right hip join. X-ray revealed a posterior dislocation of the Right hip joint. The clinical altitude of the affected lower limb will be:
(A) External rotation, extension & abduction
(B) Internal rotation, flexion & adduction
(C) Internal rotation, extension & abduction

(D) External rotation, flexion & abduction


Answer: (B)

565. Pipkin fracture is defined as:
(A) Head of radius fracture
(B) Head of femur fracture
(C) Fracture dislocation of ankle
(D) Fracture neck of femur
(E) Fracture of C6 Spine

Answer: (B)

566. In traumatic injury of the hip:
(A) Anterior dislocation is common
(B) Posterior dislocation is common
(C) Visceral injury usually associated with fracture femur
(D) Open reduction is always necessary

Answer: (B)

567. Following is true in treatment of post, dislocation:
(A) Closed reduction under anaesthesia
(B) Open reduction
(C) Skeletal traction
(D) Soft tissue

Answer: (A)

568. In anterior dislocation of hip, the posture of lower limb will be
(A) Abduction, externally rotated and extension
(B) Abduction, externally rotated and flexion
(C) Abducted externally rotated and flexion
(D) Adducted, internally rotated and flexion

Answer: (B)

569. Flexion, abduction and external rotation with limb length discrepancy is seen in:
(A) Posterior dislocation of hip
(B) Central dislocation of hip
(C) Anterior dislocation of hip
(D) Fracture neck of femur (c)

Answer: (C)

570. Flexion, abduction and external rotation at hip joint with limb length discrepancy is seen in
(A) Fracture neck of femur
(B) Anterior dislocation of hip
(C) Posterior dislocation of hip
(D) None

Answer: (B)

571. Deformity in anterior dislocation of hip is
(A) Ext. rotation, abduction, flexion
(B) Ext. rotation, adduction flexion
(C) Int. rotation, abduction flexion
(D) Int. rotation, adduction flexion

Answer: (A)

572. A 32 year old male presented to the casually with pain in the left hip region following RTA. On examination there is Shortening of left lower limb by 7 cm along with an obvious Flexion and External rotation deformity. A mass is palpable in the left gluteal region which moves with movement of the femur. Most likely X-ray finding would be:
(A) Posterior dislocation of Hip with Neck in full profile
(B) Dislocation of hip with lesser trochanter n full profile
(C) Fracture roof of acetabulum with central dislocation
(D) Acetabular fracture with posterior dislocation of hip

Answer: (B)

573. In per rectal examination, femoral head is palpable in
(A) Anterior dislocation of hip
(B) Posterior dislocation of hip
(C) Central dislocation of hip
(D) Lateral dislocation of hip

Answer: (C)

574. Line joining ant. sup iliac spine to ischial tuberosity and passes a greater trochanter
(A) Nelaton’s line
(B) Showmakers line
(C) Cniene’s line
(D) Perkins line

Answer: (A)

575. Trendelenberg test is positive due to injury to:
(A) Sup gluteal nerve
(B) Inf gluteal nerve
(C) Obutrator nerve
(D) Tibial nerve

Answer: (A)

576. “Trendelenburg sign” is positive in damage of the following nerve :
(A) Inferior gluteal nerve
(B) Pudendal nerve
(C) Superior gluteal nerve
(D) Posterior tibial nerve

Answer: (C)

577. Trendelenbrug’s test positive in all EXCEPT:
(A) Posterior dislocation of hip
(B) Poliomyelitis
(C) # Neck of femur
(D) Tuberculosis of hip joint

Answer: (D)

578. Trendelenberg’s sign is –ve in Inter-Trochanteric fracture because of:
(A) Gluteus medius
(B) Gluteus maximus
(C) Gluteus minimus
(D) Tensor fascia lata

Answer: (D)

579. All of the following names are associated with tests/operations around the h I joint except
(A) Bryant
(B) Shenton
(C) Mc. Murray
(D) Salter
(E) Nelton

Answer: (D)

580. Telescopic test is useful to diagnose
(A) Perthe’s disease
(B) Intracapsular fracture neck of femur
(C) Malunited Trochanteric fracture
(D) Ankylosis of hip joint

Answer: (B)

581. Subrochanteric fractures of femur can be treated by all of the following methods except:
(A) Skeletal traction on Thomas’ splint.
(B) Smith Petersen Nail.
(C) Condylar blade plate.
(D) Ender’s nail.

Answer: (B)

582. The femur is fractured at birth at
(A) Upper third of shaft
(B) Middle third of shaft
(C) Lower third of shaft
(D) Neck region

Answer: (A)

583. In upper one third femoral shaft fracture, the displacement of proximal segment is
(A) Flexion abduction and external rotation
(B) Flexion, adduction and external rotation
(C) Flexion, abduction and internal rotation
(D) Flexion, adduction and internal rotation

Answer: (C)

584. Maximum shortening of lower limb is seen in:
(A) # shaft femur
(B) # Neck femur
(C) # intertochanter
(D) Transcervical #

Answer: (A)

585. Not true about fracture shaft femur in infant is:
(A) Child abuse is commonest cause
(B) Heals rapidly
(C) Traction is all that usually required
(D) Fat embolism is common
(E) Sclera must be examined

Answer: (D)

586. Fracture shaft of femur in children of less than 2 years old is treated by:
(A) Open reduction
(B) External fixation
(C) Gallow’s traction
(D) Closed reduction

Answer: (C)

587. A 3 year old child presents with fracture of femoral shaft had immobilized on traction constantly for two months. The next step of management is:
(A) Hip Spica and if necessary internal fixation
(B) Gallow traction for 2 months
(C) Open reduction and Kuntscher’slover leaf intramedullary nailing or plating
(D) Traction by Thomas splint

Answer: (A)

588. Fracture femur is children is treated by:
(A) Open reduction
(B) Gallow’s splint
(C) Intra medullary nailing
(D) Closed reduction &splintage

Answer: (B)

589. A 4 year old female brought to casualty department with multiple fracture ribs, and inconspicuous history from parents. On examination show multiple bruise and healed fractures. The provable diagnosis is:
(A) Polytrauma for evaluation
(B) Flail chest
(C) Munchausen’ syndrome
(D) Battered baby syndrome

Answer: (D)

590. Fracture of femur at the level of isthmus is best treated by
(A) Intramedullary nail fixation
(B) Plate and screws
(C) Closed method
(D) External fixation

Answer: (A)

591. Intramedullary fixation is ideal in a case of fracture of shaft of femur when there is
(A) A transverse fracture
(B) A compound fracture
(C) Soft tissue interposition between the fractured ends
(D) Such a fracture in a child

Answer: (A)

592. Treatment of choice for old non-united fracture of shaft of femur
(A) Compression plating
(B) Bone grafting
(C) Nailing
(D) Compression plating with bone grafting

Answer: (D)

593. Treatment of Non-union of # shaft femur
(A) Open reduction with external fixation
(B) Excision of the bone
(C) Bone grafting with internal fixation with K-Nail
(D) All of the following.

Answer: (C)

594. Best treatment of 3 weeks old, fracture shaft femur with nonunion is
(A) Bone graft with internal fixation
(B) External fixation
(C) Internal fixation only
(D) Prosthesis

Answer: (A)

595. Fracture shaft of femur in adult unites by
(A) 3 to 4 weeks
(B) 3 to 5 weeks
(C) 3 to 4 months
(D) 3to 6 months
(E) None of these

Answer: (C)

596. Clinical feature of fat embolism includes all except:
(A) Tachypnoea
(B) Systemic hypoxia may occur
(C) Fat globules in urine are diagnostic
(D) Manifests after several days of trauma
(E) Petechaie in the anterior chest wall

Answer: (ALL)

597. True about Post traumatic fat embolism syndrome:
(A) Fracture mobility is a risk factor
(B) Associated diabetes pose a risk
(C) Bradycardia occurs
(D) Thrombocytopenia
(E) On ABG PaO2< 60 mm Hg on FIO2< 0.4

Answer: (A, D, E)

598. True about Post traumatic fat embolism syndrome is A/E
(A) Bradycardial occur
(B) Systemic hypoxia may occur
(C) Fracture mobility is a risk factor
(D) Associated diabetes pose a risk
(E) More common in closed fracture
(F) Liposuction is used for treatment

Answer: (D, F)

599. Factors favoring fat embolism in trauma patient:
(A) Diabetes Mellitus
(B) Mobility of joint
(C) Resp. failure
(D) Hypovolemic shock

Answer: (B, D)

600. Commonest sit of fracture leading to fat-embolism is:
(A) Tibia #
(B) Femur #
(C) Humerus #
(D) Ulna #

Answer: (B)

601. A 30 year old man had road traffic accident and sustained fracture of femur. Two days later he developed sudden breathlessness. The most probable cause can be:
(A) Pneumonia.
(B) Congestive heart failure.
(C) Bronchial asthma.
(D) Fat Embolism.

Answer: (D)

602. A 64 year old hypertensive obsese female was undergoing surgery for fracture femur under general anaesthesia. Intra-operatively her end-tidal carbon dioxide decreased to 20 from 40 mm of Hg. followed by hypotension and oxygen saturation of 85%. What could be the most probable cause?
(A) Fat embolism
(B) Hypovolemia
(C) Brochospasm
(D) Myocardial infarction

Answer: (A)

603. Ramesh sing, a 40 yrs man, was admitted with fracture shaft femur following a road traffic accident. On 2n day he became disoriented. He was found to be tachypnoeic, and had conjunctivalpetechiae.
(A) Pulmonary embolism
(B) Sepsis syndrome
(C) Fat embolism
(D) Hemothorex

Answer: (C)

604. The management of fat embolism includes all of the following except:
(A) Oxygen
(B) Heparinization
(C) Low molecular weight dextran
(D) Pulmonary Embolectomy

Answer: (D)

605. Fat embolism syndrome characterized by :
(A) Tachycardia
(B) Hypoxemia
(C) Fat globules in urine
(D) Thrombocytosis

Answer: (A)

606. True about fat embolism
(A) Seen one week after injury
(B) Patechie
(C) Bradycardia
(D) Tachycardia
(E) ↑ Incidence in multiple #

Answer: (B, D, E)

607. A 22 year old male is admitted with fracture of the left femur. Two days later, he becomes mildly confused, has a respiratory rate of 40/min and scattered petechial rash on his upper torso. Chest X-ray shows patchy alveolar opacities bilaterally. His arterial blood gas analysis is abnormal. The most likely diagnosis is
(A) Cerebral oedema with early neurogenic pulmonary oedema
(B) Pulmonary thrombo-embolism
(C) Chest contusion
(D) Fat embolism

Answer: (D)

608. Fat embolism may ensue following
(A) Fracture of spine and ribs
(B) Fractured fibula
(C) Fracture of skull bone
(D) Fracture of long bones only

Answer: (D)

609. Which of the following is true about fat embolism?
(A) Usually ensues after fracture of the lower limb
(B) Uncommon complication of fracture
(C) Spontaneously reversible process
(D) All of the above

Answer: (A)

610. After an operation on femur bone, chest X-ray shows widespread mottling throughout the lung field like a snowstorm. It is diagnostic of
(A) Fat embolism
(B) Shock lung
(C) Bronchopneumonia
(D) Atelectasis

Answer: (A)

611. Fat embolism is characterized by
(A) Petechealhaemorrhages
(B) Closed fractures of femur
(C) Aggregation of chylomicrons
(D) Fall in the haemoglobin
(E) Fat globules in sputum & urine

Answer: (ALL)

612. True about fat embolism:
(A) Petechia in the anterior chest wall
(B) Bradycardia
(C) Fat globules in urine
(D) Occurs after 1st week of poly trauma
(E) Thrmbocytopenia

Answer: (A, C, E)

613. A person with multiple injuries develops fever, restlessness, tachycardia, tachypnea and perimumbilical rash. The likely diagnosis is:
(A) Air embolism
(B) Fat embolism
(C) Pulmonary embolism
(D) Bacterial pneumonitis

Answer: (B)

614. In fracture of femur popliteal artery is commonly damaged by
(A) Proximal fragment
(B) Distal fragment
(C) Muscle haematoma
(D) Tissue swelling

Answer: (B)

615. The classical example of muscular violence is
(A) # of fibula
(B) # of patella
(C) # of clavicle
(D) all of the these

Answer: (B)

616. Transverse fracture of the patella with separation of fragments is best treated by
(A) Closed reduction with cylinder cast
(B) Open reduction with screw fixation of the fragments
(C) Blind fixation of the two fragments with Kirschner wire
(D) Open reduction with Kirschner wire fixation of the fragment an tension band wiring

Answer: (D)

617. In transverse fracture of the patella, the treatment is
(A) Excision of a small fragment
(B) Wire fixation
(C) Plaster cylinder
(D) Patellectomy

Answer: (B)

618. A comminuted fracture of the Patella should be treated by
(A) Inserting screws and wires
(B) Physiotherapy alone
(C) Removal of all the patella
(D) Removal of smallest piece only
(E) Plastering & Immobilization

Answer: (C)

619. Treatment of displaced transverse fracture of patella:
(A) POP
(B) Tension band wiring
(C) Screw
(D) Patellectomy

Answer: (B)

620. Displaced transverse # patella, Rx is:
(A) Patellectomy
(B) Tension band wiring
(C) POP cast
(D) Non operative

Answer: (B)

621. Tube (Cylinder) cast s applied for the fracture of –
(A) Shoulder
(B) Hip
(C) Pelvis
(D) Knee

Answer: (D)

622. What is acceptable angulation after reducing the fracture tibia
(A) 5
(B) 10
(C) 15
(D) 20

Answer: (A)

623. In fracture neck of fibula, the following nerve is involved
(A) Common peroneal nerve
(B) Anterior tibial nerve
(C) Posterior tibial nerve
(D) Medial popliteal nerve

Answer: (A)

624. Healing below knee joint is slow because of
(A) Decreased subcutaneous fat
(B) Increased movement
(C) Weight bearing
(D) Poor vascularity

Answer: (D)

625. Non union is a common feature of fracture of
(A) Supracondylar humerus
(B) Clavicle
(C) Lower tibia
(D) Coracoid process

Answer: (C)

626. Patellar tendon bearing P.O.P cast is indicated in the following fracture:
(A) Patella
(B) Tibia
(C) Medial malleolus
(D) Femur

Answer: (B)

627. Which of the following causes acute compartment syndrome most frequently
(A) Fractures
(B) Postischemic swelling
(C) Exercise initiated syndrome
(D) Soft tissue injury

Answer: (A)

628. Compartment syndrome is treated by
(A) Fasciotomy
(B) Bicarbonate
(C) Chloride rich fluid
(D) Early aggressive fluid

Answer: (A)

629. A patient develops compartment syndrome (Swelling, pain and numbness) following manipulation & plaster for fracture of both bones of leg. What is the best treatment?
(A) Split the plaster
(B) Infusion of low molecular wt dextran
(C) Elevate the leg after splitting the plaster
(D) Do operative decompression of fascial compartment

Answer: (D)

630. Characteristic features of the acute compartment syndrome in the lower leg include all of the following except
(A) Acute pain on employing the stretch test
(B) Normal pulses
(C) Normal sensation distally
(D) Venous occlusion

Answer: (C)

631. In posterior compartment syndrome which passive movement causes pain?
(A) Dorsiflexion of foot
(B) Foot inversion
(C) Toe dorsiflexion
(D) Toe planter flexion

Answer: (C)

632. A patient has 2 months POP cast for tibial fracture left leg. Now he needs mobilization with a single crutch. You will use this crutch on which side:
(A) Left side
(B) Right side
(C) Any side
(D) Both side

Answer: (B)

633. The stability of the ankle joint is maintained by all of the following, except
(A) Plantar cancaneonavicular (spring) ligament
(B) Deltoid ligament
(C) Lateral ligament
(D) Shape of the superior talar articular surface

Answer: (A)

634. The most commonly affected component of the lateral collateral ligament complex in an “ankle sprain” is the:
(A) Middle component
(B) Anterior component
(C) Posterior component
(D) Deeper component

Answer: (B)

635. Ankle sprain is due to:
(A) Rupture of anterior talo-fibular ligament
(B) Rupture of poster talo-fibular ligament
(C) Rupture of deltoid ligament
(D) Rupture of Clacaneo-fibular ligament

Answer: (A)

636. Inversion injury at the ankle can cause all of the following EXCEPT:
(A) Fracture tip of lateral melleolus
(B) Fracture base of the 5th metatarsal
(C) Sprain of Ext. Digitorumbrevis
(D) Fracture of sustentaculamtali.

Answer: (C)

637. The mechanism of injury of transverse fracture of medial malleolus results due to
(A) Abduction injury
(B) Adduction injury
(C) Rotation injury
(D) Direct injury

Answer: (A)

638. True about Eponymous fractures is A/E
(A) A. Montegia # is # of the proximal third of ulna with radial head dislocation
(B) B. Galeazzi # is # of the distal third of the radius with dislocation of the distal radio-ulnar joint
(C) C. Colles # is # at cortico-cancellous junction of the distal-end of radius with dorsal tilt
(D) D. Potts # Trimalleolar ankle #
(E) E. Bennet’s # is Oblique intra-articular # of the base of the 1st metacarpal

Answer: (D)

639. Cotton’s fracture is
(A) Avulsion fracture of C7
(B) Trimaoleolar
(C) Bimalleolar
(D) Burst fracture of atlas
(E) None of these

Answer: (B)

640. Fracture involving both the malleoli is
(A) Cotton’s fracture
(B) Pott’s fracture
(C) Duputren’s fracture
(D) Dupuytren’s fracture

Answer: (B)

641. Pott’s fracture is fracture of
(A) Lower end of tibia
(B) Lower end of tibia + fibula
(C) Lower end of tibia + Calcaneum
(D) Calcaneum + Talus

Answer: (B)

642. Avascular necrosis is a complication of
(A) Fracture of talus
(B) Fracture of medial condyle of femur
(C) Olecranon fracture
(D) Radial head fracture

Answer: (A)

643. Avascular necrosis is a complication of
(A) Distal fracture both bones leg
(B) Distal fracture fibula
(C) Bimalleolar
(D) Fracture talus

Answer: (D)

644. MC comp. of # talus is
(A) Avascular necrosis
(B) Non union
(C) Osteoarthritis of ankle joint
(D) Osteoarthritis of neck of talus
(E) Non union

Answer: (A)

645. Fracture of talus without displacement in x-ray would lead to:
(A) Osteoarthritis of ankle
(B) Osteonecrosis of head of talus
(C) Avascular necrosis of body of talus
(D) Avascular necrosis of neck of talus
(E) Non union

Answer: (A, C, E)

646. In which type of fracture, the tuber-joint angle is reduced to about half
(A) Crush fracture of calcaneum
(B) Fracture neck of humerus
(C) Dislocation of shoulder
(D) Split fracture of calcaneum

Answer: (A)

647. Giassane’s angle intraarticular fracture of calcaneum:
(A) Reduced
(B) Increased
(C) Not changed
(D) Variable

Answer: (B)

648. Bohler’s angle is decreased in fracture of
(A) Calcaneum
(B) Talus
(C) Navicular
(D) Cuboid

Answer: (A)

649. Bohler angle measurement gives the reference for-
(A) Calcaneus
(B) Talus
(C) Navicular
(D) Cuboid

Answer: (A)

650. Neutral triangle is seen radiologically in:
(A) Neck femur
(B) Proximal humerus
(C) Calcaneus
(D) Talus

Answer: (C)

651. # of calcaneous management depending upon:
(A) Type of fracture
(B) Subtalar joint dislocation
(C) Duration of presentation
(D) Degree of displacement

Answer: (ALL)

652. Calcaneum is associated most commonly with which #
(A) # rib
(B) # vertebrae
(C) # skull
(D) # fibula

Answer: (B)

653. Fall on heel with fracture oscalcis is associated with commonly:
(A) Fracture clavicle
(B) Fracture vertebra
(C) Fracture femur
(D) Posterior dislocation of hip

Answer: (B)

654. Least common complication of fall from height is
(A) Fracture base of skull
(B) Fracture calcaneum
(C) Fracture fibula
(D) Fracture 12th thoracic vertebra

Answer: (C)

655. Fatigue fractures (Stress fractures) are most commonly seen in:
(A) Metatarsals
(B) Tibia
(C) Fibula
(D) Neck of femur

Answer: (A)

656. What type of fracture of petrous temporal bone has got the highest chance of facial nerve paralysis?
(A) Longitudinal
(B) Transverse
(C) Oblique
(D) All have equal incidence

Answer: (B)

657. Motorcyclist’s fracture:
(A) Ring fracture
(B) Comminuted fracture
(C) Separation of suture between anterior and posterior half of skull.
(D) Fracture base of skull

Answer: (A)

658. The commonest cause of spinal cord injuries in our country is:
(A) Road traffic accident
(B) Fall from a height
(C) Fall into well
(D) House collapse

Answer: (B)

659. Dislocation without fracture is seen in:
(A) Sacral spine
(B) Lumbar spine
(C) Cervical spine
(D) Thoracic

Answer: (C)

660. ‘Whip-lash’ injury is caused due to:
(A) A fall from a height
(B) Acute hyperextension of the spine.
(C) A blow on top to head.
(D) Acute hyper flexion of the spine.

Answer: (B)

661. Regarding whiplash injury, a true statement includes
(A) Contusion of the spinal cord and fracture of vertebra
(B) Fracture of vertebral body
(C) Spinal cord injury without vertebral fracture
(D) Vertebral fracture without spinal cord injury

Answer: (C)

662. All are true regarding whiplash injury except
(A) Lumbar spine is commonly involved
(B) Fractures are not common
(C) Paresthesia and chronic pain
(D) Hyperextension injury
(E) Sprains and strains without radiological findings

Answer: (A)

663. Jefferson’s # is:
(A) C1
(B) C2
(C) C2 C1
(D) C2 C3

Answer: (A)

664. Jefferson # is:
(A) # of atlas
(B) # of axis
(C) # of spinous process of C7
(D) # of any cervical vertebra

Answer: (A)

665. All of the following are true about fracture of the atlas vertebra, except:
(A) Jefferson fracture is the most common type.
(B) Quadriplegia is seen in 80% cases.
(C) Atlantooccipital fusion may sometimes be needed.
(D) CT scans should be done for diagnosis.

Answer: (B)

666. Hangman’s fracture is fracture of C2
(A) Dens fracture
(B) Lamina
(C) Pars interarticularis
(D) Spinous process

Answer: (C)

667. True regarding Hangman’s fracture is
(A) Odotoid process fracture of C2
(B) Spondylolisthesis of C2 over C3
(C) Whiplash injury
(D) Fracture of hyoid bone

Answer: (B)

668. Hangman’s fracture is
(A) Subluxation of C5 over C6
(B) Fracture dislocation of C2
(C) Fracture dislocation of ankle joint
(D) Fracture of odontoid

Answer: (B)

669. Regarding Hangman’s fracture true is
(A) High post admission mortality
(B) Most common axis fracture
(C) Surgical treatment is necessary
(D) Union almost always occurs

Answer: (D)

670. Burst fracture of cervical spine is due to
(A) Whiplash injury
(B) Fall of weight on neck
(C) Vertical compression injury
(D) Car accident

Answer: (C)

671. Tear droop fracture of lower cervical spine implies:
(A) Wedge compression fracture
(B) Axial compression fractures
(C) Flexion-rotation injury with failure of anterior body
(D) Flexion compression failure of body

Answer: (D)

672. Most common type of injury to spinal cord is-
(A) Flexon
(B) Extension
(C) Compression
(D) Flexon-rotation

Answer: (D)

673. Burst Fracture of spine is a-
(A) Compression Fracture
(B) Extension injury
(C) Direct injury
(D) Flexion-rotation

Answer: (A)

674. In a Pt. with head injury, unexplained hypotension warrants evaluation of:
(A) Upper cervical spine
(B) Lower cervical spine
(C) Thoracic spine
(D) Lumbar spine

Answer: (B)

675. The compression fracture is commonest in
(A) Cervical spine
(B) Upper thoracic spine
(C) Lower thoracic spine
(D) Lumbosacral region

Answer: (C)

676. Seat belt injury is
(A) Tear drop fracture
(B) Wedge fracture
(C) Chance fracture
(D) Whiplash injury

Answer: (C)

677. Chance fracture is/are
(A) Mainly ligamentous injury
(B) Neurological involvement is common
(C) Flexion compression injury
(D) Seat belt injury

Answer: (D)

678. Spinal shock is associated with:
(A) Increased spinal reflexes
(B) Absent spinal reflexes
(C) Loss of autonomic reflexes
(D) Bizarre reflexes

Answer: (B, C)

679. In spinal shock
(A) Knee jerk is the first reflex to return
(B) High thoracic lesions are commonly associated with more severely associated with more severe neurological deficits
(C) Failure of return of cord activity within 48 hours in a very poor prognostic sign
(D) Both B & C

Answer: (D)

680. Return of Bulbocavernous reflex in spinal shock
(A) Sign of recovery from spinal shock
(B) Partial lesion of spinal cord
(C) Complete transection of spinal cord
(D) Incomplete transection of spinal cord

Answer: (A)

681. Complete transaction of the spinal cord at the C7 level produces all of the following effects except:
(A) Hypotension
(B) Limited respiratory effort
(C) Anaesthesia below the level of the lesion
(D) Areflexia below the level of the lesion

Answer: (D)

682. A 40 years old male after RTA, attains spinal injury. His lower limb power is greater than that of upper limb and sacral sensations are present. Type of spinal cord lesion is:
(A) Central cord syndrome
(B) Anterior cord syndrome
(C) Posterior cord syndrome
(D) Complete spinal cord injury

Answer: (A)

683. A patient presented with Saddle anaesthesia, bladder & bowel are normal and muscle power is normal. The diagnosis is
(A) Cauda equine syndrome
(B) L3-L4 root involvement
(C) Conusmedullaris lesion
(D) L4-L5 disc prolapsed

Answer: (C)

684. Symmetrical areflexicblader bowel & lower limb occur in
(A) Cauda equine syndrome
(B) Conusmedullaris syndrome
(C) Nerve root damage
(D) Brown sequerd syndrome

Answer: (B)

685. Cause of atonic bladder is:
(A) Injury to sacral plexus
(B) Injury to upper thoracic cord
(C) Pregnancy
(D) UTI

Answer: (A)

686. A paralysed bladder following spinal injury is best managed by:
(A) Gibbon’s catheter
(B) Malicot catheter
(C) Foley’s catheter
(D) Metallic catheter

Answer: (C)

687. A patient involved in a road traffic accident presents with quadriparesis, sphincter disturbance, sensory level up to the upper border of sternum and a respiratory rate of 35/minute. The likely level of lesion is
(A) C1-C2
(B) C4-C5
(C) T1-T2
(D) T3-T4

Answer: (B)

688. Wrist flexion and finger extension test the following nerve root.
(A) C6
(B) C7
(C) C8
(D) T1

Answer: (B)

689. Little finger of the hand corresponds to which dermatome?
(A) C6 dermatome
(B) C7 dermatome
(C) C8 dermatome
(D) T1 dermatome

Answer: (C)

690. Which movement is not affected in L5 root involvement
(A) Knee extension
(B) Knee flexion
(C) Toe extension
(D) Thigh adduction

Answer: (D)

691. In case of unconscious patient spinal injury is assessed by
(A) Absence of response to painful stimulus
(B) Absence of deep reflexes
(C) Abdominal respiration
(D) Spinal tenderness

Answer: (A, B, C)

692. Patient develops myelopathy post trauma. What dose of methyl prednisolone is to be given:
(A) 30 mg/kg within 3hrs
(B) 45 mg/kg with 6hrs
(C) 60 mg/kg within 9hrs
(D) 75 mg/kg within 12hrs

Answer: (A)

693. Emergency treatment required in:
(A) Fracture humerus
(B) Fracture pelvis
(C) Vertical compression injury
(D) Car accident

Answer: (C)

694. In spinal cord injury, the patient should be transferred with p ad and bandage in the following position:
(A) Supine
(B) Prone
(C) Lateral
(D) Semi prone

Answer: (A)

695. Position for transport of a patient with fracture lumbar spine in a patient:
(A) Neutral
(B) Hyper flexion
(C) Alternating
(D) Hyperextension

Answer: (A)

696. Careless handling of a suspected case of cervical spine injury may result in:
(A) Injury to the spinal cord leading to quadriparesis or quadriplegia
(B) Intracranial haemorrhage with cerebral imitation or unconsciousness.
(C) Cervical haematoma with compression of brachial vessels
(D) Complete paralysis of the affected upper extremity

Answer: (A)

697. A scooter is hit from behind. The rider is thrown off and he lands with his head hitting the kerb. He does not move, complains of severe pain in the neck and is unable to turn his he ad. Well-meaning onlookers rush up to him and try to make him sit up. What would be the best course of action in this situation?
(A) He should be propped up and given some water to drink
(B) He should not be propped up but turned on his face and rushed to the hospital
(C) He should be turned on his back and a support should be placed behind his neck and transported to the nearest hospital
(D) He should not be moved at all but carried to the nearest hospital in the same position in which he has been since his fall.

Answer: (C)

698. The most important step in primary management of a patient with fracture vertebral column:
(A) Careful transport of patient
(B) Maintenance of airway
(C) Treatment of shock
(D) None of the above

Answer: (B)

699. On accident there is damage of cervical spine, first line of management is:
(A) X-ray
(B) Turn head to side
(C) Maintain airway
(D) Stabilize the cervical spine

Answer: (C)

700. Bulbocavernosus reflex is elicited by A/E
(A) Glans pens
(B) Clitoris
(C) Tug of foley catheter
(D) Peri anal

Answer: (D)

701. A young woman met with an accident and had mild quadriparesis. Her lateral X-ray cervical spine revealed C5-C6 fracture dislocation. Which of the following is the best line of management?
(A) Immediate anterior decompression
(B) Cervical traction followed by instrument fixation
(C) Hard cervical collar and bed rest.
(D) Cervical laminectomy.

Answer: (B)

702. Percutaneous vertebroblasty is indicated in all except
(A) Tuberculosis
(B) Metastatis
(C) Osteoporosis
(D) Hemangioma

Answer: (A)

703. Substance that is used for vertebroplasty is:
(A) Poly methyl methacrylate
(B) Poly ethyl methacrylate
(C) Poly methyl ethacrylate
(D) Poly ethyl ethacrylate

Answer: (A)

704. Disc prolapse commonly occurs at:
(A) L4-L5
(B) L5-S1
(C) C5-C6
(D) C4-C5
(E) C3-C4

Answer: (A, B)

705. Commonest site of Disc prolapse is
(A) C5-C6
(B) T8-T9
(C) L4-L5
(D) L5-S1

Answer: (C)

706. The most common site of Disc prolapse is
(A) L2-L3
(B) L3-L4
(C) L4-L5
(D) L5-S1

Answer: (C)

707. The most common site for Prolapse of intervertebral disc is
(A) Cervical region
(B) Lower thoracic region
(C) Upper thoracic region
(D) Lumber region

Answer: (D)

708. After L4-S1 the next commonest site of intervertebral disc prolapse is
(A) C6-C7
(B) T12-L1
(C) L1-L2
(D) L2-L3

Answer: (A)

709. The most common cause of acute sciatica is due to
(A) Trauma
(B) Secondaries of spine
(C) Acute prolapsed intervertebral disc
(D) Tuberculosis of spine

Answer: (C)

710. A building contractor suddenly complains of lower backache which increase on bending down. He has
(A) Renal colic
(B) Tuberculosis of spine
(C) Disc prolapse
(D) Fibrositis’

Answer: (C)

711. The most important single special investigation in lumbar disc prolapse is
(A) Epidurography
(B) Myelography
(C) MRI
(D) Discography
(E) Spinal venography

Answer: (C)

712. Management in case of rupture of disc at L5, S1 is
(A) Emergency removal of disc
(B) Joint fusion
(C) Immobilization for 2 weeks with spinal brace
(D) Traction

Answer: (C)

713. A 44-year –old man presented with acute onset of low backache radiating to the right lower limb. Examination revealed SLRT <40° on the right side, weakness of extensor halluces longus on the right side, sensory loss in the first web space of the right foot and brisk knee jerk. Which of the following is the most likely diagnosis:
(A) Prolapsed intervertebral disc L4-5
(B) Spondylolysis L5-S1
(C) Lumbar canal stenosis
(D) Spondylolisthesis L4-5

Answer: (A)

714. A previously healthy 45 years old laborer suddenly develops acute lower back pain with right-leg pain & weakness of dorsiflexion of the right great toe. Which of the following is true:
(A) Immediate treatment should include analgesics muscle relaxants & back strengthening exercises
(B) The appearance of the foot drop indicate early surgical intervention
(C) If the neurological sign resolve with in 2 to 3 weeks but low back pain persists, the proper treatment would include fusion of affected Lumbar vertebra.
(D) If the neurological signs fail to resolve within 1 week, Lumbar laminectomy and excision of any herniated nucleus pulposus should be done.

Answer: (B)

715. A middle aged lady presents with complaints of lower back pain. On examination there is weakness of extension of right great toe with no sensory impairment. An MRI of the lumbosacral spine would most probably reveal a prolapsed intervertebral disc at what level?
(A) L3-L4
(B) L4-L5
(C) L5-S1
(D) S1-S2

Answer: (B)

716. A patient while lifting a heavy weight presents with sudden onset pain in the lower back radiating along the postero-lateral thigh and lateral leg to the big toe with numbness. The most likely diagnosis is:
(A) L5 fracture
(B) L3-L4 Disc prolapsed
(C) L4-L5 Dis prolapsed
(D) L5-S1 Disc prolapsed

Answer: (C)

717. Which of the following is not recommended in the treatment of Chronic Low Back Pain:
(A) NSAIDS
(B) Bed Rest for 3 months
(C) Exercises
(D) Epidural steroid Injection

Answer: (B)

718. All of the following are included as y ellow flag sings of low back pain, except:
(A) History of systemic steroids use
(B) Reliance on Passive Treatment
(C) Social Isolation
(D) Belief that back pain is severely disabling

Answer: (A)

719. Athlete is sitting on the edge of table with knees flexed at 90 degree. When he extends his knee fully, what will happen to the tibial tuberosity in relation to patella
(A) No change
(B) Movement of TT towards medial border of patella
(C) Movement of TT towards lateral border of patella
(D) Movement of TT towards centre of patella

Answer: (C)

720. In “Bounce home” test of knee joint, end feel is described as all except?
(A) Bony
(B) Empty
(C) Springy
(D) Firm

Answer: (B)

721. Best diagnostic procedure for ant. Cruciate ligament injury is
(A) Lachman’s test
(B) Pivot shift test
(C) Anterior drawer test
(D) Mc Murray’s test

Answer: (A)

722. Lachman test is positive in
(A) Anterior cruciate ligament injury
(B) Posterior Cruciate ligament injury
(C) Medial meniscus injury
(D) Lateral meniscus injury

Answer: (A)

723. What would be the most reliable test for an acutely injured knee of a 27 year old athlete
(A) Anterior drawer test
(B) Posterior drawer test
(C) Lachman test
(D) Steinmann test

Answer: (C)

724. Which one of the following tests will you adopt while examining a knee joint where you suspect an old tear of anterior cruciate ligament?
(A) Posterior drawer test
(B) Mc Murray test
(C) Lachman test
(D) Pivot shift test

Answer: (C)

725. Which of the following is the SAFEST test to be performed in a patient with acutely injured knee joint?
(A) Lachman test
(B) Pivot shift test
(C) Mc Murray’s test
(D) Apley’s grinding test

Answer: (A)

726. In anterior cruciate ligament tear, which of these tests are positive:
(A) Lachman test
(B) McMurry’s test
(C) Anterior drawer test
(D) Posterior drawer test
(E) Apley’s test

Answer: (A)

727. Drawer sign seen in
(A) Cruciate ligament injury
(B) Scurvy
(C) Perthes’s disease
(D) Hyperparathyroidism

Answer: (A)

728. Positive pivot shift test in knee is because of injury to:
(A) Post cruciate ligament
(B) Ant. Cruciate ligament
(C) Medial collateral ligament
(D) Posterior elbow ligament

Answer: (B)

729. Which activity will be difficult to perform for a patient with an anterior cruciate deficient knee joint?
(A) Walk downhill
(B) Walk uphill
(C) Sit cross leg
(D) Getting up from sitting

Answer: (A)

730. Posterior cruciate ligament-true statement is
(A) Attached to the lateral femoral condyle
(B) Intra synovial
(C) Prevents posterior dislocation of tibia
(D) Relaxed in full flexion

Answer: (C)

731. About posterior cruciate ligament true statement is-
(A) Prevent posterior displacement of Tibia
(B) Attaches to lateral femoral condyle.
(C) Intra synovial
(D) Inserted on medial femoral condyle.

Answer: (A)

732. The pivot test is for
(A) Anterior cruciate ligament
(B) Posterior cruciate ligament
(C) Median meniscus
(D) Lateral meniscus

Answer: (A)

733. A twisting injury of knee in flexed position would result in injury to all except
(A) Meniscal tear
(B) Capsular tear
(C) Anterior cruciate ligament
(D) Fibular collateral ligament

Answer: (D)

734. Torsion of knee results in injury most commonly to:
(A) Anterior cruciate ligament
(B) Medial meniscus
(C) Fibular collateral ligament
(D) Tibial collateral ligament

Answer: (D)

735. Structural integrity of collateral ligaments are tested by:
(A) Varus/valgus stress test in full flexion
(B) Varus/valgus stress test in full extension
(C) Varus/valgus stress test in 30° of flexion
(D) Varus/valgus stress test in 90° of flexion

Answer: (C)

736. A patient met with Road traffic Accident with subsequent injury to the left knee. Dial test was positive. What could be the cause:
(A) Medial Collateral Ligament Injury
(B) Medial Meniscal Injury
(C) Lateral Meniscus Tear
(D) Posterolateral Corner Injury

Answer: (D)

737. Physiological locking involves
(A) Internal rotation of femur over stabilized tibia
(B) Internal rotation of tibia over stabilized femur
(C) External rotation of tibia over stabilized femur
(D) External rotation of femur over stabilized tibia

Answer: (A)

738. Which type of injury causes more damage to the semilunar cartilage in the Knee
(A) Flexion and extension at the ankle
(B) Rotation on a flexed knee
(C) Rotation on an extended knee
(D) Squatting position

Answer: (B)

739. Injury to the medial meniscus is rather impossible when the knee joint does not
(A) Extend
(B) Flex
(C) Rotate
(D) Abduct adduct

Answer: (C)

740. The most common type of mode of meniscal injury is
(A) Extension of knee
(B) Flexion of knee
(C) Flexion and rotation movement
(D) Extension and rotation of movement

Answer: (C)

741. Which of the following statements about ‘Menisci’ is not true:
(A) Medial meniscus is more mobile than lateral
(B) Lateral meniscus covers more tibial articular surface than lateral
(C) Medial meniscus is more commonly injured than lateral
(D) Menisci are predominantly made up of type I Collagen

Answer: (A)

742. Medial meniscus of knee joint is injured more often than the lateral meniscus because the medial meniscus is relatively
(A) More mobile
(B) Less mobile
(C) Thinner
(D) Attached lightly to femur

Answer: (B)

743. Bucket handle tear at knee joint is due to:
(A) Injury to medial collateral ligament
(B) Injury to lateral collateral ligament
(C) Injury to ligamentum patellae
(D) Injury to menisci

Answer: (D)

744. Locking of knee joint can be caused by:
(A) Osgood shalter
(B) Loose body in knee joint
(C) Tuberculosis of knee
(D) Medial meniscal partial tear

Answer: (D, B)

745. A patient gives a H/O twisting strain and locking of the knee joint, the most likely diagnosis is:
(A) Avulsion of tibial tubercle
(B) Meniscal tear
(C) Tearing of lateral collateral ligament
(D) Tear of anterior cruciate ligament

Answer: (B)

746. Which of the following correct in medical meniscus tear
(A) Rotation of femur on tibia
(B) Menisci do not heal
(C) Locking and unlocking episodes
(D) Menisci should be excised
(E) All of the about are correct

Answer: (E)

747. It is wise to keep and repair the meniscus rather than removing it when the injury is to which of the following?
(A) Medial part of meniscus
(B) Mid part of meniscus
(C) lateral part of meniscus
(D) Associated with collateral ligament injury

Answer: (C)

748. McMurray’s test is positive in injury of:
(A) Anterior cruciate ligament
(B) Posterior cruciate ligament
(C) Medical meniscus injury
(D) Lateral Meniscus injury
(E) Popliteal bursitis

Answer: (C, D)

749. Mc. Murray’s sign is seen in injury to
(A) Medial meniscus
(B) Medial collateral ligament
(C) Anterior cruciate ligament
(D) Posterior cruciate ligament

Answer: (A)

750. All of the following features in the knee are recognized to be consistent with a torn medial meniscus, except
(A) Excessive forward glide
(B) Giving way
(C) Locking
(D) Mc. Murray’s sign

Answer: (A)

751. A 18 years old boy was playing football, when he suddenly twisted his knee on the ankle and he fell down. He got up after 10 minutes and again started playing, but next day his knee was swollen and he could not move it. The most probable cause is:-
(A) Medial meniscus tear
(B) Anterior cruciate ligament tear
(C) Medical collateral ligament injury
(D) Posterior cruciate ligament injury

Answer: (A)

752. Investigation for injury of knee cartilage
(A) Aspiration
(B) Arthrography
(C) X-ray
(D) Arthroscopy

Answer: (D)

753. Which is the investigation of choice for a sport injury of the knee?
(A) Ultrasonography
(B) Plain radiography
(C) Arthrography
(D) Arthroscopy

Answer: (D)

754. Athletic sustained an injury around the knee joint suspecting cartilage damage, which of the following is an investigation of choice?
(A) Pain X ray
(B) Clinical examination
(C) Arthroscopy
(D) Arthrotomy

Answer: (C)

755. Commonest dangerous complication of posterior dislocation of knee is
(A) Popliteal artery injury
(B) Sciatic nerve injury
(C) Ischaemia of lower leg compartment
(D) Femoral artery injury

Answer: (A)

756. Management of hemarrthrosis
(A) Compression bandage
(B) Needle aspiration
(C) P.O.P cast
(D) Rest to the joint
(E) Synovectomy in recurrent cases

Answer: (ALL)

757. The most common site for ligamentous injuries are those of the
(A) Shoulder joint
(B) Elbow
(C) Knee joint
(D) Ankle joint

Answer: (D)

758. Injury around the ankle joint occur at
(A) Inversion of foot
(B) Eversion of foot
(C) Internal rotation of foot
(D) External rotation of foot

Answer: (A)

759. The most common ligament injured around ankle joint is
(A) Anterior talofibular
(B) Deltoid ligament
(C) Posterior talofibular
(D) Spring ligament

Answer: (A)

760. Most common cause of insertional tendonitis of tendoachilles is:
(A) Overuse
(B) Improper shoe wear
(C) Runner and jumpers
(D) Steroid injections

Answer: (A)

761. Ruptured tendon is most commonly seen in
(A) Stab injury
(B) soft Tissue tumour
(C) Overuse
(D) Congenital defect

Answer: (C)

762. The operative procedure known as “micro-fracture” is done for the
(A) Delayed union of femur.
(B) Non union of tibia
(C) Loose bodes of ankle joint
(D) Osteochondrial defect of femur

Answer: (D)

763. Indications of arthoplasty:
(A) Osteoarthritis
(B) Rheumatoid arthritis
(C) Ankylosingspondylosis
(D) Gout
(E) Fracture neck femur

Answer: (ALL)

764. Major indication (s) for arthroplasty:
(A) Osteoarthritis of hip
(B) Ankylosis of elbow
(C) Ununitedtibial fracture
(D) Ununited femoral neck fracture
(E) TB spine

Answer: (A, B, D)

765. Metal on Metal articulation should be avoided in:
(A) Osteonecrosis
(B) Young female
(C) Inflammatory arthritis
(D) Revision surgery

Answer: (B)

766. Aseptic loosening in cemented total hip replacement, occurs as a result of hypersensitivity response to:
(A) Titanium debris
(B) High density polythene debris
(C) N. N-Dimethyltryptamine (DMT)
(D) Free radicals

Answer: (B)

767. Watson Jones operation is done for?
(A) Neglected Club foot
(B) Muscle paralysis
(C) Valgus deformity
(D) Hip replacement

Answer: (D)

768. Watson Jones operation is done for:
(A) Polio
(B) Muscle paralysis
(C) Neglected clubfoot
(D) Chronic ankle instability.

Answer: (D)

769. A patient developed breathlessness and chest pain, on second postoperative after a total hip replacement. Echo-cardiography showed right ventricular dilatation and tricuspid regurgitation. What is the most likely diagnosis.
(A) Acute MI
(B) Pulmonary embolism
(C) Hypertensive shock
(D) Cardiac tamponade

Answer: (B)

770. What is the most common cause of death after total hip replacement?
(A) Infection
(B) Deep vein thrombosis
(C) Pulmonary embolism
(D) Pneumonia

Answer: (C)

771. A sixty five year old lady presents with a long standing history of pain and swelling in her right knee. Pain is significantly interfering with her activities of daily living. Radiological evaluation shows grade III changes of osteoarthritis. Which of the following is the recommended line of management?
(A) Conservative management
(B) Arthroscopic lavage washout
(C) Partial knee replacement
(D) Total knee replacement

Answer: (D)

772. A 68 yr old man came with pain and swelling of right knee. Ahlbeck grade 2 osteoarthritic changes were found on investigation. What is the further management:
(A) Conservative
(B) Arthroscopic washout
(C) High tibial osteotomy
(D) Total knee replacement

Answer: (D)

773. All of the following statements about High Tibial osteotomy are true, Except:
(A) Magnitude of correction achieved is greater than 30°
(B) Indicated in Unicompartmental osteoarthritis
(C) Performed through cancellous bone
(D) Recurrence is a long term complication

Answer: (A)

774. Site for 1st order bone grafting:
(A) Pelvis
(B) Tibial metaphysis
(C) Medial malleolus
(D) Femoal condyle
(E) Greater trochanter

Answer: (A)

775. Standard site of primary bone graft:
(A) Pelvis
(B) Greater trochanter
(C) Medial malleolus
(D) Lateral malleolus
(E) Fibula

Answer: (A)

776. Cancellous bone graft taken from:
(A) Femoral condyles
(B) Pelvis
(C) Greater trochanter
(D) Tibial metaphysic

Answer: (ALL)

777. Which of the following is ideal site for harvesting bone graft?
(A) Iliac crest
(B) Distal end of humerus
(C) Distal end of femur
(D) Fibula

Answer: (A)

778. Which of the following is/are not included in management of intra-articular fracture:
(A) Arthrodesis
(B) Excision
(C) Aspiration
(D) K Wire
(E) Plaster of Paris cast

Answer: (NONE)

779. Usual treatment (s) of fracture involving articular surface is/are
(A) Arthrodesis
(B) Excision
(C) Skeletal traction
(D) Internal fixation
(E) P.O.P slab

Answer: (ALL)

780. Which of the following condition should be given most priority in case of fracture:
(A) Open fracture
(B) Dislocated fracture
(C) Vascular injury
(D) Malunited fracture
(E) Compartment syndrome

Answer: (A, B, C, E)

781. In the management of long bone fracture following can be done:
(A) Intramedullary nailing
(B) Plating
(C) External fixation
(D) Tension band wiring
(E) Screw

Answer: (A, B, C)

782. Long bone # fixation done with:
(A) Intramedullary Nail
(B) Compression plate
(C) External fixation
(D) Screw
(E) Tension band wiring

Answer: (A, B, C)

783. Action of intramedullary ‘K’ nail is
(A) Two-point fixation
(B) Three-point fixation
(C) Compression
(D) Weight concentration

Answer: (B)

784. All of the following are indications for open reduction and internal fixation of fractures except
(A) Compound fracture
(B) Unsatisfactory closed reduction
(C) Multiple trauma
(D) Intra-articular fracture

Answer: (A)

785. Which fracture in children requires open reduction:
(A) Fracture tibial epiphysis
(B) Fracture shaft of femur
(C) Fracture both bones forearm
(D) Fracture femoral condyle

Answer: (A)

786. Open reduction & internal fixation is done for all of the following #s except:
(A) Patella #
(B) Olecranon #
(C) Volar Barton’s #
(D) # Lateral condyle of humerus

Answer: (C)

787. Treatment of choice for fracture lower 1/4th of tibia in non-union with multiple scarred wounds and discharging sinuses and about 4 cm shortening of leg?
(A) Ilizarov fixator
(B) Plate
(C) External fixation
(D) Intramedullary nail

Answer: (A)

788. Surgical excision is contraindicated in:
(A) Olecranon process
(B) Patella
(C) Head of radius
(D) Lateral condyle humerus

Answer: (D)

789. Contraindication for skin traction:
(A) Dermatitis
(B) Vascularity compromised status of limb
(C) Abrasions
(D) Hypopigmentation (vitiligo)
(E) Bony deformity

Answer: (A, B, C)

790. Gallows traction is used in fracture of:
(A) Head of femur
(B) Neck of femur
(C) Shaft of femur
(D) Humerus

Answer: (C)

791. Skeletal traction is given by:
(A) K-wire
(B) Pavlik harness
(C) Denham Pin
(D) Steinmann’s pin
(E) Rush pin, Cancellos screw

Answer: (A, D)

792. All of the following are used for giving skeletal traction, except:
(A) Steimann’s pin
(B) Kirschner’s wire
(C) Bohler’s stirrup
(D) Rush pin

Answer: (D)

793. During reconstruction of an amputated limb, which of the following is done first:
(A) Vein ligation
(B) Artery ligation
(C) Nerve ligation
(D) Fixation of bone

Answer: (D)

794. Which of the following structures is fixed first during implantation of an amputated digit:
(A) Bone
(B) Artery
(C) Vein
(D) Nerve

Answer: (A)

795. Pollicization is
(A) Amutation of thumb
(B) Equalization of fingers
(C) Toe to thumb transplantation
(D) Reconstruction of thumb

Answer: (D)

796. Which is not a deep heat therapy
(A) Short wave diathermy
(B) Ultrasound therapy
(C) Infrared therapy
(D) Microwave therapy

Answer: (C)

797. Consider the following nerve injuries
(1) Axonotmesis
(2) Neurotmesis
(3) Neuropraxia the correct sequence of these injuries in terms of ‘increasing’ severity of damage to the nerve fibres and sheath, is
(A) 1, 2 and 3
(B) 3, 2 and 1
(C) 2, 1 and 3
(D) 3, 1 and 2

Answer: (D)

798. Rate of regeneration of severed nerve is
(A) 0.1 mm/day
(B) 1 mm/day
(C) 1 cm/day
(D) None

Answer: (B)

799. “Tinel’s sign” indicates
(A) Neurofibroma
(B) Injury to peripheral nerves
(C) Atrophy of nerves
(D) Regeneration of nerves

Answer: (D)

800. Tourniquet paralysis is an unfortunate complication leads to
(A) Neuropraxia
(B) Axonotmesis
(C) Neurotmesis
(D) None of the above

Answer: (A)

801. Nerve suturing in a clean cut injury is done best in
(A) 6 hours
(B) 12 hours
(C) After one day
(D) After two day

Answer: (A)

802. In Seddon’s classification, complete division of nerve is
(A) Neuropraxia
(B) Axonotmesis
(C) Neurotmesis
(D) None of the above

Answer: (C)

803. Following indicate better prognosis is in injury except
(A) Neuroproxia
(B) Younger age
(C) Pure motor nerve injury
(D) Proximal injury

Answer: (D)

804. Prognosis after secondary nerve suturing is better in pure than in mixed ones. Based on this criterion, which one of the following nerves should be given the best result after suturing in identical conditions?
(A) Common peroneal nerve
(B) Radial nerve
(C) Ulnar nerve
(D) Median nerve

Answer: (B)

805. Best results of nerve suturing are seen in
(A) Median nerve
(B) Ulnar nerve
(C) Sciatic nerve
(D) Radial nerve

Answer: (D)

806. All are true regarding brachial plexus injury, except:
(A) Preganglionic lesions have a better prognosis than postganglionic lesions
(B) Erb’s palsy causes paralysis of the abductors and external rotators of the shoulder
(C) In Klumpke’s palsy, Horner’s syndrome may be present on the ipsilateral side
(D) Histamine test is useful to differentiate between the preganglionic and postganglionic lesions

Answer: (A)

807. Most common cause of neurological deficit in upper limb is
(A) Polio
(B) Erb’s palsy
(C) C1-C2 dislocation
(D) Fracture dislocation of cervical spine

Answer: (B)

808. A pole vaulter had a fall during pole vaulting and had paralysis of the arm. Which of the following investigations gives the best recovery prognosis:
(A) Electromyography
(B) Muscle biopsy
(C) Strength Duration Curve
(D) Creatine phosphokinase levels

Answer: (C)

809. A 20 year old male involved in single vehicle rollover accident presented with inability to abduct the shoulder and flex the elbow. There was associated wasting of the supraspinatus, deltoid and biceps muscle. Which of the following is the most likely diagnosis:
(A) Shoulder Dislocation
(B) Upper Trunk Injury
(C) Lateral Cord Injury
(D) Medial Cord Injury

Answer: (B)

810. In a full term newly born male baby, on routine check up soon after delivery has weak Moro’s reflex on right hand but intact Grasp’s reflex with weak abduction and supination movement of hand is found. This will suggest that most likely injury would be at
(A) C5, 6nerve
(B) C7, C8 nerve
(C) C8, T1 nerve
(D) T1, T2 nerve

Answer: (A)

811. The nerve roots involved in Erb’s palsy are
(A) C3 C4
(B) C4 C5
(C) C5 C6
(D) C6 C7

Answer: (C)

812. Which of the following statement is wrong about Erb’s palsy?
(A) Abductors of the shoulder are weak
(B) Upper part of brachial plexus is involved
(C) Supinators are normal
(D) Internal rotation of arm

Answer: (C)

813. Erb’s point is located at the junction of
(A) Anterior primary rami of C5 and C6
(B) Anterior primary rami of C8 and T1
(C) Superior and middle trunk of brachial plexus
(D) None of the above

Answer: (A)

814. Erb’s palsy is due to involvement of
(A) Upper trunk of brachial plexus
(B) Middle trunk of brachial plexus
(C) Lower trunk of brachial plexus
(D) Long thoracic nerve

Answer: (A)

815. A young boy who was driving motorcycle at a high speed collided with a tree & was thrown on his right shoulder. Though there was no fracture, his right arm was medially rotated and forearm pronated. The following facts concerning this patient are correct, except
(A) The injury was at Erb’s point
(B) A lesion of C5 and C6 was present
(C) The median and ulnar nerves were affected
(D) Supraspinatus, Infraspinatus, Subclavius& Biceps brachii were paralyzed

Answer: (C)

816. All of the following muscles undergo paralysis after injury to C5 and C6 spinal nerves except:
(A) Biceps.
(B) Corachobrachialis.
(C) Brachialis.
(D) Brachioradialis.

Answer: (D, B)

817. A 19 year old boy fell from the motar bike on his shoulder. he doctor diagnosed him a case of Erbs paralysis. The following signs and symptoms will be observed except:
(A) Loss of abduction at shoulder joint
(B) Loss of lateral rotation
(C) Loss of pronation at radioulnar joint.
(D) Loss of flexion at elbow joint.

Answer: (C)

818. Lower branch of brachial plexus injury leads to
(A) Erb’s palsy
(B) Klumpke’s palsy
(C) Bell’s palsy
(D) Wrist drop

Answer: (B)

819. The lesion in Klumpke’s paralysis is at
(A) Cervical plexus
(B) Lower brachial
(C) Upper brachial
(D) Sacral plexus

Answer: (B)

820. The mode of injury of Klumpke’s paralysis is
(A) Forcible increase in the neck shoulder angle (As during child birth)
(B) Forcible rotation of upper arm
(C) Forcible hyperabduction of upper arm (falling person trying to catch an object)
(D) Fracture shaft humerus

Answer: (C)

821. A 45 years male present with abrupt onset, pain weakness, loss of contour of shoulder and wasting of muscle of arm on 5th day of tetanus toxoid immunization in deltoid. Likely cause is
(A) Radial nerve entrapment
(B) Thoracic outlet syndrome
(C) Brachial neuritis
(D) Hysteria

Answer: (C)

822. All of the following are features of musculocutaneous nerve injury at axilla except:
(A) Loss of flexion of shoulder
(B) Loss of flexion at elbow
(C) Loss of supination of forearm
(D) Loss of sensation on radial side of forearm

Answer: (A)

823. All of the following features can be observed after the injury to axillary nerve, except:
(A) Loss of rounded contour of shoulder
(B) Loss of sensation along lateral side of upper arm
(C) Loss of overhead abduction
(D) Atrophy of deltoid muscle

Answer: (C)

824. Which of the following nerve injuries produce the deformities?
(I) Upper trunk I. Wrist drop
(II) Ulnar nerve II. Porter’s tip hand
(III) Axillary nerve III. Claw hand
(IV) Radial nerve IV. Flatterning of the shoulder
(A) I(i), II(ii), III(iii), IV(iv)
(B) I(ii), II(iii), III(iv), IV(i)
(C) I(iii), II(iv), III(ii), IV(i)
(D) I(iv), II(iii), III(ii), IV(i)

Answer: (B)

825. Match the followings:
(I) Erb’s paralysis I. Lower trunk
(II) Klumpke’s paralysis II. Axillary nerve injury
(III) Crutch paralysis III. Radial nerve injury
(IV) Fractures surgical neck humerus IV. Upper trunk injury
(A) I(iv), II(iii), III(ii), IV(i)
(B) I(iii), II(ii), III(iv), IV(i)
(C) I(iv), II(i), III(iii), IV(ii)
(D) I(i), II(iii), III(iv), IV(ii)

Answer: (C)

826. Match the following
(I) Erb’s paralysis
(II) Klumpke’s paralysis
(III) Crutch paralysis
(IV) Fracture surgical neck humerus
(i) Lower trunk injury
(ii) Axillary nerve injury
(iii) Radial nerve injury
(iv) Upper trunk injury
(A) I(iv), II(iii), III(ii), IV(i)
(B) I(iii), II(ii), III(iv), IV(i)
(C) I(iv), II(i), III(iii), IV(ii)
(D) I(i), II(iii), III(iv), IV(ii)

Answer: (C)

827. A patient presented with claw hand after a Supracondylar fracture was reduced and plaster applied. The diagnosis is:
(A) Medial nerve injury
(B) Volkmann’s ischaemic contracture
(C) Ulnar nerve injury
(D) Dupuytren’s contracture

Answer: (B)

828. Dislocation of which one of the following carpal bones can present as medal nerve palsy?
(A) Scaphoid
(B) Hamate
(C) Lunate
(D) Trapezium

Answer: (C)

829. Injury of median nerve at wrist is best detected by
(A) Action of abductor pollicisbrevis
(B) Action of flexor pollicisbrevis
(C) Loss of sensation of radial half of palm
(D) Loss of sensation of tip of ring finger

Answer: (A)

830. Pointing index sign in seen in __ nerve palsy
(A) Ulnar
(B) Radial
(C) Median
(D) Axillary

Answer: (C)

831. Anterior interosseous nerve is a branch of
(A) Musculocutaneous
(B) Radial
(C) Median
(D) Ulnar

Answer: (C)

832. Median nerve is injured during
(A) Elbow dislocation
(B) Fracture lateral epicondyle of humerus
(C) Fracture medial epicondyle of humerus
(D) Supracondylar fracture of humerus

Answer: (A, D)

833. Median nerve injury at the wrist causes
(A) Claw hand
(B) Loss of apposition of thumb
(C) Policeman’s tip deformity
(D) Saturday night palsy

Answer: (B)

834. ‘Ape thumb deformity’ is observed in lesions of
(A) Radial nerve injury
(B) Ulnar nerve injury
(C) Median nerve injury
(D) Circumflex humeral nerve injury
(E) Posterior interosseous nerve injury

Answer: (C)

835. Compression of a nerve within the carpal tunnel products inability to:
(A) Abduct the thumb
(B) Adduct the thumb
(C) Flex the distal phalanx of the thumb
(D) Oppose the thumb

Answer: (D)

836. Median nerve lesion at the wrist causes all of the following, Except:
(A) Thenar Atrophy
(B) Weakness of adductor Pollicis
(C) Weakness of 1st and 2ndLumbricals
(D) Weakness of Flexor PollicisBrevis

Answer: (B)

837. Median nerve injury at wrist, is commonly tested by:
(A) Contraction of abductor pollicisbrevis
(B) Contraction of flexor pollicisbrevis
(C) Loss of sensation on palm
(D) Loss of sensation on ring finger

Answer: (A)

838. A cut injury on wrist causes inability of thumb to touch the tip of little finger, the nerve likely to be damaged is:
(A) Ulnar
(B) Radial
(C) Median
(D) Deep branch of ulnar nerve

Answer: (C)

839. A boy presents with complaints of hypoaesthesia and wasting of thenar eminence. The nerve most likely to damaged in this patient:
(A) Musculocutaneous nerve
(B) Median nerve
(C) Ulnar nerve
(D) Radial nerve

Answer: (B)

840. Nerve damaged due to lunate dislocation (in carpal tunnel):
(A) Median & ulnar
(B) Median
(C) Ulnar
(D) Radial

Answer: (B)

841. Ape thumb deformity is seen in involvement of :
(A) Median nerve
(B) Ulnar nerve
(C) Radial nerve
(D) Axillary nerve

Answer: (A)

842. Tardy ulnar nerve palsy is due to:
(A) Cubitus valgus
(B) Fixation of nerve in the groove by osteoarthritis
(C) Excision of elbow joint
(D) Fracture of internal condyle

Answer: (A)

843. Trady ulnar nerve palsy is seen with
(A) Lateral humeral condyle fracture
(B) Supracondylar fracture
(C) Medial humeral condyle fracture
(D) Fracture capitulum

Answer: (A)

844. The “Card test” tests the function of
(A) Median nerve
(B) Ulnar nerve
(C) Axillary nerve
(D) Radial nerve

Answer: (B)

845. Froment’s sign in present in paralysis of ____ injury
(A) Ulnar
(B) Radial
(C) Median
(D) Axillary

Answer: (A)

846. Claw hand is seen in
(A) Cervical rib Carpal tunnel syndrome
(B) Multiple scelerosis
(C) Radial nerveinjury
(D) Ulnar nerve injury

Answer: (D)

847. Following an incised wound in the front of wrist, the subject is unable to oppose the tips of the little finger and the thumb. The nerve (s) involved is/are
(A) Ulnar nerve alone
(B) Median nerve alone
(C) Median nerve ulnar nerves
(D) Radial and ulnar nerves

Answer: (C)

848. “Ulnar paradox’ is related with the following
(A) Lumbricals
(B) Intrinsic muscle
(C) EPL
(D) Ulnar half of FDP

Answer: (B)

849. Ulnar nerve injury at wrist results in
(A) Unable to grasp card between the fingers
(B) Claw hand
(C) Inability to abduct or adduct the fingers
(D) All of the above

Answer: (D)

850. Disability of hands is maximum with a lesion of
(A) Median nerve at elbow
(B) Median nerve at wrist
(C) Ulnar nerve at elbow
(D) Ulnar nerve at wrist
(E) Radial nerve lesion

Answer: (D)

851. Ulnar nerve severed above elbow causes
(A) Complete loss of sensation in 4th and 5th finger
(B) Paralysis of all lumbricals
(C) Paralysis of all interosseia
(D) Paralysis of flexor carpi ulnaris
(E) Paralysis of flexor profundus

Answer: (B, C, D, E)

852. A factory worker has laceration proximal to wrist with no opposition of thumb and fingers, loss of sensation over lateral aspect of thumb and fingers. Diagnosis is injury to
(A) Median nerve + Ulnar nerves
(B) Median + Radial nerves
(C) Radial + Ulnar nerves
(D) Median nerve only

Answer: (A)

853. Claw hand occurs in
(A) Volkmann’s ischaemic Contracture
(B) Dupuytren’s contracture
(C) Ulnar nerve injury
(D) Sudeck’s atrophy

Answer: (C)

854. Froment’s sign is characteristically seen in
(A) Ulnar nerve injury
(B) Median nerve injury
(C) Radial nerve injury
(D) Intercostobrachial nerve injury

Answer: (A)

855. Froment’s sign is positive due to paralysis of
(A) Opponenspollicis
(B) Abductor pollicis
(C) Adductor pollicis
(D) Adductor halluces longus

Answer: (C)

856. A boy presents with injury to medial epicondyle of the humerus. Which of the following would not be seen
(A) Weakness of the ulnar deviation and flexion
(B) Complete paralysis of the IIrd and IVth digits
(C) Atrophy of the hypothenar eminence.
(D) Decreased sensation of the hypothenar eminence.

Answer: (B)

857. All of the following muscles are paralyzed in trauma to median nerve at the wrist except
(A) Adductor pollicis
(B) Abductor pollicis
(C) First two lumbicals
(D) Muscles of the Thenar eminence

Answer: (A)

858. Ulnar nerve injury at wrist involves following except:
(A) Plamarinterossei
(B) Opponenspollicis
(C) Dorsal interossei
(D) Adductor pollicis

Answer: (B)

859. Low ulnar nerve palsy is characterized by:
(A) Claw hand
(B) Sensory loss of medial four digits
(C) Weakness of grips
(D) Inability to extend at M.C.P. joint
(E) Inability to abduct the thumb

Answer: (A, C)

860. Injury to the Ulnar nerve at the wrist causes paralysis of
(A) Apposition of the thumb
(B) Abduction of the carpo-metacarpal joint of the thumb
(C) Adduction of the thumb
(D) Flexion of the MCP joint of the middle finger

Answer: (C)

861. Claw hand is caused by lesion of –
(A) Ulnar nerve
(B) Median nerve
(C) Axillary nerve
(D) Radial nerve

Answer: (A)

862. In a patient with Claw Hand due to Leprosy, the deformity would be classified as:
(A) Grade 0
(B) Grade I
(C) Grade II
(D) Grade III

Answer: (C)

863. A patient with leprosy presents with clumsiness of hand. His ulnar nerve is affected. Clumsiness is due to palsy of which muscle:
(A) Extensor carpi ulnaris
(B) Abductor pollicisbrevis
(C) Oppenenspollicis
(D) Inerosseous muscle

Answer: (D)

864. A patient comes to the emergency department after alcohol binge previous night and sleeping on arm chair. In the morning he is unable to move his hand and diagnosis of ulnar nerve palsy is made. What is the next line of management
(A) Wait and Watch
(B) Knuckle Bender Splint
(C) Immediate exploration of nerve
(D) Do EMG study after 2 days

Answer: (B)

865. A person is not able to extend his metacarpophalangeal joint. This is due injury to which nerve
(A) Ulnar nerve
(B) Radial nerve injury
(C) Medial nerve injury
(D) Post. Introsseus nerve injury

Answer: (D)

866. A 30-year-old male underwent excision of the right radial head. Following surgery, the patient developed inability to extend the fingers and thumb of the right hand. He did not have any sensory deficit. Which one of the following is the most likely cause?
(A) Injury to Posterior interosseus nerve
(B) Iatrogenic injury to common extensor origin
(C) Injury to anterior interosseus nerve
(D) High radial nerve palsy

Answer: (A)

867. Which of the following statements about ‘Low’ Radial nerve palsy is not true:
(A) Loss of nerve supply to brachioradialis
(B) Loss of nerve supply to extensor carpi radialisbrevis
(C) Loss of nerve supply to extensor pollicisbrevis
(D) Loss of sensation over first dorsal web space

Answer: (A)

868. Commonest cause of wrist drop is
(A) Intramuscular injection
(B) Fracture humerus
(C) Dislocation of elbow
(D) Dislocation of shoulder

Answer: (B)

869. Injury to which of the following nerves is most likely to result in Wrist drop?
(A) Radial
(B) Ulnar
(C) Median
(D) Musculocutaneous

Answer: (A)

870. Saturday night palsy involves nerve
(A) Radial
(B) Ulnar
(C) Median
(D) Axillary
(E) Any of the above

Answer: (A)

871. In fracture of distal half of humerus, the nerve injured is
(A) Axillary
(B) Median
(C) Radial
(D) Ulnar

Answer: (C)

872. After injury to radial nerve in radial groove, first sign of recovery is
(A) Tinel’s sign
(B) Power in extensor corpiradialislongus
(C) Power in Brachioradialis
(D) Power in Triceps

Answer: (A)

873. Feature of radial nerve injury at spiral groove
(A) No wrist drop
(B) Sensory loss over deltoid
(C) Thumb, finger, wrist drop
(D) Wrist droop + Extensors of Forearm paralysis

Answer: (C)

874. Radial nerve is most commonly injured in fracture of
(A) Mid shaft of humerus
(B) Neck of humerus
(C) Lateral condyle of humerus
(D) Lower 1/3rd of humerus

Answer: (A)

875. Writ drop results in injury of
(A) Ulnar nerve
(B) Median nerve
(C) Posterior interosseus nerve
(D) Radial nerve

Answer: (D)

876. Following structures pass through spiral groove of humerus
(A) Radial nerve
(B) Radial artery
(C) Anterior circumflex humeral artery
(D) Posterior circumflex humeral artery

Answer: (A, E)

877. Injury to radial nerve – mid humerus level following occurs:
(A) Paralysis of extensors of hand & wrist
(B) Equally diagnosed by sensory and motor testing
(C) Can be approached by splitting latissimusdorsi
(D) Paralysis of Triceps

Answer: (A)

878. Damage to the radial nerve in the spinal groove spares which muscle
(A) Lateral head of triceps
(B) Long head of triceps
(C) Medial head of triceps
(D) Anconeus

Answer: (B)

879. Injury to radial nerve in lower part of spiral groove:
(A) Spares nerve supply to extensor carpi radialislongus
(B) Results in paralysis of anconeus muscle
(C) Leaves extensions at elbow joint intact
(D) Weakness pronation movement

Answer: (C)

880. Cock up splint is used in management of
(A) Ulnar nerve palsy
(B) Brachial plexus palsy
(C) Radial nerve palsy
(D) Combined ulnar & Median nerve palsy

Answer: (C)

881. Non dynamic splint is:
(A) Banjo
(B) Opponens
(C) Cock up
(D) Brand

Answer: (C)

882. Cock up splint is used in:
(A) Median nerve injury
(B) Radial nerve injury
(C) Ulnar nerve injury
(D) Volkman’s ischemic contracture.

Answer: (B)

883. A 25-yr-old lady sustained a lacerated wound on the back of right thigh by a horn of a bull. The wound was sutured two months later she developed foot drop and an ulcer on the dorsum of the foot. The most likely diagnosis is
(A) Chronic ischaemia to limbs due to Popliteal artery injury
(B) Partial injury to sciatic nerve
(C) Complete division of sciatic nerve
(D) Injury to hamstring muscles

Answer: (B)

884. Section of lateral Popliteal nerve at the neck of fibula causes:
(A) Foot drop
(B) Trophic ulcers on toes
(C) Atrophy of calf muscle
(D) Pain in the leg

Answer: (A)

885. Foot drop result because of injury to
(A) Superficial peroneal nerve
(B) Deep peroneal nerve
(C) Posterior tibial nerve
(D) Anterior tibial nerve

Answer: (B)

886. Sudden hyperflexion of high over abdomen (Mcrobert’s procedure), which of the following nerve is commonly involved:
(A) Common peroneal nerve
(B) Obturator nerve
(C) Lumbosacral trunk
(D) Lateral cutaneous nerve of thigh.

Answer: (D)

887. Injury to the common peroneal nerve at the lateral aspect of head of fibula results in all of the following except:
(A) Weakness of ankle dorsi-flexion.
(B) Foot drop.
(C) Loss of ankle reflex.
(D) Sensory impairment on lateral aspect of leg extending to the dorsum of foot.

Answer: (C)

888. Common peroneal nerve is related to
(A) Shaft of tibia
(B) Neck of fibula
(C) Lower tibio-fibular joint
(D) Shaft of fibula

Answer: (B)

889. Carpel tunnel syndrome is due to compression of :
(A) Radial nerve
(B) Ulnar nerve
(C) Palmer branch of the Ulnar nerve
(D) Median nerve

Answer: (D)

890. Which of the following does NOT predispose to carpal tunnel syndrome
(A) Hypertension
(B) Hypothyroidism
(C) Pregnancy
(D) Acromegaly

Answer: (A)

891. True about carpel tunnel syndrome:
(A) Paraesthesia of the lateral 2 fingers
(B) Atropy of the thenar eminence
(C) Atropy of the hypothenar eminence
(D) Claw hand
(E) Tinel sign is positive

Answer: (A, E)

892. Phalen’s test is positive in
(A) Carpal tunnel syndrome
(B) De Quervain’s disease
(C) Tennis elbow
(D) Ulnar bursitis

Answer: (A)

893. Consider the following statements about Carpal tunnel syndrome
(1) It may occur in Acromegaly
(2) It may occur in pregnancy
(3) It causes delayed ulnar nerve conduction
(4) It may be associated with wasting of abductor pollicisbrevis of these statements
(A) 1, 2 and 3 are correct
(B) 2, 3 and are correct
(C) 1, 2 and 4 are correct
(D) 1, 3 and 4 are correct

Answer: (C)

894. In carpal tunnel syndrome, features are of
(A) Compression of ulnar nerve
(B) Compression of median nerve under the flxor retinaculum
(C) Anaesthesia over thenar eminence
(D) Atrophy of hypothenar muscles

Answer: (B, C)

895. Carpal tunnel syndrome can be caused by the following except?
(A) Diabetes Mellitus
(B) Amyloidosis
(C) Addison’ Disease
(D) Hypothyroidism

Answer: (C)

896. True about carpal tunnel syndrome is A/E
(A) More common in males
(B) Acromegaly can cause
(C) Positive tinel’s sign
(D) Nerve conduction study done
(E) Transverse carpal ligament are cut to relieve symptom

Answer: (A)

897. A 52 year old lady complaints of nocturnal pain over the thumb, index finger and middle finger. Which of the following tests is not required to reach a diagnosis:
(A) Phalen’s test
(B) Finkelstein test
(C) Tinel’s sign
(D) Tourniquet test

Answer: (B)

898. True about carpal tunnel syndrome:
(A) Occur in pregnancy
(B) Affects medial 3½ finger
(C) Associated with Hypothyroidism
(D) Froment sign positive
(E) Median nerve involvement is present

Answer: (A, C, E)

899. Entrapment neuropatheis commonly affect the following nerves except:
(A) Tibial
(B) Femoral
(C) Lateral cutaneous nerve of thigh
(D) Common digital nerve

Answer: (B)

900. MeralgiaParaesthetic involves
(A) Axillary nerve
(B) Sural nerve
(C) Median nerve
(D) Lateral cutaneous nerve of high

Answer: (D)

901. Most common cause of tarsal tunnel syndrome:
(A) Osteoarthritis
(B) Ankylosing spondylitis
(C) Psoriatic arthritis
(D) Rheumatoid arthritis

Answer: (D)

902. Commonest cause for neuralgic pain in foot is:
(A) Compression of communication between medial and lateral planter nerves
(B) Exaggeration of longitudinal arches
(C) Injury to deltoid ligament
(D) Shortening of planter aponeurosis

Answer: (A)

903. In a patient with a history of burning pain localized to the plantar aspect of the foot, the differential diagnosis must include:
(A) Peripheral vascular disease
(B) Tarsal coalation
(C) Tarsal tunnel syndrome
(D) Planter fibromatosis

Answer: (C)

904. The classical flexion and rotation deformities at hip and knee joint, as a sequelae of poliomyelitis, are due to the contracture of
(A) Tensor fascia lata
(B) Gastrocnemius
(C) Tendo Achilles
(D) Hamstrings

Answer: (A)

905. All are possible during contraction of Tensor fasciae latae except
(A) Adduction of Hip joint
(B) Flexion of Hip joint
(C) Abduction of Hip joint
(D) Flexion of the knee

Answer: (A)

906. In a post polio case, ilio-tibial tract contracture is likely to result in:
(A) Extension at the hip and knee
(B) Extension at the hip
(C) Flexion at the hip and the knee
(D) Extension at the knee

Answer: (C)

907. Test for tight iliotibial band is:
(A) Ober’s test
(B) Osber’s test
(C) Simmand’s test
(D) Charnley’s test

Answer: (A)

908. Post Poliomyelitis, a patient has grade II power in Gastrocnemius, grade III is Peroneus, grade IV in Tibialis Anterior. The deformity is
(A) Calcaneovalgus
(B) Equinovarus
(C) Calcaneovarus
(D) Genu valgus

Answer: (C)

909. In 3 year child with polio paralysis, tendon transfer operation is done at
(A) 2 months after the disease
(B) 2 years after the disease
(C) 6-12 months after the disease
(D) After skeleton maturation

Answer: (B)

910. You have treated the simple and undisplaced fracture of shaft of right tibia in a nine year girl with above knee plaster cast. Parents want to know the prognosis of union of the fractured limb which was affected by poliomyelitis four years ago. What is the best possible advice will you offer to the parents?
(A) Fracture will unite slowly
(B) Fracture will not unite
(C) Fracture will unite normally
(D) Fracture will unite on attaining puberty

Answer: (C)

911. Stance phase muscle among the following is
(A) Quadriceps
(B) Hamsring
(C) Gastrocnemius-soleus
(D) Tibialis anterior
(E) Peroneus longus

Answer: (A)

912. Which of the commonest fracture in children?
(A) Fracture clavicle
(B) Supracondylar fracture
(C) Green stick fracture of lower end of radius
(D) All of the above

Answer: (C)

913. Which statements pertaining to green stick fracture is except?
(A) Any fracture (#) in child
(B) Is generally incomplete
(C) # only in rickety children
(D) All of the above

Answer: (B)

914. Which of the following statements pertaining to green stick fracture is true?
(A) Any fracture in a child
(B) Fracture only in rickety children
(C) Only if there is no deformity
(D) All of the above

Answer: (C)

915. A 6-year-old child falls in right-sided forearm region and develops fracture in dorsal surface of mid region of radius. The best treatment is
(A) Antibiotics & sedative
(B) Bone plating and external fixation
(C) Slab with wait for bone remodeling
(D) Break the cortex other side and immobilization by POP

Answer: (D)

916. In children, all are true except:
(A) Dislocations are rare
(B) Comminuted fractures are common
(C) Thick periosteum
(D) Soft bones

Answer: (B)

917. In children best remodeling is seen in fracture with
(A) Angulation in diaphysis
(B) Angulation in metaphysis
(C) Rotation in diaphysis
(D) Rotation in metaphysic

Answer: (B)

918. An 8yrs child is brought by parents to the casualty with a spiral fracture of Femur and varying degree of Ecchymosis all over body. The Etiology is:
(A) Hit & run accident
(B) Battered Baby Syndrome
(C) Hockey Stick injury
(D) Osteogenicimperfecta

Answer: (B)

919. A one year old child presented with multiple fractures seen in various stages of healing. The most probable diagnosis in the case is:
(A) Scurvy
(B) Rickets
(C) Battered baby syndrome
(D) Fall from height

Answer: (C)

920. Type VI Rang’s injury induces
(A) Transverse fracture of metaphysic with longitudinal extension into physis.
(B) Open injury with loss of physis.
(C) Thurston Holland’s sign
(D) Perichondrial ring injury
(E) Intraarticular fractures

Answer: (D)

921. Perichondrial ring is
(A) Seen around foramen magnum
(B) Seen around epiphyseal plate
(C) More prominent in adults
(D) Shear strength increases with age
(E) Seen around all fetal diaphysis and joint cavity

Answer: (B)

922. An 8 year old boy with a history of fall from 10 feet height complains of pain in the right ankle. X-ray taken at that time are normal without any fracture line. But after 2 years, he developed a calcaneovalgus deformity. The diagnosis is
(A) Undiagnosed malunited fracture
(B) Avascular necrosis talus
(C) Tibial epiphyseal injury
(D) Ligamentous injury of ankle joint

Answer: (C)

923. Epiphyseal enlargement occurs in
(A) Paget’s disease
(B) Sheurmann’s disease
(C) Epiphyseal dysplasia
(D) Hemophilia

Answer: (D)

924. Epiphyseal dysgenesis is a feature of
(A) Hyperparathyroidism
(B) Hypoparathyroidism
(C) Hypothyroidism
(D) Hyperthyroidism

Answer: (C)

925. Phocomelia is best described as
(A) Defect in development of long bones
(B) Defect in development of flat bones
(C) Defect of intramembranous ossification
(D) Defect of cartilage replacement by bone

Answer: (A)

926. Phocomelia is
(A) Absence of short bones
(B) Complete absence of extremities
(C) Defects of long bones of limb
(D) Partial absence of extremities

Answer: (C)

927. Madelung’sdeoformity involves the following:
(A) Knee
(B) Wrist
(C) Hip
(D) Elbow

Answer: (B)

928. Cause of congenital pseudoarthrosis is
(A) Intrauterine fracture
(B) Neurofibromatosis
(C) Fibrous dysplasia
(D) Unknown

Answer: (B)

929. Musculoskeletal abnormalities in neurofibromatosis is
(A) Hypertrophy of limb
(B) Scoliosis
(C) Pseudo arthrosis
(D) All

Answer: (D)

930. The common features of Neurofibromatosis include al, except
(A) Optic glioma
(B) Dumbellneurofibroma
(C) Scoliosis
(D) Periventricular calcifications

Answer: (D)

931. Congenital pseudo arthrosis is seen in the following
(A) Hip joint
(B) Femur
(C) Radius – ulna
(D) Tibia-fibula

Answer: (D)

932. Pseudoarthrosis can be due to all except
(A) Congenital
(B) Post inflammatory
(C) Trauma
(D) None of the above

Answer: (B)

933. Pseudoarthrosis is seen in all of the following except
(A) Idiopathic
(B) Fracture
(C) Osteomylitis
(D) Neurofibromatosis

Answer: (C)

934. Pseudoarthrosis of tibia is best treated by
(A) Internal fixation
(B) Internal fixation and bone grafting
(C) Above knee POP cast
(D) Below knee POP cast

Answer: (B)

935. Causes of a painless limp since infancy includes
(A) Congenital dislocation of hip
(B) Infantile coax vara
(C) Poliomyelitis
(D) All of the above

Answer: (D)

936. All the following are causes of a painful limb except
(A) Slipped femoral epiphysis
(B) TB of the hip
(C) Perthes disease
(D) Infantile CoxaVara

Answer: (D)

937. Antalgic hip gait is related to which of the following
(A) Waddling gait
(B) Trendelenberg gait
(C) Painful hip gait
(D) Short leg gait

Answer: (C)

938. Congenital Coxavara is treated by
(A) Fixation by SP Nail
(B) Osteotomy
(C) Bone grafting
(D) Traction
(E) Splint

Answer: (B)

939. Coxavara is found in
(A) Perthe’s disease
(B) Tuberculosis
(C) Rickets
(D) Rheumatoid arthritis

Answer: (A)

940. Transient synovitis (toxic synovitis) of the hip is characterized by all of the following, except:
(A) May follow upper respiratory infection
(B) ESR and white blood cell counts are usually normal
(C) Ultrasound of the joint reveals widening of the joint space
(D) The hip is typically held in adduction and internal rotation

Answer: (D)

941. A 7year old boy with abrupt onset of pain in hip with hip held in abduction. Hemogram is normal. ESR is raised. What is the next line of management.
(A) Hospitalize and observe
(B) Ambulatory observation
(C) Intravenous antibiotics
(D) USG guided aspiration of hip

Answer: (D)

942. True about Perthe’s disease:
(A) Congenital in nature
(B) Occur in Children b/w 4-10 yr age group
(C) Involve knee joint
(D) Involve distal femur
(E) Involve head of femur

Answer: (B, E)

943. Pethe’s disease is
(A) Fracture of femoral shaft
(B) Osteochondritis of femoral epiphysis
(C) Infarction of femoral head
(D) Fracture dislocation of femoral neck.

Answer: (B)

944. A 10 years old male with pain in left hip & limping on examination restricted abduction & internal rotation, probable diagnosis is
(A) Septic arthritis of hip
(B) Tuberculosis arthritis of hip
(C) Cong dislocation of hip
(D) Perthes disease

Answer: (D)

945. Perthe’s disease is treated by:
(A) High dose of calcium with steroids
(B) Total hip replacement
(C) Supervised containment of femora head in acetabulum
(D) Relieving weight bearing

Answer: (C)

946. Legg-Calves-Perthes’ disease is
(A) Mild pyogenic arthritis
(B) Slipped femoral epiphysis
(C) Avascular necrosis of head of femur
(D) Low grade tuberculosis of hip

Answer: (C)

947. The commonest cause of limp in a child of seven years is:
(A) T.B. hip
(B) C.D.H
(C) Perthe’s disease
(D) Slipped upper femoral epiphysis

Answer: (C)

948. Legg-Calve-Perthes’ disease is
(A) Femoral epiphyseal osteochondritis
(B) Oteochondritis of tibial tubercle
(C) Slipped upper femoral epiphysis
(D) Spinal osteochondritis

Answer: (A)

949. The late radiographic changes seen in active Perthes’ disease are all except
(A) Narrow joint space
(B) A widened femoral neck
(C) An irregular density of the epiphysis
(D) An irregular epiphyseal line

Answer: (A)

950. Legg-Calve-Perthes disease is commonly seen in the age group of
(A) 1-3 years
(B) 3-10 years
(C) 10-20 years
(D) 20 year & above

Answer: (B)

951. Which one of the following is the investigation of choice for evaluation of suspectedPerthes’ disease?
(A) Plain X-ray
(B) Ultrasonography (US).
(C) Computed Tomography (CT).
(D) Magnetic Resonance Imaging (MRI).

Answer: (D)

952. A six year old boy presents to the emergency department with a painful limp. Clinical examination reveals tenderness in the femoral triangle and some limitation of hip movements. An X-ray was done which was normal. Which of the following should be the next course of action?
(A) Wait and Watch/Observation
(B) Ultrasonography
(C) Aspiration
(D) MRI Scan

Answer: (D)

953. Regarding Perth’s disease all of the following are correct except
(A) Osteonecroticosteochondritis
(B) Decreased bone age
(C) Male predominance
(D) Restricted abduction
(E) Age of onset has no affect on course

Answer: (E)

954. True about perthe’s disease is A/E:
(A) Avascular necrosis of femoral head
(B) Onset before 10 years of age
(C) Osteotomy is used for treatment
(D) Limb shortening
(E) Joint space obliterated

Answer: (E)

955. A 12 year old with rapid increase in weight and height over the past one year presents with difficulty in sitting cross legged and squatting. The knee would go into axilla every time the flexes her hip and knee. he most likely diagnosis is:
(A) Perths disease
(B) Slipped Capital Femoral Epiphysis
(C) Congenital Coxa Vera
(D) Synovitis of knee

Answer: (B)

956. An 11 year old 70 kg. child presents with limitation of abduction and internal rotation. There is tenderness in scarpas triangle. On flexing the hip the limb is abducted. The diagnosis is:
(A) Perthes disease
(B) slipped capital femoral epiphyses
(C) Observation hip
(D) Tuberculosis hip

Answer: (B)

957. 14 year old boy with 78 kg weight & hypothyroidism developed sudden onset of severe pain & tenderness on left hip as a result of minor fall. Most likely diagnosis is
(A) Fracture neck femur
(B) SCFE
(C) Perthes
(D) Hip Dislocation

Answer: (B)

958. Trethowan’s sign is seen in
(A) Perthe’s disease
(B) CDH
(C) SCFE
(D) Fracture neck femur

Answer: (C)

959. Ortolani’s test is done for
(A) Congenital dislocation of hip
(B) Traumatic dislocation of hip
(C) Rheumatoid arthritis
(D) Tuberculous arthritis

Answer: (A)

960. Which of the following movement is restricted in congenital dislocation of hip (CDH), in infant?
(A) Flexions
(B) Extension
(C) Rotation
(D) Full abduction in 90° flexion

Answer: (D)

961. CDH is due to
(A) Large acetabulum
(B) Rotation of femur
(C) Small neck femur
(D) Small femoral head

Answer: (D)

962. Which of the following test is useful in diagnosis of congenital dislocation of hip?
(A) Barlow’s test
(B) Thomas test
(C) Hibb’s test
(D) Laguerres test

Answer: (A)

963. Barlow’s test is done for testing:
(A) CDH in child
(B) CDH in infancy
(C) Femoral neck fracture
(D) Slipped femoral epiphysis

Answer: (B)

964. In a newborn child, abduction and internal rotation produces a click sound. It is
(A) Otorolani, sign
(B) Telescoping sign
(C) Lachman’s sign
(D) Mc. Murray’s sign

Answer: (A)

965. In congenital dislocation of hip, not true is
(A) Real shortening
(B) Telescoping
(C) Trendelenburg test
(D) Head of femur downwards
(E) Asymmetrical crease

Answer: (D)

966. Congenital dislocation of hip is more commonly seen in
(A) Caucasians
(B) Negroes
(C) Japanese
(D) Eskimos

Answer: (A)

967. Commonest deformity in congenital dislocation of hip
(A) Small head of femur
(B) Angle of torsion
(C) Decreased neck shaft angle
(D) Shallow acetabulum

Answer: (A, D)

968. True about CDH:
(A) Usually b/I
(B) Genu valgum
(C) Waddling gait
(D) Shenton’s line broken
(E) Spine lordosis present

Answer: (C, D, E)

969. All of the following statements are true about development dysplasia (DDH) of the hip, except:
(A) It is more common in females
(B) Oligohydraminos is associated with a higher risk of DDH
(C) The hourglass appearance of the capsule may prevent a successful closed reduction
(D) Twin pregnancy is a known risk factor

Answer: (D)

970. The treatment of Congenital dislocation of hip after childhood is
(A) Leave alone
(B) Surgery should be done if symptomatic
(C) Conservative measures to be taken
(D) None of the above

Answer: (B)

971. Von Rosen Splint in used in
(A) CTEV
(B) CDH
(C) Fracture shaft of femur
(D) Fracture tibia

Answer: (B)

972. Absolute indication of X-ray of pelvis in case of congenital dislocation of hip are all EXCEPT
(A) Positive family history
(B) Breach presentation
(C) Unstable hip
(D) Shortening of limb

Answer: (D)

973. True about B/L CDH:
(A) Exaggerated lordosis
(B) B/L genu valgum
(C) Wadding gait
(D) Shenton’s line broken
(E) Short stature

Answer: (A, C, D)

974. Seen in B/L congenital hip dislocation is/are:
(A) Short strature
(B) Wandering acetabulum
(C) Waddling gait
(D) Lumbar Iordosis

Answer: (A, D, C)

975. Congenital bilateral dislocation of hip shows
(A) Waddling gait
(B) Lordosis
(C) +veTrendelenburg test
(D) +ve non Rosen’s sign

Answer: (B, C)

976. Which of the following is seen in bilateral congenital dislocation of hip
(A) Waddling Gait
(B) Shenton’s line is broken
(C) Trendelenderg test positive
(D) Allis test positive

Answer: (A, B, C)

977. Traumatic dislocation of epiphyseal plate of femur occurs:
(A) Medially
(B) Laterally
(C) Posteriorly
(D) Rotationally
(E) Anteriorly

Answer: (B, E)

978. Commonest presentation of congenital dislocation of knee is
(A) Varus
(B) Valgus
(C) Flexion
(D) Hyperextension

Answer: (D)

979. True about Genu Valgum:
(A) Physiological condition
(B) Pathological condition
(C) Both physiological & pathological
(D) Same as bow-legs
(E) Known as knock knee

Answer: (A, B, C, E)

980. Most common cause of genuvalgum in children is
(A) Osteoarthritis
(B) Rickets
(C) Paget’s disease
(D) Rheumatoid arthritis

Answer: (B)

981. Pappu, 7 years old young boy, had fracture of lateral condyle of femur. He developed malunion as the fracture was not reduced anatomically. Malunion will produce:
(A) Genu vulgum
(B) Genu varum
(C) Genu recurvatum
(D) Dislocation of knee

Answer: (A)

982. Which statement is true regarding gene varum (bowleg):
(A) In infants, it may be considered normal
(B) Occurs due to epiphyseal dysplasia
(C) Seldom associated with tibial angulation
(D) Affects only tibia but never femur
(E) Does not require any treatment

Answer: (A, B, C, E)

983. Blount’s disease is:
(A) Genu valgus
(B) Tibia vera
(C) Flat foot
(D) Genu recurvatum

Answer: (B)

984. All are false regarding genu varum except
(A) Orthosis is a must only during weight
(B) Orthosis is recommended during day time
(C) Orthosis is recommended during night time
(D) Orthosis is recommended full time

Answer: (D)

985. Critical age of osteotomy for genu varus is
(A) 4 years
(B) 6 years
(C) 8 years
(D) 10 years

Answer: (A)

986. Blount’s disease is characterized by:
(A) Genu valgum
(B) Genu varum
(C) Genu recurvatum
(D) Meniscal injury

Answer: (B)

987. Blount’s disease is associated with all of the following except
(A) Genu varum
(B) Genu Recurvatum
(C) Internal Tibial Torsion
(D) External Tibial Torsion

Answer: (D)

988. An athletic teenage girl complains of anterior knee pain on climbing stairs and on getting u after prolonged sitting. Which of the following is the most likely diagnosis?
(A) Chondramalacia Patellae
(B) Plica syndrome
(C) Bipartite Patella
(D) Patellofemoralosteoardin

Answer: (A)

989. Nail patella syndrome is characterized by:
(A) Iliac horn
(B) Sacral horn
(C) Absent patella
(D) Knee deformity
(E) Dislocation of patella

Answer: (A, C, D, E)

990. Rocker bottom foot is seen in:
(A) Congenital vertical talus
(B) Excessive correction of Grice procedure
(C) Arthrogryposis
(D) Holding club foot in too long corrected position
(E) Fore dorsiflexion against equinusvarus

Answer: (A)

991. CTEV is caused by all except
(A) Neurological disorder
(B) Mostly idiopathic
(C) Spinafibida
(D) Cubitusvarus
(E) Arthrogryposis multiplex

Answer: (A, B, C, E)

992. Club foot is commoner among
(A) Males
(B) Binovular twins
(C) Females
(D) Uniovular twins

Answer: (A)

993. The most common congenital anomaly among the following is encountered in our country
(A) Congenital Pseudoarthrosis of Tibia
(B) Congenital dislocation of hip
(C) Congenital talipesequinovarus
(D) Multiple congenital contractures

Answer: (C)

994. The club foot characteristically involves
(A) Foot and ankle
(B) Foot, ankle and leg
(C) Foot only
(D) Foot, ankle, leg and knee joint

Answer: (B)

995. The most common deformity seen in club foot is
(A) Talipesequinovarus
(B) Equines
(C) Equino valgus
(D) Calcaneoequinovarus

Answer: (A)

996. Various congenital deformity of the ankle joint occurs at
(A) Calcanionavicular joint
(B) Talocalcaneal joint
(C) Talonavicular joint
(D) Tarsonavicular joint

Answer: (C)

997. Most important pathology in club foot is:
(A) Congenital talonavicular dislocation
(B) Tightening of Tendoachilles
(C) Calcaneal fracture
(D) Lateral derangement

Answer: (A)

998. Talipesequinovarus is
(A) Equinus, inversion, abduction
(B) Equines, inversion, adduction
(C) Equines, eversion, abduction
(D) Equines, eversion, abduction

Answer: (B)

999. Which of the following is true of Talipescalcaneovalgus?
(A) Dorsiflexed and inverted
(B) Dorsiflexed and everted
(C) More serious than CTEV
(D) Plantar flexed

Answer: (B)

1000. The crucial component of the club foot deformity is
(A) Contracture of Tendoachilles
(B) Under development of calf and personeal muscles
(C) Subluxation of the talo-navicular joint
(D) Lateral derangement of tarsal bones

Answer: (C)

1001. The world “Talipes” refers to
(A) Long feet with spidery toes
(B) Flat feet
(C) Club feet
(D) Hammer toes

Answer: (C)

1002. All are true statement regarding talipesequinovarus except:
(A) Cubitus valgus
(B) Inversion of the foot
(C) Abduction of the forefoot
(D) Arthrogryposis multiplex congenital causes it
(E) Treatment should start after 3 month

Answer: (A, C, E)

1003. A newborn child presents with inverted foot & dorsum of foot can not touch anterior tibia. The most probable diagnosis is:
(A) CTEV
(B) Congenital vertical talus
(C) Arthrogryposis Multiplex
(D) Flat foot

Answer: (A)

1004. The ideal treatment of bilateral idiopathic clubfoot in a newborn is:
(A) Manipulation by mother
(B) Manipulation and Dennis Brown splint
(C) Manipulation and casts
(D) Surgical release

Answer: (A)

1005. Club foot in a new born in is treated by
(A) Surgery
(B) Manipulation by the mother
(C) Dennis Brown splint
(D) Strapping

Answer: (B)

1006. Treatment of clubfoot in new born is
(A) Manipulation and corrective splint
(B) Corrective splint
(C) Nothing to be done for 6 months
(D) Surgical

Answer: (A)

1007. Early CTEV is treated by
(A) CTEV cast from 1st postnatal day
(B) Manipulation
(C) Manipulation & Strapping
(D) Operative repair as early as possible

Answer: (C)

1008. Treatment of club foot should begin:
(A) As soon as possible after birth
(B) 1 month after birth
(C) 1 year after birth
(D) None of the above

Answer: (A)

1009. Treatment of club foot in new born is:
(A) Observation till 6 months of age
(B) Corrective splint application
(C) Manipulation alone
(D) Manipulation & corrective splint

Answer: (D)

1010. Treatment of club foot in new born is
(A) Observation till 6 months of age
(B) Corrective splint application
(C) Manipulation and corrective splint
(D) Tendon releases operation

Answer: (C)

1011. In correction of clubfoot by manipulation, which deformity should be corrected first:
(A) Forefoot adduction
(B) Varus
(C) Upper end tibia
(D) Calcaneum

Answer: (A)

1012. Child 3¼ years is treated for CTEV by
(A) Triple arthrodesis
(B) Postero medial soft tissue release
(C) Lateral wedge resection
(D) Tendo Achilles lengthening and posterior capsulatomy

Answer: (B)

1013. Treatment for chronic cases of club foot is:
(A) Triple arthrodesis
(B) Dorso medial release
(C) Amputation
(D) None

Answer: (A)

1014. Club foot seen in a 15 year old could be treated successfully by a:
(A) Appropriate footwear
(B) Soft tissue operation
(C) Triple arthrodesis
(D) Quadrple fusion

Answer: (C)

1015. ‘Pseudoarthrosis’ in Triple fusion is seen at the joint of
(A) Calcaneocuboid
(B) Calcaneonavicular
(C) Naviculocuboid
(D) Talonavicular

Answer: (D)

1016. Triple fusion involves the following
(A) Subtalar, calcaneocuboid, talonavicular
(B) Subtalar, calceneum, tibiofibular
(C) Subtalar, tibiofibular, calcaneocuboid
(D) Tibiofibular, cuboid, calcaneum

Answer: (A)

1017. Triple arthrodesis involves:
(A) Calcaneocuboid, talonavicular and talocalcaneal
(B) Tibiotalar, calcaneocuboid and talonavicular
(C) Ankle joint, calcaneocuboid and talonavicular
(D) None of the above

Answer: (A)

1018. Triple arthrodesis is NOT done before-skeletal maturation because of:
(A) Shortening of foot
(B) Recurrence of deformity
(C) Inadequate fusion
(D) Complete correction not possible

Answer: (C)

1019. Rocker bottom foot is due to
(A) Over corrected club foot
(B) Poliomyelitis
(C) Vertical talus
(D) Deformities of spine

Answer: (A, C)

1020. In Klippel-Feil syndrome, the patient has all of the following clinical features except:
(A) Low hair line
(B) Bilateral Neck webbing.
(C) Bilateral shortness of sterno mastoid muscles
(D) Gross limitations of neck movements.

Answer: (C)

1021. The characteristic triad of Klippel-Feil syndrome includes all of the following, Except
(A) Short neck
(B) Low hair line
(C) Limited neck movements
(D) Elevated scapula

Answer: (D)

1022. Sprengel’s shoulder is due to deformity
(A) Scrapula
(B) Humerus
(C) Clavicle
(D) Vertebra

Answer: (C)

1023. Sprengel’s deformity of scapula
(A) Undescended/Elevated scapula
(B) Undescended neck of scapula
(C) Exostosis scapula
(D) None of the above

Answer: (A)

1024. Sprengel’s deformity is
(A) Absence of clavicle
(B) Acomioclavicular dislocation
(C) Congenital elevation of scapula
(D) Recurrent dislocation of shoulder

Answer: (C)

1025. Spina bifida occulta is
(A) Infection of the spine
(B) Traumatic fracture of the spine
(C) Congenital fusion of the body of the Vertebra
(D) Congenital on fusion of the spinal lamina

Answer: (D)

1026. All the following are true in infantile torticollis, EXCEPT:
(A) It arises before birth
(B) There is facial asymmetry
(C) Commonest form of wryneck
(D) Infarction of sterno-cleidomastoid muscle

Answer: (A)

1027. Which of the following is not true is case of congenital tortiocollis
(A) Seen only in cases of breech vaginal delivery
(B) It can disappear spontaneously
(C) It is also known as sternomastoidtumour
(D) Untreated, neglected cases can result in plagiocephaly.

Answer: (A)

1028. About congenital torticollis all are true except:
(A) Always associated with breech extraction
(B) Spontaneous resolution in most cases
(C) 2/3rd cases have palpable neck mass at birth
(D) Uncorrected cases develop plagiocephaly

Answer: (A)

1029. Progression of congenital scoliosis is least likely in which of the following vertebra anomalies
(A) Fully segmented Hemivertebra
(B) Wedge vertebra
(C) Block vertebra
(D) Unilateral unsegmented bar with Hemivertebra

Answer: (C)

1030. HariVerdhman, 9 yrs old child, presents with scoliosis, hairy tuft in the skin of back and neurological deficit. Plain X-rays reveal multiple vertebral anomalies & a vertical bony spur overlying lumbar spine on AP view. The most probable diagnosis is:
(A) Dorsal dermal sinus
(B) Diastometamyelia
(C) Tight filumterminale
(D) Caudal regression syndrome

Answer: (B)

1031. Vertebra plana seen in:
(A) Eosinophilia granuloma
(B) Trauma
(C) Paget’s disease
(D) Malignancy
(E) Ewing’s sarcoma

Answer: (A)

1032. RisserLocaliser cast is used in the management of:
(A) Kyphosis
(B) Spondylolysthesis
(C) Idiopathic scoliosis
(D) Lordosis

Answer: (C)

1033. Bone dysplasia is due to
(A) Faulty nutrition
(B) Faulty development
(C) Trauma
(D) Parathyroid tumour

Answer: (B)

1034. The following is false of Achondroplasia
(A) Autosomal dominant
(B) Mental retardation
(C) Due to gene mutation
(D) Shortening of limbs present

Answer: (B)

1035. Mode of inheritance of Achondroplasia is
(A) Autosomal dominant
(B) Autosomal recessive
(C) X-liked dominant
(D) X-linked recessive

Answer: (A)

1036. All are true of Achondroplasia except
(A) Autosomal dominant
(B) Shortening of limbs is present
(C) Usually due to gene mutation
(D) Mental retardation

Answer: (D)

1037. “Trident hand” seen in
(A) Achondroplasia
(B) Mucopolysaccharoidosis
(C) Diphysealachlasia
(D) Cleido-cranial dystosis

Answer: (A)

1038. A short statured patient brought to Orthopaedics OPD with a X ray showing flattened vertebral with beak. The probable diagnosis is
(A) Achondroplasia
(B) Ochronosis
(C) Eosinophilic granuloma
(D) Calve’s disease

Answer: (A)

1039. The features of Achondroplasia include all, except
(A) Defective head
(B) Mental retardation
(C) Autosomal recessive
(D) Familial

Answer: (C)

1040. Absent lateral 1/3rd of clavicle is seen in:
(A) Hyperparathyroidism
(B) Turner’s syndrome
(C) Fibrous dysplasia
(D) Cleidocranialdysostosis

Answer: (D)

1041. Cleidocranialdysostosis may show:
(A) Wide foramen magnum
(B) Absence of clavicles
(C) Coax vara
(D) All of the above

Answer: (D)

1042. A 9-year old child with high arched palate has shoulders meeting in front of his chest. he has
(A) Erb’s palsy
(B) Cleidocranialdysostosis
(C) Chondro-Osteodystrophy
(D) Cortical hyperostosis

Answer: (B)

1043. Absent clavicles are seen in
(A) Cleidocranialdysostosis
(B) Achondroplasia
(C) Moriquo’s disease
(D) Oliver’s disease

Answer: (A)

1044. The characteristics of Morquio’s disease include
(A) Spinal kyphosis
(B) Subnormal/normal intelligence
(C) Excessive excretion of keratosulphate in urine
(D) Dwarfism
(E) All of the above

Answer: (E)

1045. Osteogenesis imperfect is due to
(A) Defective mineralization of bone
(B) Fracture mobilization of bone
(C) Abnormal collagen defect
(D) Excessive osteoid formation

Answer: (C)

1046. Osteogenesis imperfect is due to the following
(A) Excessive Osteoblastic activity
(B) Defective Osteoid formation
(C) Defective osteoclast function
(D) Defective mineralisation of bone

Answer: (B)

1047. Osteogenesis imperfect is defect in
(A) Bone
(B) Calcification
(C) Cartilage
(D) Collagen

Answer: (D)

1048. Osteogenesisimperfecta
(A) Autosomal dominant (AD)
(B) Autosomal Recessive (AR)
(C) Both AD and AR
(D) Sex linked dominant
(E) None of the above

Answer: (C)

1049. Multiple bone fractures are seen in
(A) Rickets
(B) Osteomalacia
(C) Scurvy
(D) Osteogenesisimperfecta

Answer: (D)

1050. In which of the following condition bilateral symmetrical fractures occur?
(A) Rickets
(B) Osteopetrosis
(C) Osteogenesisimperfecta
(D) Fluorosis

Answer: (C)

1051. Multiple bone fracture in a new born is seen in
(A) Scurvy
(B) Syphilis
(C) Osteogenesisimperfecta
(D) Morquio’s syndrome

Answer: (C)

1052. Wormian bones are seen in
(A) Osteogenesisimperfecta
(B) Scheurmanns disease
(C) Paget’s disease
(D) Osteoclastoma

Answer: (A)

1053. Wormian bones are not noted in
(A) Fibrous dysplasia
(B) Osteogenesisimperfecta
(C) Cretinism
(D) Rickets

Answer: (B)

1054. Not true about Osteogenesisimperfecta
(A) Impaired healing of fracture
(B) Deafness
(C) Laxity of joints
(D) Fragile fracture

Answer: (A)

1055. Following are true about Osteogenesisimperfeta except
(A) Usually green stick fracture
(B) Generally do not unite without T/t
(C) Fracture frequency decreases during puberty
(D) Fracture seen intrauterine life and during parturition

Answer: (B)

1056. All are seen in osteogenesisimperfecta except
(A) Blue sclera
(B) Bilateral Hip dislocation
(C) Lax ligament
(D) Osteoporosis

Answer: (B)

1057. Not true about Osteogenesisimperfecta is
(A) Deafness
(B) Fractures
(C) Cataract
(D) Blue Sclera

Answer: (C)

1058. All are features of Osteogenesisimperfecta except:
(A) Blue sclera
(B) Multiple # s
(C) Cataract
(D) Hearing loss

Answer: (C)

1059. Osteogenesisimperfecta is characterized by:
(A) Blue sclera
(B) Also known as brittle bone disease
(C) Collagen defect
(D) A/W otosclerosis
(E) Autosomal dominant

Answer: (A, B, E)

1060. True about osteogenesisimperfecta:
(A) Autosomal dominant
(B) Known as marble bone disease
(C) Blue sclera
(D) Associated with otosclerosis
(E) Defect in collagen-I

Answer: (A, C, D, E)

1061. Parental determination of osteogenesisimperfecta is done by:
(A) Acid phosphatase
(B) Alkaline phosphatase
(C) Abnormal Pro-α chain
(D) None of these

Answer: (C)

1062. Brittle bones disease is
(A) Osteoporosis
(B) Osteopetrosis
(C) Osteogenesisimperfecta
(D) Osteomalacia

Answer: (C)

1063. Marble bone disease is
(A) Osteitisfragils
(B) Osteopetrosis
(C) Osteosarcoma
(D) Paget’s disease

Answer: (B)

1064. Albers schonberg disease is:
(A) Osteopetrosis
(B) Osteoporosis
(C) Osteochondritis
(D) Osteomalacia

Answer: (A)

1065. A 3 year male presented with progressive anemia, hepatosplenomegaly and osteomyelitis of jaw with pathological fracture, x-ray shows chalky white deposits on bone, probable diagnosis is
(A) Osteopetrosis
(B) Osteopoikilocytosis
(C) Alkaptonuria
(D) Myositis-ossificansprogressiva

Answer: (A)

1066. Raju, a 10 yrs old child, presents with predisposition to fractures, anemia, hepatosplenomegaly and a diffusely increased radiographic density of bones. The most likely diagnosis is
(A) Osteogenesisimperfecta
(B) Pyenodysotosis
(C) Myelofibrosis
(D) Osteopetrosis

Answer: (D)

1067. All of the following features are characteristic of Osteopetrosis except:
(A) Pancytopenia
(B) Abnormally slow rates of fracture healing
(C) Compression of cranial nerves
(D) Mandibular osteomyelitis

Answer: (B)

1068. Muscle most commonly affected by congenital absence is
(A) Pectoralis major
(B) Semi membranosus
(C) Teres minor
(D) Gluteus maximus

Answer: (A)

1069. Bone density on X-ray is/are seen in:
(A) Uric acid crystal deposition
(B) # and collapse of cancellous bone
(C) Increased thickening of trabeculae
(D) Periosteal reaction
(E) AVN

Answer: (B, C, D, E)

1070. Increased Bone density in X-ray seen in:
(A) Increased thickening of trabeculae
(B) Fracture & Collapse of cancellous bone
(C) Defective mineralization
(D) Myossitisossificans
(E) Relative disuse atrophy & surrounding bone response

Answer: (A, B)

1071. Increased bone density in X-ray seen in:
(A) Collapse cancellous bone
(B) Periosteal reaction
(C) Paget’s disease
(D) AVN
(E) Osteomyelitis

Answer: (A, B, C, D)

1072. There is considerable morbidity and mortality due to bone disease, more frequently amongst women. The most common cause of bone disease in India is:
(A) Nutritional Deficiency
(B) Steroid Induced osteoporosis
(C) Paget’s Disease
(D) Sarcoidosis

Answer: (A)

1073. Increased bone density occurs in:
(A) Cushing syndrome
(B) Hypoparathyroidism
(C) Fluorosis
(D) Hyperthyroidism

Answer: (C)

1074. Increased density in skull vault is seen in:
(A) Hyperparathyroidism
(B) Multiple myeloma
(C) Fluorosis
(D) Renal osteodystrophy

Answer: (C)

1075. What is the diagnostic radiological finding skeletal fluorosis
(A) Sclerosis of sacroiliac joint
(B) Interroseous membrane ossification
(C) Osteosclerosis of vertebral body
(D) Ossification of ligaments of knee joint

Answer: (B)

1076. The following is a radiological feature of Skeletal Fluorosis
(A) Intervertebral ligament calcification
(B) Sacral sclerosis
(C) Osteolytic lesion in pelvis
(D) None

Answer: (A)

1077. Manifestations of Fluorosis includes
(A) Stiffness of back ligaments
(B) Caries teeth (due to deficiency of fluorine)
(C) Genu valgum
(D) Dental changes
(E) Stiffnesss of bones and tendons (a, d)

Answer: (A, D, E)

1078. Jaw tumour is seen in
(A) Osteoporosis
(B) Osteomalacia
(C) Osteopetrosis
(D) Caffey’s disease

Answer: (D)

1079. Caffey’s disease is
(A) Renal osteodystrophy
(B) Infantile cortical hyperostosis
(C) Osteomyelitis of jaw in children
(D) Chronic osteomyelitis in children

Answer: (B)

1080. Caffey’s disease occurs in
(A) Infants below 6 months
(B) Above years
(C) Above 10-20 years
(D) 20-40 years

Answer: (A)

1081. Osteomyelitis of Jaw is seen in
(A) Osteomalacia
(B) Osteopoikilosis
(C) Osteoporosis
(D) Caffey’s disease

Answer: (D)

1082. “Rugger Jersey Spine” is seen in:
(A) Fluorosis
(B) Achondroplasia
(C) Renal osteodystrophy
(D) Marfan’s syndrome

Answer: (C)

1083. Rugger jersey spine in CRF is due to:
(A) Osteomalacia
(B) Trauma
(C) Hyperparathyroidism
(D) Aluminiumosteodystrophy
(E) Osteopetrosis

Answer: (C)

1084. Metabolic bone disease is caused by excess intake of which vitamin:
(A) A
(B) B
(C) C
(D) D
(E) E

Answer: (A)

1085. Hypervitaminosis of which of the following will cause bony abnormalities:
(A) Vit. A
(B) Vit. D
(C) Vit. C
(D) Vit. E
(E) Vit. K

Answer: (A, B, D)

1086. A Bald child with swollen abdomen, hyperosteous bones with mental retardation has
(A) Hypervitaminosis A
(B) Hypervitaminosis D
(C) Down’s syndrome
(D) Tuberous sclerosis

Answer: (B)

1087. Action of Vitamin D is that it
(A) Stimulates bone marrow
(B) Increases calcium loss
(C) Stimulates absorption of calcium
(D) Stimulates osteoclasts.

Answer: (C)

1088. Osteomalacia is associated with:
(A) Decreases in osteoid volume
(B) Decrease in osteoid surface
(C) Increase in osteoid maturation time
(D) Increase in mineral apposition rate

Answer: (C)

1089. Drug induced Osteomalacia is known to be associated with the use of:
(A) Steroids
(B) Heparin
(C) Phenytoin
(D) Gentamycin

Answer: (C)

1090. The cause of osteomalacia is
(A) Phenytoin
(B) Malabsorption
(C) Vincristine
(D) Cushing’s syndrome

Answer: (A, B)

1091. The commonest cause of osteomalacia in our country is:
(A) Repeated systemic infections
(B) Idiopathic steatorrhoea
(C) Dieteic deficiency of vitamin D & calcium
(D) Poor protein intake
(E) Chronic renal disease

Answer: (C)

1092. Osteomalacia causes include
(A) Phenytoin
(B) Malabsorption
(C) Indoor stay
(D) Vincristine
(E) Cushing’s syndrome

Answer: (A, B, C)

1093. Osteomalacia/Rickerts may be seen in A/E
(A) Neurofibroma
(B) Osteoblastoma
(C) Hemangiopericytoma
(D) Ewings sarcoma

Answer: (D)

1094. Rickets is seen in A/E
(A) Dietary deficiency
(B) Hypocalcemia
(C) Malabsorption
(D) Fanconi’s syndrome

Answer: (B)

1095. Basic pathological defect in rickets is
(A) Decreased osteoblastic activity
(B) Non functional osteoclast
(C) Defective osteoclasticresoprtion of uncalcified osteoid and cartilage
(D) Defective proliferation of physis.

Answer: (C)

1096. Bony lesion in Rickets is due to
(A) Increased parathoromone leading to osteoclastic activity
(B) Reduced dietary calcium absorption
(C) Increased calcium excretion
(D) None

Answer: (B)

1097. Decreased mineralization of Epiphyseal plate in a growing child is seen in:
(A) Rickets
(B) Osteomalacia
(C) Scurvy
(D) Osteoporosis

Answer: (A)

1098. Defective mineralization of proximal zone of cartilage and epiphysis is a feature of:
(A) Rickets
(B) Scurvy
(C) Osteomalacia
(D) Syphilis

Answer: (A)

1099. Rickets of vitamin D deficiency is associated with
(A) Subperiostealhaemorrhage
(B) Ricketrirosary at costochondral junction produced by subluxation of sternal plate
(C) Apt to develop during growth in low birth weight infants
(D) Inorganic serum phosphate concentration 4.5-6.5 mg/dl

Answer: (B)

1100. In nutrional rickets changes occurs at the:
(A) Metaphysis
(B) Epiphysis
(C) Diaphysis
(D) All of these

Answer: (A)

1101. Rickets in infancy is characterized by the following except:
(A) Craniotabes
(B) Rachitic rosary
(C) Wide open fontanelles
(D) Bow legs

Answer: (D)

1102. Biochemical changes seen in osteomalacia:
(A) Serum Ca2+↑
(B) Serum Ca2+↓
(C) Serum Normal ca2+
(D) ↑ ALP
(E) ↓ 1, 25 (OH) 2D

Answer: (B, C, E)

1103. Biochemical abnormality seen in nutritional rickets:
(A) Hyperphosphaturia
(B) Hyperphosphatemia
(C) ALP
(D) PTH
(E) Hypocalcemia

Answer: (E)

1104. Note seen in Rickets
(A) Cupping of ends of bones
(B) Irritability
(C) Craniotabes
(D) Decreased alkaline phosphatase

Answer: (D)

1105. The metabolic indicator of Rickets is
(A) Low serum Ca
(B) Low PO4
(C) ↑Alkaline phosphatase
(D) ↑ Urinary hydroxyl proline

Answer: (C)

1106. Which of the following is a persistant biochemical marker of rickets:
(A) S. Ca++
(B) S. Alkaline phosphate
(C) S. Acid phosphate
(D) S. phosphate

Answer: (B)

1107. A diet deficient in calcium will most commonly result in
(A) Osteomalacia
(B) Rickets
(C) Osteoporosis
(D) Osteitisfibrosa

Answer: (A)

1108. Pseudo fracture is seen in:
(A) Pseudo parathyoridism
(B) Hypoparathyroidism
(C) Osteomalacia
(D) Osteoporosis

Answer: (C)

1109. Which is true about Osteomalacia?
(A) Alkaline phosphatase is raised
(B) Serum phosphate is low
(C) Serum calcium is low or normal
(D) All of the above

Answer: (D)

1110. Alkaline phosphatase is elevated in all, EXCEPT:
(A) Rickets
(B) Osteomalacia
(C) Hypoparathyroidism
(D) Hypophosphatemia

Answer: (C)

1111. Increased alkaline phosphate is seen in:
(A) Multiple myeloma
(B) Primary hyperparathyroidism
(C) Chronic renal failure
(D) Osteoporosis

Answer: (B, C)

1112. During a routine check up, a 67-year-old man is found to have a level of serum alkaline phosphatase three times the upper limit of normal. Serum calcium and phosphorus concentrations and liver function test results are normal. He is asymptomatic. The most likely diagnosis is:
(A) Metastatic bone disease
(B) Primary hyperparathyroidism
(C) Paget’s disease of bone
(D) Osteomalacia

Answer: (C)

1113. A patient Shweta with raised serum alkaline phosphatase and raised parathormone level along with low calcium and low phosphate level is likely to have:
(A) Primary hyperparathyroidism
(B) Paget’s disease
(C) Osteoporosis
(D) Vitamin D deficiency

Answer: (D)

1114. A 65 year old female presented with # of T12 vertebra with increased alkaline phosphatase and paratharmone level along with low calcium and low phosphate level, diagnosis is
(A) Osteoporosis
(B) Paget’s disease
(C) Primary hyperparathyroidism
(D) Vitamin D deficiency

Answer: (D)

1115. The characteristic finding in osteomalacia is
(A) ↓ P
(B) ↓ Ca
(C) ↓ Ca& ↑ P
(D) ↓ Ca& ↓ P

Answer: (D)

1116. All are true regarding Osteomalacia
(A) ↑ Serum Ca++
(B) ↑ Serum Phosphate
(C) Normal Serum calcium
(D) Osteosclerotic lesions
(E) Pseudofracture seen

Answer: (C, E)

1117. Milkman’s fracture is?
(A) Pseudofracture in adults
(B) Fracture of Clavicle in children
(C) Fracture humerus
(D) Fracture first Metacarpal

Answer: (A)

1118. A young patient presents with enlargement of costochondral junction and with the white line of Fraenkel at the metaphysis. The diagnosis is
(A) Scurvy
(B) Rickets
(C) Hyperparathyroidism
(D) Osteomalacia

Answer: (A)

1119. In scurvy there is deficit in
(A) Collagen
(B) Fibroblasts
(C) Elastic fibres
(D) All of the above

Answer: (A)

1120. Vitamin required for collagen is
(A) Vitamin A
(B) Vitamin C
(C) Vitamin D
(D) Vitamin E

Answer: (B)

1121. Vitamin C deficiency leads to
(A) Defective mineralization
(B) Defective Osteoid formation
(C) Normal collagen and Bone matrix
(D) X-ray shows normal evidence

Answer: (B)

1122. Scurvy affects bone by
(A) Decreasing mineralization of bone
(B) Less formation of osteoid matrix
(C) Loss of calcium from mineralized matrix
(D) Affection of zone of provisional calcification only

Answer: (B)

1123. Ring sign is seen in:
(A) Rickets
(B) Barlow’s disease
(C) Scurvy
(D) Fibrous dysplasia

Answer: (C)

1124. Which is/are X-ray finding of infantile scurvy?
(A) A dense line between metaphysis and epiphyseal cartilage
(B) A clear band of rarefaction on the diaphysial side
(C) Both of these
(D) None of these

Answer: (C)

1125. Metaphyseal fracture is commonly seen in
(A) Osteogenesisimperfecta
(B) Scurvy
(C) Rickets
(D) None

Answer: (B)

1126. Soft tissue calcification occurs in all except:
(A) Hyper parathyroidism
(B) Scleroderma
(C) Hyperthyroidism
(D) Hyper vitaminosis D

Answer: (C)

1127. Osteitisfibrosacystica is due to
(A) Paget’s disease of bone
(B) Polyostotic fibrous dysplasia
(C) Parathyroid adenoma
(D) All of the above

Answer: (C)

1128. Most common cause of primary hyperparathyroidism is:
(A) Solitary adenoma
(B) Chief cell hyperplasia
(C) Multiple adenoma
(D) Werner’s syndrome

Answer: (A)

1129. Which is not a feature of Osteitisfibrosacystica?
(A) Fracture
(B) Tetany
(C) Increased serum calcium
(D) Increased alkaline phosphatase

Answer: (B)

1130. Sub-Periosteal erosions of middle phalanges at the radial aspect in characteristic of
(A) Hypothyroidism
(B) Hyperthyroidism
(C) Hypoparathyroidism
(D) Hyperparathyroidism

Answer: (D)

1131. Subperiosteal erosion is seen in
(A) Scurvy
(B) Hyperparathyroidism
(C) Hypoparathyroidism
(D) Rickets

Answer: (B)

1132. In hyperparathyroidism bone resorption is seen in all except
(A) Jaws
(B) Metacarpals
(C) Ribs
(D) End of long bones

Answer: (A)

1133. Soft tissue calcification with hypercalcaemia is abserved in:
(A) Hyperparathyroidism
(B) Alkaptonuria
(C) Gout
(D) Cushing’s disease

Answer: (A)

1134. Tufting of the terminal phalanges is seen in:
(A) Hypoparathyroidism
(B) Hyperparathyroidism
(C) Hyperthyroidism
(D) Hypothyroidism

Answer: (B)

1135. Resorption of the terminal phalanx is not seen in:
(A) Hyperparathyroidism
(B) Reiter’s syndrome
(C) Scleroderma
(D) Psoriasis

Answer: (A)

1136. Erosion of ungula tufts in terminal phalanges of hand may be due to:
(A) Rheumatoid arthritis
(B) Gout
(C) Hyperparathyroidism
(D) Psoriasis

Answer: (D)

1137. Brown Tumor is seen in
(A) Hypothyroidism
(B) Hyperthyroidism
(C) Hypoparathyroidism
(D) Hyperparathyroidism

Answer: (D)

1138. Sclerotic lesion on the bone is seen in all except
(A) Osteitisfibrosa
(B) Osteopetrosis
(C) Melorheostosis
(D) Caffey’s disease

Answer: (A)

1139. Osteitisfibrosacystica is seen in:
(A) Hyperparathyroidism
(B) Hypoparathyroidism
(C) Hypothyroidism
(D) Hyperthyroidism

Answer: (A)

1140. Absence of lamina dura in the alveolus occurs in
(A) Rickets
(B) Osteomalacia
(C) Deficiency of vitamin C
(D) Hyperparathyroidism

Answer: (D)

1141. Sclerosis of bone is seen in all except:
(A) Secondaries from prostate
(B) Fluorosis
(C) Hyperparathyroidism
(D) Osteopetrosis

Answer: (C)

1142. Brown tumors are seen in:
(A) Hyperparathyroidism
(B) Pigmented villonodularsynovitis
(C) Osteomalacia
(D) Neurofibromatosis

Answer: (A)

1143. A 50 year old man presented with multiple pathological fractures. His serum calcium was 11.5 mg/dl and phosphate was 2.5 mg/dl. Alkaline phosphatase was 940 I.U/dl. The most probable diagnosis is
(A) Osteoporosis
(B) Osteomalacia
(C) Multiple Myeloma
(D) Hyperparathyroidism

Answer: (D)

1144. Treatment of choice for Paget’s disease of the bone is
(A) Vitamin D
(B) Immobilization of the limb
(C) Surgical treatment
(D) Calcitonin

Answer: (D)

1145. Paget’s disease also known as:
(A) Osteitisdeformans
(B) Osteitisfibrosacystica
(C) Osteochonoritis
(D) Osteomalacia
(E) Marble bone disease

Answer: (A)

1146. Paget’s disease of bone commonly affects:
(A) Skull
(B) Vertebra
(C) Pelvis
(D) Femur
(E) Humerus

Answer: (A, B, C, D)

1147. Paget’s disease of one commonly affects:
(A) Skull
(B) Vertebra
(C) Pelvis
(D) Phalanges
(E) Humerus

Answer: (A, B, C)

1148. Pain in pagets disease is relieved best by
(A) Simple analgesics
(B) Narcotic analgesics
(C) Radiation
(D) Calcitonin

Answer: (D)

1149. The complications of Paget’s disease is:
(A) Osteogenic sarcoma
(B) Deafness
(C) Heart failure
(D) All of the above

Answer: (D)

1150. Deafness in case of Paget’s disease is due to
(A) Thickened cranium
(B) Narrowing of foramina of skull
(C) Brain compression
(D) Otosclerosis

Answer: (B)

1151. Drug therapy of Paget’s disease (OsteitisDeformans) include all except:
(A) Alendronate
(B) Etidronate
(C) Calcitonin
(D) Plicamycin

Answer: (D)

1152. Following are features of Paget’s disease except
(A) Deformity of bones
(B) Secondary osteosarcoma
(C) Lowered serum alkaline phosphatase
(D) Surgery

Answer: (C)

1153. The histopathologic feature of Paget’s disease includes
(A) Simultaneous osteoblastic activity at places
(B) Osteoclasticresorption
(C) Replacement of bone marrow by fibro vascular tissue
(D) All of the above

Answer: (D)

1154. Which of the following is a primary defect in Paget’s disease?
(A) Osteoblast
(B) Osteoclast
(C) Osteocyte
(D) Fibroblast

Answer: (B)

1155. All of the following statements regarding Paget’s disease are correct except
(A) Females are affected more than males
(B) If frequently leads to Osteogenic sarcoma
(C) Serum alkaline phosphates level is ↑ed
(D) All called as Osteitisdeformans

Answer: (A)

1156. Calcitonin is used in the treatment of
(A) Post menopausal osteoporosis
(B) Malignant hyper calcemia
(C) Paget’s disease
(D) All of the above

Answer: (C)

1157. A 67 year old man on biochemical analysis found to have three fold rise of level of serum alkaline phosphatase that upper limit of norm value during a routine checkup but serum calcium and phosphorous concentration and liver function test results and normal. He is asymptomatic. The probable cause is
(A) Multiple myeloma
(B) Paget’s disease of bone
(C) Primary hyperparathyroidism
(D) Osteomalacia

Answer: (B)

1158. Calcitonin is used in
(A) Paget’s disease
(B) Hyperparathyroidism
(C) Osteosclerosis
(D) Vitamin D intoxication

Answer: (A)

1159. A patient presents with bone pains and on investigating calcium and phosphorus levels in serum were found to be normal except with elevation of serum alkaline phosphatase. The probable diagnosis is
(A) Osteomalacia
(B) Paget’s disease
(C) Osteoporosis
(D) Hyperparathyroidism

Answer: (B)

1160. Which is not true about Paget’s disease of bone?
(A) Often there is no symptom
(B) Alkaline phosphates is low
(C) Thickening of bone due to widening of cortex
(D) All of these

Answer: (B)

1161. Which of the following is not true regarding Paget’s disease of bone?
(A) Serum alkaline phosphatase is low
(B) In frequently leads to Osteogenic sarcoma
(C) Males are affected more than females
(D) Involved area shows rapid formation and resorption of bone

Answer: (A)

1162. Paget’s disease
(A) Is painless
(B) Begins as an osteoblastic
(C) Is frequently complicated by osteogenic sarcoma
(D) Does not affect bones singly (e.g. Only the clavicle)

Answer: (C)

1163. 60 years old male with bony abnormally at upper tibia associated with sensori neural hearing loss. On laboratory examination serum alkaline phosphatase levels are (440 mU/1) elevated and serum Ca++ and PO4− cotton wool spots in x-ray skull. Diagnosis is:
(A) Fibrous dysplesia
(B) Paget disease
(C) Osteosclerotic metastasis
(D) Osteoporosis

Answer: (B)

1164. True about paget’s disease:
(A) Common in young girl 10-16yr of age
(B) ↑ALP
(C) Associated with hypercalcemia& hypophosphatemia
(D) More common in female
(E) Associated with chondrosarcoma

Answer: (B, E)

1165. Which of the following is seen in osteoporosis:
(A) Low Ca, high PO4, high alkaline phosphatase
(B) Low Ca, low PO4, low alkaline phosphatase
(C) Normal Ca, normal PO4, normal alkaline phosphatase
(D) Low Ca, low PO4, normal alkaline phosphatase

Answer: (C)

1166. True about osteoporosis:
(A) ↓with age
(B) Due to diet deficiency in Ca
(C) Commonly presented with #
(D) Cod-fish vertebra seen

Answer: (C, D)

1167. Osteoporosis is seen in all the following except
(A) Thyrotoxicosis
(B) Rheumatoid arthritis
(C) Hypoparathyroidism
(D) Steroid therapy

Answer: (C)

1168. All are causes of Osteoporosis, except:
(A) Thyrotoxicosis
(B) Hypothyroidism
(C) Chronic heparin therapy
(D) Old age

Answer: (B)

1169. Osteoporosis may be seen in all except:
(A) Hyperparathyroidism
(B) Hypoparathyroidism
(C) Thyrotoxicosis
(D) Heparin administration

Answer: (B)

1170. Osteoporosis is caused by all except:
(A) Sarcoidosis
(B) Old age
(C) Hypoparathyroidism
(D) Steroid therapy

Answer: (C)

1171. Osteoporosis is seen in:
(A) Thyrotoxicosis
(B) Cushing’s disease
(C) Menopause
(D) All of the above

Answer: (D)

1172. All of the following conditions may be responsible for osteoporosis except
(A) Steroid therapy
(B) Prolonged weightlessness in spaceship
(C) Hyperparathyroidism
(D) Hypoparathyroidism

Answer: (D)

1173. Osteoporosis is seen in following except
(A) Old age
(B) Prolonged Heparin administration
(C) Hyperthyroidism
(D) Steroid intake

Answer: (C)

1174. The most common manifestation of osteoporosis is:
(A) Compression fracture of the spine
(B) Asymptomatic, detected incidentally by low serum Calcium
(C) Bowing of legs
(D) Loss of weight

Answer: (A)

1175. The most common site of fracture of the following bone in senile osteoporosis is
(A) Vertebra
(B) Neck of femur
(C) Radius
(D) Shaft of femur

Answer: (A)

1176. Osteoporosis is characterized most commonly by
(A) Fracture vertebra
(B) Backache
(C) Bowing of legs
(D) Abdominal pain

Answer: (A)

1177. All of the following fractures are associated with osteoporosis except:
(A) Colles’ fracture
(B) Fracture of clavicle
(C) Vertebral fracture
(D) Fracture neck of femur

Answer: (B)

1178. Not a complication of menopause:
(A) Fracture spine
(B) Colles fracture
(C) Fracture neck of femur
(D) Supra condylar fracture humerus

Answer: (D)

1179. Risk factors for osteoporosis
(A) Late menopause
(B) COPD
(C) Obesity
(D) Smoking
(E) OCP

Answer: (D, B)

1180. Bone Density is best studied by:
(A) CT scan
(B) DEXA scan
(C) MRI scan
(D) Bone scan

Answer: (B)

1181. Drug of choice for senile osteoporosis is
(A) Estrogens
(B) Androgens
(C) Calcitonin
(D) Ethidronate

Answer: (A)

1182. Which of the following is useful in the treatment of Osteoporosis?
(A) Potassium
(B) Sodium chloride
(C) Calcium
(D) Phosphate

Answer: (C)

1183. A 65-year-old female patient presents with
(A) Calcium + Oestrogen + Progesterone
(B) Oestrogen + Progesterone
(C) Calcium + Ostrogen
(D) Calcium

Answer: (A)

1184. Calcium content of bone is increased in
(A) Prolonged immobilization
(B) Glucocorticoid’s administration
(C) Hyperparathyroidism
(D) Estrogen supplementation in post menopausal women

Answer: (D)

1185. Treatment of postmenopausal osteoporosis:
(A) Tamoxifen
(B) Progesterone
(C) Estrogen
(D) Alendronate
(E) Calcitonin

Answer: (C, D, E)

1186. Rx of postmenopausal osteoporosis:
(A) Calcitonin
(B) Alendronate
(C) Progesterone
(D) Tamoxifience
(E) Androgen

Answer: (A, B)

1187. Bone density is decrease in which of the following
(A) Osteoporosis
(B) AVN of bone
(C) Osteopetrosis
(D) Fracture and collapse of cancellous bone
(E) SLE

Answer: (A)

1188. Denosumab- a monoclonal antibody against RANKL receptor is used in treatment of
(A) Rheumatoid arthritis
(B) Osteoporosis
(C) Osteoarthritis
(D) SLE

Answer: (B)

1189. All of the following agents decreased bone resorption in osteoporosis, except:
(A) A lendronate
(B) Etidronate
(C) Strontium
(D) Terparatide

Answer: (D)

1190. Osteoporosis is characterized by:
(A) Increased serum alkaline phosphatase
(B) Decreased bone density
(C) Wasting of muscles
(D) Looser’s zone seen
(E) Decreased serum Ca2+

Answer: (B)

1191. Traumatic fracture showing avascular necrosis:
(A) Femoral neck
(B) Surgical neck of humerus
(C) Body of talus
(D) Cuboid

Answer: (A)

1192. Avascular necrosis seen in
(A) # surgical neck of humerus
(B) # neck of scapula
(C) # neck of talus
(D) # neck of femur
(E) # neck f scaphoid/radius

Answer: (C, D)

1193. Posttraumatic avascular necrosis occur in fracture of
(A) Neck femur
(B) Surgical neck humerus
(C) Neck talus
(D) Waist scaphoid
(E) Neck radius

Answer: (A, C)

1194. Most common site of avascular necrosis amongst the following is:
(A) Medial candyle of femur #
(B) Talus #
(C) Olecranon #
(D) Head of the radius #

Answer: (B)

1195. A vascular necrosis can be a possible sequelae of fracture of all of the following bones, except:
(A) Femur neck
(B) Scaphoid
(C) Talus
(D) Calanuem

Answer: (D)

1196. Avascular necrosis of head of the femur is most common in
(A) Subcapital #
(B) Basal #
(C) Fracture intertochantric
(D) Trans cervical fracture

Answer: (A)

1197. The most common site of fracture neck of femur that causes avascular necrosis is
(A) Subcapital
(B) Intertrochanteric
(C) Transcervical
(D) Basal

Answer: (A)

1198. Avascular necrosis of head of femur can occur in
(A) Sickle cell anaemia
(B) Caisson’s disease
(C) Intracapsular fracture neck
(D) Trochanteric fracture

Answer: (A, B, C)

1199. Osteonecrosis of femoral head seen in:
(A) Sickle cell disease
(B) Gaucer’s disease
(C) Polycythemia
(D) Hyperparathyroidism
(E) Fracture neck of femur

Answer: (A, B, C, E)

1200. Avascular necrosis of femur seen in:
(A) Steroid therapy
(B) Gaucher’s disease
(C) Fracture
(D) Pancreatitis

Answer: (ALL)

1201. Pathological changes in Caison’s disease is due to:
(A) N¬2
(B) O2
(C) CO2
(D) CO

Answer: (A)

1202. Avascular necrosis is seen in:
(A) Sickle cell disease
(B) Thalassemia
(C) Polychthemia
(D) Hyperparathyroidism

Answer: (A, C, B)

1203. Aseptic necrosis can occur in:
(A) Perthes disease
(B) Slipped capital femoral epiphysis
(C) Steroid administration
(D) Caisson’s disease
(E) Gaucher’s disease

Answer: (ALL)

1204. A 50-year-old man sustained posterior dislocation of lest hip in an accident. Dislocation was reduced after 3 days. He started complaining of pain in left hip after 6 months. X-ray of the pelvis were normal. The most relevant investigation at this stage will be-
(A) CRP Levels in blood
(B) Ultrasonography of hip
(C) Arthrography of hip
(D) MRI of hip

Answer: (D)

1205. A patient is using oral steroids for a period of 5 years and patient complaints of pain in the both hip regions. Which one of the following is a diagnostic modality of conformation of diagnosis?
(A) Plain X ray
(B) CT scan
(C) MRI
(D) Isotope Bone scan

Answer: (C)

1206. A woman of 45, a known cause of pemphigus vulgaris on a reglar treatment with controlled primary disease presented with pain in the right hip and knee. Examination revealed no limb length discrepancy but the patient has tenderness in the Scarpa’s triangle and limitation of adduction and internal rotation of the right hip joint as compared to the other side. The most probable diagnosis is
(A) Stress fracture of neck of femur
(B) Avascular necrosis of femoral head
(C) Perthe’s disease
(D) Transient synovitis of hip

Answer: (B)

1207. A thirty one year old male with Nephrotic syndrome complains of pain in right hip joint of 2 months duration. The movements at the h ip are free but painful terminally. The most likely diagnosis is
(A) Tuberculosis of hip
(B) Avascular necrosis of femoral head
(C) Chondrolysis of hip
(D) Pathological fracture of femoral neck

Answer: (B)

1208. A patient of nephrotic syndrome taking steroids for 6 years presented with a limp gait and of limitation of abduction and internal rotation. He most probably has:
(A) Renal osteodystrophy
(B) Avascular necrosis of hip
(C) Septic arthritis
(D) Osteomylitis of hip jt

Answer: (B)

1209. A 45 year old was given steroids after renal transplant. After 2 years he had difficulty in walking and pain in both hips. Which one of the following most likely cause?
(A) Primary Osteoarthritis.
(B) Avascular necrosis.
(C) Tuberculosis.
(D) Aluminum toxicity.

Answer: (B)

1210. Which joint is commonly involved in Osteochondritis Dissecans
(A) Ankle joint
(B) Knee joint
(C) Wrist joint
(D) Elbow joint

Answer: (B)

1211. Most common site of osteochondritis dessicans
(A) Lateral part of the medial femoral condyle.
(B) Medial part of the medial femoral condyle.
(C) Lateral part of the lateral femoral condyle.
(D) Medial part of the lateral femoral condyle.

Answer: (A)

1212. Kinebock’s disease is due to avascular necrosis of:
(A) Femoral neck
(B) Medial cuneiform bone
(C) Lunate bone
(D) Scaphoid bone

Answer: (C)

1213. What is Kineboch’s disease
(A) Osteochondritis of lunate
(B) Osteochondritis of neck of femur
(C) Osteochondritis of navicular
(D) Osteochondritis of tibial condyle

Answer: (A)

1214. Kinebock’s disease is due to avascular necrosis of:
(A) Femoral neck
(B) Medial cuneiform bone
(C) Lunate bone
(D) Scaphoid bone
(E) Medial condyle of tibia

Answer: (C)

1215. Which of the following is true about manifestation of Kienbocks disease:
(A) Constant tenderness is present
(B) Seen in old age
(C) Restriction of joint movement is late sequale
(D) Tenderness in lunate bone
(E) Ulna is shortened

Answer: (C, D, E)

1216. Osteocondritis is Osgood Schlatter disease affect which bone?
(A) Capitulum’s
(B) Matatarsal
(C) Navicular
(D) Tibial tuberosity

Answer: (D)

1217. Osgood Shalttller disease
(A) Involve the knee joint
(B) Pelvis
(C) Wrist joint.
(D) Cervical spine

Answer: (A)

1218. Osgood-Shclatter disease is
(A) A traction injury of femoral epiphysis
(B) A traction injury of posterior epiphysis of oscalcis
(C) A traction injury of vertebral epiphysis
(D) A traction injury of the tibial tubercle of the tibial epiphysis
(E) None of the above is true

Answer: (D)

1219. Osgood Schlatters disease affects:
(A) Proximal (upper) tibia
(B) Lower tibia
(C) Distal femur
(D) Proximal (upper) femur
(E) Patella

Answer: (A)

1220. Osteochondiritis is not seen in – disease
(A) Pelligrini – Stieda
(B) Panner’s
(C) Clave’s
(D) Kohler’s

Answer: (A)

1221. Islene’s disease is osteochondritis of
(A) 2nd Metacarpal
(B) 5th Metacarpal
(C) 2nd Metatarsal
(D) 5th Metatarsal

Answer: (D)

1222. Frieberg’s Osteochondritis is
(A) 2nd Metatarsal head
(B) 5th metatarsal head
(C) 2nd Metatarsal base
(D) 5th Metatarsal base

Answer: (A)

1223. Proximal interphalangeal, distal interphalangeal&Ist carpometacarpal joint involvement and sparing of wrist is a feature of
(A) Rheumatoid arthritis
(B) Pseudogout
(C) Psoraticarthropathy
(D) Osteoarthritis

Answer: (D)

1224. Least common site to be involved in osteoarthritis amongst the following is:
(A) Hip joint
(B) Knee joint
(C) Carpometacarpal joint of thumb
(D) Metacarpophalangeal joint

Answer: (D)

1225. In patient with osteoarthritis of knee joint, atrophy occur most commonly in which muscle
(A) Qudriceps only
(B) Hamstrings only
(C) Bot (a) and (b)
(D) Gastrocnemius

Answer: (A)

1226. A 35 year old male patient develops involvement of proximal and distal interphalangeal joints and Istcanpometacarpal joints with sparing of wrist and metacarophalangeal joint. The Diagnosis is
(A) Osteoarthritis
(B) Psoriatic arthropathy
(C) Rheumatoid arthritis
(D) Pseudogout

Answer: (A)

1227. Osteo arthritis following is not a predisposing factor
(A) Diabetes mellitus
(B) Defective joint position
(C) Weight bearing joints
(D) Incongruity of articular surfaces
(E) Old age

Answer: (A)

1228. In Fracture acetabulum, late complication is
(A) Osteoarthritis
(B) Tardy sciatic nerve palsy
(C) Recurrent Dislocation
(D) None of the above

Answer: (A)

1229. True about osteoarthritis except:
(A) Commonly found in adult before 50 yrs.
(B) Heberden’sodules are found
(C) Single joint involvement
(D) Lower limb deformity is seen
(E) Ankylosis occurs

Answer: (A)

1230. Osteoarthritis all are true except:
(A) Occurs after 50 years of age
(B) Single joint may be affected
(C) Heberden’s node present
(D) Lower limb deformity seen
(E) Ankylosis occurs

Answer: (NONE)

1231. Treatment of Osteo arthritis include all except:
(A) Graded muscle exercises
(B) Replacement of articular surface
(C) Correction of deformities
(D) Increase the weight bearing by the affected joint.
(E) Rest to the joint in acute phase

Answer: (D)

1232. Severe disability in primary osteoarthritis of hip is best managed by:
(A) Arthrodesis
(B) Arthroplasty
(C) Mc Murray’s osteotomy
(D) Intra-articular hydrocortisone and physiotherapy

Answer: (B)

1233. The most common site of primary osteroarthrosis is
(A) Hip joint
(B) Knee joint
(C) Ankle joint
(D) Shoulder joint

Answer: (B)

1234. Osteoarthrosis does not affect
(A) Knee joint
(B) Hip joint
(C) Interphalangeal joint
(D) Metacarpophalangeal joint
(E) Should joint

Answer: (D)

1235. Osteoarthritis commonly involves:
(A) Proximal IP joint
(B) Distal IP joint
(C) Ist carpometacarpal joint
(D) Wrist joint
(E) Distal radio-ulnar joint

Answer: (A, B, C)

1236. Involvement of PIP jt, DIP jt and the carpo-mcpjt of base of thumb with sparing the wrist is seen in:
(A) Rheumatoid arthritis
(B) Osteoarthritis
(C) Psoriatic arthritis
(D) Pseudogout

Answer: (B)

1237. Heberden’sArthropathy affects:
(A) Lumbar spine.
(B) Symmetrically large joints.
(C) Sacroiliac joints.
(D) Distal interphalangeal joints.

Answer: (D)

1238. 40 years patient having arthritis of PIP and DIP along with carpometacarpal joint of thumb and sparing of wrist and metacarpophalangeal joint, most likely diagnosis is
(A) Rheumatoid arthritis
(B) Osteoarthritis
(C) Psoriatic arthritis
(D) Pseudogout

Answer: (B)

1239. Earliest radiological sing of the Osteoarthritis is
(A) Narrowing of joint space
(B) Osteophyte formation
(C) Cystic lesion on cancellous bone
(D) Sclerosis in subchondral bone

Answer: (A)

1240. Which is characteristic F/O osteo arthritis on X-ray?
(A) Spurring
(B) Subchondral sclerosis
(C) Dimunition of cartilage space
(D) All of these

Answer: (D)

1241. Osteoarthritis is associated with all of the following except:
(A) Decrease joint space
(B) Subchondral sclerosis
(C) Osteophyte formation
(D) Ca++ deposition in joint space

Answer: (D)

1242. Osteoarthritis of Knee involves –
(A) Hamstring
(B) Quadriceps
(C) Both (a) and (b)
(D) Gastrocneimius

Answer: (B)

1243. The cause of rheumatoid arthritis is
(A) Familial
(B) Immunological
(C) Infective
(D) Traumatic

Answer: (B)

1244. The following is involved in Rheumatoid arthritis:
(A) Synovial fluid
(B) Synovial membrane
(C) Cartilage
(D) Subchondral bone

Answer: (B)

1245. Which of the following is inflammatory arthritis
(A) Rheumatoid arthritis
(B) Osteoarthritis
(C) Osteochondritis
(D) All of the above

Answer: (A)

1246. In rheumatoid arthritis, pathology starts in the
(A) Articular cartilage
(B) Capsule
(C) Synovium
(D) Muscles

Answer: (C)

1247. Inflammatory arthritis is/are all except
(A) Osteoarthritis
(B) Rheumatoid arthritis
(C) Psoriatic arthritis
(D) Gout arthritis
(E) Lymes arthritis

Answer: (A)

1248. All of the following are seen in inflammatory polyarthritis, except
(A) New bone formation
(B) Erythema
(C) Increased ER
(D) Morning stiffness more than one hour

Answer: (A)

1249. Distal interphalangeal joint is not involved in:
(A) Rheumatoid arthritis
(B) Psoriatic arthritis
(C) Multicentrichisteocytosis
(D) Neuropathic arthropalsty

Answer: (A)

1250. Which joint is spared in Rheumatoid arthritis
(A) MP joints of hand
(B) DIP joints of finger
(C) PIP joints of finger
(D) Atlanto-axial joint

Answer: (B)

1251. The most common arthritis that affects the wrist is
(A) Osteoarthritis
(B) Tuberculous arthritis
(C) Rheumatoid arthritis
(D) Gout

Answer: (C)

1252. Swan neck deformity is a feature of:
(A) Syphilitic arthritis
(B) Gouty arthritis
(C) Rheumatoid arthritis
(D) Osteo arthritis
(E) TB or Psoriatic arthritis

Answer: (C)

1253. Swan-neck deformity is
(A) Flexion of Metacarpophalangeal joint and extension at interphalangeal joint
(B) Extension at Proximal interphalangeal joint and flexion at Distalinterphalangeal joint
(C) Flexion at proximal interphalangeal joint and extension at distal interphalangeal joint
(D) Extension at Metacarpophalangeal joint and flexion interphalangeal joint

Answer: (B)

1254. Hyperextension of PIP joints and hyperflexion of DIP joint is known as:
(A) Trigger finger
(B) Boutoiner’sdeformtiy
(C) Swan neck deformity
(D) Mallet finger

Answer: (B)

1255. “Wind-sweep deformity” is seen in
(A) Ankylosing spondylitis
(B) Scurvy
(C) Rheumatoid arthritis
(D) Rickets

Answer: (C, D)

1256. A burn patient develop claw hand. Joint affected will be:
(A) Flexion at proximal Interphalangeal joint
(B) Flexion at Distal Interphalangeal joint
(C) Thumb Abduction
(D) Flexion at Metacarpophalangeal joint
(E) Extension at Metacarpophalangeal joint

Answer: (A, B, E)

1257. Butonniere deformity occur due to:
(A) Flexion of Proximal interphalangeal joint
(B) Flexion at Distal interphalangeal joint
(C) Extension at Ditalinterphalangeal joint
(D) Extension at Metarcarpophalangeal joint
(E) Flexion at Metarcarpophalangeal joint

Answer: (A, C)

1258. What is pathonomic feature of rheumatoid arthritis?
(A) Rheumatoid factor
(B) Rheumatoid nodule
(C) Morning stiffness
(D) Ulnar drift of fingers

Answer: (B)

1259. RA factor is used mainly
(A) Screening patients for Rheumatoid arthritis
(B) Predicting multisystem disease
(C) Predicting severity of disease
(D) Monitoring treatment

Answer: (B, C)

1260. Rheumatoid factor in rheumatoid arthritis is important because:
(A) RA factor is associated with bad prognosis
(B) Absent RA factor rules out the diagnosis of Rheumatoid arthritis
(C) It is very common in childhood Rheumatoid arthritis
(D) It correlates with disease activity

Answer: (A)

1261. Type of anemia seen in Rheumatoid arthritis is:
(A) Microcytic hypochromic anaemia
(B) Macrocytic hyperchromicanaemia
(C) Normocytic hypochromic anaemia
(D) Normocytic normochromic anaemia

Answer: (D)

1262. Joint not involved in Rheumatoid arthritis according to 1987 modified ARA criteria?
(A) Knee
(B) Ankle
(C) Tarsometatarsal
(D) Metatarsophalangeal

Answer: (C)

1263. All the following are true about Rheumatoid arthritis except:
(A) Positive for Anti-IgG antibody
(B) Juxta-articular osteoporosis
(C) Morning stiffness
(D) C Reactive protein indicates better prognosis

Answer: (D)

1264. Para-articular erosion are most commonly seen in:
(A) Osteoarthritis
(B) Rheumatoid arthritis
(C) Gout
(D) Acute suppurative arthritis

Answer: (B)

1265. Pain in small joints in an elderly lady is most likely due to:
(A) Rheumatic arthritis
(B) Rheumatoid arthritis
(C) Psoriatic arthritis
(D) Reiter’s disease

Answer: (B)

1266. All are features of Rheumatoid arthritis, EXCEPT:
(A) Osteosclerosis of joint
(B) Soft tissue swelling
(C) Narrowing of joint space
(D) Periarticular osteoporosis

Answer: (A)

1267. True regarding felty’s syndrome is all, EXCEPT:
(A) Splennomegaly
(B) Rheumatoid arthritis
(C) Neutropenia
(D) Nephropathy

Answer: (D)

1268. Which of the following is true regarding Rheumatoid arthritis:
(A) Typically involves small and large joints symmetrically but spares the cervical spine
(B) Causes pleural effusion with low sugar
(C) Pulmonary nodules are absent
(D) Enthesopathy prominent

Answer: (B)

1269. All of the following are well known features of Rheumatoid arthritis except
(A) Bilateral hip arthritis
(B) Erosion of distal interphalangeal joints
(C) Pleural effusion
(D) Hypocomoplementemia

Answer: (B)

1270. Which is true regarding rheumatoid arthritis
(A) Small and large joins are affected mostly
(B) Younger females are affected more commonly
(C) Diagnosed only if Rheumatoid factor is positive
(D) Life expectancy is unchanged
(E) Hepatosplenomegaly is common

Answer: (A)

1271. Extra articular C/F Rheumatoid Arthritis:
(A) Splenomegaly
(B) Fever
(C) Subcutaneous nodules
(D) Pleural effusion

Answer: (ALL)

1272. In R.A. (Rheumatoid Arthritis) true is:
(A) M.C. in young
(B) R. F. is diagnostic
(C) Small & big joints involved
(D) Felty syndrome associated

Answer: (C, D)

1273. True about Rheumatoid arthritis:
(A) Involves peripheral joints more commonly than axial joints
(B) Does not decrease life expectancy
(C) A/wvasculitis
(D) Adrenal hyperplasia

Answer: (A, C)

1274. A woman presented with right shoulder pin and Rheumatoid factor test came as negative, but the pain responded to the Prednisolone therapy. The diagnosis include
(A) Osteopetrosis
(B) Seronegative Rheumatoid arthritis
(C) Polymylagia
(D) Polymylagiarheumtica

Answer: (B)

1275. Still’s disease is
(A) Post traumatic bone formation in the lateral ligament of the knee
(B) Spastic diplegia
(C) Rheumatoid arthritis is child hood
(D) Rheumatoid arthritis in the elderly.

Answer: (C)

1276. Treatment of Rheumatoid arthritis include all except
(A) Give rest to the joints
(B) Correction of deformities
(C) Synovectomy
(D) Exercises
(E) Immuno suppressive drugs

Answer: (D)

1277. The following are rheumatoid disease modifying drugs except:
(A) Chloroquine
(B) Gold
(C) Penicillamine
(D) BAL

Answer: (D)

1278. Gold salts can be used is
(A) Ankylosing spondylitis
(B) Rheumatoid arthritis
(C) Osteoarthritis
(D) Behcet’s syndrome

Answer: (B)

1279. Indication of systemic steroids in rheumatoid arthritis is:
(A) Mononeuritis multiplex
(B) Carpul tunnel syndrome
(C) Presence of deformities
(D) Articular cartilage involvement

Answer: (A)

1280. Multiple loose bodies are seen in:
(A) Osteochondritisdessicans
(B) Synovial chondrocalcinosis
(C) Osteoarthritis
(D) Rheumatoid arthritis
(E) Osteogenesisimperfecta

Answer: (A, B, C)

1281. The following is the commonest cause of loose body in a joint
(A) Osteoarthritis
(B) Osteochondral fracture
(C) Synovial chondromatosis
(D) Ostochondritisdissecans

Answer: (D)

1282. One of the following is to be considered as differential diagnosis for foreign body in plain X-ray of knee joint:
(A) Fabella
(B) Calcified bursa
(C) Patella
(D) Chondromatosis

Answer: (D)

1283. True about osteochondromatosis:
(A) Malignant
(B) Manifest at adult
(C) Seen at 4-6 years after disease
(D) Common in women

Answer: (B, C, D)

1284. Disease where distal Interphalangeal joint is characteristically involved
(A) Psoriatic arthritis
(B) Rheumatoid
(C) SLE
(D) Gout

Answer: (A)

1285. Terminal interphalangeal joints of hands are commonly involved in:
(A) Psoriatic arthropathy
(B) Rheumatoid arthritis
(C) Still diseases
(D) Ankylosing spondylitis

Answer: (A)

1286. A 35-year old male develops involvements of PIP, DIP and metacarpopahalangeal joints with sparing of wrist and carpometacarpal joints. The probable diagnosis is:
(A) Psoriatic arthopathy
(B) Osteo arthritis
(C) Rheumatoid arthritis
(D) Pseudo gout

Answer: (A)

1287. In Psoriatic arthropathy, treatment of choice is
(A) Methotrexate
(B) PUVA therapy
(C) Corticosteroids
(D) Indomethacin

Answer: (A)

1288. All are true regarding psoriatic arthritis except:
(A) Arthritis mutilans
(B) DIP involvement
(C) Ankylosis of small joints
(D) Sacroilitis
(E) Lengthening of digit k/a telescoping

Answer: (E)

1289. In Reiter’s disease, untrue is
(A) Conjunctivitis
(B) Ulcer on palm & soles
(C) Interstitial lung disease
(D) After sexual contact

Answer: (C)

1290. Which one of the following is not associated with HLA B 27?
(A) Ankylosing spondylitis
(B) Reiter’s syndrome
(C) Sjogren’s syndrome
(D) Psoriatic arthritis

Answer: (C)

1291. Ankylosing spondylitis in associated with:
(A) HLA-B27
(B) HLA-B-8
(C) HLA-DW4/DR4
(D) HLA-DR3

Answer: (A)

1292. All are seronegative ( spondyloepiphyseal) arthritis with ocular manifestations, except-
(A) Ankylospondilitis
(B) Ritter’s disease
(C) Rheumatoid arthritis
(D) Psoriatic arthritis

Answer: (C)

1293. All the following disease are associated with HLA B-27 & Uveitis, except:
(A) Bechcet’s syndrome
(B) Psoriasis
(C) Ankylosing Spondylitis
(D) Reiter’s syndrome

Answer: (A)

1294. HLA B27 is associated with:
(A) Rheumatoid arthritis
(B) Ankylosing spondylitis
(C) Rheumatic arthritis
(D) Gouty arthritis

Answer: (B)

1295. Positivity of HLA B 27 in Ankylosing spondylitis
(A) 10%
(B) 96%
(C) 78%
(D) 100%

Answer: (B)

1296. Bamboo spine is seen in:
(A) Tuberculosis
(B) Rheumatoid arthritis
(C) Ochronosis
(D) Ankylosingspondylosis

Answer: (D)

1297. The early X-ray changes of Ankylosing spondylitis would be
(A) Disc space narrowing
(B) Anterior osteophyte formation
(C) Sacroiliac joint erosion
(D) Facetal joint ankylosis

Answer: (C)

1298. Ankylosing spondylitis the joint least commonly involved is
(A) Elbow
(B) Sacroiliac
(C) Ankle
(D) Spinal

Answer: (A)

1299. A young male presents with joint pains and backache. X-ray of spine shows evidence of sacroilitis. The most likely diagnosis is
(A) Rheumatoid arthritis
(B) Ankylosing spondylitis
(C) Polyarticular juvenile arthritis
(D) Psoriatic arthropahty

Answer: (B)

1300. One of the following is true regarding Ankylosing spondylitis
(A) Common is females
(B) Sacrolitis is common
(C) Anti DNA and antinuclear antibodies are present
(D) Symmetrical peripheral arthritis

Answer: (B)

1301. Which is not involved in Ankylosing spondylitis
(A) Knee & Ankle
(B) Sacroilac jt.
(C) Wrist & elbow
(D) Spine

Answer: (C)

1302. Which is true about Ankylosing spondylitis?
(A) Sacroliac joints unusually involved
(B) Peripheral joint is rarely involved
(C) Common manifestation if females
(D) Iritis is seen more frequently

Answer: (D, B)

1303. One of the following is not associated with Ankylosing spondylitis
(A) Pain more in the early morning period
(B) Pain is relieved in lying down position
(C) Morning stiffness more than 3 months
(D) May be associated with uveitis

Answer: (B)

1304. Which of the following agent has/have proven to be a value in treatment of Ankylosing spondylitis?
(A) Phenylbutazone
(B) Intramuscular gold
(C) Naproxen
(D) Penicillamine

Answer: (A)

1305. The treatment of choice for Ankylosing spondylitis is
(A) Phenylbutazone
(B) Radiotherapy
(C) Steroid
(D) All of the above

Answer: (A)

1306. Which joint is commonly affected in Ankylosing spondylitis?
(A) Ankle
(B) Knee
(C) Shoulder
(D) All of the above

Answer: (C)

1307. Enthesopathy is commonly found in:
(A) Rheumatoid arthritis
(B) Rheumatic fever
(C) Ankylosing spondylitis
(D) Osteoarthritis

Answer: (C)

1308. All are true regarding Ankylosing spondylitis except
(A) sacroilitis
(B) HLA-B27
(C) Iridocyclitis
(D) More common in females
(E) Spinal fractures

Answer: (D)

1309. Earliest investigation for diagnosis of Ankylosing spondylitis:
(A) MRI STIR sequence
(B) Bone scan
(C) CT scan
(D) X-ray
(E) USG

Answer: (A)

1310. A 65 yrs old man had H/o of back pain sine 3 months. ESR is raised. He also has marked stiffness on examination & mile restriction of chest movements. On X-ray, syndesmophytes are present in vertebrae. Diagnosis is:
(A) Ankylosing spondylitis
(B) Degenerative osteoarthritis of spine
(C) Ankylosinghyperosteosis
(D) Lumbar canal stenosis

Answer: (A)

1311. Bleeding into joint cavities is not common in
(A) Christmas disease
(B) Hemophilia
(C) ITP
(D) None of above

Answer: (C)

1312. All are features of haemophilic knee joint, EXCEPT:
(A) Juxta articular osteosclerosis
(B) Sub chondral cyst formation
(C) Widening of intercondylarnoth
(D) Squarring of patella

Answer: (A)

1313. Most common site for pseudotumour like growth inhaemophilicarthropathy is
(A) Quadriceps femoris
(B) Hamstring muscle
(C) Gastrocnemius
(D) Ilioopsoas

Answer: (A)

1314. Arthroscopy is contraindicated in
(A) Chronic joint disease
(B) Loose bodies
(C) Haemophilia
(D) Meniscal tear

Answer: (C)

1315. True about treatment of haemarthosis
(A) Aspiration
(B) POP
(C) Traction
(D) Compression bandage.

Answer: (A, B, D)

1316. Neuropathic joint may arise in
(A) Syringomyelia
(B) Tabesdorsalis
(C) Leprosy
(D) All of the above

Answer: (D)

1317. Following are the recognized causes of Charcot’s (Neuro-pathic) joint, except for:
(A) Peripheral neuritis
(B) Syringomyelia
(C) Tabesdorsalis
(D) Hysterical joint

Answer: (D)

1318. Most common charcot’s joints involved in diabetes mellitus are those of:
(A) Shoulder
(B) Ankle
(C) Knee
(D) Foot

Answer: (D)

1319. Clutton’s joints are:
(A) Syphilitic joints
(B) End stage Tuberculous joints
(C) Associated with trauma
(D) Usually painful

Answer: (A)

1320. The most common cause of Neuropathic joints is:
(A) Leprosy
(B) Diabetes
(C) Rheumatoid arthritis
(D) Syphilis

Answer: (B)

1321. Neuropathic joint of foot and ankle is most commonly due to
(A) CTEV
(B) Hansen’s disease
(C) Polio
(D) Mycetoma

Answer: (B)

1322. Charcot’s joints have all of the following characteristics except
(A) Copious effusion in the joint
(B) Painful limitation of joint movements
(C) Hypermobility of joint
(D) Osteophyte formation

Answer: (B)

1323. A 60-year old man with diabetes mellitus presents with painless, swollen right ankle joint. Radiographs of the ankle show destroyed joint with large number of loose bodies. The most probable diagnosis is:
(A) Charcot’s joint
(B) Clutton’s joint
(C) Osteoarthritis
(D) Rheumatoid arthritis

Answer: (A)

1324. In a patient suffering from tabesdorsallscharcot’s joint occurs most commonly at
(A) Elbow
(B) Tarsometatarsal
(C) Wrist
(D) Knee

Answer: (D)

1325. Dissociative sensory loss occurs in
(A) Tuberculosis of spine
(B) Disc prolapse
(C) Tabesdorsalis
(D) Syringomyelia

Answer: (D)

1326. The joint commonly involved in syphilitic arthritis is
(A) Hip
(B) Shoulder
(C) Wrist
(D) Knee

Answer: (D)

1327. Tertiary syphilitic arthritis most frequently involve
(A) Spine
(B) Hip
(C) Ankle
(D) Knee

Answer: (D)

1328. Joint least affected by Neuropathy
(A) Shoulder
(B) Hip
(C) Wrist
(D) Elbow

Answer: (A)

1329. Neuropathic joints of ankle and foot are most commonly caused by:
(A) Polio
(B) Club foot
(C) Mycetoma
(D) Hansen’s disease

Answer: (D)

1330. Painless effusion in joints in Congenital Syphilis are called
(A) Clutton’s joint
(B) Charcot’s joint
(C) Barton’s joint
(D) Chronic osteomyelitis

Answer: (A)

1331. False about Charcot’s joint in diabetes mellitus is
(A) Limitation of movements with bracing
(B) Arthrodesis
(C) Total ankle replacement
(D) Arthrocentesis

Answer: (C)

1332. Erosion of bone is seen with all of the following except
(A) Gout
(B) SLE
(C) Psoriasis
(D) Rheumatoid arthritis

Answer: (B)

1333. Which is NOT characterized by bony lesion
(A) Gout
(B) SLE
(C) Psoriasis
(D) Rheumatoid arthritis

Answer: (B)

1334. Erosive arthritis is not noted in
(A) Amyloidosis
(B) Hyperparathyroidism
(C) Psoriasis
(D) Sickle-cell disorder

Answer: (B)

1335. Deforming Polyarthritis is associated with all of the following except
(A) Rheumatoid arthritis
(B) Psoriatic arthritis
(C) Behcet’s syndrome
(D) Ankylosing spondylitis

Answer: (C)

1336. Synovial fluid of low viscosity seen in:
(A) Gout
(B) Septic arthritis
(C) TB
(D) Osteoarthritis
(E) Rheumatoid arthritis

Answer: (A, B, C, E)

1337. False statement about gout:
(A) Can lead to chronic arthritis
(B) Negatively bifringent crystals are present
(C) More common in premenopausal females
(D) Fbeuxostat is used in treatment
(E) Serum uric acid level may be normal during attack

Answer: (C)

1338. Which of the following is not affected in gout?
(A) Muscle
(B) Skin
(C) Cartilage
(D) Tendon
(E) Bursa

Answer: (A)

1339. Tophi of gout is fund in:
(A) Articular cartilage
(B) Synovium
(C) Skin
(D) Muscle
(E) Joint capsule

Answer: (A, B, C, E)

1340. Clinical features of Gout are all except
(A) Chronic patient with renal complication allopurinol is drug of choice
(B) +vebirefringent crystal in the Synovial fluid
(C) Hyperuricemia
(D) Mostly affects smaller joint

Answer: (B)

1341. A 35-year-old businessmen presents suddenly with severe pain, swelling and redness in left big toe in early morning. Most likely diagnosis is
(A) Rheumatoid arthritis
(B) Gouty arthritis
(C) Pseudogout
(D) Septic arthritis

Answer: (B)

1342. All of the followings drugs are used in an attack of Gout except
(A) Aspirin
(B) Idomethacin
(C) Colchicines
(D) Phenlbutazone

Answer: (A)

1343. Sudden attack of acute pain in great toe is due to
(A) Rheumatoid arthritis
(B) Gout
(C) Rheumatic fever
(D) Psoriatic arthritis

Answer: (B)

1344. “Gouty arthritis” usually involves first:
(A) Ankle
(B) Great toe
(C) Thumb
(D) Shoulder joint

Answer: (B)

1345. MC joint involved in Gout:
(A) Knee
(B) Hip
(C) MP joint of the big toe
(D) MP joint of thumb.

Answer: (C)

1346. In a patient of gouty arthritis best investigation to detect it
(A) Serum uric acid
(B) Uric acid in urine
(C) Urate crystal in synovial fluid
(D) Serum calcium level

Answer: (C)

1347. Biochemical marker in early gouty arthritis is:
(A) S. uric acid
(B) Increased urinary uric crystal
(C) Synovial fluid urate crystal
(D) Alkaline phosphate

Answer: (C)

1348. What is not rue about gout:
(A) Abrupt increase in serum urate levels is more common a cause for acute gout than an abrupt fall in urate levels
(B) Patient may be asymptomatic with high serum uric acid for years
(C) Development of arthritis correlates with level of serum uric acid
(D) Uric acid crystals are best seen by polarizing light microscope

Answer: (A)

1349. Specific test for gout is:
(A) Raised serum uric acid level
(B) Raised uric acid in synovial fluid of joint
(C) Raised urea level
(D) Raised urease enzyme level

Answer: (B)

1350. All are true about gouty arthritis except
(A) Arthritis is manifested after long attack of hyperuricemia
(B) There is good correlation between level of uric acid and severity of gouty arthritis
(C) Synovial analysis is diagnostic
(D) Allopurinol is treatment of choice in acute gout

Answer: (B)

1351. Periosteal reaction is not common in
(A) Syphilis
(B) Gout
(C) Osteomyelitis
(D) Tuberculousdactylitis

Answer: (B)

1352. In a gouty arthritis, the characteristic X-ray findings includes
(A) Osteoporosis
(B) Erosion of joint
(C) Soft tissue calcification
(D) Narrowing of joint space

Answer: (B)

1353. A person has severe pain & swelling in great toe. True statements are:
(A) Allopurinol used in acute control of gout
(B) Colchicine acts slowly
(C) Colchicine cause gastrointestinal disturbances
(D) High serum uric acid level may be not present
(E) Joint Fluid aspiration is done for investigation

Answer: (C, D, E)

1354. Characteristic crystals in pseudogout are:
(A) Calcium pyrophosphate
(B) Sodium monourate
(C) Potassium monourate
(D) Sodium pyrophosphate

Answer: (A)

1355. How to differentiate gout with pseudogout:
(A) Large joint involvement
(B) Birefringent (Particles) crystals
(C) Serum uric acid normal
(D) Associated with hyperparathyroidism
(E) Pain is very intense

Answer: (B, E)

1356. Calcification of menisci is seen in:
(A) Hyperparathyroidism
(B) Pseudogout
(C) Renal Osteodystrophy
(D) Acromegaly

Answer: (B)

1357. Soft tissue calcification around the knee is seen in
(A) Scurvy
(B) Scleroderma
(C) Hyperparathyroidism
(D) Pseudogout

Answer: (D)

1358. The most commonly involved joint in pseudo gout
(A) Knee
(B) Great toe
(C) Hip
(D) Elbow

Answer: (A)

1359. Subluxation of atlanto-occipital joint is seen in all except
(A) Gout
(B) Parpharyngeal abscess
(C) Rheumatoid arthritis
(D) Ankylosing spondylitis

Answer: (A)

1360. Chondrocalcinosis is seen in:
(A) Ochronosis
(B) Hypoparathyroidism
(C) Rickets
(D) Hypervitaminosis D

Answer: (A)

1361. The earliest manifestation of Alkaptonuria is
(A) Ankylosis of lumbodorsal spine
(B) Ochronotic arthritis
(C) Prostatic calculi
(D) Pigmentation of tympanic membrane
(E) All of the above

Answer: (B)

1362. A lady presents with right knee swelling. Aspiration was done in which CPPD crystals were obtained. Next best investigation is:
(A) ANA
(B) RF
(C) CPK
(D) TSH

Answer: (D)

1363. In a patient with gouty arthritis, synovial fluid aspiration will show:
(A) Monosodium Urate crystals
(B) Calcium Pyrophosphate crystals
(C) Mononuclear Leucocytosis
(D) Polymorphonuclear Leukocytosis

Answer: (A)

1364. X-ray of a young man shows heterotopic calcification around bilateral knee joints. Next investigation would be
(A) Serum phosphate
(B) Serum calcium
(C) Serum PTH
(D) Serum Alkaline phosphatase

Answer: (A)

1365. All of the following are associated with CV junction anomalies except:
(A) Rheumatoid Arthritis
(B) AnkyloisngSpondylosis
(C) Odonotoiddysgenesis
(D) Basilar degeneration

Answer: (B)

1366. All of the following statements about synovial fluid are true, except:
(A) Secreted primarily by type A synovial cells
(B) Follows Non-Newtonian Fluid Kinetics
(C) Contains Hyaluronic acid
(D) Viscosity is variable

Answer: (A)

1367. Which of the following statements about changes in articular crtilage with aging is not true:
(A) Total proteoglycan content is decreased
(B) Synthesis of proteoglycans is decreased
(C) Enzymtic degradation of proteoglycans is increased
(D) Total water content of cartilage is decrease

Answer: (C)

1368. Heterotropic calcification is seen in all except
(A) Ankylosing spondylitis
(B) Gouty arthritis
(C) Forrerstier’s disease
(D) Traumatic paraplegia
(E) After revision of surgery

Answer: (NONE)

1369. Common vertebral level of spondylolisthesis is
(A) L4-5
(B) L3-4
(C) L1-2
(D) T12-L1
(E) L5-S1

Answer: (E)

1370. In spondylolisthesis, there is fracture of vertebra in
(A) Spinous process
(B) Neural arch pars inter articularis
(C) Tansverse process
(D) Body

Answer: (B)

1371. True about spondylolisthesis is/are
(A) Congenital defect of posterior arch
(B) Slipping of L over S1
(C) Progressive slipping
(D) Abnormal congenital development

Answer: (A, B, C)

1372. Cervical spondylosis is more common at
(A) C1-C2
(B) C2-C3
(C) C6-C7
(D) C4-C5

Answer: (C)

1373. Cervical spondylosis
(A) Most frequently results from an incidence of acute trauma
(B) Causes compression of nerve roots to produce an upper motor neuron lesion in the lower limbs
(C) Produces pain and Parasthesia over the lateral aspect of the forearm and thumb when affecting the 6th cervical nerve
(D) Most frequently affects the upper cervical vertebrae

Answer: (C)

1374. In cervical spondylosis which part of vertebral body involved
(A) Inferior articular facet
(B) Pars interarticularis
(C) Superior articular facet
(D) All of the above

Answer: (D)

1375. Osteophytes developing at the joint at Luscka characteristically compresses spinal nerves at
(A) Intervertebral foramen
(B) Anterior part of body
(C) Posterior part of body
(D) Paradural areas

Answer: (A)

1376. Gradual painful limitation of shoulder movements in an elderly suggest that the most probable diagnosis is
(A) Arthritis
(B) Osteoarthritis
(C) Periarthritis
(D) Myositis Ossificans
(E) Fracture – dislocation

Answer: (C)

1377. Which of the following movements are restricted in Frozen shoulder?
(A) Abduction & Internal rotation
(B) Adduction & external rotation
(C) All range of movements
(D) Only abduction

Answer: (C)

1378. Painful are syndrome is seen in all except:
(A) Complete tear of supraspinatus
(B) #grater tuberosity
(C) Subacromial bursitis
(D) Suprasinatus tendinitis

Answer: (A)

1379. Which movements at shoulder gets restricted when supraspinatous torn?
(A) Flexion
(B) Adduction
(C) Abduction
(D) Rotation only

Answer: (C)

1380. ‘Tennis elbow’, is characterized by:
(A) Tenderness over the medial epicondyle
(B) Tendinitis of common extensor origin
(C) Tendinitis of common flexor origin
(D) Painful flexion and extension

Answer: (B)

1381. Tennis elbow is
(A) Olecranon bursitis
(B) Pain over the medial epicondyle
(C) Pain over the lateral epicondyle
(D) Myositis ossificans

Answer: (C)

1382. Pain & tenderness over the lateral condyle of humerus with a painful dorsi flexion of the wrist is indicative of:
(A) Golfer’s Elbow
(B) Tennis Elbow
(C) Pitcher’s Elbow
(D) Cricket Elbow

Answer: (B)

1383. A 40-year-old man was repairing his wooden shed on Sunday morning. By afternoon, he felt that the hammer was becoming heavier and heavier. He felt pain in lateral side of elbow and also found that squeezing water out of sponge hurt his elbow. Which of the muscles are most likely involved.
(A) Biceps brachii and supinator
(B) Flexor digitorumsuperficialis
(C) Extensor carpi radialislongus and brevis
(D) Triceps brachii and anconeous

Answer: (C)

1384. DeQuervian’s disease classically affects the:
(A) Flexor pollicislongus and brevis
(B) Extensor carpi radialis and extensor pollicislogus
(C) Abductor pollicislongus and brevis
(D) Extensor pollicisbrevis and abductor pollicislongus

Answer: (D)

1385. In de Quervian’s disease the following tendons are involved
(A) Abductor pollicislongus + Extensor pollicisbravis
(B) Abductor pollcisbrevis + Extensor pollicislongus
(C) Adductor pollicisbrevis + Extensor policislongus
(D) Extensor pollicislongus + Flexor pollicislongus

Answer: (A)

1386. Finkelstein’s test is associated with
(A) Dequervians disease
(B) Dupuytren’s Contracture
(C) Carpal tunnel syndrome
(D) Any of the above

Answer: (A)

1387. Which sing is positive in de Quervain’s disease?
(A) Phalen’s sign
(B) Froment’s sign
(C) Cozen’s sign
(D) Finkelstein’s sign

Answer: (D)

1388. About De Quervian’s disease, which of the following is correct:
(A) Pain after straining at thumb base
(B) Involve External pollicislongus
(C) Tense mass may visible
(D) Steroid is used to relieve symptoms
(E) Involve Abductor pollicislongus

Answer: (A, C, D)

1389. Dupuytren’s contracture is:
(A) Thickening of palmar fascia
(B) Base of little finger involved first
(C) Seen in cirrhotics
(D) Seen in epileptics on hydantoin
(E) All of the above

Answer: (E)

1390. Dupuytrens contracture is fibrosis of
(A) Palmar fascia
(B) Forearm muscles
(C) Sortorius fascia
(D) None of the above

Answer: (A)

1391. The following structure is involved in Duputren’s contracture
(A) Thickening of the palmar fascia
(B) Thickening of the dorsal fascia
(C) Contracture of the flexor tendons
(D) Post burns contracture

Answer: (A)

1392. True about Duputren’s contracture
(A) Slight preponderance in males
(B) Slight preponderance in females
(C) Much more common in males
(D) Much more common in females

Answer: (C)

1393. Dupuytren’s Contracture of the hand commonly starts in
(A) Thumbs
(B) Index finger
(C) Middle finger
(D) Ring finger
(E) Little finger only

Answer: (D)

1394. All are true of dupuytren’s contracture except
(A) Usually 4th finger is involved
(B) Bilateral disease is are
(C) Surgical release is useful
(D) May be associated with Pyroine disease

Answer: (B)

1395. The best treatment for Dupuytren’s contracture is
(A) Fasciotomy
(B) Fasciectomy
(C) Incision and release
(D) Fasciectomy + Skin transplantation

Answer: (D)

1396. Dupuytren’s contracture occur in:
(A) Diabetes Mellitus
(B) Alcohol
(C) Epilepsy
(D) Rheumatoid Arthritis
(E) Chronic Pulmonary disease

Answer: (A, B, C, E)

1397. True about Dupuytren’s contracture:
(A) A/W peyronie’s disease
(B) First affect index finger
(C) Nodule formation & thickening of palmar fascia
(D) Amputation may be required

Answer: (A, C, D)

1398. A 65 yr old alcoholic suffering from diabetes has flexion deformity of the right little finger over the metacarpophalangeal joint of around 15 degrees. The ideal management would be:
(A) Observation
(B) Percutaneous fasciotomy
(C) Partial (selective) fasciectomy
(D) Complete fasciectomy

Answer: (A)

1399. Cause of trigger finger is
(A) Thickening of the fibrous tendon sheath
(B) Following local trauma
(C) Unaccustomed activity
(D) All of the above

Answer: (D)

1400. In trigger finger the level of tendon sheath constriction is found at the level of:
(A) Middle phalanx
(B) Proximal intephalangeal joint
(C) Proximal phalanx
(D) Metacarpophalangeal joint

Answer: (D)

1401. Trigger finger is
(A) A feature of carpal tunnel syndrome
(B) Injury to fingers while operating gun
(C) Stenosis tenovaginitis of flexor tendon or affected finger
(D) Any of the above

Answer: (C)

1402. Trigger finger occurs in
(A) Rheumatoid arthritis
(B) Trauma
(C) Osteosarcoma
(D) Osteoarthritis

Answer: (A)

1403. Trigger finger is due to
(A) Tenovaginitis
(B) Synovitis
(C) Bursitis
(D) Fibrositis

Answer: (A)

1404. Trigger finger is caused by
(A) Rheumatoid arthritis
(B) Tenosynovitis
(C) Colles’ fracture
(D) Osteoarthritis

Answer: (B)

1405. The level of the constricting nodule in case of a trigger finger is at
(A) Neck of the corresponding metacarpal bone
(B) Metacarpophalangeal joint
(C) Proximal interphalangeal joint
(D) Distal interphalangeal joint

Answer: (B)

1406. Pulley involved in trigger finger
(A) A1
(B) A2
(C) A3
(D) A4

Answer: (A)

1407. Kanaval’s sign is positive in
(A) Tenosynovitis
(B) Carpel tunnel syndrome
(C) Trigger finger
(D) Dupuytren’s contracture

Answer: (A)

1408. True regarding mallet finger is
(A) Avulsion of tendon at the base of the middle phalanx
(B) Avulsion of extensor tendon at the base of the distal phalanx
(C) Fracture of distal phalanx
(D) Fracture of the proximal phalanx

Answer: (B)

1409. Mallet finger is due to avulsion of extensor tendon of:
(A) Proximal phalanx
(B) Middle phalanx
(C) Distal phalanx
(D) Metacarpals
(E) Any other phalanx

Answer: (C)

1410. A 30 year old man involved in a fisticuff, injured his middle finger and noticed slight flexion of DIP joint. X-rays were normal. The most appropriate management at this stage is-f
(A) Ignore
(B) Splint the finger in hyperextension
(C) Surgical repair of the flexor tendon
(D) Buddy strapping

Answer: (B)

1411. True about ganglion:
(A) Common in volar aspect
(B) Seen adjacent to tendon sheath
(C) Communicates with joins cavity and tendon sheath
(D) It is Unilocular

Answer: (B, C, D)

1412. Compound palmar ganglion is
(A) Tuberculosis affection of ulnar bursa
(B) Pyogenic affection of ulnar bursa
(C) Non specific affection of ulnar bursa
(D) Ulnar bursitis due to compound injury

Answer: (A)

1413. House maids knee is bursitis of:
(A) Prepatellar bursa
(B) Infrapatellar bursa
(C) Olecranon
(D) Ischial bursa

Answer: (A)

1414. Usual site of Tubercular bursitis:
(A) Prepatelar
(B) Subdeltoid
(C) Subpatellar
(D) Trochanteric
(E) None

Answer: (D)

1415. Site of TB bursitis:
(A) Prepatellar
(B) Subacromial
(C) Subdeltoid
(D) Subpatellar
(E) Trochanteric

Answer: (C, E)

1416. Which of the following cysts is medially situated
(A) Housemild’s knee
(B) Clergyman’s knee
(C) Bursa anserine
(D) Semimembranosus bursitis
(E) Morrant Baker’s cyst

Answer: (C)

1417. Hallux valgus means
(A) Outward deviation of great toe
(B) Inward deviation of great toe
(C) Outward deviation of fifth toe
(D) Inward deviation of fifth toe

Answer: (A)

1418. Hallux valgus is associated with all except
(A) An exostosis on the medial side of the head of the first metatarsal
(B) A bunion
(C) Osteoarthritis of the metatarsophalangeal joint
(D) Over-riding or under-riding of the second toe by the third

Answer: (A)

1419. In Hallux valgus surgery, the patients who are likely to be most satisfied are;
(A) Those with pain
(B) Those with hammertoe
(C) Those with metatarsus prims varus
(D) Young age

Answer: (A)

1420. The primary pathology in Athletic Pubalgia is:
(A) Abdominal muscle strain
(B) Rectus femoris strain
(C) Gluteus medius strain
(D) Hamstring strain

Answer: (A)

1421. Thoracic Outlet Syndrome is best Diagnosed By:
(A) CT scan
(B) MRI
(C) Digital subtraction angiography
(D) Clinical examination

Answer: (D)

1422. True about Osteomyelitis is A/E
(A) Staph. Aureus is most common
(B) Epiphysis most commonly involved region
(C) In sickle cell anemia – salmonella is causative organism
(D) Sequestrum is a piece of dead bone
(E) Involucrum is dense sclerotic bone overlying a seuestrum

Answer: (B)

1423. Acute Osteomyelitis is most commonly caused by
(A) Staphylococcus aureus
(B) Actinomyces bovis
(C) Nocardia asteroids
(D) Borrelia Vincentii

Answer: (A)

1424. The most common organism causing osteomyelitis in drug abusers is
(A) E. coli
(B) Pseudomonas
(C) Klebsiella
(D) Staph. Aureus

Answer: (B)

1425. Commonest cause of hematogenous osteomyelitis
(A) Streptococcus
(B) Staph aureus
(C) Salmonella
(D) H. influenza

Answer: (B)

1426. True about HIV, Osteomyelitis is all EXEPCT:
(A) Necrosis absent
(B) Often bilateral
(C) Periosteal new bone formation
(D) Most common cause is staph. Aureus

Answer: (D)

1427. Acute Osteomyelitis of long bones commonly affects the
(A) Epiphysis
(B) Diaphysis
(C) Metaphysis
(D) Articular surface

Answer: (C)

1428. When osteomyelitis disseminates by Hematogenous way the most affected part of bone is
(A) Metaphyses
(B) Epiphyses
(C) Diaphyses
(D) Any of the above

Answer: (A)

1429. The most common source of bone and joint infection is
(A) Direct spread
(B) Percutaneous
(C) Lymphatic
(D) Haematogenous

Answer: (D)

1430. Chronic osteomyelitis is diagnosed mainly by
(A) Sequestrum
(B) Bone fracture
(C) Deformity
(D) Brodie’s abscess

Answer: (A)

1431. About Sequestrum not true is
(A) Infection nidus
(B) Lighter than live bone
(C) Dead piece of bone
(D) Heavier than live bone & trabeculated

Answer: (D)

1432. Involucrum means
(A) Fragment of dead bone
(B) Hole formed in the bone during the formation of a draining sinus
(C) Osteomyelitis of spine
(D) Periosteal new bone formation around necrotic sequestrum

Answer: (D)

1433. Involucrum is found
(A) Underneath the sequestrum
(B) Around the sequestrum
(C) At metaphysis
(D) Beneath the periosteum

Answer: (B)

1434. Non-healing sinus is a common clinical feature is chronic osteomyelitis. The most common frequent cause for this presentation is
(A) Resistant organisms
(B) Retained foreign body
(C) Presence of sequestrum
(D) Intraosseous cavities

Answer: (C)

1435. A dead piece of bone is known as
(A) Involucrum
(B) Sequestrum
(C) Cloacae
(D) All of these

Answer: (B)

1436. All are associated with chronic osteomyelitis except
(A) Amyloidosis
(B) Sequestrum
(C) Metastatic abnormality
(D) Myositis ossificans

Answer: (D)

1437. Which of the following terms in inappropriate to the condition of Osteomyelitis?
(A) Cloacae
(B) Involucrum
(C) Sequestrum
(D) Myelocoele

Answer: (D)

1438. When does the lesion of Osteomyelitis appear on X-ray
(A) 2 hour
(B) 24 hours
(C) 1 week
(D) 2 week

Answer: (D)

1439. Complications of acute osteomyelitis:
(A) Malignancy
(B) Fracture of the affected bone
(C) Sepsis
(D) Chronicity

Answer: (B, D)

1440. True regarding acute osteomyelitis in a child:
(A) Diagnosis by X-ray is 8-10 days after onset
(B) There is diffuse tenderness at the site
(C) Antibiotic therapy should be at least for 4 weeks
(D) Salmonella is the most common cause

Answer: (A, B, C)

1441. Which is false regarding acute osteomyelitis?
(A) Staphylococcus is usual organism
(B) Rest and elevation relieves pain
(C) Parenteral antibiotics are given
(D) Surgery is the only treatment

Answer: (D)

1442. Acute Hematogenous osteomyelitis is treated with all except:
(A) Antibiotics
(B) Splinting
(C) Analgesics
(D) Surgery

Answer: (D)

1443. The deal treatment for acute Osteomyelitis of long bones is
(A) Antibiotics only
(B) Drilling of bone
(C) Decompression
(D) Antibiotics and delayed decompression

Answer: (A)

1444. Instillation treatment in Osteomyelitis is
(A) Continuous suction + continuous drainage
(B) Intermittent suction + continuous drainage
(C) Continuous suction + intermittent suction
(D) Intermittent suction + intermittent drainage

Answer: (A)

1445. Earliest site of bone involvement in heatogenous osteomyelitis:
(A) Metaphysis
(B) Diaphysis
(C) Epiphysis
(D) Point of entry of the nutrient artery

Answer: (A)

1446. Sclerosis of a long bone may suggest
(A) Osteoid osteoma
(B) Sclerosing Osteomyelitis
(C) Both are correct
(D) None of the above

Answer: (C)

1447. Metaphyseal lesion in children include
(A) Fracture
(B) Osteomyelitis
(C) Dislocation
(D) Ewing’s tumour
(E) Osteosarcoma

Answer: (B, E)

1448. Sabre tibia seen in
(A) Tuberculous Osteomyelitis
(B) Syphilitic osteitis
(C) Rickets
(D) Paget’s disease

Answer: (B)

1449. Brodies abscess usually involves
(A) Long bones
(B) Short bones
(C) Pelvic bones
(D) Flat bones

Answer: (A)

1450. What is Brodie’s abscess
(A) Long standing localized pyogenic abscess in the bone
(B) Cold abscess
(C) Subperiosteal abscess
(D) Soft tissue abscess

Answer: (A)

1451. Which of the following is NOT TRUE regarding tubercular osteomyelitis?
(A) It is a secondary TB
(B) Periosteal reaction is seen
(C) Sequestration is uncommon
(D) Inflammation is minimum

Answer: (B)

1452. Tom smith’s arthritis is due to:
(A) Pyogenic infection in infancy
(B) TB
(C) RA
(D) OA

Answer: (A)

1453. Tom Smith arthritis manifests as;
(A) Increase hip mobility and unstability
(B) Hip stiffness
(C) A + D
(D) shortening of limb

Answer: (C)

1454. Chondrolysis occurs commonly in
(A) T.B. arthritis
(B) Syphilitic arthritis
(C) Chondrosrcoma only
(D) Septic arthritis of infancy

Answer: (D)

1455. Acute supurative arthritis is associated with all except:
(A) May be caused by penetrating wound
(B) May be caused by a compound fracture involving a joint
(C) May be due to blood borne infection
(D) Causes the joint to be held in the position of ease
(E) Tends to end with the formation of a fibrous ankylosis

Answer: (E)

1456. Bony ankylosis result from
(A) Pyogenic arthritis
(B) Tuberculosis arthritis
(C) Osteoarthritis
(D) Rheumatic arthritis

Answer: (A)

1457. Most common cause of bony ankylosis is
(A) Rheumatoid arthritis
(B) Pyogenic arthritis
(C) Traumatic arthritis
(D) Osteoarthritis

Answer: (B)

1458. Most common cause of nongonococcal septic arthritis is
(A) Staph-aureus
(B) H. influenza
(C) Pseudomonas
(D) Streptococcus

Answer: (A)

1459. Septic arthritis in a 2 year old child is often caused by
(A) Hemophilous influezae
(B) Staphylococcus aureus
(C) Gonococci
(D) Pneumococci

Answer: (B)

1460. In actinomycosis of the spine, the abscess usually erodes:
(A) Intervertebral disc
(B) Into the pleural cavity
(C) Into the reteropritoneal space
(D) Towards the skin

Answer: (D)

1461. Most common site of Actinomycosis amongst the following is:
(A) Tibia
(B) Rib
(C) Mandible
(D) Femur

Answer: (C)

1462. Madura foot is
(A) Tuberculous infection of the bone
(B) Madura mycotic infection of the bone
(C) Parasitic infection of the bone
(D) Water borne disease of the foot

Answer: (B)

1463. A patient with swelling foot, pus discharge, multiple sinuses. KOH smear shares filamentous structures. Diagnosis is
(A) Osteomyelitis
(B) Madulra Mycosis
(C) Anthrax
(D) None

Answer: (B)

1464. A man aged 40 yrs presents with swelling over foot with multiple sinuses. What is most probable diagnosis:
(A) TB
(B) Maduromycosis
(C) Actinomyecetoma
(D) Osteomyelitis
(E) Tetanus unilateral

Answer: (B)

1465. Tuberculosis osteomyelitis is most likely:
(A) Paucibacillary and haematoganous
(B) Multibacillary and haematoganous
(C) Multibacillary and lymphatogenous
(D) Paucibacillary and lympatogenous

Answer: (A)

1466. Commonest site of skeletal tuberculosis is
(A) Tibia
(B) Radius
(C) Humerus
(D) Vertebrae

Answer: (D)

1467. The commonest infective lesion of the spine in India as:
(A) Pyogenic infection
(B) Fungal
(C) T.B.
(D) Typhoid

Answer: (C)

1468. Tuberculosis of the spine is known as:
(A) Pott’s disease
(B) Scheuermann’s disease
(C) Perthes disease
(D) Frieberg’s disease

Answer: (A)

1469. Tuberculosis spine; most common site is:
(A) Sacral
(B) Cervical
(C) Dorsolumbar
(D) Lumbosacral

Answer: (C)

1470. Commonest site for tuberculous spondylitis is:
(A) T12/L1
(B) C6-7
(C) L4-5
(D) S1-2

Answer: (A)

1471. Pott’s spine is commonest in spine
(A) Cervical
(B) Thoracic
(C) Lumbar
(D) Sacral

Answer: (B)

1472. The most common type of spinal Tuberculosis is
(A) Anterior
(B) Posterior
(C) Central
(D) Paradiscal

Answer: (D)

1473. Tuberculosis of the spine starts in:
(A) Vertebral body
(B) Nucleous pulposus
(C) Annulus fibrosis
(D) Paravertebral fasica

Answer: (A)

1474. Tuberculosis of the spine commonly affects all of the following parts of the vertebra except:
(A) Body
(B) Lamina
(C) Spinous process
(D) Pedicle

Answer: (C)

1475. Tuberculosis of the spine is thought to originate from
(A) By extension from the para vertebral structures
(B) In the cancellous vertebral body
(C) In the ligamentous structures
(D) In the nucleus pulposus

Answer: (B)

1476. Early features of spinal tuberculosis in child includes
(A) Pain on sudden movement
(B) Gradual deformity
(C) Night cries
(D) Sudden deformity

Answer: (A)

1477. Earliest feature of spinal tuberculosis is
(A) Gibbus
(B) Muscle spasm
(C) Pain
(D) Psoas abscess

Answer: (C)

1478. In TB spine the first symptom is
(A) Decreased sensation
(B) Decreased motor power
(C) Pain
(D) Increased deep tendon reflexes.

Answer: (C)

1479. First symptom in tuberculous cord compression is:
(A) Sensory change
(B) Decrease tendon reflex
(C) Spasticity
(D) Lower limb weakness

Answer: (C)

1480. Which of the following causes para vertebral abscess?
(A) Brucella
(B) Tuberculosis
(C) Kala-azar
(D) Typhoid

Answer: (B)

1481. Cold abscess in chest wall is most common due to
(A) T. B spine
(B) T. B rib
(C) T. B pelvis
(D) T. B pleura

Answer: (A)

1482. Paraplegia due to Tuberculosis of spine most commonly occurs at
(A) Cervical spine
(B) Upper thoracic spine
(C) Lower thoracic spine
(D) Lumbar spine

Answer: (B)

1483. Paraplegia is common in Tuberculosis of dorsal spine, because
(A) Incidence of Tuberculosis is commoner in dorsal spine
(B) Natural tendency for kyphosis
(C) Canal is narrow
(D) Spinal cord ends at L1
(E) All of the above

Answer: (E)

1484. In tuberculosis of spine, which one of the following is not a cause for Paraplegia?
(A) Stretching of spinal cord in gibbus deformity
(B) Spinal artery compression
(C) Compression by granulation tissue
(D) Oedema of spinal cord

Answer: (B)

1485. The most common cause of Paraplegia of early onset of Tuberculosis of spine is
(A) Spinal artery thrombosis
(B) Sudden collapse of vertebra
(C) Granulation tissue pressing on cord
(D) Cold abscess pressing on the cord

Answer: (D)

1486. The most common cause of Kyphosis in a male is
(A) Congenital
(B) Tuberculosis
(C) Trauma
(D) Secondaries

Answer: (B)

1487. The most common sequelae of tuberculous spondylitis in an adolescent is:
(A) Fibrous Ankylosis
(B) Bony-Ankylosis
(C) Pathological dislocation
(D) Chronic osteomyelitis

Answer: (B)

1488. In Bony ankylosis, there is
(A) Painless, No movement
(B) Painful complete movement
(C) Painless complete movement
(D) Painful incomplete movement

Answer: (A)

1489. Clinical features of TB spine are:
(A) Night sweat
(B) Reduced appetite
(C) Loss of lordosis
(D) Back pain

Answer: (A, B, C, D)

1490. Clinical features of TB spine are A/E
(A) Loss of lordosis
(B) Night sweat
(C) Loss of appetite
(D) Fever (evening rise)
(E) Back Pain
(F) None

Answer: (F)

1491. Clinical features of T.B. Spine are A/E:
(A) Loss of lordosis
(B) Night sweats
(C) Weight gain
(D) Evening rise of temperature
(E) ↓ed appetite

Answer: (C)

1492. True about TB spine is all except:
(A) Early paraplegia is good prognosis
(B) Insidious onset paraplegia is good prognosis
(C) Dorsolumbart spine is commonest site
(D) It is commonest site for TB of bone in the body

Answer: (B)

1493. Poor prognostic factors in pott’s paraplegia:
(A) Acute onset of paraplegia
(B) Sudden progression of paraplegia
(C) Motor paralysis alone
(D) Long standing paraplegia
(E) Paraplegia in children

Answer: (B, D)

1494. Indicator (s) of poor prognosis in the pott’s spine is/are:
(A) Healed vertebral lesion
(B) Grade IV Pott’s spine
(C) Kyphotic angle >600
(D) Short duration
(E) Acute onset

Answer: (B, C, E)

1495. The 1st sign of TB is
(A) Narrowing of intervertebral space
(B) Rarefaction of vertebral bodies
(C) Destruction of laminae
(D) Fusion of spinous processes

Answer: (A)

1496. Earliest features of TB vertebra
(A) Decreased joint space
(B) Soft tissue swelling
(C) Decreased movements
(D) Pain

Answer: (A)

1497. Earliest radiological sign of Spinal tuberculosis is
(A) Wedging of vertebra
(B) Syndesmophyte formation
(C) Formation of paravertebral abscess
(D) Decreased joint space

Answer: (D)

1498. A 35 yr old lady has chronic backache. On X ray she had a D12 collapse but Intervertebral disc space is maintained. All are possible cause except:
(A) Multiple myeloma
(B) Osteoporosis
(C) Metastasis
(D) Tuberculosis

Answer: (D)

1499. Compression of single vertebral with narrow joint space is characteristic of
(A) Caries spine
(B) Fracture spine
(C) Prolapsed intervertebral disc
(D) Scondaries spine

Answer: (A)

1500. A 46-year-old, known alcoholic, presented with pain in the dorsal spine. On examination there is tenderness at te dorso-lumbar junction. Radiograph shows destruction of the 12th dorsal vertebra with loss of disc space between D12-L1 vertebrae. The most probable diagnosis is
(A) Metastatic spine disease
(B) Pott’s spine
(C) Missed trauma
(D) Multiple myeloma

Answer: (B)

1501. X-ray showing decreased Intervertebral space and presence of Para vertebral shadow. What could be the diagnosis?
(A) Tuberculosis of spine
(B) Ankylosing spondylitis
(C) Eosinophilic granuloma
(D) Multiple myeloma

Answer: (A)

1502. Treatment of Pott’s paraplegia is
(A) Lateral decompression
(B) Posterior decompression
(C) Excision of vertebra
(D) Antituberculous drugs

Answer: (A, D)

1503. Tuberculosis of spine is best treated by
(A) Anterior fusion
(B) Posterior fusion
(C) Antero-lateral decompression
(D) Aspiration of Para spinal abscess and instillation of streptomycin

Answer: (C)

1504. The ideal surgical treatment for Pott’s paraplegia is
(A) Laminectomy and decompression
(B) Anterior decompression and bone grafting
(C) Anterolateral decompression
(D) Costotransversectomy

Answer: (B)

1505. Surgical treatment in Pott’s spine is indicated if there is
(A) Progressive loss of function in spite of medical treatment
(B) No improvement in motor power in spite of 3 months of treatment
(C) There is no improvement in fever in 3 months of treatment
(D) Patient who is an adult or middle aged

Answer: (B)

1506. Indicator 9s) of poor prognosis in the pott’s spine is/are:
(A) Healed vertebral lesion
(B) Grade IV pott’s spine
(C) Kyphotic angle >60°
(D) Short duration
(E) Acute onset

Answer: (A, B, C)

1507. All of the following movements of the hip are painful in a patient with psoas abscess, except
(A) Extension
(B) Adduction to abduction
(C) Abduction to adduction
(D) Fixed flexion to further full flexion.

Answer: (D)

1508. A psoas abscess present on the right side, the correct statement is
(A) Opposite hip flexion relieves pain
(B) Same side hip flexion relieves pain
(C) Same side extension relieves pain
(D) Kyphosis may occur

Answer: (B)

1509. Spina Ventosa results from
(A) Sarcoidosis
(B) Tuberculosis
(C) Histiocytosis X syndrome
(D) Both A + B but not C

Answer: (D)

1510. Caries sica is seen in:
(A) Hip
(B) Shoulder
(C) Knee
(D) None of the above

Answer: (B)

1511. Short long bones of hand and foot are commonly infected by the following organism
(A) Pyogenic
(B) Tuberculous
(C) Fungal
(D) All of the above

Answer: (B)

1512. The most common site of skeletal tuberculosis is
(A) Hip + Spine
(B) Knee + Hip joints
(C) Knee joint
(D) Cervical spine.

Answer: (A)

1513. The most common cause of Monoarthritis in children is
(A) Septic arthritis
(B) Tuberculous arthritis
(C) Osteoarthritis
(D) Rheumatoid arthritis
(E) Any of the above

Answer: (B)

1514. Tuberculous arthritis in advanced cases lead to:
(A) Bony ankylosis
(B) Fibrous ankylosis
(C) Loose joints
(D) Charcots joints

Answer: (B)

1515. All are features of joint Tuberculosis except
(A) Synovium is involved
(B) Synvial fluid has < 20% blood sugar
(C) Kissing arthritis – subchondral bone is involved
(D) Pain is a common feature
(E) Lymphocyte / Moncyte ratio is decreased

Answer: (B)

1516. Triple deformity of knee is classically seen in
(A) Fracture patella
(B) Tuberculosis
(C) Rheumatic arthritis
(D) Rheumatoid arthritis

Answer: (B)

1517. The most appropriate for a 20 year old man suffering from old tuberculosis arthritis of knee with triple deformity is
(A) Plaster immobilization
(B) Joint clearance and traction
(C) Total knee replacement
(D) Charnleys’ Arthrodesis

Answer: (D)

1518. Treatment of triple deformity is
(A) ATT
(B) ATT + Immobilization
(C) ATT + Immobilization + Debridement
(D) None

Answer: (B)

1519. A fiver year old child is suffering from painful restriction of all movements of hip joint, the most likely cause in our country is
(A) Congenital coax vara
(B) Tuberculosis arthritis
(C) Perthe’s disease
(D) Sequelae of septic arthritis of infancy
(E) Psoas abscess

Answer: (B)

1520. Wandering acetabulum is seen in
(A) Fracture acetabulum
(B) Dislocation of femur
(C) CDH
(D) Tuberculosis of hip

Answer: (D)

1521. Malignant bone tumors are all except
(A) Osteosarcoma
(B) Gaint cell tumor (Osteoclastoma)
(C) Aneurysmal bone cyst
(D) Chondrosarcoma
(E) Ewing’s stumor

Answer: (B, C)

1522. Fibrous bony lesions is/are:
(A) Desmoplastic fibroma
(B) Benign fibrous histocytoma
(C) Fibrous dysplasia
(D) Osteosarcoma
(E) Admantinoma

Answer: (A, C, B)

1523. Benign bone tumor is/are:
(A) Osteoid osteoma
(B) Chondroblastoma
(C) Endothelial hemangioma
(D) Osteoclastoma
(E) Osteochondroma

Answer: (A, B, D, E)

1524. Bone forming tumors are:
(A) Osteosarcoma
(B) Osteoid osteoma
(C) Giant cell tumour
(D) Osteoblastoma
(E) Chondrosarcoma

Answer: (A, B, D)

1525. Benign bone tumors are:
(A) Osteoid osteoma
(B) Osteochondroma
(C) Chondroblastoma
(D) Chondromyxooid fibroma
(E) Multiple myeloma

Answer: (A, B, C, D)

1526. All of the following tumor are benign tumour except:
(A) Chondroma
(B) Chordoma
(C) Osteochondroma
(D) Enchondroma

Answer: (B)

1527. According to a newer hypothesis Ewings sarcoma arises from:
(A) Epiphysis
(B) Diaphysis
(C) Medullary cavity
(D) Cortex

Answer: (C)

1528. Classification system of bone tumors is:
(A) Enneking
(B) Manchester
(C) Edmonton
(D) TNM

Answer: (A)

1529. According to Enneking system not true regarding an active benign tumor is
(A) Intracapsular
(B) Margin of reactive bone
(C) Thick rim of reactive bone
(D) Extended curettage is treatment

Answer: (C)

1530. Non neoplastic lesion simulating bone tumor are all except?
(A) Fibrous dysplasia
(B) Bone island
(C) Bone infract
(D) Hurler syndrome

Answer: (D)

1531. Bone tumor arising from diaphysis:
(A) Chondroblastoma
(B) Fibrous dysplasia
(C) Giant cell tumor
(D) Ewing sarcoma
(E) Aneurysmal bone cyst

Answer: (B, D)

1532. Chondroblastoma is a tumour of:
(A) Epiphysis
(B) Metaphysis
(C) Diaphysis
(D) Flat bone

Answer: (A)

1533. Bone tumors arising from diaphysis:
(A) Chondrosarcoma
(B) Weing’s tumor
(C) Osteoclastoma
(D) Chondroblastom
(E) Osteoid osteoma

Answer: (B)

1534. Which of the following occurs in epiphysis:
(A) Osteoclastoma
(B) Chondroblastoma
(C) Osteochodroma
(D) Ewing’s sarcoma
(E) Chondrosarcoma

Answer: (A, B)

1535. The following tumors are seen in metaphysis:
(A) Osteomyelitis
(B) Osteosarcoma
(C) Chondrosarcoma
(D) Osteoclastoma
(E) Wring’s sarcoma

Answer: (A, B, C )

1536. Most common site of osteogenic sarcoma is:
(A) Femur, upper end
(B) Femur, lower end
(C) Tibia, upper
(D) Tibia, lower end

Answer: (B)

1537. Solitary bone cyst is most common in the
(A) Upper end of humerus
(B) Lower end of humerus
(C) Upper end of fibula
(D) Lower end of femur

Answer: (A)

1538. Chordoma can occur over all the following sites, except:
(A) Rib
(B) Clivus
(C) Sacrum
(D) Vertebral body

Answer: (A)

1539. True about bone tumour is:
(A) Multiple myeloma-more than 55 years age and above
(B) Osteogenic sarcoma-fourth decade
(C) Chondrosarcoma – first decade
(D) Osteoclastoma – fifth decade

Answer: (A)

1540. Years old boy, LEAST common cause of proximal lytic lesion of head to femur is:
(A) Plasmacytoma
(B) Metastasis
(C) Histiocytosis
(D) Bone tumour

Answer: (A)

1541. Which of the following is the most common mutation in Ewing’s sarcoma:
(A) Translocation X: 18
(B) Translocation 11 : 22
(C) Activative mutation of G5alpha surface protein
(D) Missense mutation in EXT1

Answer: (B)

1542. A 11 year old boy presented with the complaints of pain in the right arm near the shoulder. X-ray examination revealed an expansile lytic lesion in the upper third of humerus. The most likely diagnosis is
(A) Giant cell tumour
(B) Unicameral bone cyst
(C) Osteochondroma
(D) Parostealostesarcoma

Answer: (B)

1543. Ramu, an 8-year old boy presented with pain in the arm. On x-ray his upper end of humerus demonstrates an expansile lesion in the metaphysis with breech of the overlying cortex. Most likely diagnosis is:
(A) Aneurysmal bone cyst
(B) Unicameral Bone cyst
(C) Chondroblastoma
(D) Osteoclastoma

Answer: (A)

1544. True about aneurysmal bone cyst:
(A) Endothelial lining
(B) Blood filed spaces
(C) Found in upper end of tibia
(D) Pulsatile seen in older age
(E) Seen in older age

Answer: (B, C, D)

1545. Simple bone cyst is seen most commonly in:
(A) Tibia
(B) Radius
(C) Femur
(D) Humerus

Answer: (D)

1546. Babloo a 10 year old boy presents with # of humerus. X ray reveals a lytic lesion at the upper end. Likely condition is
(A) Unicameral bone cyst
(B) Osteosarcoma
(C) Osteoclastoma
(D) Aneurysmal bone cyst

Answer: (A)

1547. A 8 year child present with fracture humerus after trivial injury X-ray shows lytic lesion diagnosis is
(A) Osteoclastoma
(B) Osteomyelitis
(C) Unicameral bone cyst
(D) Aneurysmal bone cyst

Answer: (C)

1548. A classical expansive lytic lesion in the transverse process of a vertebra is seen in:
(A) Osteosarcoma
(B) Osteomyelitis
(C) Unicameral bone cyst
(D) Aneurysmal bone cyst

Answer: (B)

1549. A classical expansile lytic lesion in the transverse process of a vertebra is seen in:
(A) Osteosarcoma
(B) Aneurysmal bone cyst
(C) Osteoblastoma
(D) Osteocalsoma
(E) Metastasis

Answer: (B)

1550. Which of the following conditions is least likely to present as an eccentric osteolytic lesion
(A) Aneurysmal bone cyst
(B) Giant cell tumor
(C) Fibrous cortical defect
(D) Simple bone cyst

Answer: (D)

1551. Differential diagnosis of simple bone cyst are:
(A) Giant cell tumour
(B) Non ossifying fibroma
(C) Enchondroma
(D) Fibrous dysplasia
(E) Eosinophilic granuloma

Answer: (B, C, D, E)

1552. True about simple bone cyst:
(A) Seen in young adult
(B) Present as well demarcated radiolucent lesions
(C) Shaggy fibrous layering
(D) Pathological fracture seen
(E) Commonest site is diaphysis

Answer: (B, C, D)

1553. Histology of unicameral bone cyst
(A) Blood filled cavities
(B) Endothelial lining
(C) Giant cells
(D) Bone formation

Answer: (C, D)

1554. Histological features of unicameral bone cysts are
(A) Blood filled cystic spaces
(B) Endothelial cell lining
(C) Fibrous tissue with cystic spaces
(D) Pseudocyst
(E) Single cavity with connective tissue lining

Answer: (E)

1555. Secondary aneurysmal bone cyst arise in
(A) Osteoclastoma
(B) Chondroblastoma
(C) Fibrous dysplasia
(D) None

Answer: (A, B, C)

1556. An 8 year old boy presents with a gradually progressing swelling and pain since 6 months over the upper tibia. On X-ray, there is a lytic lesion with selerotic margins in the upper tibial metaphysis. The diagnosis is
(A) Osteogenic sarcoma
(B) Osteoclastoma
(C) Brodie’s abscess
(D) Ewing’s sarcoma

Answer: (C)

1557. Epiphyseal tumor is-
(A) Osteoclasoma
(B) Chondromyxoid fibroma
(C) Osteosarcoma
(D) Ewing sarcoma

Answer: (A)

1558. In a young boy, x-ray of upper end of tibia shows a lytic lesion. The least likely diagnosis is
(A) Giant cell tumour
(B) Osteosarcoma
(C) Solitary bone cyst
(D) Tuberculosis

Answer: (A)

1559. When size of osteoclastoma exceeds the size of metaphysis:
(A) Tumor will be covered by cortex.
(B) Tumor will be covered by fibrous capsules
(C) Covered by thin layered of bone
(D) It is limited to metaphysis
(E) Covered by periosteum

Answer: (A, B, C)

1560. The differential diagnosis of lesion, histologically resembling giant cell tumour in the small bones of the hands or feet, includes all of the following except:
(A) Aneursymal bone cyst
(B) Fibrosarcoma
(C) Osteosarcoma
(D) Hyperparathyroidism

Answer: (B)

1561. Variant of Giant tumor is?
(A) Ossifying fibroma
(B) Non ossifying fibroma
(C) Osteosarcoma
(D) Chondroblastoma

Answer: (B, C, D)

1562. Treatment of histologically confirmed giant cell tumour:
(A) Excision, bone grafting and Chemical Cautery
(B) Excision
(C) Radiotherapy
(D) Chemical excision

Answer: (ALL)

1563. Osteoclastoma is treated with:
(A) Total bone replacement
(B) Excision
(C) Curettage
(D) Arthrodesis
(E) Chemotherapy

Answer: (A, B, C, D)

1564. Soap bubble appearance at lower end of radius, the treatment of choice is:
(A) Local excision
(B) Excision and bone grafting
(C) Amputation
(D) Radiotherapy

Answer: (B)

1565. Osteogenic sarcoma arise from:
(A) Epiphysis
(B) Metaphysis
(C) Growth plate
(D) Epiphyseal cortex
(E) Diaphysis

Answer: (B)

1566. What is the most common site of osteosarcoma:
(A) Lower end of femur
(B) Upper end of humerus
(C) Lower end of tibia
(D) Upper end of femur

Answer: (A)

1567. Which of the following bone tumour present secondaries in lung with pneumothorax
(A) Osteosarcoma
(B) Ewing’s sarcoma
(C) Osteoclastoma
(D) Chondroblastoma

Answer: (A)

1568. A pt presents with pheumothorax. Examination shows a swelling over knee. Chest x-ray shows lung nodules, give your most probable diagnosis
(A) Osteosarcoma
(B) Ewing sarcoma
(C) Multiple myeloma
(D) Osteoclastoma

Answer: (A)

1569. Radiological inv. shows sun ray appearance; dx is
(A) Osteosarcoma
(B) GCT
(C) Osteomyelitis
(D) Ewing’s sarcoma

Answer: (A)

1570. True regarding osteosarcoma is
(A) Occurs because of proliferation of osteoclasts
(B) Sunray appearance in x-ray indicates new bone formation
(C) Affects mainly males in 5th or 6th decade
(D) Lymphatic metastasis is most common

Answer: (B)

1571. Radiological features of osteosarcoma:
(A) New bone formation
(B) Sun ray appearance
(C) Codman’s triangle
(D) Soap bubble appearance
(E) Onion pool appearance

Answer: (A, B, C)

1572. 7 year old child presents with a lesion in upper tibia. X-ray shows radiolucent area with Codman’s triangle and Sunray appearance. Diagnosis is
(A) Ewing’s sarcoma
(B) Osteosarcoma
(C) Osteoid osteoma
(D) Chondrosarcoma

Answer: (B)

1573. A15 year old boy presented with mass in lower femur, which on X-ray was found to have Codman’s triangle & sunray appearance. The diagnosis is
(A) Ewing’s Tumour
(B) Osteosarcoma
(C) Osteoclastoma
(D) Osteomyelitis

Answer: (B)

1574. Sunburst appearance found in:
(A) Osteosarcoma
(B) Ewing’s sarcoma
(C) Osteoclastoma
(D) Osteoid osteoma
(E) Chondrosarcoma

Answer: (A)

1575. All of the following investigations are needed for the diagnosis if osteosarcoma, except –
(A) MRI of femur
(B) Bone marrow biopsy.
(C) Bone scan
(D) CT chest.

Answer: (B)

1576. Which of the following malignant tumors is radioresistant?
(A) Ewing’s sarcoma
(B) Retinoblastoma
(C) Osteosarcoma
(D) Neuroblastoma

Answer: (C)

1577. Management plan for osteogenic sarcoma of the lower end of femur must include:
(A) Radiotherapy , amputation, chemotherapy
(B) Surgery alone
(C) Chemotherapy + Limb Salvage Surgery + Chemotherapy
(D) Chemotherapy + radiotherapy

Answer: (C)

1578. True about osteosarcoma:
(A) Involves epiphysis of long bones
(B) Most commonly involve knee & distal femur
(C) Spread to lung through hematogenous route
(D) Exclusively found in adolescent & early adult life
(E) X-ray sunray appearance

Answer: (B, C, E)

1579. All are the predisposing factors of osteogenic sarcoma except:
(A) Paget’s disease of bone
(B) Radiation
(C) Viral infection
(D) Bone infarction

Answer: (C)

1580. True about parosteal osteosarcoma:
(A) Same prognosis as medullary type
(B) Never go to medulla
(C) May involve medulla
(D) Lies on cortical surface
(E) Radiotherapy is treatment of choice

Answer: (C, D)

1581. Features of parosteal osteosarcoma include:
(A) It invades the medullary cavity
(B) Easily diagnosed by x-ray
(C) Prognosis is same as in other forms of osteosarcoma
(D) En bloc resection/amputation is the treatment of choice
(E) Seen most commonly in younger age groups

Answer: (A, B, D)

1582. Which of the following is a pulsatile tumor:
(A) Osteosarcoma
(B) Chondrosarcoma
(C) Osteoclastoma
(D) Ewing’s sarcoma

Answer: (A)

1583. ‘T-10 Protocol’ for treatment of osteosarcoma includes all of the following, Except:
(A) High Dose Methotrexate
(B) Bleomycin, Cyclophosphamide, Doxorubicin (BCD)
(C) Vincristine
(D) Etoposide

Answer: (D)

1584. A 2 yr old boy with Ewing’s sarcoma is having radiotherapy & chemotherapy. Which of the following indicates poor prognosis:
(A) Bet 2 microglobulin
(B) Fever
(C) Thrombocytosis
(D) Young age

Answer: (B)

1585. mic-2 mutation is associated with
(A) Osteosarcoma
(B) Ewing sarcoma
(C) Alveolar soft tissue sarcoma
(D) Dematofibrosarcomprotruberance

Answer: (B)

1586. 8 year old child has fever with pain and swelling in mid-thigh. Lamellated appearance and Codman’s triangle is seen on X-ray. Histopathology shows small round cell tumor positive for MIC-2. Diagnosis is?
(A) Osteosarcoma
(B) Ewing’s sarcoma
(C) Chondroblastoma
(D) Multiple myeloma

Answer: (B)

1587. True about Ewing’s sarcoma is all except:
(A) 5% cases revels t(11-22)
(B) Arise from medullary cavity of tubular bone
(C) Arise from diaphysis
(D) N-myc chromosome

Answer: (B, C)

1588. A child 10 years of age presents with a mass on his left thigh. The mass seems to be rising from diaphysis of femur, and involving the soft tissue of thigh. The child is having fever also. Give your most probable diagnosis
(A) Osteosarcoma
(B) Ewing’s sarcoma
(C) Chondro sarcoma
(D) Malignant fibrous histiocytoma

Answer: (B)

1589. A 12 year old girl complains of pain persisting in his leg for several weeks with a low grade fever. A radiograph reveals a mass in the diaphyseal region of the left femur with overlying cortical erosion and soft tissue extension. A biopsy of the lesion shows numerous small round cells, rich in PAS positive diastase sensitive granules. The most likely histological diagnosis is
(A) Osteogenic sarcoma
(B) Osteoblastoma
(C) Ewing’s sarcoma
(D) Chondroblastoma

Answer: (C)

1590. A 15-year –old boy is injured while playing cricket. X-rays of the lef rule out of a possible fracture. The radiologist reports the boy has an evidence of aggressive bone tumor with both bone destruction and soft tissue mass. The bone biopsy reveals a bone cancer with neural differentiation. Which of the following is the most likely diagnosis?
(A) Chondrablastoma
(B) Ewing’s sarcoma
(C) Neuroblastoma
(D) Osteosarcoma

Answer: (B)

1591. A 7 year old boy with h/o trauma 2 months back now presents with fever & acute pain over thigh. On x-ray femoral shaft shows lesions with multiple laminated periosteal reaction next line of management is
(A) CRP measurement
(B) Core biopsy
(C) Tc99 MDP scan
(D) MRI

Answer: (B)

1592. Most common site of origin of adamantinoma is
(A) Mandible near molar tooth
(B) Middle alveolar margins
(C) Hard palate
(D) Mandible near symphisismenti

Answer: (A)

1593. Most common lesion of the mandible is:
(A) Ameloblastoma
(B) Squamous cell ca
(C) Osteosarcoma
(D) Osteoclstoma

Answer: (A)

1594. Most common site of admantinoma of the long bones is:
(A) Femur
(B) Ulna
(C) Tibia
(D) Fibula

Answer: (C)

1595. True about Ameloblastoma:
(A) Cystic lesion
(B) Rapidly growing
(C) Malignant disease
(D) MC site is Tibia
(E) Presented in children

Answer: (A, E)

1596. A 33-year-old man presented with a slowly progressive swelling in the middle 1/3rd of his right tibia. X-rays examination revealed multiple sharply demarcated radiolucent lesions separated by areas of dense and sclerotic bone. Microscopic examination of a biopsy specimen revealed island of epithelial cells in a fibrous stroma. Which of the following is the most probable diagnosis?
(A) Adamantinoma
(B) Osteofibrous dysplasia
(C) Osteosarcoma
(D) Fibrous cortical defect

Answer: (A)

1597. A 19 year young patient with sclerotic lesion at diaphysis. The diagnosis is:
(A) Osteoid osteoma
(B) Ewing’s sarcoma
(C) Osteoclastoma
(D) Metastasis

Answer: (A)

1598. Babu a 19 yrs old ale has a small circumscribed sclerotic swelling over diaphysis of femur; likely diagnosis is
(A) Osteoclastoma
(B) Osteosarcoma
(C) Ewing’s sarcoma
(D) Osteoid osteoma

Answer: (D)

1599. True statement (s) regarding Osteiodosteoma is/are:
(A) Malignant
(B) MC bone involvement-tibia
(C) Local excision or curettage cure
(D) Bone pain relieved by aspirin
(E) X-ray shows density surrounding radiolucent lesion

Answer: (B, C, D, E)

1600. True about non-ossifying fibroma of bone:
(A) Present until 3rd& 4th decade
(B) Eccentric
(C) Prominent sclerotic margin
(D) Histologically giant cell with areolar
(E) Metaphyseal lesion

Answer: (B, C, D, E)

1601. True about non-ossifying fibroma:
(A) Prominent at 2nd and 4th decade
(B) Prominent sclerotic margins
(C) Fibrous tissue with areolar tissue
(D) Centrally located
(E) Risk of malignancy

Answer: (B, C)

1602. Characteristic radiological feature of fibrous dysplasia is:
(A) Cortical Thickening
(B) Cortical calcification
(C) Ground glass appearance
(D) Bone enlargement

Answer: (C)

1603. Which of the following is not a benign bone tumor
(A) Osteoid osteoma
(B) Chondroma
(C) Enchondroma
(D) Chordoma

Answer: (D)

1604. Chordoma commonly involves:
(A) Dorsal spine
(B) Clivus
(C) Lumbar spine
(D) Sacrum
(E) Cervical spine

Answer: (B, D)

1605. Most common tumour in hand
(A) Exostosis
(B) Giant cell tumour
(C) Enchondroma
(D) Synovial sarcoma

Answer: (C)

1606. All are true about enchondroma except
(A) Usually asymptomatic
(B) Ollier’s disease may be associated
(C) Occurs only in short bones
(D) Radiolucent lesion with flecks of calcification
(E) Treatment is not always necessary

Answer: (C)

1607. True about Hereditary Osteohondromatosis:
(A) Occur mainly in Neonates
(B) Occur in first decade
(C) Does not until 4-6yr
(D) Occur after Puberty
(E) No age predication

Answer: (B)

1608. Most common benign tumor of the bone is
(A) Giant cell tumor
(B) Simple bone cyst
(C) Osteochondroma
(D) Enchondroma

Answer: (C)

1609. All of the following are the causes of sudden increase in pain in osteochondroma, except:
(A) Sarcomatous change
(B) Fracture
(C) Bursitis
(D) Degenerative changes

Answer: (D)

1610. All the statements are true about exotosis, except:
(A) It occurs at the growing end of bone
(B) Growth continues after skeletal maturity
(C) It is covered by cartilaginous cap
(D) Malignant transformation may occur

Answer: (B)

1611. Which of the following statements is true about osteochondromatosis:
(A) Usually affects long bones, but can also occur in skull and pelvis
(B) Usual site is metaphyseal region
(C) Also known as multiple exostoses, diaphysealaclasis
(D) It doesn’t interfee with general body stature
(E) Autosomal dominant in inheritance

Answer: (A, B, C, E)

1612. Which one of the following bone tumors typically affects the epithysis of a long bone?
(A) Osteosarcoma
(B) Ewing’ sarcoma
(C) Chondroblastoma
(D) Chondromyxoid fibroma

Answer: (C)

1613. Chondroblastoma most commonly occurs in:
(A) Metaphysis
(B) Diaphysis
(C) Epiphysis
(D) Medullary cavity

Answer: (C)

1614. Dense calcification is found in
(A) Osteosarcoma
(B) Chondroblastoma
(C) Synovial sarcoma
(D) Osteoblastoma

Answer: (B)

1615. A 15 year-old boy presented with painful swelling over the left shoulder. Radiograph of the shoulder showed an osteolytic area with stippled calcification over the proximal humeral epiphysis. Biopsy of the lesion revealed an immature fibrous matrix with scattered giant cells. Which of the following is the most likely diagnosis?
(A) Gaint Cell Tumor
(B) Chondroblastoma
(C) Osteosarcoma
(D) Chondromyxoid fibroma

Answer: (B)

1616. A 45 years male presented with an expansile lesion in the centre of femoral metaphysis. The lesion shows Endosteal scalloping & punctuate calcification. Most likely diagnosis is:
(A) Osteosarcoma
(B) Chondrosarcoma
(C) Simple bone cyst
(D) Fibrous Dysplasia

Answer: (B)

1617. True about hemangioma of bone
(A) Mostly symptomatic
(B) Peak incidence in 5th decade
(C) Constitute 10-20% of total bone tumor
(D) Overgrowth of bone occurs
(E) Hematogenous spread

Answer: (B, C, D)

1618. Which of the following statements is true regarding hemangioma of the bone:
(A) Occurs commonly in skull bones
(B) Requires observation as I is permalignant
(C) Hamartomatous in origin
(D) Forms 10-20% of the bone tumors
(E) Local gigantism occurs when it occurs in an extremity

Answer: (A, C, E)

1619. All are true for multiple myeloma except
(A) Hypercalcemia
(B) Hyperuricemia
(C) Increased S. Alkaline phosphatase
(D) Monoclonal M Band
(E) Bone marrow plasma cells <5%

Answer: (D, E)

1620. Lytic beveled lesions are seen in skull X-ray, most likely cause is-
(A) Multiple myeloma
(B) Eosinophilic granuloma
(C) Metastasis
(D) Osteosarcoma

Answer: (A)

1621. A patient with pain in back. Lab investigation shows elevated ESR. X-ray skull shows multiple punched out lytic lesions. Most imp. investigation to be done is:
(A) Serum acid phosphatase
(B) CT head with contrast
(C) Whole body scan
(D) Serum electrophoresis

Answer: (D)

1622. A 60 yrs old male has bone pain, vertebral collapse, fracture pelvis, the probable diagnosis is
(A) Multiple myeloma
(B) Scecondaries
(C) TB
(D) Hemangioma f bone

Answer: (A)

1623. Hypercalcemia& hyperglycemia is associated with:
(A) Multiple & Myeloma
(B) Ewing’s sarcoma
(C) Osteosarcoma
(D) Chondrosarcoma

Answer: (A)

1624. Three most common cancers metastasizing to bone
(A) Kidney
(B) Thyroid
(C) Breast
(D) Prostate
(E) Lung

Answer: (C, D, E)

1625. Which of the following usually produces osteoblasticsecondaries
(A) Carcinoma lung
(B) Carcinoma breast
(C) Carcinoma urinary bladder
(D) Carcinoma prostate

Answer: (D)

1626. A patient developed paraplegia. On routine examination and X-ray it was found that there are osteoblastic lesion in his spine. Most probable diagnosis is
(A) Carcinoma thyroid
(B) Ca. Prostate
(C) Breast Ca.
(D) Pancreatic Ca.

Answer: (B)

1627. Expansile lytic osseous metastases are characteristics of primary malignancy of:
(A) Kidney
(B) Bronchus
(C) Breast
(D) Prostate

Answer: (A)

1628. Metastases least common in:
(A) Skull
(B) Pelvis
(C) Vertebrae
(D) Proximal part of long bones of the upper limb
(E) Small bones of the hand

Answer: (E)

1629. Metastatic tumor least common in
(A) Pelvis
(B) Ribs
(C) Small bone of lower limb
(D) Vertebra
(E) Skull

Answer: (C)

1630. Bone metastases is common in which of the following:
(A) Nephroblastoma
(B) Neuroblastoma
(C) RCC
(D) Clear cell sarcoma

Answer: (B, C)

1631. Most common cause of bone malignancy:
(A) Secondaries
(B) Osteosarcoma
(C) Ewing’s sarcoma
(D) Osteoclastoma
(E) Chondrosarcoma

Answer: (A)

1632. Osteosclerotic bone metastasis is found most commonly in which carcinoma:
(A) Kidney
(B) Thyroid
(C) Lung
(D) Prostate

Answer: (D)

1633. True about Bone metastasis:
(A) 5% bone metastasis are symptomatic
(B) Higher serum levels of alkaline phosphatase
(C) Most common secondary in females is breast
(D) Prostate produce osteosclerotic lesion
(E) Commonly involves hand & feet bones

Answer: (B, C, D)

1634. Bone metastases commonly seen in A/E
(A) Skull
(B) Pelvis
(C) Vertebrae
(D) Small bones of hand
(E) Proximal part of long bones of upper limb

Answer: (D)

1635. Which soft tissue sarcoma commonly gives to bone secondary:
(A) Fibrosarcoma
(B) Liposarcoma
(C) Osteosarcoma
(D) Neurofibroma
(E) Synovial sarcoma

Answer: (C, E)

1636. Most common soft tissue tumour in a child:
(A) Rhabdomyosarcoma
(B) Histiocytoma
(C) Fibrosarcoma
(D) Liposarcoma

Answer: (A)

1637. Regional lymph nodes are involved in which of these:
(A) Ewing sarcoma
(B) Osteosarcoma
(C) Adamantinoma
(D) Synovial cell sarcoma

Answer: (D)

1638. All of the following statements about synovial cell sarcoma, are true,, Except
(A) Originate from synovial lining
(B) Occur more often at extra articular sites
(C) Usually seen in patients less than 50 years of age
(D) Knee and foot are common sites involved

Answer: (A)

1639. A 30 years lady presents with pain and tenderness in index finger just under the nail. She was unable to wash her hands with cold water. Patient does not reveal any history of trauma or injury. What could be a probable finding?
(A) Sausage digits
(B) Ridging of nail, discolouration and pin – head tenderness
(C) Stiffness of whole hand
(D) Hypersensitivity of finger

Answer: (B)

Medical PG Physiology

Medical PG Physiology

1. Lowest threshold potential in a motor nerve fibre is at

(A) Dendrite

(B) Body

(C) Axon hillock 

(D) Axon

Answer: (C)

2. Heat loss from the body depends mostly on

(A) Thermoregulatory centre 

(B) Warming of air during inspiration 

(C) On the environmental temperature 

(D) Radiation and evaporation 

Answer: (C)

3. All are true regarding capillaries except

(A) Have large total cross-sectional area 

(B) Contain larger quantity of blood than veins 

(C) Site of gaseous exchange 

(D) Lined by endothelium 

Answer: (B)

4. Mouth-to-mouth respiration provides an oxygen concentration of

(A) 16% 

(B) 20% 

(C) 22% 

(D) 24% 

Answer: (A)

5. Hyaline membrane contains

(A) Albumin

(B) Fibrin

(C) Globulin

(D) WBCs

Answer: (B)

6. Surfactant is secreted by

(A) Type-I pneumocytes 

(B) Type-II pneumocytes 

(C) Bronchial goblet cells

(D) Endothelium of pulmonary vasculature 

Answer: (B)

7. Which is the best parameter for analysis of Hypoxic hypoxia

(A) Arterial PO₂ 

(B) Arterial PCO₂ 

(C) Venous PO₂ 

(D) A-V Difference 

Answer: (A)

8. Which compound shifts the Oxygen dissociation curve to the right

(A) 1, Phosphoglycerate 

(B) 2, 3 DPG 

(C) 1, 3 DPG 

(D) Glyceraldehyde

Answer: (B)

9. All of the following cause Hyperventilation except

(A) Decreased pH in CSF 

(B) Decreases plasma HCO₃ 

(C) CO poisoning 

(D) Increased adrenergic levels 

Answer: (C)

10. Sertoli cells secrete

(A) Testosterone

(B) Estrogen

(C) Androstenedione

(D) Inhibin

Answer: (D)

11. Secretion of Prolactin is affected by 

(A) GnRH analogue 

(B) Dopamine

(C) Serotonin

(D) FSH

Answer: (B)

12. All are feature of Pyramidal tract lesion except

(A) Clasp knife rigidity 

(B) Increased tone 

(C) Babinski sing + ve

(D) Involuntary movement 

Answer: (D)

13. Features of Neuroglia cells include all except

(A) Protoplasmic astrocytes are found in grey matter 

(B) Oligodendrocytes are derived from ectoderm 

(C) Microglia are mesodermal in origin 

(D) Central neuroglial cells are derived from Schwann cells 

Answer: (D)

14. Kinesthetic sensation is

(A) Transmitted by the β-type of sensory nerve 

(B) Located in Merkl’s disc 

(C) Transmitted by Meissner’s corpuscles 

(D) Means abnormal perception of sensation 

Answer: (A)

15. Cerebellar connection  to other parts of the brain is projected through which cell

(A) Golgi cells 

(B) Basket cells 

(C) Purkinje cells 

(D) Oligodendrocytes 

Answer: (C)

16. Output from the cerebellum is solely from

(A) Basket cells 

(B) Granular cells 

(C) Treitz cells 

(D) Purkinje cells 

Answer: (D)

17. Renin secretion is increased by all of the following except

(A) Na⁺ in PCT 

(B) Afferent arteriolar low pressure 

(C) Na⁺ in DCT 

(D) Sympathetic nerve stimulation 

Answer: (A)

18. Sperms acquire motility in 

(A) Epididymis

(B) Seminal vesicle 

(C) Rete testes 

(D) Vas deferens 

Answer: (A)

19. Human insulin differs from beef insulin by how many number of amino acid residues 

(A) 1

(B) 2 

(C) 3

(D) 4

Answer: (A)

20. Normal excretion of protein in urine per day is

(A) 100 mg 

(B) 150 mg 

(C) 400 mg 

(D) 600 mg 

Answer: (B)

21. Normal gastric juice contains all except

(A) Na⁺ 

(B) K⁺

(C) Ca⁺⁺ 

(D) Mg⁺⁺ 

Answer: (C)

22. Which of the following is true about gastric emptying

(A) Decreased by cholecystokinin 

(B) Decreased by gastrin 

(C) Increased by secretin 

(D) Increased by GIP 

Answer: (A)

23. ‘D’ cells of the pancreas secrete

(A) Insulin

(B) Glucagon

(C) Intestinal peptide 

(D) Somatostatin

Answer: (D)

24. Spontaneous release of acetylcholine at the neuromuscular junction produces

(A) Miniature end-plat potential 

(B) Action potential 

(C) Post-tetanic potential 

(D) Resting membrane potential 

Answer: (A)

25. Myasthenia gravis is a disorder of 

(A) Motor neuron 

(B)  Neuromuscular junction 

(C) Peripheral nerve 

(D) Spinal cord 

Answer: (B)

26. Oxygen affinity is increased by all of the following except

(A) Alkalosis

(B) Hypoxia

(C) Increased HbF

(D) Hypothermia

Answer: (B)

27. Which of the following does NOT stimulate peripheral chemoreceptors

(A) Hypoxia

(B) Hypocapnia

(C) Hypocapnia

(D) Low perfusion pressure 

Answer: (B)

28. Ventilation perfusion ratio is maximum at 

(A) Apex of lung 

(B) Base of lung 

(C) Right posterior lobe of lung 

(D) Equal in all areas of lung 

Answer: (A)

29. Highest conduction rate in m/s is observed in 

(A) SA node 

(B) AV node 

(C) Bundles of His 

(D) Purkinje system 

Answer: (D)

30. The order of activation after stimulation of Purkinje fibres is:

(A) Endocardium, Septum, epicardium 

(B) Endocardium, epicardium, Septum 

(C) Septum, endocardium, epicardium 

(D) Septum, epicardium, endocardium 

Answer: (C)

31. Cardiac output in L/min divided by heart rate equals

(A) Cardiac efficiency 

(B) Mean stroke volume 

(C)Cardiac index 

(D) Mean arterial pressure

Answer: (B)

32. Pulmonary micro-circulation differs from systemic circulation in having

(A) Resistance low, pulsatile flow high 

(B) Resistance low, capillary pressure low 

(C)  Resistance high, pulsatile flow low 

(D) Resistance high, capillary pressure high

Answer: (B)

33. Blood supply during exercise is increased in:

(A) Cutaneous circulation 

(B) Hepato-splanchnic circulation 

(C) Renal circulation 

(D) Coronary circulation 

Answer: (D)

34. BMR depends on

(A) Body weight 

(B) Surface area 

(C) Amount of adipose tissue 

(D) Amount of lean body mass 

Answer: (B)

35. Ketone bodies are produced in the 

(A) Liver

(B) Kidney

(C) Muscle

(D) GIT

Answer: (A)

36. Which of the following does NOT form a filtration barrier in nephrons

(A) Podocytes

(B) Mesangium

(C) Endothelial cell 

(D) Basement membrane 

Answer: (B)

37. Which of the following occurs along with glucose transport into a cell

(A) Sodium symport 

(B) Sodium anteport 

(C) Potassium transport 

(D) Amino acid transport 

Answer: (A)

38. Appreaciation of shape and size of an object placed in the hand is lost in the lesion of 

(A) Tractus gracilis 

(B) Tractus Cuneatus 

(C) Lateral spinothalamic tract 

(D) Spino-reticular tract 

Answer: (B)

39. Unidirectional flow of a nerve impulse is at 

(A) Synapse

(B) Axon

(C) Dendrites

(D) All of the above 

Answer: (A)

40. Excitable tissue at rest is least permeable to

(A) Na⁺ 

(B) K⁺ 

(C) Ca⁺⁺ 

(D) Cl 

Answer: (A)

41. Which is True about capillaries

(A) Stores 25% of blood volume 

(B) Stores 15% of blood volume 

(C) Stores 5% of blood volume 

(D) Stores 95% of blood volume 

Answer: (C)

42. QRS complex on ECG tracing indicates

(A) Atrial depolarization 

(B) Ventricular depolarization 

(C) Atrial repolarization 

(D) Ventricular repolarization 

Answer: (B)

43. Mean arterial pressure is

(A) Systolic + (Diastolic BP)/2 

(B) Systolic + 1/3rd Pulse pressure 

(C) Diastolic + (Systolic BP)/2 

(D) Diastolic + 1/3rd Pulse pressure 

Answer: (D)

44. Total alveolar ventilation volume (in L/min) is 

(A) 1.5 

(B) 3.5 

(C) 4.2 

(D) 5.0

Answer: (C)

45. The fraction of inspired air in mouth-to-mouth respiration is

(A) 0.16

(B) 0.19 

(C) 0.21 

(D) 0.26 

Answer: (A)

46. Anemic hypoxia is due to

(A) ↓PO₂ in arterial blood 

(B) ↑PO₂ in arterial blood 

(C) ↑PCO₂ in arterial blood 

(D) ↓O₂ content in arterial blood 

Answer: (D)

47. Erythropoietin secretion occurs when

(A) ↓Tissue pO₂ concentration 

(B) ↑ Tissue pH 

(C) ↑ Tissue pCO₂ concentration

(D) ↑Hemoglobin 

Answer: (A)

48. Anion gap is mostly due to

(A) Proteins

(B) Sulphates

(C) Phosphates

(D) Nitrates

Answer: (A)

49. Glucose symport occurs with 

(A) Na⁺ 

(B) Ca⁺⁺ 

(C) K⁺ 

(D) Cl⁻ 

Answer: (A)

50. Endogenous triglycerides are maximum in 

(A) VLDL

(B) Chylomicron

(C) LDL

(D) HDL

Answer: (A)

51. Gall bladder contraction is stimulated by

(A) Gastrin

(B) Vagus

(C) Secretion

(D) Cholecystokinin

Answer: (D)

52. The enzymes secreted by endocrinal part of pancreas are all except

(A) Somatostatin

(B) Chymotrypsin

(C) Glucagon

(D) Insulin

Answer: (B)

53. Flocculonodular lobe has direct connections with 

(A) Red nucleus 

(B) Inferior olivary nucleus 

(C) Vestibular nucleus 

(D) Dentate nucleus 

Answer: ()

54. True about function of brain are all except

(A) Sensitive to hypoxia 

(B) Dependent on glucose 

(C) Uses fatty acids in starvation 

(D) Does not store energy 

Answer: (C)

55. Melatonin is secreted by 

(A) Hypothalamus

(B) Adrenal cortex 

(C) Pineal gland

(D) Melanocytes

Answer: (C)

56. Spirometry can demonstrate and measure all of the following except:

(A) Tidal volume 

(B) Residual volume 

(C) Vital capacity 

(D) Inspiratory reserve capacity 

Answer: (B)

57. Surfacant is made up of :

(A) Fibrin

(B) Mucoprotein

(C) Mucoprotein

(D) Fibrinogen

Answer: (C)

58. The important feature of 2.3 Diposphoglycerate includes :

(A) Higher concentration in adult blood 

(B) Contribution to Bohr effect 

(C) Increases affinity of O₂ to haemoglobin 

(D) Associated with foetal blood to promote oxygenation. 

Answer: (A)

59. Set of data which correctly defines restrictive lung disease is :

(A) ↑FRC, ↑compliance of lung tissue 

(B) ↑FEV₁­/FVC, ↓compliance of lung tissue 

(C) ↓FEV₁/FVC, ↓compliance of lung tissue 

(D) ↑TLC, RV is ↓ 

Answer: (B)

60. Aerobic capacity is maximally increased by :

(A) Regular 3 minute exercises 

(B) Spurts of exercises 

(C) Prolonged exercises

(D) Sternous exercises 

Answer: (A)

61. Which of the following variants of hypoxia does not stimulate peripheral chemoreceptors :

(A) Hypoxic hypoxia 

(B) Anaemic hypoxia 

(C) Stagnant hypoxia 

(D) Histotoxic hypoxia 

Answer: (B)

62. Blood in splanchnic area during exercise is decreased due to :

(A) Venoconstriction with decreased blood flow 

(B) Venodilation with decreased blood flow 

(C) Venodilation with increased blood flow 

(D)  Venodilation with normal blood flow 

Answer: (A)

63. Most important stimulant for bile secretion is :

(A) Cholecystokinin

(B) Secretin

(C) Bile acid 

(D) Bile salt

Answer: (D)

64. Corpus luteum is maintained by :

(A) Progesterone

(B) LH

(C) FSH

(D) Estrogen

Answer: (B)

65. Weber Fechner law is related with :

(A) Amplitude

(B) Surface area 

(C) Number of sensory fibre involvement 

(D) Stimulus discrimination 

Answer: (D)

66. Modality that is lost on the ipsilateral side in Brown sequard syndrome is

(A) Pain

(B) Temperature

(C) Crude touch 

(D) Proprioception

Answer: (D)

67. Amyotrophic lateral sclerosis involves :

(A) Upper motor neuron and lower motor neuron both

(B) Posterior column only 

(C) Lower motor neuron only 

(D) Raphae nucleus 

Answer: (A)

68. All of the following are known functions of hypothalamus except :

(A) Temperature regulation 

(B) Increased in heart rate with exercise 

(C) Food intake 

(D) Hypophyseal control 

Answer: (B)

69. All of the following statements about adrenals are true except :

(A) Chromoffin granules are seen in pheochromocytoma 

(B) Adrenal medulla normally secretes epinephrine & excess of norepinephrine

(C) Tumours of adrenal medulla secrete nor epinephrine in excess of epinephrine 

(D) Adrenal medulla is not essential for life 

Answer: (B)

70. Increased ratio of insulin to glucagon causes :

(A) Decreased levels of cyclic AMP 

(B) Decreased levels of lipoprotein lipase 

(C) Decreased amino acid synthesis 

(D) Enhanced lipolysis in adipose tissue 

Answer: (A)

71. True statement regarding extracellular fluid to intracellular fluid is :

(A) ECF is rich K+ 

(B) ECF is more than ICF 

(C) ECF is rich in organic anion 

(D) High Na:K+ ratio is seen 

Answer: (D)

72. Most diffusible ion in excitable tissue is :

(A) Na⁺ 

(B) K⁺ 

(C) PO⁻₊ 

(D) Cl⁻ 

Answer: (B)

73. True regarding transport across a cell membrane is :

(A) Cl⁻with glucose symport 

(B) Na⁺ with glucose anteprot 

(C) Na⁺ with glucose symport 

(D) K⁺ with glucose symport 

Answer: (C)

74. First change observed in distal part, of a cut nerve is :

(A) Axonal degeneration 

(B) Sprouting 

(C) Myelin degeneration 

(D) Schawn cells proliferation

Answer: (A)

75. QRS complex indicates :

(A) Atrial repolarization 

(B) Atrial depolarization 

(C) Ventricular repolarization 

(D) Ventricular depolarization 

Answer: (D)

76. Isovolumic relaxation phase of the cardiac cycle ends with :

(A) Peak of ‘C’ waves 

(B) Opening of A.V. valve 

(C) Closure of semilunar valve 

(D) Begning of “T” wave 

Answer: (B)

77. In the physiological state :

(A) Vein contain 25% of blood 

(B) Capillaries contain 25% blood 

(C) Vein contains 5% of blood 

(D) Capillaries contains 5% blood 

Answer: (D)

78. Erythropoitin level are increased by :

(A) ↓PO₂ 

(B) ↓PCO₂ 

(C) ↓Hb 

(D) ↓pH 

Answer: (A)

79. Total lung capacity depends upon :

(A) Size of airway 

(B) Closing volume 

(C) Lung compliance 

(D) Residual volume 

Answer: (C)

80. Pulmonary function changes seen in Emphysema are :

(A) ↑TLC 

(B) ↓RV 

(C) ↑FEV₁ 

(D) ↑VC 

Answer: (A)

81. Ventilation perfusion ratio is maximum at :

(A) Apex of lung 

(B) Base of lung 

(C) Posterior lobe of lung 

(D) Middle of the lung 

Answer: (A)

82. Alveolar-arterial tension gradient increases in all of the following conditions except :

(A) Hypoventilation

(B) Rt-Lt shunt 

(C)  Diffusion defect 

(D) Ventilation perfusion abnormality 

Answer: (A)

83. Gallbladder contraction is stimulated by :

(A) Gastrin

(B) Secretin

(C) Vagus

(D) Cholycystokinin

Answer: (D)

84. Features of pyramidal tract lesion are all except :

(A) Clasp knife rigidity 

(B) Involuntary movements 

(C) Positive Babiniski sign 

(D) Exaggrated reflexes 

Answer: (B)

85. Motor aphasia refers to defect in :

(A) Peripheral speech apparatus 

(B) Verbal expression 

(C) Auditory comprehension 

(D) Verbal comprehension 

Answer: (B)

86. Ability to perceive shape and size is lost due to lesion of:

(A) Tractus gracilis

(B) Tractus cuniatus 

(C) Spinothalamic tract 

(D) Spinoreticular tract 

Answer: (B)

87. Group B muscle fibre are :

(A) Sympathetic pregaglionic 

(B) Sympathetic postganglionic 

(C) Parasympathetic preganglionic 

(D) Parasympathetic post ganglionic 

Answer: (A)

88. In hippocampus EEG waves are :

(A) α wave 

(B) β wave 

(C) Theta wave 

(D) Delta wave

Answer: (C)

89. Flocculo nodular lobe has direct connection with :

(A) Red nucleus 

(B) Inferior olivary nucleus 

(C) Vestibular nucleus 

(D) Dentate nucleus 

Answer: (C)

90. In moderate exercise the respiratory rate is increased due to response of :

(A) Propioception receptor in the joints 

(B) ↑PCO₂in arterial blood 

(C) ↑PO₂in arterial blood 

(D) J-receptor stimulation 

Answer: (A)

91. Endogenous non-shivering thermogens are secreted by all except :

(A) Liver 

(B) Spleen

(C) Heart

(D) Small intestine 

Answer: (B)

92. CO­2 is transported in blood mainly as :

(A) Bicarbonate

(B) Carbimino-compounds 

(C) Free CO₂ 

(D) Plasma-protein combination 

Answer: (A)

93. In  upper air way obstruction all of the following changes are seen except :

(A) Decreased Maximum breathing capacity 

(B) RV decreased 

(C) Decreased FEV 

(D) Decreased vital capacity 

Answer: (B)

94. CCK-PZ causes all of the following except :

(A) Gall bladder contraction 

(B) Pancreatic enzyme secretion 

(C)

(D) Decreased lower esophageal sphinictor 

Answer: (C)

95. Delta cells of pancreas secretes :

(A) Glucagon

(B) Insulin

(C) Somatostatin

(D) Pancreatic polypeptide

Answer: (C)

96. Which of the following is true about Nephron function

(A) Ascending thick limb is permeable to water 

(B) Descending thin limb is impermeable to water 

(C) Osmolality of intra-tubular content in DCT is more than surrounding interstitution 

(D) Osmolality of intratubular content in PCT is isotonic to surrounding interstitium 

Answer: (D)

97. Twitch of a single motor unit is called :

(A) Myoclonic-jerk 

(B) Fasciculation

(C) Tremor

(D) Chorea

Answer: (B)

98. Sympathetic stimulation causes all of the following except :

(A) Increase in heart rate 

(B) Increase in blood pressure 

(C) Increase in total peripheral resistance 

(D) Increase in venous capacitance 

Answer: (D)

99. G-CSF and GMCSF in haematopoisis causes :

(A) Leucocytosis

(B) Erythrocytosis

(C) Leucopenia

(D) Thrombocytosis

Answer: (A)

100. Pain-sensitive intracranial structure is :

(A) Piamater

(B) Pial vassels 

(C) Duramater 

(D) Brain matter 

Answer: (C)

101. Parasympathetic stimulation causes :

(A) Decrease GI secretion 

(B) Bronchodilation 

(C) Sweat-secretion

(D) Pupillary constriction

Answer: (D)

102. EEG waves from hippocampus are :

(A) α- Wave 

(B) β-Wave 

(C) Theta-Wave 

(D) Delta-Wave 

Answer: (C)

103. CSF/plasma glucose ratio is :

(A)  0.2-0.4 

(B) 0.6-.08 

(C) 1.2-1.6 

(D) 1.6-2.2 

Answer: (B)

104. Sperm acquires motility in :

(A) Seminal vesicle 

(B) Testes 

(C) Epidydimis

(D) Ejaculatory duct

Answer: (C)

105. Antibodies against sperms may develop after :

(A) Trauma

(B) Infection

(C) Vasectomy

(D) Orchidectomy

Answer: (C)

106. The gene coding for androgen receptors is located on :

(A) Short term of X- chromosome 

(B) Short arm of Y-chromosome 

(C) Long arm of X-chromosome 

(D) Long arm of Y-chromosome 

Answer: (C)

107. Cartoid sinus pressure causes:

(A) Reflex bradycardia 

(B) Breathing

(C) Heart rate 

(D) BP

Answer: (A)

108. Vitamin B12 is absorbed in:

(A) Duodenum

(B) Jejunum

(C) IIeum

(D) Stomach

Answer: (C)

109. Loop of Henle handles the following ions except:

(A) Na⁺ 

(B) K⁺ 

(C) Cl⁻ 

(D) Urea

Answer: (B)

110. Functional residual capacity of lung is defined as

(A) Volume expired after normal expiration 

(B) Volume remaining after forced expiration 

(C) ERV + RV 

(D) Tidal volume + volume inspired forcefully 

Answer: (B)

111. Ionophores have following actions except:

(A) Abolish proton gradient 

(B) Inhibit ADP to ATP conversion 

(C) Hydrophilic in character 

(D) Abolish pH gradient 

Answer: (C)

112. Apnoea is defined as:

(A) Stoppage of heart beat 

(B) Cessation of respiration 

(C) Irregular respiration 

(D) RR

Answer: (B)

113. Cardiac index ratio is determined by:

(A) CO & surface area 

(B) SV & surface area 

(C) Surface area only 

(D) Peripheral resistance 

Answer: (A)

114. Beri beri is caused by deficiency of

(A) Riboflavin

(B) Pantothenic acid 

(C) Folic acid

(D) Thiamine

Answer: (D)

115. True about cytokines are all except 

(A) It is polypeptide 

(B) Have autocrine and paracrine effects 

(C) Involved in Leukocytic movements 

(D) It takes part in intrinsic enzymatic reaction 

Answer: (D)

116. Parathyroid hormone is responsible for all actions except 

(A) Absorption of phosphorous increase 

(B) Vit D absorption increases 

(C) Mobilizes calcium from bone 

(D) Increase intestinal absorption of calcium 

Answer: (A)

117. Life span of neutrophil is :

(A) 6 hours 

(B) 6 days 

(C) 10 days 

(D) 15 days 

Answer: (A)

118. Helper and cytotoxic cells belong to:

(A) T cells 

(B) B cells 

(C) Monocytes

(D) Macrophages

Answer: (A)

119. Arterial blood O2 in ml of O2 per dL:

(A) 12.1 

(B) 19.8 

(C) 15.6 

(D) 27.8 

Answer: (B)

120. Longest transit time in GIT is seen in:

(A) Stomach

(B) Jejunum

(C) Colon

(D) Ileum

Answer: (C)

121. Protein synthesis occurs in:

(A) Smooth ER 

(B) Golgi bodies 

(C) Rough ER 

(D) Lysosomes 

Answer: (C)

122. Following are true about HCO3-except :

(A) Extracellular concentration 25 m mol 

(B) Intracellular concentration 10 m mol 

(C) 7.5% solution gives 2 nmols 

(D) In kidney, HCO 3-is produced by carbonic anhydrase 

Answer: (C)

123. Cardiac index is defined as:

(A) Stroke volume M²/BSA 

(B) C.O.P per unit body surface area 

(C) Syst press/M² BSA 

(D) End diastolic volume 

Answer: (B)

124. Chyme is propulsed forward in small intestine by:

(A) Segmentation

(B) Haustrations

(C) Migratory motor complexes (MMC)

(D)  Peristalsis

Answer: (D)

125. Pacemaker of the G.I.T is located in:

(A) Cardiac end of stomach 

(B) Long muscle of small intestine 

(C) Pyloric end of stomach 

(D) Fundus of stomach 

Answer: (D)

126. Not a vit. K dependent factor :

(A) II

(B) VII

(C) IX

(D) XII

Answer: (D)

127. Activity of factor VIII procoagulant is deficient in:

(A) Haemophilia

(B) Von Willibrand’s disease 

(C) ITP

(D) Sickle cell anemia 

Answer: (A)

128. Volume determining preload is :

(A) End diastolic volume of ventricles 

(B) End systolic volume 

(C) Volume of blood in aorta 

(D) Ventricular ejection volume 

Answer: (A)

129. True about Hb dissociation curve is :

(A) Acidosis shifts O₂ dissociation curve to right 

(B) ↑CO₂ shifts the curve to left 

(C) Hypoxia shifts curve to left  

(D) 2.3 DPG has no effect no curve 

Answer: (A)

130. Following are true about G protein except:

(A) G channels 

(B) Phosphorylase formation 

(C) Made up of 4 units 

(D) Related to ras oncogene 

Answer: (C)

131. True about facilitated diffusion are A/E:

(A) Occur in direction of concentration gradient 

(B) Does not require energy 

(C) Occur in direction opposite to electrical gradient 

(D) Facilitated by charge of molecule 

Answer: (C)

132. Least conduction velocity is seen in :

(A) AV node 

(B) Purkinje fibres 

(C) Bundle of his 

(D) Ventricular myocardial fibres 

Answer: (A)

133. Elimination waste product from a normal person requires minimal amount of urine of :

(A) 100 ml 

(B) 500 ml 

(C) 1000 ml 

(D)  2000 ml 

Answer: (B)

134. Epinephrine action in liver :

(A) Glycogenoloysis

(B) Gluconeogenesis

(C) Glycolysis

(D) Lipolysis

Answer: (A)

135. Enterogastric reflex is stimulated by all except :

(A) Alk. content of small int. 

(B) Hyperosmolarity of chyme 

(C) Distention of duodenum 

(D) None

Answer: (A)

136. Gaucher’s disease is due to deficiency of :

(A) β-glucosidase

(B) Sphingomyelin

(C) Aldolase

(D) G6PD

Answer: (A)

137. Diet devoid of carbohydrates causes :

(A) Ketosis

(B) DM

(C) No effect 

(D) Obesity

Answer: (A)

138. Blood pressure is defined as the product of :

(A) Systolic pressure × pulse 

(B) Diastolic pressure × pulse rate 

(C) Pulse pressure × pulse rate 

(D) Cardiac output × peripheral resistance 

Answer: (D)

139. In exercising muscle; true about metabolism is : 

(A) Same in aerobic and anaerobic 

(B) Fatty acids used mainly 

(C) Glycogen and creatine kinase used aerobically 

(D) All of the above 

Answer: (C)

140. Nitrogen washout method is used for estimating:

(A) Dead space volume 

(B) Functional residual capacity 

(C) Tidal volume 

(D) Diffusion capacity 

Answer: (B)

141. Repolarization in isolated muscle piece fibre proceeds from:

(A) Epicardium to endocardium 

(B) Epicardium to endocardium 

(C) Left to right 

(D) Right to left 

Answer: (B)

142. In sickle cell anemia replacement is :

(A) Valbglu in b6 

(B) Glu by val in b6 

(C) Hist val b8 

(D) 6A 

Answer: (B)

143. ADT text is for :

(A) Bence jounces protein 

(B) Fetal Hb 

(C) Albumin

(D) Myoglobin

Answer: (B)

144. Vasopressin acts by :

(A) Water transport across collecting duct 

(B) Water absorption at meduallary ducts 

(C) Water secretion at loop of Henle 

(D) Water transport at PCT 

Answer: (A)

145. If one of your nerves is compressed and this leads to paresthesia for some time, the type of nerve fiber affected is probably:

(A) A α 

(B) A α δ 

(C) C

(D) B

Answer: (A)

146. In normal adult Vd/Vt ratio is :

(A) 20

(B) 0.35 

(C) 40

(D) 50

Answer: (B)

147. The cardiac output can be determined by all except:

(A) Fick’s principle 

(B) V/Q ratio 

(C) Echocardiography 

(D) Thermodilution 

Answer: (B)

148. True about a blood flow in various organs:

(A) Liver > Kidney > brain > heart 

(B) Liver > brain > kidney > heart 

(C) Kidney > brain > heart > liver 

(D) Liver > heart > brain > kidney 

Answer: (A)

149. The ODC is shifted to the left by :

(A) Acidosis

(B) Raised pH 

(C) Raised CO2 

(D)Raised temperature 

Answer: (B)

150. Administration of pure of O2 to hypoxic patients is dangerous because :

(A) Apnea occurs due to hypostimulation of peripheral chemoreceptors 

(B) Pulmonary edema 

(C) DPG

(D) Convulsions

Answer: (A)

151. The feature of JVP in cardiac tamponade is

(A) Prominent x descent with prominent ‘y’ descent 

(B) Prominent x descent with absent ‘y’ descent 

(C) Absent x descent with prominent ‘y’ descent 

(D) Absent x descent with absent ‘y’ descent 

Answer: (B)

152. Sweating a result of exertion is mediated through 

(A) Adrenal hormones 

(B)  Sympathetic cholinergic

(C) Sympathetic adrenergic 

(D) Parasympathetic cholinergic 

Answer: (B)

153. Hyperbaric oxygen is dangerous because it :

(A) Decreases displacement of O₂ from Hb 

(B)  Decreases respiratory drive 

(C) Enzyme damage 

(D) Is toxic to tissures 

Answer: (D)

154. Pressure on carotid sinus cause :

(A) Hyperapnea

(B)  Reflex bradycardia 

(C) Tachycardia

(D) Dyspnea

Answer: (B)

155. Hpocalcemia due to calcitonin is by :

(A) Increased excretion in kidney 

(B) Decreased bone resorption

(C) Decreased intestinal absorption 

(D) Decreased renal reabsorption 

Answer: (B)

156. Most susceptible to hypoxia is :

(A) Thalamus

(B) Caudate

(C) Cerebellum

(D)  Hippocampus

Answer: (D)

157. Calcium absorption is from :

(A) Proximal small intestine 

(B) Distal lleum 

(C) Middle small intestine 

(D) Ascending colon 

Answer: (A)

158. Unlikely to be involved in lesion of anterior spinal artery is :

(A) Pain and temp

(B) Vibration and proprioception 

(C) Pyramidal tract 

(D) Sphincters

Answer: (B)

159. Best test for GFR is with :

(A) Inulin

(B) Hippuric acid 

(C) Cratinine

(D) PAH

Answer: (A)

160. Basal cardiac output in an adult in nearly :

(A) 7.5 litre 

(B) 5 litre 

(C) 12 litre 

(D) 10 litre 

Answer: (B)

161. ACTH level is highest during :

(A) Early morning 

(B) Evening

(C) Afternoon

(D) Night

Answer: (A)

162. Acromegaly is due to excess of :

(A) Somatomedin

(B) Growth hormone 

(C) Somatostatin

(D) Insulin

Answer: (B)

163. All are vitamin K dependent clotting factors of hepatic origin except :

(A) II

(B) VII

(C) VIII

(D) X

Answer: (C)

164. Reperfusion injury is caused by :

(A) Vitamin E 

(B) Superoxide ion 

(C) Calcium ion 

(D) Magnesium ion 

[bg_colMagnesium ion lapse view=”button-orange” color=”#4a4949″ icon=”arrow” expand_text=”Show Answer” collapse_text=”Hide Answer” ]

Answer: (B)

[/bg_collapse]

165. All are controlled by autonomic nervous system except :

(A) Aldosterone

(B) Insulin

(C) GH

(D) Somatostatin

Answer: (A)

166. cAMP action mediates all except :

(A) Glucagon

(B) FSH

(C) LH

(D) Estrogen

Answer: (D)

167. Maximum contraction of gall bladder is seen with :

(A) CCK

(B) Secretin

(C) Gastrin

(D) Enterogastrone

Answer: (A)

168. Physiological gastrectomy is :

(A) Ligate all major arteries 

(B) Antrectomy

(C) Upper 1/3 of stoma resected 

(D) Ligation of 4-out of five arteries 

Answer: (A)

169. Exocytosis :

(A) Extrusion of cell bound vesicles 

(B) Intrusion of liquid particles 

(C) Instrusion of solid particles 

(D) All of the above 

Answer: (A)

170. O2 consumption is minimal when temp. gradient between skin and ext. env. is :

(A) < 0.5⁰ 

(B) > 2.5⁰ 

(C) < 1.5⁰ 

(D) > 3.5⁰ 

Answer: (A)

171. Max. blood supply to liver is by :

(A) Portal v. 

(B) Hepatic A. 

(C) Splenic A. 

(D) Mesenteric A. 

Answer: (A)

172. What is true of iron :

(A) It is stored in ferritin 

(B) It is absorbed by Transferrin in the intestine 

(C) Spleen is major storage organ 

(D) Fe++ is excreted in urine 

Answer: (A)

173. Erythropoietin is secreted by A/E :

(A) Hemangioblastoma

(B) Hepatoma

(C) Renal cell carcinoma

(D) Adrenocortical tumours 

Answer: (D)

174. Which is true about rennin :

(A) It helps to convert angiotensinogen to Angiotensin – I 

(B) Secreted by PCT 

(C) ↑GFR causes ↑ secretion of renin 

(D) ↓plasma Na⁺ & H₂O 

Answer: (A)

175. Nitric Oxide is produced in :

(A) Endothelium

(B) Plasma

(C) Platelets

(D) Serum 

Answer: (A)

176. Maximum water absorption takes place in :

(A) Proximal convoluted tubule 

(B) Distal convoluted tubule 

(C) Collecting duct 

(D) Loop of Henle 

Answer: (A)

177. True about Insulin action is :

(A) Causes neoglucogenesis 

(B) Not useful for growth & development 

(C) Required for transport of glucose, aminoacid, K⁺ & Na⁺ 

(D) Catabolic hormone 

Answer: (C)

178. Aggrecan is 

(A) Receptor over platelets 

(B) A molecule present in osteoid tissue 

(C) Granules in leukocytes 

(D) Important component of cartilage 

Answer: (D)

179. Which of the following is most prone to hypoxic injury :

(A) Thalamus

(B) Hippocampus

(C) Caudate nucleus 

(D) Cerebellum

Answer: (B)

180. Function of hepatic stellate cells is/are :

(A) Formation of sinusoides 

(B) Vit-A storage 

(C) Increases blood perfusion 

(D) Increases blood perfusion 

Answer: (B)

181. Function of peritoneum are A/E :

(A) Lubrication

(B) Hormone release 

(C) Pain sensitive 

(D) Enzymatic digestion

Answer: (B)

182. Central cyanosis is seen if :

(A) Methemoglobin 0.5 gm/dl 

(B) O₂ saturation < 85% 

(C) O₂ saturation < 94% 

(D) Hb – 4 gm% 

Answer: (B)

183. Which of the following act through tyrosine kinase receptor :

(A) Insulin

(B) Glucagon

(C) GH

(D) FSH

Answer: (A)

184. True about Glutathione reductase :

(A) Sulphur containing enzyme 

(B) Important in methemoglobinemia 

(C) Free radical scavenger 

(D) All

Answer: (C)

185. Thymidylated RNA is present in :

(A) mRNA

(B) rRNA

(C) tRNA

(D) 16-s-RNA 

Answer: (C)

186. Which of the following organ is not involved in Calcium metabolism :

(A) Lung

(B) Liver

(C) Spleen

(D) Skin

Answer: (C)

187. Volume of air taken in and given out during normal respiration is reffered to as :

(A) IRV

(B) TV

(C) ERV

(D) VC

Answer: (B)

188. In a patient with a transplanted heart which of these are the reasons for increased cardiac output during exercise :

(A) Reinnervation of transplanted heart by vagus 

(B) Intrinsic mechanism 

(C) Epinephrine from medulla 

(D) Bainbridge reflex 

Answer: (C)

189. Hormones secreted by adrenal medulla are :

(A) Glucagon

(B) Cortisol

(C) Norepinephrine

(D) Insulin

Answer: (C)

190. UMN includes :

(A) Pyramidal cells

(B) Peripheral nerves 

(C) Anterior horn cells 

(D) Glial Cells 

Answer: (A)

191. Functions of basal ganglia include :

(A) Gross motor 

(B) Skilled movements 

(C) Emotions

(D) Maintenance of equilibrium 

Answer: (B)

192. All of these cause hyperglycemia except :

(A) GH

(B) Cortisol

(C) Catecholamines

(D) Insulin

Answer: (D)

Medical PG Pathology

Medical PG Pathology

1. Programmed cell death is also called as :

(A) Apoptosis

(B) Necrosis

(C) Degeneration

(D) Calcification

Answer: (A)

2. Haemorrhagic infarct may be seen in :

(A) Brain

(B) Lung

(C) Spleen

(D) Heart

Answer: (B)

3. All of the following constitute familial cancer syndrome except :

(A) Xeroderma-pigmentosum

(B) Retinoblastoma

(C) Neurofibromatosis

(D) MEN-I

Answer: (A)

4. Howel-Jolly bodies may be seen after :

(A) Hepatectomy

(B) Splenectomy

(C) Pancreatectomy

(D) Cholecystectomy

Answer: (B)

5. ‘Intermediate form’ of Non hodgkin’s lymphoma is :

(A) Small noncleaved cell

(B) Diffuse, small cleaved cell

(C) Lymphoblastic

(D) Large cell immunoblastic

Answer: (B)

6. ‘Hairy cell leukemia’ is a neoplastic proliferation of :

(A) T.cells

(B) B. cells

(C) Myeloid cells

(D) Macrophages

Answer: (B)

7. Characteristic feature of Rheumatic carditis is :

(A) Pericarditis

(B) Endocarditis

(C) Myocarditis

(D) Pancrditis

Answer: (C)

8. Most common tumour of heart is :

(A) Myxoma

(B) Rhabdomyosarcoma

(C) Fibroma

(D) Leiomyosarcoma

Answer: (A)

9. Almost always associated with one of the following conditions :

(A) Diabetes-mellitus

(B) Analgesic-nephropathy

(C) Chronic pyelonephirits

(D) Post streptococcal GN

Answer: (A)

10. Which of the following types of glomerulonephiritis is most likely to cause CRF all except

(A) Post streptococcal glomerulonephritis

(B) Membranous GN

(C) Membrano proliferative GN

(D) Focal segmental glomerulosclerosis

Answer: (A)

11. In Leprosy most common renal lesion seen is :

(A) MGN

(B) MPGN

(C) Focal glomeruloselerosis

(D) Diffuse glomerulosclerosis

Answer: (A)

12. Centrilobular necrosis of liver may be seen with :

(A) Phosphorus

(B) Arsenic

(C) CCl₄

(D) Ethanol

Answer: (C)

13. Kaposi sarcoma associated with gut may be seen in

(A) Nonhodgkin’s lymphoma

(B) HIV-infection

(C) Fungal infection

(D) Keratoacanthosis

Answer: (B)

14. Most common histological types of lung carcinoma in India is:

(A) Squamous cell CA

(B)Adeno carcinoma

(C) Small cell CA

(D) Large cell CA

Answer: (A)

15. Which of the following histological type lung carcinoma has worst prognosis :

(A) Squamous cell CA

(B) Adenocarcinoma

(C) Alveolar-carcinoma

(D) Small cell carcinoma

Answer: (D)

16. Characteristic pathological feature of Parkinson’s disease is presence of :

(A) Lewy bodies

(B) Babes nodule

(C) Neuro fibrillary tangle

(D) Negri-bodies

Answer: (A)

17. Pathological manifestation of chronic alcoholism include all of the following except :

(A) Piecemeal necrosis

(B) Balloning degeneration

(C) Microvesicular fatty changes

(D) Central hyaline sclerosis

Answer: (A)

18. Pale infract is not seen in

(A) Heart

(B) Lung

(C) Kidney

(D) Spleen

Answer: (B)

19. In an inflammatory response, Macrophages are usually derived from

(A) Monocytes

(B) Reticuloendothelial cells

(C) Neutrophils

(D) Lymphocytes

Answer: (A)

20. Heart failure cells contain

(A) Hemosiderin

(B) Lipofuschin

(C) Myoglobin

(D) Albumin

Answer: (A)

21. The most common site of Myocardial infarction is

(A) Anterolateral

(B) Interventricular septum

(C) Posterior wall

(D) Inferior wall

Answer: (A)

22. Most common cause of Abdominal aorta aneurysm is

(A) Atherosclerosis

(B) Trauma

(C) Marfan’s syndrome

(D) Congenital

Answer: (A)

23. Commonest cause of SVC obstruction is

(A) Fibrosis of SVC

(B) Bronchogenic carcinoma

(C) Pericardial fibrosis

(D) hepatocellular carcinoma

Answer: (B)

24. The cells seen after 72 hours in the Infracted area in MI are

(A) Neutrophils

(B) Lymphocytes

(C) Macrophages

(D) Monocytes

Answer: (C)

25. The least common site of Berry aneurysm is

(A) Anterior communicating artery

(B) Vertebral artery

(C) Basilar artery

(D) Middle cerebral artery

Answer: (B)

26. The commonest tumor of the myocardium is

(A) Myxoma

(B) Rhabdomyoma

(C) Sarcoma

(D) Fibroma

Answer: (A)

27. All of the following decrease in Nephrotic syndrome except

(A) Fibrinogen

(B) Thyroxin

(C) Transferrin

(D) Albumin

Answer: (A)

28. All are features of RPGN except

(A) Rapid recovery

(B) Crescent formation

(C) Hypertension

(D) Non-selective proteinuria

Answer: (A)

29. Which characteristic feature is seen in the kidney in Malignant hypertension

(A) Hyaline necrosis

(B) Fibrinoid necrosis

(C) Medial wall hyperplasia

(D) Micro-aneurysm

Answer: (B)

30. Most common factor in development of Diabetic nephropathy is

(A) Previous kidney disease

(B) Associated hypertension

(C) Duration of diabetes

(D) Control with treatment

Answer: (C)

31. The HLA important in IDDM is

(A) HLA-A₃

(B) HLA-B₂₇

(C) HLA-DR₃/DR₄

(D) HLA-W₁

Answer: (C)

32. The commonest Soft tissue tumor in adults is

(A) Lipoma

(B) Histiocytoma

(C) Fibroma

(D) Leiomyoma

Answer: (A)

33. Malignancy of the Breast is likely to be associated with

(A) Sclerosing adenosis

(B) Atypical epithelial hyperplasia

(C) Cystic change

(D) Apocrine metaplasia

Answer: (B)

34. All of the following Paraneoplastic syndromes are seen in Carcinoma lung except

(A) Hypertrophic osteodystrophy

(B) Myasthenia gravis

(C) Cushing’s syndrome

(D) Hypoglycemia

Answer: (D)

35. The commonest site of Pleomorphic adenoma is

(A) Parotid salivary gland

(B) Submandibular salivary gland

(C) Sublingual gland

(D) Submaxillary gland

Answer: (A)

36. MEN type I includes tumors of all except

(A) Parathyroid

(B) Pituitary

(C) Pancreas

(D) Medullary carcinoma of thyroid

Answer: (D)

37. Primary pleural tumor is

(A) Mesothelioma

(B) Myxoma

(C) Lipoma

(D) Fibroma

Answer: (A)

38. Mucin is not secreted by

(A) Goblet cell

(B) Paneth cell

(C) Brunner’s gland

(D) Crypts of Lieberkuhn

Answer: (B)

39. Pseudopolyps are features of 

(A) Crohn’s disease

(B) Ulcerative colitis

(C) Celiac sprue

(D) Whipple’s disease

Answer: (B)

40. Transport of lipids from the intestine to other tissues is by

(A) Chylomicrons

(B) LDL

(C) HDL

(D) VLDL

Answer: (A)

41. Dystrophic calcification is seen in

(A) Milk-alkali syndrome

(B) Atheromatous plaque

(C) Hyperparathyroidism

(D) Vitamin A intoxication

Answer: (B)

42. Cytokeratin is seen in

(A) Neurofibroma

(B) Lymphoma

(C) Carcinoma

(D) Fibrosarcoma

Answer: (C)

43. Pale infarct is not seen in

(A) Lung

(B) Liver

(C) Spleen

(D) Heart

Answer: (A)

44. Major fibril protein in Primary Amyloidosis is 

(A) AL

(B) AA

(C) Transthyretin

(D) Procalcitonin

Answer: (A)

45. Laminin is found in the

(A) Outer nuclear membrane

(B) Inner nuclear membrane

(C) Basement membrane

(D) Endothelium

Answer: (C)

46. The most common cause of SVC syndrome is

(A) Thrombosis

(B) Extrinsic compression

(C) Mediastinal lymphoma

(D) Teratoma

Answer: (B)

47. In Marfan’s syndrome, Aortic aneurysm occurs most commonly in:

(A) Ascending aorta

(B) Descending aorta

(C) Abdominal aorta

(D) Arch of aorta

Answer: (A)

48. McCallum’s patch is diagnostic of

(A) Infective endocarditis

(B) Rheumatic endocarditis

(C) Myocardial infarction

(D) Tetralogy of Fallot(ToF)

Answer: (B)

49. Calcification of the aortic valve is seen in

(A) Aortic stenosis

(B) Aortic regurgitation

(C) Marfan’s syndrome

(D) Hurler’s syndrome

Answer: (A)

50. Which is a component of ToF

(A) Aorta arises from RV

(B) LVH

(C) VSD

(D) ASD

Answer: (C)

51. The following carcinomas are associated with DIC except

(A) Stomach

(B) Pancreas

(C) Pancreas

(D) Breast

Answer: (D)

52. The pathological change in malignant hypertension is

(A) Hyperplastic arteriosclerosis

(B) Cystic medial necrosis

(C) Benign nephrosclerosis

(D) Hyaline arteriosclerosis

Answer: (A)

53. All of the following may be involved in PAN except

(A) Kidney

(B) Heart

(C) Liver

(D) Lung

Answer: (D)

54. Pathological changes of diabetic nephropathy are all except

(A) Fibrin caps and capsular drops

(B) Intercapillary glomerulosclerosis

(C) Focal sclerosis

(D) Capillary basement membrane thickening

Answer: (C)

55. True about Light microscopic changes in Minimal Change Glomerulonephritis is

(A) No abnormality

(B) Fusion of foot process

(C) Absence of Immunoglobulins

(D) Absence of complement

Answer: (A)

56. The following are seen in chronic obstructive pulmonary disease(COPD) except

(A) Hemoptysis

(B) Cor Pulmonale

(C) Pneumothorax

(D) Aplastic anemia

Answer: (D)

57. Deficiency of the ‘intrinsic factor of Castle’ causes

(A) Megaloblastic anemia

(B) Pernicious anemia

(C) Cooley’s anemia

(D) Aplastic anemia

Answer: (B)

58. The most pre-cancerous condition for carcinoma colon is

(A) Familial polyposis

(B) Hamartomatous polyps

(C) Juvenile polyps

(D) Hyperplastic polyps

Answer: (A)

59. In Peutz-Jeghers syndrome, polyps are seen in

(A) Colon

(B) Rectum

(C) Small bowel

(D) Stomach

Answer: (C)

60. Which is NOT a component of Werner’s syndrome

(A) Pituitary

(B) Adrenal

(C) Parathyroid

(D) Pancreas

Answer: (B)

61. Malignancy in pheochromocytoma is indicated by

(A) Vascular invasion

(B) Mitotic figures

(C) Capsular invasion

(D) Metastasis

Answer: (D)

62. An Autosomal Dominant condition is

(A) Albinism

(B) Huntington’s chorea

(C) Hurler’s syndrome

(D) Hunter’s syndrome

Answer: (B)

63. Which is not a type III hypersensitivity reaction

(A) TB

(B) Rheumatoid arthritis

(C) SLE

(D) Arthus reaction

Answer: (A)

64. Russell’s body is found in 

(A) WBC

(B) RBC

(C) Mast cell

(D) Plasma cell

Answer: (D)

65. The function of oligodendrocyte is

(A) Myelinates the CNS

(B) Nutrition of nervous tissue

(C) Lining the cavities of the CVS

(D) Behave like macrophages

Answer: (A)

66. Triple helix structure is found in

(A) Keratin

(B) Collagen

(C) Proline

(D) Cellulose

Answer: (B)

67. Function of Langerhans cells are

(A) Antigen presentation

(B) Phagocytosis

(C) Autoimmune

(D) None of the above

Answer: (A)

68. Russell bodies are found in

(A) Multiple Myeloma

(B) Gonadal tumor

(C) Parkinsonism

(D) Intracranial neoplasms

Answer: (A)

69. The correct statement regarding fat distribution in cold-water channel swimmers is

(A) Preferentially deposited in subcutaneous tissue

(B) Accumulated around the abdominal viscera

(C) Concentrated retroperitoneally

(D) Deposited preferentially in the liver

Answer: (A)

70. Macro-vesicular fatty liver is seen in

(A) Protein-Energy malnutrition

(B) Viral hepatitis

(C) Acute fatty liver of Pregnancy

(D) Reye’s syndrome

Answer: (A)

71. Vitamin K-dependent factor is

(A) II

(B) III

(C) IV

(D) VI

Answer: (A)

72. The most common pre-malignant condition of oral carcinoma is

(A) Leukoplakia

(B) Erythroplakia

(C) Lichen planus

(D) Fibrosis

Answer: (A)

73. The following are predisposing factors for Esophageal carcinoma except

(A) Plummer-Vinson syndrome

(B) Tylosis palmaris

(C) Chronic Achalasia

(D) Benzene therapy

Answer: (D)

74. Amongst the following, the most common site for Leiomyoma is

(A) Stomach

(B) Small Intestine

(C) Duodenum

(D) Colon

Answer: (A)

75. Skip granulomatous lesions are seen in

(A) Ulcerative colitis

(B) Crohn’s disease

(C) Whipple’s disease

(D) Reiter’s disease

Answer: (B)

76. Toxic megacolon is most commonly associated with

(A) Ulcerative colitis

(B)  Crohn’s disease

(C) Whipple’s disease

(D) Reiter’s disease

Answer: (A)

77. Clinical features of minimal change glomerulonephritis are all except

(A) Hypertension

(B) Edema

(C) Selective proteinuria

(D) Fever

Answer: (A)

78. All are steroid resistant except

(A) Post-streptococcal glomerulonephritis

(B) Minimal change glomerulonephritis

(C) RPGN

(D) Recurrent hematuria

Answer: (B)

79. Post-infective glomerulonephritis present as 

(A) ARF

(B) Nephrotic syndrome

(C) Nephritic syndrome

(D) Asymptomatic hematuria

Answer: (C)

80. An important factor for chronic pyelonephritis is

(A) Obstruction

(B) Vesico-ureteral reflux

(C) Pelvi-ureteric junction obstruction

(D) Catheter induced

Answer: (B)

81. Flea-bitten kidney is seen in all except

(A) SABE

(B) Malignant hypertension

(C) Hemolytic Uremic Syndrome

(D) Diabetic nephropathy

Answer: (D)

82. A testicular tumor marker is

(A) α-Fetoprotein

(B) Ectopic hormones

(C) CEA

(D) Testosterone

Answer: (A)

83. Which Hodgkin’s disease is associated with best prognosis

(A) Lymphocytic depletion

(B) Lymphocyte depletion

(C) Mixed cellularity

(D) Nodular sclerosis

Answer: (A)

84. Ectoptic ACTH production is seen in

(A) Small cell carcinoma is lung

(B) Anaplastic carcinoma of lung

(C) Squamous cell carcinoma of lung

(D) Adenocarcinoma of lung

Answer: (A)

85. The nucleus involved in Alzheimer’s disease is 

(A) Basal nucleus of Mayernet

(B) Raphe nucleus

(C) Superior salivary nucleus

(D) Basal lobe of cerebellum

Answer: (A)

86. The most common site for myocardial infarction is 

(A) Left atrium

(B) Left ventricle

(C) Right atrium

(D) Right ventricle

Answer: (B)

87. Coagulative necrosis may be seen in all of the following except :

(A) Myocardial infarction

(B) Thermal injury

(C) Tuberculosis

(D) Zenker’s degeneration necrosis

Answer: (D)

88. Dystrophic calcification is seen in :

(A) Atheromatous plaques

(B) Gastric mucosa

(C) Normal tissue

(D) Lung

Answer: (A)

89. Langerhans cells are :

(A) Antigen presenting cells

(B) Phagocytosing cells

(C) Autoimmune cells

(D) Cell in acute infection

Answer: (A)

90. Increased capillary permeability is caused by all of the following except :

(A) Anaphylatoxin

(B) 5 hydroxytryptamine

(C) Renin

(D) Histamines

Answer: (C)

91. Tumour positive for cytokeration are likely to be:

(A) Adenoma

(B) Melanoma

(C) Carcinoma

(D) Lipoma

Answer: (C)

92. Pale infarcts are seen at all of the following sites except :

(A) Heart

(B) Spleen

(C) Kidney

(D) Lung

Answer: (D)

93. All of the following statements are true for cell againg except :

(A) Enlargement of telomere

(B) ↓number of mitochondria

(C) Glycolysation of DNA

(D) Glycolysation of RNA

Answer: (A)

94. All of the following gases contribute to green house effect except :

(A) Ozone

(B) Methane

(C) N₂O

(D) Chloroflorocarbons

Answer: (C)

95. Most commonly employed stain for amyloidosis is :

(A) Congored with polarized light

(B) Congored with non polarized light

(C) Sudan black

(D) Giemsa staining

Answer: (A)

96. All of the following statements regarding amyloidosis are true except :

(A) Multiple myeloma – shows AL type deposits

(B) Secondary amyloidosis-shows AA type deposits

(C) Renal amyloidosis commonly presents with hypertension

(D) Renal amyloidosis commonly presents with mida proteinuria

Answer: (C)

97. All of the following are decreased in Nephrotic syndrome except :

(A) Transferrin

(B) Ceruloplasmin

(C) Albumin

(D) Fibrinogen

Answer: (D)

98. All of the following are causes of granular contracted kidney except :

(A) Benign nephrosclerosis

(B) Chronic pylonephritis

(C) Diabetes mellitus

(D) Chronic glomerulonephritis

Answer: (C)

99. Salt losing nephritis s a feature of :

(A) Interstitial nephritis

(B) Amyloidosis of kidney

(C) Post streptococcal GN

(D) PEM

Answer: (A)

100. Common pathological changes seen in kidney in benign hypertension are :

(A) Fibronoid necrosis

(B) Microaneurysm

(C) Hyaline arteriosclerosis

(D) Thining of walls

Answer: (C)

101. All of the following conditions may show Mallory Hyaline changes except :

(A) Wilson disease

(B) Indian childhood cirrhosis

(C) Primary biliary cirrhosis

(D) Hepatitis E

Answer: (D)

102. Liver biopsy is carried out at 8th intercostals space in order to :

(A) Prevent pleural damage

(B) Prevent lung injury

(C) Pass through hepatorenal pouch

(D) Pass through bare area of liver

Answer: (B)

103. Thick mucous coat is :

(A) Protective in ulcer patients

(B) Not protective in ulcer patients

(C) Easily destroyed by antacids

(D) Commonly associated with carcinomatous change

Answer: (A)

104. Protective epithelium :

(A) Has regenerative power

(B) Does not have regenerative power

(C) Does not shed off easily

(D) Bleeding is common

Answer: (A)

105. Most common carcinoma of breast is 

(A) Intra-ductal carcinoma

(B) Colloid carcinoma

(C) Lobular carcinoma

(D) Sarcoma phylloides

Answer: (A)

106. Commonest type of lung carcinoma in nonsmokers is 

(A) Squamous cell CA

(B) Small cell CA

(C) Adeno CA

(D) Alveolar cell CA

Answer: (C)

107. Diagnostic feature in rheumatic heart disease is

(A) Aschoff’s nodule

(B) Mc Callumn patch

(C) Bread & butter pericarditis

(D) Shaggy vegetation

Answer: (A)

108. Least common site for berry aneurysm is :

(A) Vertebral artery

(B) Basilar artery

(C) Junction of anterior cerebral artery and internal carotidartery

(D) Posterior cerebral artery

Answer: (A)

109. In Alzheimer’s diseases the likely site of lesion is the :

(A) Basal nucleus of Mayernet

(B) Pons

(C) Basal ganglia

(D) Partial lobes of cerebrum

Answer: (A)

110. ‘Biological membranes’ are characterized by all of the following :

(A) They are symmetric bileaflet structures

(B) Their constituent lipid and protein moities are held together by covalent interactions.

(C) They are rigid assemblies of protein, lipid and carbohydrates

(D) Their lipid moieties are amphipathic in nature

Answer: (D)

111. Hypertriglyceridemia is seen in :

(A) LDL receptor defect

(B) Dysbetalipoprotenemia

(C) Abetalipoproteinemia

(D) all of the above

Answer: (B)

112. Apoptotic bodies are :

(A) Clumped chromatin bodies

(B) Pyknotic nucleus without organelles

(C) Cell membrane bound with organelles

(D) No nucleus with organelles

Answer: (C)

113. Langerhan’s cells are :

(A) Phagocytic cells

(B) Antigen presenting cells

(C) Seen in auto immune conditions

(D) Seen in chronic infection

Answer: (B)

114. Gittre cells are synonymous with :

(A) Ependymal cells

(B) Oligodendrocytes

(C) Astrocytes

(D) Modified CNS macrophages

Answer: (D)

115. Phagocytosis is the function of :

(A) Astrocytes

(B) Oligodendrocytes

(C) Microglia

(D) Schawan cells

Answer: (C)

116. Epitheloid granuloma may be seen in all of the following conditions except :

(A) Sarcoidosis

(B) Tuberculosis

(C) Pneumocystic carini

(D) Hodgkin’s lymphoma

Answer: (D)

117. Triple helix is found in :

(A) Cystine

(B) Collagen

(C) Pectin

(D) DNA

Answer: (B)

118. “Tertiary hyperparathyroidism’ refers to :

(A) Primary hyperparathyroidism with decrease Ca⁺⁺level

(B) Secondary hyperparathyroidism following chief cell adenoma

(C) Secondary hyperparathyroidism following intestinal malabsorption

(D) Metastasis with normal phosphate level metastasis

Answer: (B)

119. Lack of leucocyte adhesion molecules (LAM) is associated with :

(A) Delayed closure of umblical cord

(B) Normal chemotaxis

(C) Compliment opsinization

(D) Neutropenia

Answer: (A)

120. Maximum malignant potential is in :

(A) Superficial naevus

(B) Epidermal naevus

(C) Junctional naevus

(D) Intradermis naevus

Answer: (C)

121. Most common cause of aortic aneurysm is :

(A) Syphilis

(B) Marfan’s syndrome

(C) Atherosclerosis

(D) Congenital

Answer: (C)

122. Most common site of myocardial infaction is :

(A) Anterior wall of left ventricle

(B) Posterior wall of left ventricle

(C) Posterior wall of right ventricle

(D) Inferior wall of left ventricle

Answer: (A)

123. Most common type of hodgkin’s lymphoma in India is :

(A) Nodular sclerosing

(B) Lymphocyte predominance

(C) Mixed cellularity

(D) Lymphocyte depletion

Answer: (C)

124. Most malignant form of NHL is :

(A) Diffuse large cell

(B) Small cell lymphocytic lymphoma

(C) Follicular cleavage

(D) Large cell follicular

Answer: (A)

125. Fibrous histiocytoma is classified as :

(A) Haemangio pericytoma

(B) Sclerosing haemangioma

(C) Angiomyolipoma’s

(D) Angiomyosarcoma

Answer: (B)

126. Commonest site of lytic lesion in multiple myeloma is :

(A) Vertebral column

(B) Femur

(C) Clavicle

(D) Pelvis

Answer: (A)

127. Most common site of Leomyoma is :

(A) Stomach

(B) Ileum

(C) Colon

(D) Rectum

Answer: (A)

128. Common site of regional enteritis is :

(A) Colon

(B) Rectum

(C) Distal ileum and colon

(D) Caecum

Answer: (C)

129. Toxic megacolon is seen in :

(A) Chronic nonspecific ulcerative colititis

(B) Crohn’s disease

(C) Colonic diverticulosis

(D) Hamartomatous polyp

Answer: (A)

130. The pathological appearance in pseudomembranous colitis consists of :

(A) Necrosis and gangrene

(B) Small ulceration with slough

(C) Serositis with covered by membrane

(D) Excessive ulceration in serosa

Answer: (B)

131. Maximum endocapillary proliferation is a feature of :

(A) Membranous GN

(B) Measangio proliferation GN

(C) Focal sclerosis

(D) Post streptococcai GN

Answer: (D)

132. ‘Flea bitten’ appearance of kidney may be seen in all except :

(A) Malignant hypertension

(B) SABE

(C) Diabetes mellitus

(D) PAN

Answer: (C)

133. Most common pathological features in diabetes melitus is :

(A) Papillary necrosis

(B) Diffuse glomerulosclerosing

(C) Renal atherosclerosis

(D) Chronic pyelonephritis

Answer: (B)

134. Differentiating point between invasive carcinoma and carcinoma is situ is:

(A) Mitosis

(B) Penetration of basement membrane

(C) Cellular atypia

(D) Nuclear pleomorphism

Answer: (B)

135. Amyloidosis is found in the following except:

(A) Multiple myeloma

(B) Hypernephroma

(C) Thymoma

(D) Lymphoma

Answer: (C)

136. In DIC, following are seen except:

(A) Fibrinogen decreased

(B) Thrombocytopenia

(C) Normal APTT

(D) PT elevation

Answer: (C)

137. Pathological change in brain in cerebral malaria is:

(A) Cerebral edema

(B) Microvascular occlusion

(C) ICT

(D) Space occupying lesion

Answer: (B)

138. Pulmonary infarction and pulmonary edema is differentiated by

(A) Heart failure cells

(B) Necrotising alveolar sac

(C) Capillary congestion

(D) Pulmonary wedge pressure

Answer: (A)

139. Macrophage, granuloma erythrophagocytosis are found in:

(A) Ulcerative colitis

(B) Necrotising enterocolitis

(C) Regional ileitis

(D) Typhoid

Answer: (C)

140. Massive transfusion causes the following except:

(A) DIC

(B) Hypothermia

(C) Hypokalemia

(D) Hyperkalemia

Answer: (C)

141. Reed sternberg cells are found in:

(A) Hodgkin’s disease

(B) Sickle cell anaemia

(C) Thalassemia

(D) CML

Answer: (A)

142. Broad cast is characteristic of:

(A) CRF

(B) Ac GN

(C) ARF

(D) Renal TB

Answer: (A)

143. In rabies pathological lesions in CNS are

(A) Brainstem encephalitis

(B) Cranial nerve arteritis

(C) Neuronal loss

(D) Neurofibrillary tangles

Answer: (A)

144. Hemophilia manifest clinically as rise in

(A) APTT

(B) PT

(C) CT

(D) FDP

Answer: (A)

145. In malignant HTN, seen is:

(A) Hyalinised arterioles

(B) Replacement by necrotizing arteriolitis

(C) Calcification

(D) Atherosclerosis

Answer: (B)

146. True about integrin is

(A) Used in binding

(B) Oncogene

(C) Anti oncogene

(D) Component of basement membrane

Answer: (A)

147. Sezary syndrome included in category of 

(A) T cell leukaemia

(B) Lymphoma

(C) B cell leukaemia

(D) Pigmented disorder of skin

Answer: (A)

148. Non specific esterase in present in :

(A) Megakaryocytic leukaemia

(B) Lymphocytic leukaemia

(C) Erythroleukaemia

(D) AML

Answer: (D)

149. Starting point of apoptosis for programme cell death is:

(A) Activation of endonuclease

(B) Release of enzyme

(C) Accumulation of calcium

(D) Destruction by macrophages

Answer: (A)

150. Lead causes following except :

(A) Uroporphyrinuria

(B) Sideroblastic anemia

(C) Basophilic stippling

(D) Macrocytic anemia

Answer: (D)

151. Coomb’s positive hemolysis is seen in:

(A) TTP

(B) SLE

(C) ITP

(D) PNH

Answer: (B)

152. In IgA nephropathy (Berger’s disease, there are:

(A) Subepithelial deposits

(B) Subendothelial deposits

(C) Mesangial deposits

(D) Basement membrane deposits

Answer: (C)

153. The cause of scleritis is:

(A) Collagen vascular disease

(B) TB

(C) Trauma

(D) Glaucoma

Answer: (A)

154. Delayed type of hypersensitivity is seen in following except:

(A) Arthus phenomenon

(B) Contact dermatitis

(C) Tuberculin test

(D) Graft vs host reaction

Answer: (A)

155. In shock, characteristic feature is:

(A) Cardiac failure

(B) Poor perfusion of tissues

(C) Cyanosis

(D) Oedema

Answer: (B)

156. Histiocytosis X is seen in except :

(A) Hand schuller Christian disease

(B) Eosinophilic granuloma

(C) Letter-siwe syndrome

(D) Torres syndrome

Answer: (D)

157. Defect leading to thalassemia lies in

(A) Haemoglobin

(B) Osmotic fragility

(C) RBC membrane

(D) Platelets

Answer: (A)

158. Hereditary spherocytosis is due to:

(A) Spectrin deficiency

(B) Integrin defect

(C) Collagen defect

(D) Defect with glycoprotein

Answer: (A)

159. Palpable purpura is seen in following except:

(A) ITP

(B) Henoch schonlein purpura

(C) Wegener’s granulomatosis

(D) Mixed cryoglobulinemia

Answer: (A)

160. Subepithelial deposits are seen in:

(A) Membranoproliferative nephropathy

(B) Membranous GN

(C) Minimal change GN

(D) Diabetic sclerosis

Answer: (B)

161. Antinuclear antibody specific for SLE is

(A) Anti ds DNA

(B) Anti nuclear antibodies

(C) Anti centromere antibody

(D) Anti histone Ab

Answer: (A)

162. Dystrophic calcification means:

(A) Calcification in destroyed tissue with normal calcium level in blood

(B) ↑ level of Ca⁺⁺ deposits

(C) Calcification in normal tissues seen in hyperparathyroidism

(D) Calcification is destroyed tissues with hyper calcemia

Answer: (A)

163. Psammoma bodies are seen in :

(A) Thyroid carcinoma

(B) Carcinoma breast

(C) Carcinoma stomach

(D) Carcinoma testis

Answer: (A)

164. In an adult Unilateral smooth contracted kidney with hypertension is seen in:

(A) Stenosis of renal artery

(B) Chr. GN

(C) Renal cell CA

(D) Pyelonephritis

Answer: (A)

165. Non immune hemolytic anemia occurs in:

(A) SLE

(B) P. vivax infection

(C) CLL

(D) CML

Answer: (B)

166. Following are the findings in sickle cell Amemia, except :

(A) Fish vertebra

(B) Enlarged heart

(C) Splenomegaly usually seen

(D) Leukocytosis

Answer: (C)

167. True about Thrombocytopenic pupura is :

(A) Haemolysis is extravasular

(B) Normal renal function test

(C) Thrombosis in cerebral blood vessels

(D) Immediate cure following plasmapheresis

Answer: (D)

168. In sickle cell trait, number of bands found in Hb:

(A) 2

(B) 1

(C) 4

(D) 5

Answer: (A)

169. Following injury to a blood vessel, immediate haemostasis is achieved by :

(A) Fibrin deposition

(B) Vasoconstriction

(C) Platelet adhesion

(D) Thrombosis

Answer: (B)

170. Lipoprotein X is elevated in:

(A) Primary biliary cirrhosis

(B) Indian childhood cirrhosis

(C) Hyper cholesterolemia

(D) Alcoholic cirrhosis

Answer: (A)

171. Isolation of cell molecules is done by:

(A) Centrifugation

(B) Selective separation

(C) Plasmin

(D) Fibrinogen precipitate

Answer: (A)

172. Morphologically diagnostic feature of cancer cells:

(A) ↑ Size

(B) ↑ Mitotic activity

(C) ↑ Cytoplasm

(D) ↑ Lysosomes

Answer: (B)

173. How do you differentiate CA in situ from invasive CA :

(A) Penetration of Basement membrane

(B) Number of mitotic cells

(C) Metastasis

(D) Nuclear plemorphism

Answer: (A)

174. Mismatched transfusion perioperatively is indicated by:

(A) Excess bleeding

(B) Hypothermia

(C) Hyperthermia

(D) Tachycardia

Answer: (A)

175. AML is characterized by:

(A) Philadelphia chromosome

(B) Auer rods

(C) Hemolytic anemia

(D) Dohle bodies

Answer: (B)

176. Barr body is absent in:

(A) Klinefelter’s syndrome

(B) Turner’s syndrome

(C) Super female

(D) None

Answer: (B)

177. True about multiple myeloma except :

(A) ↑ Uric acid

(B) ↑ Urea

(C) ↑ Ca⁺⁺

(D) ↑ Alk. phosphatase

Answer: (D)

178. Primary role of T-Lymphocytes is:

(A) Antibody formation

(B) Delayed hypersensitivity

(C) Complement production

(D) All of the above

Answer: (B)

179. Fibroblast is derived from:

(A) Local mesenchyme

(B) Macrophage

(C) Endothelium

(D) Vessels

Answer: (A)

180. Blood storage in CPD-A result in A/E:

(A) ↑NH₃ in blood

(B) ↑2, 3 DPG

(C) Hyperkalemia

(D) Citrate toxicity

Answer: (B)

181. Cast is produced due to damage to:

(A) Nephron

(B) Tubule

(C) PCT

(D) DCT

Answer: (B)

182. Enzyme level in tissue injury is due to:

(A) Lysis of cells

(B) Enzyme secretion

(C) No inhibitor in serum

(D) All of the above

Answer: (A)

183. Massive transfusion in previous healthy adult male can cause hemorrhage due to :

(A) Increased tPA

(B) Dilutional thrombocytopenia

(C) Vitamin K deficiency

(D) Decreased fibrinogen

Answer: (B)

184. Curshmann’s crystals are seen in:

(A) Bronchial asthma

(B) Bronchiectasis

(C) Chronic bronchitis

(D) Wegners granulomatosis

Answer: (A)

185. Pleural effusion in cirrhosis all are true except :

(A) ↓ Pleural LDH serum LDH

(B) Protein 10g/L

(C) Gluc 80 mg

(D) S.G. below 1.012

Answer: (B)

186. Atherosclerosis initiation by fibroblast plaque is mediated by injury to:

(A) Smooth

(B) Media

(C) Adventitia

(D) Endothelium

Answer: (D)

187. Crescents are derived from:

(A) Epithelial cells + fibrin + macrophage

(B) Mesangium + fibrin + macrophage

(C) tubule + mesangiaum + fibrin

(D) Mesangiaum + fibrin

Answer: (A)

188. Concentric hypertrophy of left ventricle is seen in:

(A) Cong. bicuspid aortic valve

(B) MS

(C) AR

(D) HOCM

Answer: (A)

189. Spherocytosis of RBC’s is a common feature in :

(A) G-6P-D deficiency

(B) Sickle cell anemia

(C) CML

(D) All

Answer: (A)

190. CD4 lymphocyte is

(A) T. helper cell in peripheral circulation

(B) T. cytotoxic

(C) Precursor cell in bone marrow

(D) Any of the above

Answer: (A)

191. Which is not a finding in viral encephalitis :

(A) Astroglial proliferation

(B) Perivascular mononuclear infiltrate

(C) Inclusion bodies intranuclear and intracytoplasmic

(D) None of the above

Answer: (A)

192. Amyloid is found in

(A) Medullary carcinoma thyroid

(B) ALL

(C) Cirrhosis

(D) Budd chiari syndrome

Answer: (A)

193. Lacunar cells are seen in which type of Hodgkin’s lymphoma :

(A) Lymphocyte predominance

(B) Lyphocyte depletion

(C) Nodular sclerosing

(D) Mixed cellularity

Answer: (C)

194. Not true regarding waldenstorms macroglobulinemia is :

(A) Lymphadenopathy is usually present

(B) Blood viscosity increased

(C) IgM immunoglobulin increased

(D) Hypercalcemia

Answer: (D)

195. Chloroma is due to :

(A) AML

(B) CLL

(C) ALL

(D) Non Hodgkin’s lymphoma

Answer: (A)

196. In atherosclerosis, increased LDL in monocyte macrophage due to :

(A) LDL receptors on marophage

(B) LDL receptors on endothelium

(C) Lipids in LDL get auto oxidized

(D) All of the above

Answer: (A)

197. Hb A 2 concentration in thalassemia trait is :

(A) 1

(B) 1-2.5

(C) 2.5-3.5

(D) 3.5

Answer: (D)

198. Homer wright’s rossette are seen in :

(A) Neuroblastoma

(B) Astrocytoma

(C) Meningioma

(D) Pinealoma

Answer: (A)

199. Donath Landsteiner antibody is seen in :

(A) PNH

(B) Waldenstrom’s macroglobulinemia

(C) Paroxysmal cold hemoglobinuria

(D) Malaria

Answer: (C)

200. Which not true regarding coeliac disease :

(A) Total and sub-total villous atrophy

(B) Increased chance of lymphoma

(C) Anti-gliadin antibody persists even after gluten free diet

(D) ↑ disacharidases

Answer: (D)

201. Large kidney is seen in A/E :

(A) Benign nephrosclerosis

(B) Lymphoma

(C) Amyloidosis

(D) DM

Answer: (A)

202. Antitrypsin deficiency is associated with :

(A) Restrictrictive lung pathology

(B) Cystric fibrosis

(C) Emphysema

(D) Carcinoma

Answer: (C)

203. Increased liver attenuation with intracellular infiltration is seen in :

(A) Fatty liver

(B) Amyloidosis

(C) Hemochromatosis

(D) All

Answer: (A)

204. Characteristic of alcoholic liver is :

(A) Perivenular fibrosis

(B) Maolory hyaline

(C) Spotty necrosis

(D) Zonal necrosis

Answer: (A)

205. Allo graft is between :

(A) Individuals of same genetic constitution

(B) Individuals of same species with different genetic indentity

(C) Twins

(D) Members of different species

Answer: (B)

206. Negri bodies are seen in :

(A) Oligodendroglia

(B) Neuron

(C) Microglia

(D) Astrocytes

Answer: (B)

207. Lymphoepithelial change in stomach is seen in :

(A) Maltoma

(B) Coeliac ds

(C) Ipsidoma

(D) IBS

Answer: (A)

208. Nesidioblastoma is due to hyperplasia of :

(A) Alpha cell

(B) Beta cell

(C) Acinus

(D) D cells

Answer: (B)

209. Commonest sarcoma in children is :

(A) Rhabdomyosarcoma

(B) Lipoma

(C) Angiosarcoma

(D) Fibrosarcoma

Answer: (A)

210. All are true about minimal change GN except :

(A) Selective proteinuria

(B) IgG deposition in mesangium

(C) Common in age group 2-9 years

(D) Responds to steroids

Answer: (B)

211. Epimembranous deposition is seen in :

(A) Good Pasteur syndrome

(B) Membranous GN

(C) MPGN

(D) MCD

Answer: (B)

212. All of the following are helpful for diagnosis of medullary Ca thyroid except :

(A) Spindle cell stroma with few follicles

(B) Amyloid deposition

(C) Calcitonin in stroma

(D) Histological mitochondria is essential for diagnosis

Answer: (D)

213. Life-span of Neutrophil :

(A) 15 days

(B) 6 hours

(C) 6 days

(D) 10 days

Answer: (B)

214. Microangiopathic hemolytic anemia :

(A) TTP

(B) ITP

(C) Senile pupura

(D) CML

Answer: (A)

215. In sickle cell anaemia true is :

(A) Autosplenectomy due to thrombosis & infarction

(B) Microcytosis

(C) Microcardia

(D) Splenomegaly

Answer: (A)

216. Hereditary spherocytosis is due to ….deficiency :

(A) Spectrin

(B) Invertin

(C) Cytokeratin

(D) All of the above

Answer: (A)

217. Psammoma bodies are seen in following except :

(A) Serous cystadenoma of ovary

(B) Mucinous cystadenoma of ovary

(C) Meningioma

(D) Papillary carcinoma of thyroid

Answer: (B)

218. In reflux nephropathy, glomerular lesion is :

(A) Focal G.N.

(B) Membranous G. N.

(C) Membrano proliferative G.N.

(D) Minimal change disease

Answer: (A)

219. True about membranous GN are following except :

(A) Thickening of B.M

(B) Deposition between endothelium and B.M.

(C) Most common cause of nephritic syndrome in adults

(D) Seen in SLE, tumors, drugs

Answer: (B)

220. In multiple myeloma-amyloid is

(A) AL

(B) AA

(C) ATTR

(D) Aβ₂m

Answer: (A)

221. Following will cause CNS-vasculitis except :

(A) SLE

(B) Whipples disease

(C) Granulomatous vasculitis

(D) PAN

Answer: (B)

  1. Lung is characteristically involved in :

(A) Churg-Strauss syndrome

(B) HSP

(C) PAN

(D) ITP

Answer: (A)

223. DIC occurs in :

(A) Myelomonocytic leukemia

(B) Promyelocytic leukemia

(C) Monocytic

(D) Aleukemic leukemia

Answer: (B)

224. ‘Popcorn-cells’ are seen in which variety of hodgkin’s disease :

(A) Nodular sclerosis

(B) Mixed cellularity

(C) Lymphocyte predominance

(D) lymphocyte depletion

Answer: (C)

225. Glomerulonephritis associated with AIDS is :

(A) Focal segmental GN

(B) PSGM

(C) MPGN

(D) Membranous GN

Answer: (A)

226. Anti ds-DNA antibodies are commonly seen in :

(A) SLE

(B) Scleroderma

(C) PAN

(D) Dermatomyositis

Answer: (A)

227. Secondaries are common in a/e :

(A) Skull

(B) Hand & feet bones

(C) Proximal limb bones

(D) Pelvic

Answer: (B)

228. Epithelial granuloma is caused by :

(A) Neutrophil

(B) Cylotoxic T-cells

(C) Helper T-cells

(D) NK cells

Answer: (C)

229. Common type of Ca in middle 1/3rd of oesophagus :

(A) Adeno Ca

(B) Squamns Ca

(C) Adeno squamns Ca

(D) Leiomyosarcoma

Answer: (B)

230. In anoxia of liver, necrosis is seen in :

(A) Centrilobular

(B) Around the periphery

(C) Around the central vein

(D) Around the bile duct

Answer: (C)

231. Giant hypertrophy of gastric mucosa similar to brain surface is seen in :

(A) Peptic ulceration

(B) Carcinoma stomach

(C) Menetrier’s disease

(D) Leomyosarecoma

Answer: (C)

232. Platelet are stored in :

(A) 22 ± ℃

(B) −4 ℃

(C) −2 ℃

(D) −20 ℃

Answer: (A)

233. Macrocytyic anemia may be seen in all of these except :

(A) Liver disease

(B) Copper deficiency

(C) Thiamine deficiency

(D) Vitamin B₁₂ deficiency

Answer: (B)

234. Bilateral breast ca is

(A) Scirrhous ca

(B) Medullary ca

(C) Lobular ca

(D) Ductal ca

Answer: (C)

Medical PG Biochemistry

Medical PG Biochemistry

1. Structural proteins are

(A) Fibrous

(B) Globular

(C) Branched

(D) Compound

Answer: (A)

2. The Tertiary structure of protein is most commonly detected by

(A) X-ray crystallography 

(B) Spectrophotometry

(C) Electrophoresis

(D) Chromatography

Answer: (A)

3. Flexibility of protein depends on 

(A) Glycine

(B) Tryptophan

(C) Phenylalanine

(D) Histidine

Answer: (A)

4. Catabolism of H2O2 is carried out by

(A) Peroxisomes

(B) Mitochondria

(C) Endoplasmic reticulum

(D) Lysosomes

Answer: (A)

5. Hydroxyethyl starch is a

(A) Vasodilator

(B) Inotrope

(C) Plasma expander 

(D) Diuretic

Answer: (C)

6. Function of Metabolism includes all except

(A) Extraction of nutrients from food 

(B) Breakdown of substrate

(C) Equilibrium of biochemical with intracellular components 

(D) Using building blocks for synthesis 

Answer: (A)

7. Which biochemical reaction does not  occur in the mitochondria

(A) Kreb’s cycle 

(B) Urea cycle 

(C) Gluconeogenesis 

(D) Fatty acid synthesis 

Answer: (D)

 8. Energy source used by brain in later days of Starvation is 

(A) Glucose

(B) Ketone bodies 

(C) Glycogen

(D) Fatty acids 

Answer: (B)

9. In collagen synthesis, hydroxylation of Proline to Hydroxyproline requires allexcept

(A) Pyridoxal phosphate

(B) Vitamin C 

(C) Oxygen

(D) Di-oxygenase 

Answer: (A)

10. Which of the following is luminous

(A) Porphyrin

(B) Zymogen

(C) Chromatin

(D) Albumin

Answer: (A)

11. Which is not found in DNA

(A) Adenine

(B) Thymine

(C) Guanine

(D) Uracil

Answer: (D)

12. On complete hydrolysis of DNA, we will get all of the following except

(A)  Adenosine

(B) Purine base

(C) Phosphoric acid

(D) Dexose pentose sugar

Answer: (A)

13. The vitamin used for post-translational modification of glutamic acid to γ Carboxyglutamate is 

(A) Vitamin D 

(B) Vitamin E 

(C) Vitamin A 

(D) Vitamin K 

Answer: (D)

14. Bile acids are derived from 

(A) Fatty acids 

(B) Cholesterol

(C) Bilirubin

(D) Proteins

Answer: (B)

15. Aneuploidy is due to

(A) Non-disjunction at meiosis 

(B) Mosaicism

(C) Deletion

(D) Translocation

Answer: (A)

16. Alkaline phosphatase is produced by

(A) Liver

(B) Bone

(C)  Placenta

(D) All of the above 

Answer: (D)

17. A marker of Golgi apparatus is 

(A) Galactosyl transferase 

(B) Acetyl CoA synthetase 

(C) Pyruvate kinase 

(D) Malonyl CoA 

Answer: (A)

18. In haemoglobin, iron is bound to

(A) Histidine

(B) Leucine

(C) Isoleucine

(D) Valine

Answer: (A)

19. Fluoride inhibits which enzyme 

(A) Pyruvate kinase 

(B) Succinyl dehydrogenase 

(C) Enolase 

(D) Aldolase 

Answer: (C)

20. Metabolites in HMP shunt are all except

(A) Glycerol-3 phosphate

(B) Sedoheptulose-7 phosphate

(C) Glyceraldehyde-3 phosphate 

(D) Xyluose-5 phosphate 

Answer: (A)

21. NADPH is used in 

(A) Fatty acid synthesis 

(B) Ketone synthesis 

(C) Gluconeogenesis

(D) Glycolysis

Answer: (A)

22. The most essential fatty acid is

(A) Linoleic acid 

(B) Linolenic acid 

(C) Arachidonic acid 

(D) Palmitic acid 

Answer: (B)

23. Rate limiting enzyme in the synthesis of cholesterol is 

(A) HmG CoA reductase 

(B) HmG CoA synthetase 

(C) Acetyl CoA synthetase 

(D) Acetyl CoA carboxylase 

Answer: (A)

24. If starvation exceeds 7 days, the major nutritional supply of the brain comes from

(A) Fatty acids 

(B) Ketone bodies 

(C) Protein breakdown 

(D) Carbohydrate breakdown 

Answer: (B)

25. Thiamine is essential for

(A) Isocitrate dehydrogenase 

(B) Succinate dehydrogenase 

(C) Pyruvate dehydrogenase 

(D) Acetyl CoA synthetase 

Answer: (C)

26. Which vitamin is necessary for coenzyme A synthesis

(A) Pantothenic acid 

(B) Ascorbic acid 

(C) Biotin

(D)  Pyridoxine

Answer: (A)

27. Cobalt forms a component of which vitamin 

(A) Biotin

(B) Vitamin B12 

(C) Vitamin A 

(D) Tocopherol

Answer: (B)

28. Cell shape and motility are provided by

(A) Microfilaments

(B) Microtubules

(C) Golgi apparatus

(D) Mitochondria

Answer: (B)

29. A protein estimation test is confused with 

(A) Phosphates

(B) Nitrates

(C) Sulphates

(D) Bile salts 

Answer: (A)

30. Furasol DA is:

(A) Free radical 

(B) Artificial blood 

(C) CO antagonist 

(D) Used to increase O₂ delivery to tissue 

Answer: (D)

31. Dietary fibre contains: 

(A) Collagen

(B) Pectin

(C) Proteoglycans

(D) Starch

Answer: (B)

32. Biotins act on:

(A) Carboxylation

(B) Oxidative phosphorylation 

(C) Oxidative deamination 

(D) Transmethylation

Answer: (A)

33. Vitamin B12 is absorbed in the

(A) Stomach

(B) Duodenum

(C) Ileum

(D) Colon

Answer: (C)

34. Cofactor associated with the enzyme Glutathione peroxidase is

(A) Zinc

(B) Cadmium

(C) Molybdenum

(D) Selenium

Answer: (D)

35. Strongest bond amongst the following is 

(A) Hydrophobic

(B) Electrostatic

(C) Hydrogen bond

(D) Wan der wall’s 

Answer: (B)

36. Gluconeogenesis occurs in all except

(A) Glycerol

(B) Amino acid 

(C) Lactic acid

(D) Palmitate

Answer: (D)

37. Apoprotein A is found in

(A) Chylomicrons

(B) VLDL

(C) HDL

(D) LDL

Answer: (C)

38. Amino acid which lacks chirality is:

(A) Lysine

(B) Leucine

(C) Histidine

(D) Glycine

Answer: (D)

39. An amino acid which does not participate  by α helix formation is

(A) Leucine

(B) Glycine

(C) Proline

(D) Lysine

Answer: (C)

40. Trans-amination of pyruvate and glutamic acid leads to the formation of 

(A) Oxaloacetate

(B) α-ketoglutarate 

(C) Aspartate

(D) Malate

Answer: (B)

41. Which form of DNA is predominantly seen

(A) A

(B) C

(C) B

(D) Z

Answer: (C)

42. Thermo-stability in DNA is contributed mostly by

(A) A = T 

(B) G ≡ C 

(C) Molecular base

(D) Parallel arrangement 

Answer: (B)

43. Okazaki fragment helps in

(A) DNA replication 

(B) Translation

(C) Protein synthesis

(D) Transcription

Answer: (A)

44. Bromodeoxyuridine is related to DNA in

(A) Uracil

(B) Adenosine 

(C) Cytosine

(D) Thymidine

Answer: (D)

45. The first step in fatty acid synthesis involves 

(A) Acetyl CoA carboxylase 

(B) β Hydroxyl CoA dehydrogenase 

(C) Acetyl dehydrogenase 

(D) Pyruvate kinase 

Answer: (A)

46. Which of the following is denaturing substance?

(A) Gaanosine

(B) Guanidine

(C) Glutamate

(D) Glycine

Answer: (B)

47. Cyclic GMP act on

(A) Insulin

(B) Thyroxin

(C) Atrial natriuretic peptide

(D) Growth harmone 

Answer: (C)

48. True statement regarding covalent bonds is :

(A) Electrons have same spin 

(B) Electrons have opposite spin 

(C) They are weak bonds 

(D) None of the above 

Answer: (B)

49. Vitamin required for post translational modification of coagulatants is :

(A) Vitamin A 

(B) Vitamin C 

(C) Vitamin B₆ 

(D) Vitamin K 

Answer: (D)

50. Enzyme to both common in gluconegenesis and glycolysis pathway is :

(A) Phosphofructokinase

(B) Fructose 2, 6-biphosphatase 

(C) Hexokinase

(D) Glucose 6 phosphatase 

Answer: (A)

51. The major fate of glucose-6 Phosphate in tissue in a well fed state is :

(A) Hydrolysis to glucose 

(B) Conversion to glycogen 

(C) Isomerization to fructose 6 phosphate 

(D) Conversion to ribulose 5 phosphate 

Answer: (B)

52. Gluconeogensis affect A/E :

(A) Lactate

(B) Glycerol

(C) Alanine

(D) Growth hormone 

Answer: (D)

53. Property of photochromosity is seen amongst the following amino acids :

(A) Unsaturated amino acid 

(B) Aromatic amino acid 

(C) Monocarboxylic acid 

(D) Dicarboxylic acid 

Answer: (B)

54. All of the following are required for hydroxylation of proline in collagen synthesis except :

(A) O₂ 

(B) Vitamin C  

(C) Dioxygenases

(D) Pyridoxal phosphate 

Answer: (D)

55. The cellular component for protein synthesis is :

(A) Smooth endoplasmic reticulum 

(B) Rough endoplasmic reticulum 

(C)  Ribosomes

(D) Mitochondria

Answer: (C)

56. Binding of proteins to DNA is regulated by :

(A) Copper

(B) Zinc

(C) Selenium

(D) Nickle

Answer: (B)

57. hn RNA seen in :

(A) Spinal muscular dystrophy

(B) Sickle cell disease 

(C) Hutchinson chorea 

(D) α Thallasemia 

Answer: (A)

58. Restriction endonuclease is :

(A) Break single stranded DNA 

(B) Break double stranded DNA 

(C) Break peptide chain 

(D) Break RNA 

Answer: (B)

59. The most important carrier of cholesterol in plasma is :

(A) Chylomicrons

(B) HDL

(C) VLDL

(D) LDL 

Answer: (D)

60. Leucine is aminoacid with a :

(A) Nonpolar side chain

(B) Polar side chain 

(C) Negatively charged side chain 

(D) Positively charged side chain 

Answer: (A)

61. Most basic amino acid out of the following is :

(A) Alanine

(B) Arginine

(C) Histidine

(D) Lysine

Answer: (B)

62. Transmination of pyruvate with glutamate produces :

(A) oxaloacetate & aspartate 

(B) Alanine & asparate 

(C) Oxaloacetate & α-ketoglutarate 

(D) Alanine & α-ketoglutarate 

Answer: (D)

63. Selenium is co-factor for :

(A) Glutathione peroxidase 

(B) Glutathione reductase 

(C) Glutathione synthetase 

(D) Glutathione dehydrogenase 

Answer: (A)

64. Mallate shuttle is seen to occur in :

(A) Glycolysis

(B) Glycogenolysis 

(C) HMP shunt 

(D) Gluconeogenesis

Answer: (A)

65. Glucose may be synthesized from :

(A) Glycerol

(B) Adenine

(C) Guanine

(D) Palmitic acid 

Answer: (A)

66. NADPH is required for :

(A) Gluconeogenesis

(B) Glycolysis

(C) Fatty acids synthesis 

(D) Glycogenolysis

Answer: (C)

67. If chymotrypsin molecule undergoes a ser-195-ala mutation then :

(A) Chymotrypsin will not bind the substrate 

(B) Chymotrypsin will bind the substrate as well as cause cleavage 

(C) Chymotrypsin will bind the substrate but will not cause cleavage 

(D) No affect will be observed 

Answer: (C)

68. Apoprotein A is found in :

(A) Chylomicrons

(B) VLDL

(C) HDL

(D) LDL

Answer: (C)

69. Endogenous triglycerides in plasma are maximally carried in :

(A) VLDL

(B) Chylomicrons

(C) LDL

(D) HDL

Answer: (A)

70. All of the following statements are correct about metabolism in brain except :

(A) Fatty acids are utilized in starvation 

(B) 60% of total energy is utilized during resting stage 

(C) Ketone bodes are used in starvation 

(D) Has no stored energy 

Answer: (A)

71. Which enzyme involved in translation is often referred to as ‘Fidelity enzyme’ :

(A) DNA polymerase 

(B) RNA polymerase 

(C) Amino acyl-t RNA synthetase

(D) Amino acyl-reductase 

Answer: (C)

72. Okazaki segments are required for :

(A) DNA synthesis 

(B) RNA synthesis 

(C) Protein synthesis 

(D) None of the above 

Answer: (A)

73. DNA restriction is done by the following method :

(A) Paper chromatography 

(B) Electrophoresis agargel method

(C) Spectrophotometer

(D) Spectrometry

Answer: (B)

74. Strongest bond out of the following is :

(A) Electrostatitc

(B) Hydrogen

(C) Hydrophobic

(D) Vanderwall’s

Answer: (A)

75. Which of the following molecular phenomenon in Ig genes is responsible for affinity maturation of antibody response :

(A) Chain shuffling 

(B) Junctional diverity 

(C) somatic hypermutation 

(D) Altered RAA splicing 

Answer: (C)

76. Optically inactive Amino Acid is

(A) Proline

(B) Glycine

(C) Lysine

(D) Leucine

Answer: (B)

77. True statement regarding Nitric oxide is :

(A) NO is synthesized from arginine 

(B) NO is spontaneous produced from NO₂ 

(C) NO causes vasoconstriction 

(D) NO is released from mitochondria 

Answer: (A)

78. Thiamine acts as a cofactor in :

(A) Conversion of pyruvate to acetyl-CoA

(B) Transamination reactions 

(C) Oxidation in respiratory chain 

(D) Conversion of pyridoxal to pyridoxal phosphate 

Answer: (A)

79. Following constitute dietry fibres except :

(A) Pectin

(B) Cellulose

(C) Hemicellulose

(D) Riboflavin

Answer: (D)

80. Which of the following aminoacid is excreated in urine in maple syrup urine disease :

(A) Tryptophan

(B) Phenylalanine

(C) Leucine

(D) Arginine

Answer: (C)

81. Ammonia is detoxified in brain to :

(A) Urea

(B) Glutamine

(C) GABA

(D) Uric acid 

Answer: (B)

82. Gaucher’s disease is due to deficiency of enzyme :

(A) Sphingomyelinase

(B) β Glucosidase 

(C) Hexosaminidase-A 

(D) β Galactosidase 

Answer: (B)

83. Glucose can be synthesized from all of the following except :

(A) Acetoacetate

(B) Lactic Acid 

(C) Glycerol

(D) Amino acid 

Answer: (A)

84. True about polymerase chain reaction is :

(A) Enzymatic DNA amplification 

(B) Recombinent DNA amplification 

(C) Seperation of protein fragments is serum 

(D) None

Answer: (A)

85. Translation occurs in :

(A) Ribosomes

(B) Mitochondria

(C) Nucleus

(D) Cytoplasm

Answer: (A)

86. Gout is disorder of :

(A) Purine metabolism

(B) Pyrimidine metabolism 

(C) Oxalate metabolism 

(D) Protein metabolism 

Answer: (A)

87. Best enzyme marker for chronic alcoholism is :

(A) Gamma glutamyl-transferase 

(B) SGOT

(C) SGPT

(D)  Aldolase

Answer: (A)

88. In cytochrom P-450, P stands for :

(A) Structural protein 

(B) Polymer

(C) Substrate protein

(D) Pigment

Answer: (D)

89. Dietery cholesterol is delivered transported to extra hepatic tissue by :

(A) VLDL

(B) LDL

(C) Chylomicrons

(D) IDL

Answer: (B)

90. Molecular separation of two proteins with same charge can be done by:

(A) Ion exchange chromatography 

(B) Dialysis

(C) Gel diffusion chromatography 

(D) Electrophoresis

Answer: (C)

91. In Metabolic alkalosis, which is true about excretion in urine :

(A) More of NH₃ 

(B) Less of aceto acetic acid 

(C) Betahydroxy butyric acid 

(D) Less ammonia 

Answer: (D)

92. In starvation, nitrogen is carried from muscle to liver and kidney by:

(A) Alanine

(B) Aspartic acid and Serine 

(C) Glycine

(D) Asparagine

Answer: (A)

93. Ketone body formation without glycosuria seen in

(A) DM

(B) DI

(C) Prolonged starvation 

(D) Obesity

Answer: (C)

94. Bile Salts undergo xenobiotics

(A) After conjugation with taurine and glycine 

(B) After conjugation with lysine 

(C) After conjugation with betaglucuronic acid 

(D) After conjugation with derived proteins 

Answer: (A)

95. After conjugation with derived proteins

(A) Conjugated proteins

(B) Simple globular proteins 

(C) Unconjugated proteins 

(D) Derived proteins 

Answer: (B)

96. About oxidative phophorylation true is

(A) Generation of ATP’s 

(B) Generation of ADP’s 

(C) Utilization of ATP’s 

(D) Utilization of NADP 

Answer: (A)

97. Pigment providing colour to stool is:

(A) Stercobilinogen

(B) Urobilinogen

(C) Mevobilirubin

(D) Bilirubin

Answer: (A)

98. Muscle cannot make use of glycogen for energy because of deficiency of 

(A) Glucokinase

(B) Phosphoglucomutase

(C) G-6-phosphatase 

(D) Muscle phosphorylase 

Answer: (C)

99. Acetyl CoA is necessary for:

(A) Aminoacid synthesis

(B) Fatty acid synthesis

(C) Glucose storage 

(D) All

Answer: (C)

100. Vitamin required for hydroxyproline to proline conversion :

(A) Vitamin C 

(B) Vitamin E 

(C) Pyridoxal PO₄ 

(D) Biotin

Answer: (A)

101. Ferric chloride test in PKU gives

(A) Red

(B) Green

(C) Blue

(D) Yellow

Answer: (B)

102. Mousy odour urine is seen in:

(A) Maple syrup urine 

(B) Phenylketonuria

(C) Isovalericaciduria

(D) Cystinuria

Answer: (B)

103. Enzyme activity measured in beri beri is

(A) Transketolase

(B) Transaminase

(C) Decarboxylase

(D) Deaminases

Answer: (A)

104. Pompe’s disease is due to deficiency of which enzyme

(A) Branching enzyme 

(B) Glucose 6 phosphatase 

(C) Acid maltase deficiency 

(D) Muscle phosphorylase 

Answer: (C)

105. Muscles are not involved in which glycogen storage disease :

(A) I

(B) II

(C) III

(D) IV

Answer: (A)

106. Lipoprotein with scavenging action is

(A) HDL

(B) LDL

(C) VLDL

(D) None

Answer: (B)

107. Fructokinase is necessary for:

(A) Fructose -1-PO₄ 

(B) Ructose 1,6 diphosphate 

(C) Fructose 6 phosphate 

(D) Glyceraldehyde

Answer: (A)

108. Cholesterol structurally belongs to molecules:

(A) Steroid

(B) Carbohydrate

(C) Urea

(D) Peptide

Answer: (A)

109. Lesch Nyhan syndrome is due to deficiency of :

(A) Hypoxanthine phosphoribosyl transferase 

(B) Xanthine oxidase 

(C) Purine phosphorylase 

(D) Adenosine deaminase 

Answer: (A)

110. In cell membrance, following are true except:

(A) Lipids are regularly arranged 

(B) Lipids are symmetrical 

(C) Protein displaced laterally 

(D) None

Answer: (B)

111. Intact peptide bond is necessary for which test :

(A) UV diffraction 

(B) Ninhydrin

(C) Diazo reaction 

(D) All

Answer: (B)

112. Addition of water in C-C bond is by the enzyme:

(A) Hydroxylase

(B) Dehydrogenase

(C) Hydrolase

(D) Hydratase

Answer: (C)

113. In TCA cycle, CO2 is released from :

(A) Thiokinase

(B) Isocitrate dehydrogenase 

(C) Citrate dehydrogenase 

(D) Alpha ketoglutrate 

Answer: (B)

114. One of the following is having least density:

(A) Chylomicron

(B) HDL

(C) LDL

(D) VLDL

Answer: (A)

115. Succinyl Co-A is formed by :

(A) Histidine

(B) Leucine

(C) Valine

(D) Lysine

Answer: (C)

116. Haemoglobin unlike myoglobin shows :

(A) Parabolic curve of oxygen association 

(B) Co-operative effect of combine O₂ 

(C) Cooperative index of 81 

(D) Hill’s coefficient of 1 

Answer: (B)

117. Km increased but Vmax same enzyme is:

(A) Competitive

(B) Non competitive 

(C) Irreversible

(D) Uncompetitive

Answer: (A)

118. Familial hypercholesterolemia is :

(A) Deficient LDL receptors 

(B) Deficient HDL receptors 

(C) HMG CoA reductase deficiency 

(D) Deficient VLDL receptors A

Answer: (A)

119. About glycolysis true is :

(A) Occurs in mitochondria 

(B) Complete breakdown of glucose 

(C) Conversion of glucose to 3C units

(D) 3 ATP’s are used in anaerobic pathway 

Answer: (C)

120. Km value is defined as:

(A) Substrate concentration at Vmax/2 

(B) Substrate concentration at twice Vmax 

(C) Substrate concentration of thrice Vmax 

(D) Substrate concentration of one third Vmax 

Answer: (A)

121. Glutamine in blood acts as :

(A) NH₃ transporter 

(B) Toxic element 

(C) Stored energy 

(D) Abnormal metabolite 

Answer: ()

122. In non competitive antagonism the true statement is :

(A) Km value decrease; V max normal 

(B) Km value decreased; V max decreased 

(C) Km value normal; V max decreased

(D) Km value decreased; V max increased 

Answer: (C)

123. The enzyme deficient in erythropoietic porphyria is:

(A) PBG deaminase 

(B) Uroporphyrin II cosynthetase 

(C) Coprophyrin

(D) Ferrochelatase

Answer: (D)

124. Which among the following controls is an allosteric inhibitor of TCA cycle:

(A) Pyruvate dehydrogenase 

(B) Keto glutarate dehydrogenase

(C) Isocitrate dehydrogenase 

(D) Malate dehydrogenase 

Answer: (C)

125. The best method to differentiate proteins is by :

(A) Gel chromatography 

(B) Affinity chromatography 

(C) Ion exchange electrophoresis 

(D) None of the above 

Answer: (A)

126. Among the following, the maximum redox potential is for :

(A) NADH/NAD 

(B) Succinyl Co A fumarate 

(C) Ubiquinone

(D) Fe 2⁺ 

Answer: (D)

127. HDN is seen due to deficiency of vitamin :

(A) A

(B) C

(C) K

(D) E

Answer: (C)

128. The enzyme deficient in Lesch-Nyhan syndrome is :

(A) GTRT

(B) Glutaminase

(C) Transcarboxylase

(D) HGPRT

Answer: (D)

129. The immediate precursor in the formation of acetoacetate from acetyl Co A in the liver is :

(A) Mevalonate

(B) HMG CoA 

(C) Acetoacetyl CoA 

(D) 3-hydroxy-butyryl 

Answer: (B)

130. Of the following groups of amino acids produce common component of the TCA cycle is :

(A) Alaine, isoleucine, leucine, lysine 

(B) Serine, asparagine, glycine, glutamate 

(C) Isoleucine, valine, methionine

(D) Prolone, leucine, tryptophan 

Answer: (C)

131. PCR in done for :

(A) DNA amplification 

(B) RNA amplification 

(C) Gene splicing 

(D) Genecloning 

Answer: (A)

132. Minimum fasting BGL for diagnosis of diabetes mellitus is :

(A) 114 mg/dl 

(B) 120 mg/dl 

(C) 126 mg/dl 

(D) 132 mg/dl 

Answer: (B)

133. G-6-P deficiency is seen in :

(A) Von gierke’s disease 

(B) Taysach’s ds 

(C) Pompe’s ds 

(D) Andersons’s ds 

Answer: (A)

134. Glycophorin is present in :

(A) Enterocyte

(B) Hepatocyte

(C) RBC

(D) Lymphocyte

Answer: (C)

135. NADPH is required in :

(A) Gluconeogenesis

(B) Glycolysis

(C) Glycogenolysis

(D) Fatty acid synthesis 

Answer: (D)

136. Cystinuria is associated with excretion of which A.A in urine :

(A) Cystine, lysine, Tyrosine and alanine 

(B) Cytstine, lysine, ornithine and alanine 

(C) Cystine, ornithine, lysine and arginine 

(D) Cystine, ornithine, leucine and arginine 

Answer: (C)

137. In starvation, brain utilizes glucose preferentially but not in liver because :

(A) Low km value of hexokinase 

(B) Low km glucokinase 

(C) High km glucokinase 

(D) High blood flow 

Answer: (A)

138. Insulin acts on glucose  metabolism by :

(A) ↑permeability of glucose across cell membrane 

(B) ↑permeability across cells membrane against glucose gradient 

(C) ↑permeability of renal cells 

(D) ↑glucose transport to brain 

Answer: (B)

139. All will predispose to atherosclerosis except :

(A) Homocystinemia

(B) Fibrinogen

(C) Calcium

(D) Lipoprotein A 

Answer: (C)

140. Which is common metabolic process in human as well as bacteria :

(A) Purine synthesis 

(B) Nitrogen fixation 

(C) Mucolipid formation 

(D) Nonoxidative photophosphorylation 

Answer: (A)

141. Which of the following acts as cofactor after it’s modification :

(A) Vit-C 

(B) Pantothenic acid 

(C) Biotin

(D) Zinc

Answer: (B)

142. Which important component of cell wall has carbohydrate moiety :

(A) Phosphoglyceride

(B) Triacylglycerol

(C) Sphingomyelin

(D) GM₂ Gangliosides 

Answer: (D)

143. Which of the following has highest electrophoretic mobility and least lipid content :

(A) Chylomicrons

(B) HDL

(C) LDL

(D) VLDL

Answer: (B)

144. Correct sequence of enzymes required for DNA formation is :

(A) DNA polymerase → Protein unwinding enzyme → DNA ligase → DNA Isomerase → Polymerase-1

(B) Protein unwinding enzyme → Polymerase-1 → DNA ligase → DNA Isomerase → DNA polymerase 

(C) RNA polymerase → DNA polymerase III → DNA polymerase I → DNA ligase 

(D) RNA polymerase → DNA Polymerase III → DNA ligase → Exonuclease → DNA polymerase I 

Answer: (C)

145. True about enzyme specificity :

(A) Amount of enzyme required per second, per mole of product formation. 

(B) Number of sites per substrate 

(C) Amount of enzyme binding with various substrates 

(D) Amount of enzyme causing transformation of 1 μmol of substrate per minute under standard conditions 

Answer: (D)

146. Which is the smallest fundamental unit coding for DNA synthesis :

(A) Cistron

(B) Operon

(C) Replican

(D) Anticodon

Answer: (A)

147. Polypeptide chain termination is enhanced by :

(A) Stop codon 

(B) promoter

(C) Ribosomal unit 

(D) Peptidyl transferase 

Answer: (A)

148. True about lipid bilayer of cell wall :

(A) Asymmetrical arrangement of cell wall component 

(B) Lateral diffusion of ions 

(C) Symmetrical arrangement of cell wall components 

(D) Not made up of amphipathic lipids 

Answer: (A)

149. An infant has hepatorenomegaly, hypoglycemia hyperlipidemia, aciodsis & normal structured glycogen deposition in liver. What is the diagnosis :

(A) Her’s disease 

(B) Von Gerke’s ds 

(C) Cori’s ds 

(D) Anderson’s ds 

Answer: (B)

150. Which of the following enzymes is/are involved in neoglucogenesis all except

(A) Phosphoglycerate kinase 

(B) Fructose 1, 6-biphosphatase 

(C) Phosphoglucomutase 

(D) Pyruvate carboxylase 

Answer: (C)

151. Energy required for maintainenace of bodily functions of a person weighing 40 kgs is :

(A) 1000 kilocalories 

(B) 1500 kilocalories 

(C) 2000 kilocalories 

(D) 2500 kilocalories 

Answer: (A)

152. Reducing sugar in urine is seen in :

(A) Fanconi anemia 

(B) Lactose intolerance 

(C) Galactosemia

(D) Phenylketonuria

Answer: (C)

153. Highest amount of cholesterol is present in :

(A) HDL

(B) LDL

(C) VLDL

(D) IDL

Answer: (B)

154. Hypolipidemic agents act on :

(A) HMG CoA synthetase 

(B) HMG CoA oxygenase 

(C) HMG CoA reductase 

(D) HMG CoA hydratase 

Answer: (C)

155. Cholesterol is transported to extrahepatic tissues by :

(A) IDL

(B) VLDL

(C) HDL

(D) LDL

Answer: (D)

156. The hydrolytic step leading to release of polypeptide chain from ribosomes is catalysed by :

(A) Stop codons 

(B) Peptidyl transferase 

(C) Release factors 

(D) AUG codon 

Answer: (C)

157. Substrate-level phosphorylation seen in reaction catalysed by :

(A) Succinate dehydrogenase 

(B) Alfa-ketoglutarate dehydrogenase 

(C) Succinyl CoA thiokinase 

(D) Malate dehydrogenase 

Answer: (C)

158. Conversion of glycine to serine requires :

(A) Folic acid 

(B) Thiamine

(C) Vit. C 

(D) Fe²⁺ 

Answer: (A)

Medical PG Pharmacology

Medical PG Pharmacology

1. ‘First-pass effect’ is seen with which route of administration

(A)  Oral

(B) Sublingual

(C) Intramuscular

(D) Intravenous

Answer: (A)

2. All are α-adrenergic agonists except

(A) Mephentermine

(B) Ritodrine

(C) Methoxamine

(D) Phenylephrine

Answer: (B)

3. All the following affect the metabolism of Theophylline except

(A) Erythromycin

(B) Smoking

(C) Cimetidine

(D) Steroids

Answer: (D)

4. All are side effects of Lisinopril except

(A) Dizziness

(B) Jaundice

(C) Cough

(D) Angioneurotic edema

Answer: (B)

5. Adrenal steroids are contraindicated in all except

(A) Severe hypertension

(B) Osteoporosis

(C) Diabetes mellitus

(D) Cushing’s syndrome

Answer: (D)

6. Which of the following drugs is known to cause vaginal adenocarcinoma in female offspring when given to a pregnant woman?

(A) Chlorpropamide

(B) Progesterone

(C) Diethylstilbestrol

(D) Chloramphenicol

Answer: (C)

7. Which drug has the least anti-cholinergic side-effects?

(A) Imipramine

(B) Doxein

(C) Fluoxetine

(D) Clomipramine

Answer: (C)

8. L-dopa is given along with Carbidopa

(A) To prevent peripheral decarboxylation of L-dopa

(B) To reduce side effects

(C) To increase compliance

(D) To increase half-life

Answer: (A)

9. All the following displace Imipramine from protein binding sites except

(A) Aspirin

(B) Propranolol

(C) Glibenclamide

(D) Lithium

Answer: (D)

10. The dose of Digoxin should be reduced when given along with

(A) Quinidine

(B) Rifampicin

(C) Indomethacin

(D) Antacids

Answer: (A)

11. Rapidly progressive pulmonary fibrosis is seen in

(A) Paracetamol poisoning

(B) Kerosene poisoning

(C) Amiodarone poisoning

(D) Chloroquine toxicity

Answer: (C)

12. A Cardiotoxic drug is

(A) Adriamycin

(B) Cyclophosphamide

(C) 5-FU

(D) All of the above

Answer: (D)

13. Digoxin toxicity can be precipitated by all except

(A) Phenytoin

(B) Quinidine

(C) Calcium

(D) Verapamil

Answer: (A)

14. Tocainide has the following properties except

(A) Used in ventricular tachycardia

(B) Prolongs APD

(C) Used as lidocaine analogue

(D) Used orally

Answer: (B)

15. Which of the following is the drug of choice for PSVT?

(A) Lignocaine

(B) Verapamil

(C) Flecainide

(D) Tocainide

Answer: (B)

16. Furosemide causes all except

(A) Hyperuricemia

(B) Ototoxicity

(C) Hypercalcemia

(D) Hypokalemia

Answer: (C)

17. All the following are used in Motion sickness except

(A) Cyclizine

(B) Hyoscine

(C) Domperidone

(D) Meclizine

Answer: (C)

18. The commonest side-effect of Cisapride is

(A) Abdominal cramps

(B) Diarrhea

(C) Headache

(D) Convulsions

Answer: (B)

19. Side effects Clofazimine are all except

(A) Skin pigmentation

(B) GI disturbance

(C) Ichthyosis

(D) Anemia

Answer: (D)

20. Mebendazole is used in all of the following except

(A) Hook worm

(B) Round worm

(C) Strongyloides

(D) Trichuris trichura

Answer: (C)

21. The drug inhibiting DNA-dependent RNA polymerase in Mycobacteria is

(A) INH

(B) Rifampicin

(C) Ciprofloxacin

(D) Ethionamide

Answer: (B)

22. Which of the following is a common side-effect of Cisplatin?

(A) Diarrhea

(B) Vomiting

(C) Pulmonary fibrosis

(D) Alopecia

Answer: (B)

23. Megaloblastic anemia is a side-effect of long term therapy with

(A) Diazepam

(B) INH

(C) Phenobarbitone

(D) Lithium

Answer: (C)

24. Which analgesic is not used in Acute myocardial infraction?

(A) Morphine

(B) Pentazocine

(C) Pethidine

(D) Buprenorphine

Answer: (B)

25. Therapeutic index of a drug is an indicator of its

(A) Potency

(B) Safety

(C) Toxicity

(D) Efficacy

Answer: (B)

26. Side-effects of Captopril are all except

(A) Cough

(B) Hyperkalemia

(C) Renal dysfunction

(D) Hemolytic anemia

Answer: (D)

27. Which of the following is not true about enalapril?

(A) It is a pro-drug

(B) It is a di-peptide

(C) It is more effective than captopril

(D) Has less adverse effects

Answer: (B)

28. Which of the following steroids can be administered by inhalation?

(A) Beclomethasone

(B) Betamethasone

(C) Prednisolone

(D) Hydrocortisone

Answer: (A)

29. Which drug causes osteoporosis on long-term use?

(A) Estrogen

(B) Progesterone

(C) GnRH analogues

(D) Warfarin

Answer: (C)

30. Estrogen acts on

(A) Cellular membrane receptors

(B) Cytoplasmic receptors

(C) Nuclear receptors

(D) Mitochondria

Answer: (B)

31. Which of the following is not a catecholamine?

(A) Epinephrine

(B) Norepinephrine

(C) Dopamine

(D) Phenylephrine

Answer: (D)

32. Buprenorphine is a

(A) Partial agonist

(B) Pure antagonist

(C) Agonist-antagonist

(D) None of the above

Answer: (A)

33. Which of the following is False about Pentazocine?

(A) Decreased vomiting and constipation as compared to morphine

(B) Risk of addiction is less than that with morphine

(C) Risk of addiction is more than that with morphine

(D) It is agonist-antagonist

Answer: (C)

34. Which of the following drugs acts on μ receptors of the CNS?

(A) Morphine

(B) Buprenorphine

(C) Pethidine

(D) Pentazocine

Answer: (A)

35. Which of the following is NOT an inotropic drug?

(A) Dopamine

(B) Isoprenaline

(C) Amrinone

(D) Amiodarone

Answer: (D)

36. Which of the following has the shortest plasma half-life?

(A) Propranolol

(B) Esmolol

(C) Timolol

(D) Atenolol

Answer: (B)

37. Which of the following does not cause thrombocytopenia?

(A) Chlorpropamide

(B) Phenobarbitone

(C) Quinine

(D) Thaizide

Answer: (B)

38. All the following drugs cause pulmonary fibrosis except

(A) Busulfan

(B) Methotrexate

(C) Doxorubicin

(D) Bleomycin

Answer: (C)

39. Which of the following does not cause bradycardia? 

(A) Propranolol

(B) Hydralazine

(C) Clonidine

(D) Reserpine

Answer: (B)

40. All the following are true of cholestyramine except

(A) Are basic ion exchange resins

(B) Cause compensatory increase in HMG CoA reductase activity

(C) May cause constipation, steatorrhea

(D) Patient acceptability is good

Answer: (D)

41. All the following antibiotics act on the cell wall except

(A) Ampicillin

(B) Bacitracin

(C) Cycloserine

(D) Griseofulvin

Answer: (D)

42. Advantages of 3rd generation cephalosporins over 1st and 2nd generation cephalosporins is that they are

(A) β-lactamase sensitive

(B) Not orally administered

(C) Effective against Gram +ve bacteria

(D) Effective against Gram –ve bacteria

Answer: (D)

43. The following are true of fluoroquinolones except

(A) Low toxicity to host cells

(B) DNA-gyrase inhibitors

(C) Effective against Pseudomonas

(D) Rapidly develop resistance

Answer: (D)

44.  The anti-malarial drugs effective in pre-erythrocytic phase in liver are

(A) Proguanil

(B) Chloroquine

(C) Pyrimethamine

(D) Quinine

Answer: (A)

45. Long term use of chloroquine does not lead to

(A) Lichenoid eruptions

(B) Visual deterioration

(C) T wave changes in ECG

(D) Weight gain

Answer: (D)

46. Which of the following statements is False about Acyclovir?

(A) It inhibits DNA synthesis and viral replication

(B) It is effective against influenza

(C) It has low toxicity for host cells

(D) Renal impairment necessitates dose reduction

Answer: (B)

47. All the following antimicrobrial agents are used topically except

(A) Clotrimazole

(B) Griseofulvin

(C) Nystatin

(D) Miconazole

Answer: (B)

48. Which drug does not cross the placental barrier?

(A) Heparin

(B) Warfarin

(C) Lithium

(D) Morphine

Answer: ()

49. The drug of choice in PSVT is

(A) Digoxin

(B) Propranolol

(C) Lignocaine

(D) Adenosine

Answer: (D)

50. Zero order kinetics at a higher dose is seen with

(A) Phenytoin

(B) Heparin

(C) Probenecid

(D) Lithium

Answer: (A)

51. Examples of pro-drug are all except

(A) Levodopa

(B) Omeprazole

(C) Enalapril

(D) Indomethacin

Answer: (D)

52. Lithium monitoring is done because of

(A) Low therapeutic efficacy

(B) Very low therapeutic index

(C) Adverse effects

(D) Long half-life

Answer: (B)

53. All of the following are reversible anticholinesterases except

(A) Physostigmine

(B) Ambenonium

(C) Pyridostigmine

(D) Echothiophate

Answer: (D)

54. Actions of atropine are all except

(A) Bronchoconstriction

(B) Tachycardia

(C) Mydriasis

(D) CNS stimulation

Answer: (A)

55. The following are all true about Ipratropium bromide except

(A) Used by inhalation

(B) ↑ IOP

(C) Dryness of mouth

(D) Scratching in trachea

Answer: (B)

56. Test for myasthenia gravis is

(A) Succinylcholine

(B) Edrophonium

(C) Atracurium

(D) d-Tubocurarine

Answer: (B)

57. If Theophylline is used with Ciprofloxacin

(A) Toxicity of Theophylline increases

(B) Efficacy decreases

(C) Activity of Ciprofloxacin increases

(D) It decreases absorption of Theophylline

Answer: (A)

58. Amongst the following, least glucocorticoid activity is seen with

(A) Fludrocortisone

(B) Dexamethasone

(C) Triamcinolone

(D) Betamethasone

Answer: (C)

59. Flumazenil is

(A) Benzodiazepine antagonist

(B) Benzodiazepine agonist

(C) Adrenergic blocking agent

(D) Cholinesterase inhibiter

Answer: (A)

60. N-acetyl Cysteine is an antidote for poisoning due to

(A) Paracetamol

(B) Dhatura

(C) Aspirin

(D) Propranolol

Answer: (A)

61. Buprenorphine is

(A) Opioid agonist-antagonist

(B) Partial agonist

(C) Pure antagonist

(D) Partial antagonist

Answer: (B)

62. The most common side effect of chronic use of phenothiazines is

(A) Akathisia

(B) Tardive akinesia

(C) Tardive dyskinesia

(D) Muscular dystonia

Answer: (C)

63. Drugs not used in myocardial infarction are

(A) Inhibitors of platelet aggregation

(B) Thrombolytics

(C) Anticoagulants

(D) Inhibitors of Plasminogen activator

Answer: (D)

64. All the following drugs decrease the preload

(A) Glyceryl tri-nitrate

(B) ACE inhibitors

(C) Hydralazine

(D) Sodium nitroprusside

Answer: (C)

65. Food does not interfere in absorption of 

(A) Cimetidine

(B) Ranitidine

(C) Famotidine

(D) None of the above

Answer: (A)

66. All are examples of gastro-kinetic drugs except

(A) Cisapride

(B) Domperidone

(C) Erythromycin

(D) Ampicillin

Answer: (D)

67. Acetylation is seen in all except

(A) INH

(B) Hydralazine

(C) Procainamide

(D) Phenytoin

Answer: (D)

68. Which of the following is not a hepatotoxic drug?

(A) Ethambutol

(B) Rifampicin

(C) INH

(D) Cycloserine

Answer: (D)

69. All the following drugs are used an Pseudomonas infection except

(A) Pefloxacin

(B) Imipenem

(C) Aztreonam

(D) Vancomycin

Answer: (D)

70. ADH acts on 

(A) Proximal convoluted tubule

(B) Distal convoluted tubule

(C) Loop of Henle

(D) Collecting duct

Answer: (D)

71. All the following are examples of cardiotoxic drugs except

(A) Cyclophosphamide

(B) 5-FU

(C) Adriamycin

(D) Cisplatin

Answer: (D)

72. Sterility is caused by

(A) Vinca alkaloids

(B) Alkylating agents

(C) Antimetabolites

(D) Actinomycin D

Answer: (B)

73. All of the following are calcium channel blockers except

(A) Nimodipine

(B) Verapamil

(C) Flunarizine

(D) Pirenzepine

Answer: (D)

74. Postural hypotension is common with

(A) Prazosin

(B) Labetalol

(C) Sodium Nitroprusside

(D) Captopril

Answer: (A)

75. Pralidoxime acts by :

(A) Reactivating cholinesterase enzyme

(B) Promoting synthesis of cholinesterase

(C) Promoting synthesis of acetylcholine

(D) Direct action on cholinergic receptors

Answer: (A)

76. Clonidine is a :

(A) α₁ selective agonist

(B) α₂ selective agonist

(C) α₁ selective antagonist

(D) α₂ selective antagonist

Answer: (B)

77. Propranolol is indicated in all of the following conditions except :

(A) Thyrotoxicosis

(B) Varient Angina

(C) Migraine

(D) Hypertension

Answer: (B)

78. Anti-Adrenergic drug which crosses the blood-brain barrier minimally is :

(A) Propranolol

(B) Atenolol

(C) Oxprenolol

(D) Alprenolol

Answer: (B)

79. All of the following are features of sympathetic stimulation of heart except :

(A) ↑Contractility

(B) ↑Contractility

(C) ↑Refractory period

(D) ↑Conduction velocity increased

Answer: (C)

80. Antiulcer drug is :

(A) Pirenzepine

(B) Methylcellulose

(C) Ciprofloxacin

(D) Pyrimethamine

Answer: (A)

81. All of the following drugs may be used for motion sickness except :

(A) Hyoscine

(B) Dicyclomine

(C) Domperidone

(D) Scopolamine

Answer: (C)

82. Shortest acting neuromuscular blocking agent is

(A) Pancuronium

(B) Atracurium

(C) Mivacurium

(D) Vecuronium

Answer: (C)

83. Intracranial pressure may be increased by all of the following drugs except :

(A) Hypervitaminosis A

(B) Corticosteroids

(C) Quinolones

(D) Aminoglycosides

Answer: (D)

84. Which of the following antiepileptic drugs acts by the release of the inhibiting transmitter GABA?

(A) Valproic acid

(B) Diazepam

(C) Ethambutol

(D) Phenytoin

Answer: (A)

85. Furesemide and thiazides have similar properties in the following :

(A) Duration of action

(B) Site of action

(C) Effect on urate excretion

(D) Well absorbed orally

Answer: (D)

86. All of the following statements about antiangial action of nitrates are true except :

(A) ↓Myocardial O2 consumption

(B) ↓Both pre and after load

(C) ↑Total coronary flow

(D) Cause favourable redistribution of coronary flow

Answer: (C)

87. Drug of choice in PSVT is :

(A) Verapamil

(B) Propranolol

(C) D.C. shock

(D) Digoxin

Answer: (A)

88. Predominant arteriolar dilators include all of the following except :

(A) Sodium Nitroprusside

(B) Diazoxide

(C) Hydrallazine

(D) Minoxidil

Answer: (A)

89. Which of the following antihypertensive drugs is devoid of any central action?

(A) Clonidine

(B) α methyl dopa

(C) Propranolol

(D) Indapamide

Answer: (D)

90. Interstitial nephritis is most commonly seen with :

(A) Methicilline

(B) Ampicilline

(C) Amoxycilline

(D) Cloxacilline

Answer: (A)

91. Maximum amount of Photosensitivity is seen with :

(A) Ciprofloxacin

(B) Ofloxacin

(C) Pefloxacin

(D) Norfloxacin

Answer: (C)

92. Administration of one of the following drug is known to result in neuropsychiatric symptoms :

(A) Rifampcin

(B) Cycloserine

(C) Ethionamide

(D) Cephalosporine

Answer: (B)

93. Oral contraceptive failure may be seen with :

(A) Rifampicin

(B) Cimetidine

(C) Propranolol

(D) Ethambutol

Answer: (A)

94. Most common side effect of 5-Fluoro-uracil is :

(A) G.I. toxicity

(B) Bone marrow depression

(C) Cardiotoxicity

(D) Neurotoxicity

Answer: (A)

95. True statement about Omeprazole is :

(A) It may cause Leomyosarcoma

(B) It is a nitrosource

(C) May induce carcinoid tumours in rats

(D) It is more frequently used by the I.V. route than orally

Answer: (C)

96. Prolonged use of steroids may cause :

(A) Decrease in bone matrix protein

(B) Hypoglycemia

(C) Hypotension

(D) Early healing of wound

Answer: (A)

97. First drug to be used in anaphylactic shock is :

(A) Subcutaneous adranaline

(B) I.V. corticosteroid

(C) Theophylline

(D)  Antihistaminic

Answer: (A)

98. Gynaecomastia may be associated with administration of :

(A) Ranitidine

(B) Cimetidine

(C) Terfenadine

(D) Omeprazole

Answer: (B)

99. All of the following are true about competitive  inhibitor except :

(A) Resembles chemically with the agonist

(B) Bind the same receptors

(C) Reduces potency

(D) Maximum level is not reached by increasing the concentration of the drug

Answer: (D)

100. On higher doses zero order kinetics is seen :

(A) Phenytoin

(B) Propranolol

(C) Lithium

(D) Probenacid

Answer: (A)

101. Gynocostamia may be caused by all except :

(A) Cimetidine

(B) Ranitidine

(C) Ketoconazole

(D) Spironolactone

Answer: (B)

102. Hyperglycemia may be caused by all except :

(A) Nimesulide

(B) Chlorthiazides

(C) Corticosteroids

(D) Theophylline

Answer: (A)

103. All are classified as reversible anticholinesterases except :

(A) Ambenonium

(B) Physostigmine

(C) Pyridostigmine

(D) Echothiophate

Answer: (D)

104. Agent used as a diagnostic test for myasthenia gravis is :

(A) Phentolamine

(B) Edrophonium

(C) Echothiophate

(D) Glucagon

Answer: (B)

105. In treatment of cardiac failure, dobutamine acts by all of the following mechanisms except :

(A) α receptors agonism

(B) β adrenergic receptors agonism

(C) Dopamine receptor agonism

(D) Increasing force of contraction

Answer: (C)

106. Which of the following antiarrhythmics drugs causes prolonged repolarization of ventricles & ERP?

(A) Amiodarone

(B) Propranolol

(C) Verapamil

(D) Quinidine

Answer: (A)

107. All of the following are side effects of Amiodarone except :

(A) Pulmonary fibrosis

(B) Corneal microdeposits

(C) Thyroid dysfunction

(D) Osteoporosis

Answer: (D)

108. Which of the following statements regarding adenosine is not true?

(A) Used in PSVT

(B) Administered as rapid I.V. infection

(C) Has short lived side effects

(D) Disopyramidase increases its therapeutic effect

Answer: (D)

109. Most commonly postural hypotension is seen with :

(A) Prazosin

(B) Nifedipine

(C) Atenolol

(D) ACE inhibitors

Answer: (A)

110. Which of the following drug acts as a HMG-CoA reductase inhibitor?

(A) Gemfibrozil

(B) Clofibrate

(C) Lovastain

(D) Probucol

Answer: (C)

111. Low molecular weight haparin therapy is associated with all except :

(A) Less chances of bleeding

(B) Single dose per day

(C) Easy filterability by glomerular capillaries

(D) High biological interaction to plasma proteins

Answer: (D)

112. Which of the following has least glucocorticoid activity :

(A) Fludrocortisone

(B) Dexamethasone

(C) Triamcinolone

(D) Betamethasone

Answer: (C)

113. Flumazenil is a :

(A) Benzodiazepine antagonist

(B) Benzodiazepine agonist

(C) Adrenergic blocking agent

(D) Oplate antagonist

Answer: (A)

114. The most common side effect associated with chronic use of phenothiazines is :

(A) Akethesia

(B) Parkinsonism

(C) Tardivedyskinesia

(D) Muscular dystonia

Answer: (C)

115. All of the following may be seen with Neuroleptic malignant syndrome except :

(A) Hypothermia

(B) Altered consciousness

(C) Muscle rigidity

(D) Involuntary movements

Answer: (A)

116. Tetrahydrocannabinol is the active component of :

(A) Marijuana

(B) LSD

(C) Hashish

(D) Heroin

Answer: (A)

117. Low doeses of aspirin used in myocardial infarction act by :

(A) Inhibiting thromboxane synthetase

(B) Inhibit cyclooxygenase

(C) Releasing EDRF

(D) High protein binding activity

Answer: (B)

118. Following drugs may be used for pseudomonas infection except :

(A) Pefloxacine

(B) Azithromycin

(C) Imipenam

(D) Ceftazidime

Answer: (B)

119. Clindamycin acts by inhibiting :

(A) Protein synthesis

(B) DNA Gyrase

(C) Cell wall synthesis

(D) Lysosomal enzyme

Answer: (A)

120. Albendazole may be for treatment of all of the following conditions except :

(A) Entrobius

(B) Ascariasis

(C) Ankylostoma

(D) Schistosomiasis

Answer: (D)

121. Neostigmine is a :

(A) Primary ammonium compound

(B) Secondary ammonium compound

(C) Tertiary ammonium compound

(D) Quartenary ammonium compound

Answer: (D)

122. Drug of choice in Acute central anticholinergic syndrome  is :

(A) Neostigmine

(B) Physostigmine

(C) Tacrine

(D) 4-amino pyridine

Answer: (B)

123. Selective α1-A blocker is :

(A) Prazosin

(B) Terazosin

(C) Tamsulosin

(D) Indoramine

Answer: (C)

124. β1 selective agonist is :

(A) Terbutaline

(B) Albuterol

(C) Dobutamine

(D) Isoetharine

Answer: (C)

125. All of the following are selective β1 blockers except :

(A) Atenolol

(B) Metoprolol

(C) Labetalol

(D) Betaxolol

Answer: (C)

126. Selegilline is a selective inhibitor of :

(A) MAO-A

(B) MAO-B

(C) Dopamine

(D) Norepinephrine-uptake

Answer: (B)

127. ‘Vigabatrin’ a new antiepileptic agent acts by :

(A) GABA-antagonism

(B) GABA-agonism

(C) NMDA antagonism

(D) Carbonic anhydrase inhibition

Answer: (B)

128. Quinidine is a :

(A) Na⁺channel-blocker

(B) K⁺ channel blocker

(C) Ca⁺⁺ channel blocker

(D) Cl⁻ channel blocker

Answer: (A)

129. All of the following statements are true about nitrates except :

(A) It releases NO

(B) It causes vasodilatation

(C) It decreases A.V. conduction

(D) It has high first pass metabolism

Answer: (C)

130. Dipyridamole acts by :

(A) Adenosine uptake inhibition

(B) Inhibiting thromboxane A₂

(C) Stimulating PGI₂ synthesis

(D) Inhibiting PGI₂ synthesis

Answer: (A)

131. All of the following are examples of bactericidal drugs except :

(A) INH

(B) Rifampicin

(C) Ethambutol

(D) Pyrazinamide

Answer: (C)

132. All of the following are drugs are ATT except :

(A) Kanamycin

(B) Cycloserine

(C) 5-flucytosine

(D) Ofloxacin

Answer: (C)

133. Mechanism of action of erythromycin is interference with:

(A) Transcription

(B) Translation

(C) Translocation

(D) Signal transduction

Answer: (C)

134. Which of the following drugs acts on “motilin” receptors?

(A) Erythromycin

(B) Tetracycline

(C) Norfloxacin

(D) Chloramphenicol

Answer: (A)

135. All of the statements are true about FLUORO QUINOLONES, except :

(A) Suspected of having teratogenic potential

(B) Arthropathy of limb-in children may occur

(C) Increase theophylline toxicity

(D) Increase neuromuscular blocking action

Answer: (D)

136. Differences between action of DEC and Ivermectin in a case of scrotal filariasis is :

(A) DEC acts more effectively on microfilariae than Ivermectin

(B) DEC acts only on micro filariae and Ivermectin acts only on adults

(C) DEC acts on both microfilariae and adults while Ivermectin acts on adults only

(D) DEC acts on adults and Ivermectine on microfilariae

Answer: (D)

137. Cyclosporin acts by inhibiting the proliferation of 

(A) IL₁

(B) IL₂  

(C)  IL₆

(D) Macrophages

Answer: (B)

138. Side-effects of the cis-platinum include all of the following except :

(A) Nausea and vomiting

(B) Nephrotoxicity

(C) Blindness

(D) Ototoxicity

Answer: (C)

139. Milk-Alkali syndrome may  be caused by ingestion of

(A) Calcium carbonate

(B) Magnesium sulphate

(C) Aluminium trisilicate

(D) Aluminium hydroxide

Answer: (A)

140. Pancreatitis is a known side effect with administration of :

(A) L-Asparaginase

(B) Corticosteroid

(C) Cyclophosphamide

(D) Vincristine

Answer: (A)

141. Pulmonary infiltration may be seen with all of the following drugs except :

(A) 5 FU

(B) Bleomycin

(C) Busulphan

(D) Cyclophosphamide

Answer: (A)

142. SLE like syndrome is most commonly associated with administration of :

(A) Rifampicin

(B) Procainamide

(C) Digitalis

(D) Phenytoin

Answer: (B)

143. Alkalinity of urine is done in

(A) Barbiturate poisoning

(B) Lithium toxicity

(C) Alprazolam overdose

(D) Diazepam toxicity

Answer: (A)

144. Adriamycin is classified as

(A) Antibiotic

(B) Antimetabolite

(C) 5-FU antagonist

(D) Vinca alkaloids

Answer: (A)

145. In G-6-P-D- deficiency, which drug does not cause hemolysis?

(A) Rifampicin

(B) Primaquine

(C) Furazolidone

(D) Co-trimoxazole

Answer: (A)

146. The mechanism of action of barbiturate on cerebrum is

(A) Scavenger of radicles

(B) Increase BMR in active area of brain

(C) Acts as Cl⁻ channel complex

(D) Decreased intracranial pressure

Answer: (C)

147. Best used in digoxin induced arrhythmia

(A) Phenytoin

(B) Lignocaine

(C) Quinidine

(D) Procainamide

Answer: (B)

148. Adrenaline cause vaso constriction in all of the following vessels except:

(A) Gut

(B) Cerebral

(C) Cutaneous

(D) Renal

Answer: (B)

149. About acidic drug true is:

(A) Best absorbed in acidic medium

(B) Best absorbed in alkaline medium

(C) Not absorbed in acidic medium

(D) Binds to alpha glycoprotein

Answer: (A)

150. Mebendazole is effective for following except:

(A) Cysticercosis

(B) T.trichuria

(C) Trichinella

(D) Ascaris

Answer: (A)

151. Hepatitis is a side effect of following except:

(A) INH

(B) R-cin

(C) Pyrazinamide

(D) Ethambutol

Answer: (D)

152. Long term use of chloroquine causes A/E

(A) Lichenification

(B) Retinitis pigmentosa

(C) Yellow discoloration of skin

(D) Corneal deposits

Answer: (B)

153. PSVT is best treated by:

(A) Adenosine

(B) Lidocaine

(C) Phenytoin

(D) Quinidine

Answer: (A)

154. Selective alpha 1 antagonists are following except:

(A) Prazosin

(B) Terazosin

(C) Butoxamine

(D) Tamsulosin

Answer: (C)

155. In chronic renal failure which one is contraindicated?

(A) Furosemide

(B) Ethacrynic acid

(C) Triamterene

(D) Bumetanide

Answer: (C)

156. Most powerful coronary vasodilator is :

(A) Adenosine

(B) CO₂

(C) Hypoxia

(D) Hypertension

Answer: (C)

157. Granulomatous hepatitis is caused by:

(A) Allopurinol

(B) Allopurinol

(C) Furazolidone

(D) Amiodarone

Answer: (A)

158. Cromolyn sodium is used for following actions

(A) Mast cell stabilisation

(B) H₁ antihistamine

(C) Phosphodiesterase inhibitor

(D) Inhibition of cycloxygenase

Answer: (A)

159. Most common complication of dapsone is:

(A) Hemolysis

(B) Hepatitis

(C) Renal failure

(D) Eczema

Answer: (A)

160. Dosage of drug is determined by following except :

(A) Volume of distribution

(B) Half life

(C) Lipid solubility

(D) Excretion of drug

Answer: (B)

161. Aspirin is given in MI because it:

(A) ↑PT

(B) ↓ TxA2

(C) ↓ histamine

(D) ↑ fibrosis

Answer: (B)

162. Ondansetron acts by inhibiting receptors:

(A) 5 HT

(B) 5 HT₂

(C) 5 HT₃

(D) NE

Answer: (C)

163. Digitalis has inotropic action due to:

(A) Initiation of Na⁺ K⁺ ATPase

(B) Trapping Ca⁺⁺ release

(C) Inhibiting Na⁺ K⁺ ATPase

(D) Increase in intra cellular K⁺

Answer: (A)

164. Max. nicotinic effect is seen with:

(A) Pilocarpine

(B) Carbachol

(C) Bethanechol

(D) Methacholine

Answer: (B)

165. Biguanides acts by following except :

(A) ↑ Insulin release from pancreas

(B) ↑ Glycolysis

(C) ↓ Gluconeogenesis

(D) ↑ Insulin binding to its receptors

Answer: (A)

166. Drug not causing active hepatitis

(A) INH

(B) Methyldopa

(C) Chlorpromazine

(D) Oxyphenacetin

Answer: (C)

167. Glucocorticoids causes :

(A) ↓ osteoid formation

(B) IgF stimulation

(C) ↑ Ca ++ absorption from stomach

(D) Calcification of bone

Answer: (A)

168. True about lactulose is

(A) Blood ammonia

(B) 10% absorbed from gut

(C) Osmotic laxative

(D) Disaccharide of lactose and sucrose

Answer: (C)

169. OCP produces in liver

(A) Cholangio Ca

(B) Adenoma

(C) Cirrhosis

(D) Atrophic changes

Answer: (B)

170. Mannitol when given I/V causes

(A) ↓ Blood viscosity

(B) ↑ Blood viscosity

(C) ↓ GFR

(D) ↑ ICT

Answer: (B)

171. Drug contraindicated in pregnancy:

(A) Clonidine

(B) Enalapril

(C) Methyl dopa

(D) Hydralazine

Answer: (B)

172. Not true about heparin is

(A) Activates antithrombin III

(B) Small unionized molecule are not absorbed orally

(C) Protamine sulphate always used to reverse its action

(D) Release lipoprotein lipase from vessel wall and tissues

Answer: (B)

173. In drug antagonist, 1st event to occur is

(A) Deactivation of receptors

(B) Receptor configuration changes

(C) A & B

(D) None of these

Answer: (A)

174. Ketanserin is:

(A) 5 HT 2 reuptake blocker

(B) Antipsychotic

(C) Useful in movement disorder

(D) Anti tussive

Answer: (A)

175. Newer insulins are :

(A) Acidic

(B) Alkaline

(C) Neutral

(D) Monomers

Answer: (C)

176. Amatoxins in mushroom poisoning acts by inhibiting:

(A) DNA

(B) mRNA

(C) Adenosine

(D) G – proteins

Answer: (B)

177. Alpha one stimulation produces:

(A) HR ↑

(B) BP ↑

(C) HR ↓

(D) None of these

Answer: (A)

178. Hypothyroidism may be caused by:

(A) Ethionamide

(B) Thiocetazone

(C) Ethambutol

(D) Streptomycin

Answer: (A)

179. Inhibits mitosis by forming microtubules :

(A) Vinca alkaloids

(B) Docetaxel

(C) Etoposide

(D) Colchicine

Answer: (B)

180. Osmotic diuretics cause following excepts:

(A) Inhibit Renin release

(B) ↑ Intravascular volume

(C) Reduce salt reabsorption in ascending limb

(D) None of the above

Answer: (D)

181. Antiviral agent of choice in a child with bronchiolitis

(A) Vidarbin

(B) Ribavirin

(C) Acyclovir

(D) Amantadine

Answer: (B)

182. Non-myelosuppressive agent is:

(A) Vinblastine

(B) Vincristine

(C) Bleomycin

(D) Methotrexate

Answer: (C)

183. Fluroquinolones acts by inhibiting:

(A) DNA dependent RNA synthetase

(B) Cell wall synthesis

(C) DNA gyrase

(D) Protein synthesis

Answer: (C)

184. Cyclophosphamide is used in all except :

(A) Burkitt’s lymphoma

(B) Hodgkin’s

(C) Choriocarcinoma

(D) Ovarian Ca

Answer: (C)

185. Amantidine acts by action of :

(A) Inhibiting replication

(B) Inhibits mRNA

(C) Inhibits tRNA

(D) Inhibits RNA

Answer: (A)

186. Prostaglandins are not indicted in:

(A) PDA

(B) Labour

(C) Gastric ulcer

(D) Paralytic ileus

Answer: ()

187. All are true of atropine except :

(A) ↑ BP

(B) ↑ HR

(C) ↓ BP

(D) Rubor

Answer: (C)

188. The max effect of a drug is defined by its:

(A) Therapeutic index

(B) Potency

(C) Efficacy

(D) Adversity

Answer: (C)

189. Diplopia, ataxia, gingival hyperplasia and hirsutism are side effect of:

(A) Phenobarbitone

(B) Phenytoin

(C) Carbamazepine

(D) Valproic acid

Answer: (B)

190. In a person with HR = 120, rate is reduced by:

(A) Propranolol

(B) Phentolamine

(C) Phenoxybenzamine

(D) Prazosin

Answer: (A)

191. DOC in tapeworm infestation:

(A) Niridazole

(B) Niclosamide

(C) Albendazole

(D) Praziquantel

Answer: (D)

192. Enalapril acts by :

(A) Angiotensin converting enzyme inhibition

(B) Angiotensin receptor blockade

(C) Calcium channel blocker

(D) Direct vasodilation

Answer: (A)

193. Short 1/2 life on thiopentone is due to :

(A) Rapid excretion

(B) Rapid redistribution

(C) Rapid metabolism

(D) All of the above

Answer: (B)

194. Methotrexate is most useful in :

(A) Abruptio placentae

(B) Ectopic pregnancy

(C) Placenta accreta

(D) Trophoblastic disease

Answer: (D)

195. Ganglion blockers act by :

(A) Inhibition of release of acetyl choline

(B) Competing for acetyl choline receptor

(C) Preventing conduction from pre post ganglionic

(D) Inhibition of Ach destruction

Answer: (C)

196. Dale’s vasomotor reversal is due to :

(A) Alpha blocker

(B) Beta blocker

(C) ACH inhibitor

(D) All of the above

Answer: (A)

197. NSAID’s are useful  because they inhibit:

(A) Cycloxygenase

(B) Thromboxane

(C) Histamine

(D) 5-HT

Answer: (A)

198. The action of non-competitive muscle blockers is affected by :

(A) Hypocalcemia

(B) Hyponatremia

(C) Hyperthermia

(D) All of the above

Answer: (A)

199. Mechanism of action of chloramphenicol is :

(A) Preventing binding of tRNA to ribosome

(B) Polypeptide elongation

(C) Protein chain elongation

(D) Inhibiting cell wall synthesis

Answer: (A)

200. True regarding dose-response curve is :

(A) Cannot determine the potency of a drug

(B) Log dose response curve is sigmoid shaped

(C) Cannot find response to antagonist

(D) A wide range of doses can not be plotted

Answer: (B)

201. The anti-hypertensive contraindicated in pregnancy is :

(A) Enalapril

(B) Hydralazine

(C) Clonidine

(D) Alpha methyldopa

Answer: (A)

202. Mechanism of action of GABA is on :

(A) G protein

(B) Tyrosine kinase

(C) PIP/DAG

(D) 5-HT

Answer: (A)

203. Site of action of amphotericin is :

(A) Cell membrance

(B) Cytoplasm

(C) Neucleus

(D) Protein synthesis

Answer: (A)

204. Aerosolized rivavirin is used in the treatment of bronchiolitis with :

(A) RSV

(B) H.influenza

(C) Pneumococcus

(D) Streptococcus

Answer: (A)

205. Mechanism of action of zidovudine is :

(A) Protein synthesis

(B) Reverse transcriptase inhibition

(C) Nucleic acid synthesis

(D) Cell membrane

Answer: (B)

206. A patient on warfarin was given phenobarbitone. The result would have been :

(A) Increase the dose of warfarin

(B) Decrease the dose of warfarin

(C) Increase the dose of phenobarb

(D) Decrease the dose of phenobarb

Answer: (A)

207. Quinine given to a patient of falciparum malaria caused sweating and palpitation, the likely cause is :

(A) Cinchonism

(B) Hyperglycemia

(C) Hypoglycemia

(D) Hypokalemia

Answer: (C)

208. Zidovudine given for HIV in pregnancy because :

(A) Decreases chance of vertical transmission

(B) Decrease severity of infection in mother

(C) Decrease severity of infection in new born

(D) Cause no benefit

Answer: (A)

209. A young lady on OCP develops severe abdominal pain and frank psychosis. Most likely cause is :

(A) Hysteria

(B) Porphyria

(C) Thrombosis

(D) Mesenteric infarct

Answer: (B)

210. ACE inhibitors are contraindicated in :

(A) U/L renal artery stenosis with single kidney

(B) Proteinuria with DM

(C) MI

(D) Hypotension

Answer: (A)

211. Characteristic feature of agonist is :

(A) Has affinity only

(B) Has affinity as well as intrinsic activity

(C) Has intrinsic activity only

(D) Neither has affinity nor activity

Answer: (B)

212. True about metabolism of vitamin D in kidney is:

(A) Conversion of 25 to 1, 25 dihydroxy cholecalciferol

(B) Formation of 25 hydroxylase

(C) Activated form is deactivated

(D) None of these

Answer: (A)

213. Digoxin in used in CHF due to :

(A) HOCM

(B) High output failure

(C) AF with high ventricular rate

(D) All of these

Answer: (C)

214. All are used as antidepressants except :

(A) MAOI

(B) Fluoxetine

(C) Chlorpromazine

(D) Imipramine

Answer: (C)

215. Pharmacological defibrillator is :

(A) Adenosine

(B) Bretylium

(C) Lignocaine

(D) Amiodarone

Answer: (B)

216. Drug contraindicated in infectious mononucleosis is :

(A) Ampicillin

(B) Doxycycline

(C) Atropine

(D) Gentamicin

Answer: (A)

217. At pharmacological doses unwanted unavoidable effects are called :

(A) Side effects

(B) Idiosynchrotic reaction

(C) Toxicity

(D) Pharmacogenetics

Answer: (A)

218. True about triamterene is A/E:

(A) Saluretic effect is greater than thiazide

(B) Triamternece is often combined with thiazide

(C) Action is just like amiloride

(D) It is a K⁺ sparing diuretic

Answer: (A)

219. K+ channel opener is :

(A) Verapamil

(B) Nicorandil

(C) Nitroprusside

(D) Amrinone

Answer: (B)

220. Vasomotor reversal of dale seen with :

(A) Alpha blocker

(B) Beta blocker

(C) Alpha agonist

(D) Beta agonist

Answer: (A)

221. Antifolate immunosuppressant is :

(A) 5 FU

(B) Methotrexate

(C) Cyclophosphamide

(D) Cisplatin

Answer: (B)

222. MOA of allopurinol is :

(A) Increases uric acid excretion

(B) Decreases uric acid synthesis

(C) Both (a) and (b)

(D) None of these

Answer: (B)

223. Local anaesthetics act by  inhibiting :

(A) Influx of K⁺

(B) Efflux of K⁺

(C) Influx of Na⁺

(D) Efflux of Na⁺

Answer: (C)

224. Cheese reaction with MAO inhibitors is due to:

(A) Guanethidine

(B) Resperpine

(C) Cough remedies

(D) Tyramine

Answer: (D)

225. Mechanism of action of Chloramphenicol :

(A) Binds to 50S subunit

(B) Prevents chain elongation

(C) Misreading of DNA

(D) Binds to nuclear proteins

Answer: (A)

226. FK-506 is used in :

(A) Organ transplant

(B) Bronchial asthma

(C) Diabetic diarrhoea

(D) Chemotherapecutic agent

Answer: (A)

227. Cyt-P-450 is inhibited by :

(A) Phenobarbitone

(B) Cimetidine

(C) Phenytoin

(D) CCl₄

Answer: (B)

228. Gum hyperplasia, hirsutism, granulocytopenia is caused by :

(A) Phenytoin

(B) Carbamazepine

(C) Phenobarbitone

(D) Lamotrigine

Answer: (A)

229. True of tamoxifen :

(A) Binds to estrogen receptors

(B) ↑ estrogen production

(C) ↓FSH production

(D) ↑LH secretion

Answer: (A)

230. Ondansetron :

(A) 5HT₃ – Antagonist

(B) 5HT₃ – Agonist

(C) 5HT₁ antagonist

(D) 5HT1 antagonist

Answer: (A)

231. Displacement of protein bound drug :

(A) ↑↑ free drug plasma level

(B) ↑↑ side effects

(C) ↓free levels

(D) ↓ Effect

Answer: (A)

232. Which is true about Max. distribution?

(A) Highly lipophilic

(B) Blood brain barrier

(C) ↓excretion

(D) ↓ receptors

Answer: (A)

233. AZT has following side effects except :

(A) ↓ platelet

(B) Megaloblastic anemia

(C) Pancreatitis

(D) Cardiomyopathy

Answer: (C)

234. Mechanism of Acyclovir resistance :

(A) Thymidine kinase

(B) DNA – dependent RNA polymerase

(C) ↓ spectrum

(D) ↓ Side effects

Answer: (A)

235. Amoxycillin clavulanic acid  combination :

(A) ↑ spectrum

(B) ↑ 1/2 life of amoxy

(C) ↓ 1/2 life of both

(D) ↑ Side effects

Answer: (A)

236. About the use of nitrates all are true except :

(A) In HOCM-not given

(B) Acute M.I can be given

(C) CHF and acute LVF

(D) Methemoglobinemia

Answer: (D)

237. Which of the following are drug of first choice for cardiogenic shock?

(A) Dopamine

(B) Adrenaline

(C) Mephenteramine

(D) Digoxin

Answer: (A)

238. About plasma expanders all are true except :

(A) Dextran interferes with platelet aggregation

(B) Albumin is have 69 AA

(C) Half life of albumin-15 hours

(D) Amylopectin

Answer: (C)

239. In Aspirin mechanism of action :

(A) Lipooxygenase ↓

(B) Cyclooxygenase

(C) Phospholipase ↓

(D) ↑ Lipoxygenase

Answer: (B)

240. Which prostaglandin is used in NSAID induced ulcer?

(A) Misoprostol

(B) Carboprost

(C) Mirtazapine

(D) Miprinone

Answer: (A)

241. Atropine-mechanism of action in poisoning :

(A) Reactivation of choline-esterase

(B) Acts on central and peripheral post. gaglionic receptors

(C) Acts on central and peripheral cholinergic receptors

(D) Acts on peripheral cholinergic receptors only

Answer: (C)

242. Mechanism of resistance to aminoglycosides :

(A) Produces enzymes

(B) Inactivation of RNA dependent DNA polymerase

(C) Inactivation of RNA

(D) Prevents chain elongation

Answer: (A)

243. Among following least anaphylaxis is seen with :

(A) Streptokinase

(B) Urokinase

(C) Anisolated streptokinase

(D) tPA

Answer: (D)

244. Drug of choice for cardiogenic shock :

(A) Dopamine

(B) Propranolol

(C) Digitalis

(D) Milrinone

Answer: (A)

245. Which antitubercular drug metabolism is under genetic control?

(A) Rifampicin

(B) INH

(C) Cyclosporine

(D) PZM

Answer: (B)

246. Mala-N contains :

(A) Ethinyl estradiol 0.3 mg

(B) Norethisterone 1 mg

(C) D-norgestrol 0.5 mg

(D) D-norgestrol 0.3 mg

Answer: (B)

247. Absolute contraindication to OC pills :

(A) Epilepsy

(B) Migraine

(C) Pregnancy

(D) Thromboembolism

Answer: (D)

248. Antagonist to diazepam  :

(A) Phenargan

(B) Flumazenil

(C) Domperidone

(D) Bromocriptine

Answer: (B)

249. Atropine causes :

(A) Decreased cardiac output

(B) Heart block

(C) Hypertension

(D) Miosis

Answer: (A)

250. Treatment of choice for salmonella typhi :

(A) Cephalexin

(B) Gentamicin

(C) Tetracycline

(D) Ciprofloxacin

Answer: (D)

251. Drug of choice for Methicillin resistant staphylococcus aureus is (MRSA) :

(A) Amoxicillin-Clavulanate

(B) Vancomycin

(C) Flucloxacillin

(D) Clindamycin

Answer: (B)

252. A patient was started on antihypertensive medications; developed Renal failure, the drug offended is :

(A) Beta-blocker

(B) Alfa-blocker

(C) Calcium channel blocker

(D) ACE inhibitor

Answer: (D)

253. Central muscle relaxants acts by :

(A) Decreased nerve conduction

(B) Inhibits spinal polysynaptic reflexes

(C) Block conduction across NM junction

(D) CNS depression

Answer: (B)

254. Neostigmine :

(A) It is a quartarnary amounium compound

(B) Metabolised in liver

(C) It can cross the blood brain barrier

(D) Prominent effect on smooth muscles

Answer: (A)

255. Morphine cannot given in which of the following route?

(A) IM

(B) Transdermal

(C) Epidural

(D) Subarachnoid

Answer: (D)

256. β-blockers with intrinsic sympathomymetric properties are :

(A) Propranolol

(B) Oxprenolol

(C) Practolol

(D) Esmolol

Answer: (B)

257. Drug safely given in pregnancy :

(A) Antifolate

(B) Quinine

(C) Chloroquine

(D) Primaquine

Answer: (C)

258. Methotrexate causes :

(A) Inhibition of dihydrafolate reductase

(B) Inhibition of folate synthatase

(C) Not absorbed oraly

(D) Folic acid antagonizes it’s toxicity

Answer: (A)

Medical PG Microbiology

Medical PG Microbiology

1. All are Glycoproteins except

(A) Immunoglobulins

(B) Albumin

(C) hCG

(D) Complements

Answer: (B)

2. DNA is detected by

(A) Southern blot

(B) Northern blot

(C) Western blot

(D) Easter blot

Answer: (A)

3. In Respiratory and GIT infection, which is the most affected immunoglobulin

(A) IgA

(B) IgG

(C) IgM

(D) IgD

Answer: (A)

4. The commonest Primary immunodeficiency is

(A) Common variable immunodeficiency

(B) Isolated IgA immunodeficiency

(C) Wiskott-Aldrich syndrome

(D) AIDS

Answer: (B)

5. The reactions between antibody and soluble antigen is demonstrated by

(A) Agglutination

(B) Precipitation

(C) Complement fixation test

(D) Hemagglutination test

Answer: (B)

6. C-reactive protein is

(A) An antibody produced as a result of pneumococcal infection

(B) Derived from pneumoconiosis

(C) Detected by precipitation reaction

(D) Increased in pneumococcal infection

Answer: (D)

7. Bacteria which does not  strictly follow Koch’s postulate is

(A) Mycobacteria tuberculosis

(B) Mycobacterium avium intercellulare

(C) Mycobacterium leprae

(D) Bacillus anthracis

Answer: (C)

8. All the following are true about Nosocomial infections except

(A) May manifest within 48 hours of admission

(B) May develop after discharge of patient from the hospital

(C) Denote a new condition which is unrelated to the patient’s primary conditions

(D) May already present at the time of admission

Answer: (D)

9. All the following are true of Tuberculosis except

(A) For sputum to be positive, bacilli should be > 10⁴/ml

(B) Niacin test differentiates M. tuberculosis and M. bovis

(C) Pathogenicity to rabbits differentiates M. tuberculosis and M. bovis

(D) Culture techniques have low sensitivity

Answer: (D)

10. All the following are true regarding Vibrio cholerae except

(A) Transported in acidic medium

(B) Gram-negative

(C) Aerobic organism

(D) Ferments glucose

Answer: (A)

11. All the following are true about Chlamydia except

(A) Gram-positive

(B) Causes trachoma

(C) Causative organism of psittacosis

(D) Are also called basophilic viruses

Answer: (A)

12. All the following are general properties of Viruses except

(A) May contain both DNA and RNA

(B) Form extracellular infectious particles

(C) Heat labile

(D) Not affected by antibiotics

Answer: (A)

13. Which of the following is true about Rotavirus

(A) Commonly affects children

(B) Double-stranded DNA

(C) Can be grown easily on cell culture

(D) Egg shell appearance under electron microscope

Answer: (A)

14. Most common agent responsible for Bronchiolitis is

(A) RSV

(B) Adenovirus

(C) Herpes virus

(D) Influenza virus

Answer: (A)

15. A Discharging sinus is seen in

(A) Sporotrichosis

(B) Cryptococcosis

(C) Histoplasmosis

(D) Mycetoma

Answer: (D)

16. Which of the following is nottrue of Actinomycosis

(A) Demonstration of filaments

(B) Caused by actinomyces israelii

(C) Organism cannot be cultured

(D) Sulphur granules in pus

Answer: (C)

17. All of the following disease may be acquired by ingestion except

(A) Teniasis

(B) Guinea worm

(C) Toxopolasmosis

(D) Leishmaniasis

Answer: (D)

18. Which of the following Parasites does not pass through the human lung

(A) Ancylostoma

(B) Ascaris lumbricoides

(C) Trichuris trichura

(D) Stronglyloides

Answer: (C)

19. All are inhabitants of Liver except

(A) F. hepatica

(B) F. buski

(C) Clonorchis sinensis

(D) Opisthorchis felincus

Answer: (B)

20. Dark ground microscopy is used to see

(A) Refractile organisms

(B) Flagella

(C) Capsule

(D) Fimbriae

Answer: (B)

21. A single immunoglobulin molecule contains

(A) 1 light chain, 1 heavy chain

(B) 2 heavy chains, 1 light

(C) 2 light chains, 2 heavy chains

(D) 2 light chains, 1 heavy chain

Answer: (C)

22. Which one of the following is False

(A) Theobald-Smith phenomenon is a type I hypersensitivity reaction

(B) Serum sickness is a type II hyper sensitivity reaction

(C) Allograft rejection is a type IV hypersensitivity reaction

(D) Transfusion reaction is a type II hypersensitivity reaction

Answer: (B)

23. The best site to obtain a swab in asymptomatic gonorrhea is

(A) Endocervix

(B) Urethra

(C) Lateral vaginal wall

(D) Posterior fornix

Answer: (A)

24. Swarming growth on culture is characteristic of which Gram-negative organism

(A) Clostridium welchi

(B) Clostridium tetani

(C) Bacillus cereus

(D) Proteus mirabilis

Answer: (D)

25. Gram-positive organism that produces swarming growth on culture medium is

(A) Proteus mirabilis

(B) Salmonella typhi

(C) Clostridium tetani

(D) Clostridium welchii

Answer: (C)

26. Which of the following produces enterotoxin

(A) Sh. dysenteriae

(B) Sh. sonnei

(C) Sh. flexneri

(D) Sh. boydi

Answer: (A)

27. Which antigen blocks the agglutination of salmonella by O antiserum

(A) H

(B) Fimbriae

(C) Vi

(D) O

Answer: (C)

28. One of the following infections is caused by anaerobic gram-positive cocci

(A) Puerperal infection

(B) Food poisoning

(C) Endocarditis

(D) Septicemia

Answer: (A)

29. Chlamydia causes all the following diseases except

(A) Non-gonococcal urethritis

(B) Pneumonia

(C) Trachoma

(D) Parotitis

Answer: (D)

30. All of the following viral genes associated with HIV infection code for structural proteins except

(A) gag gene

(B) env gene

(C) pol gene

(D) tat gene

Answer: (D)

31. Which of the following is a marker of HIV infection in blood

(A) DNA polymerase

(B) RNA polymerase

(C) Reverse transcriptase

(D) DNA isomers

Answer: (C)

32. All the following are true about HIV infection except

(A) Caused by an enveloped RNA virus

(B) Rate of killing is directly proportional to T₄ molecules on cell surface

(C) Increased release of acid labile interferon

(D) Decreased delayed hypersensitivity activity reaction

Answer: (C)

33. All the following viruses cause pneumonia except

(A) Cytomegalovirus

(B) Mumps

(C) Measles

(D) Retrovirus

Answer: (B)

34. EB virus belongs to which group

(A) Retrovirus

(B) Herpes virus

(C) RNA virus

(D) Pox virus

Answer: (B)

35. All of the following are true about the papovavirus except

(A) They are non-enveloped icosahedral viruses

(B) Produce papilloma and warts in humans

(C) RNA virus

(D) SV-40 is oncogenic

Answer: (C)

36. Which of the following is False about mycetoma

(A) Can affect lower and upper extremities

(B) Caused by actinomycetes and filamentous fungi

(C) Diagnosis is by examination of pus

(D) Uncommon in India

Answer: (D)

37. Which is False regarding Cryptococcus neoformans

(A) Grows at 5℃ and 37℃

(B) It has 4 serotypes

(C) Urease negative

(D) Causes superficial skin infection

Answer: (C)

38. Cryptococcus is least likely to cause infection of

(A) Skin

(B) Bone

(C)  Brain

(D) Kidney

Answer: (D)

39. Which one of the following is associated with infection of the CNS

(A) Toxoplasma

(B) Cryptococcus

(C) Treponema

(D) Treponema

Answer: (D)

40. Which if the following diseases is transmitted by egg ingestion

(A) Teniasis

(B) Trichinosis

(C) Hydatidosis

(D) Strongyloidosis

Answer: (C)

41. All the following are TRUE about Brugia malayi except

(A) The intermediate hosts in the India are Mansoni

(B) The tail tip is free from nuclei

(C) Nuclei are blurred, so counting is difficult

(D) Adult worm is found in the lymphatic system

Answer: (B)

42. Differentiation of N.gonorrhea and N. meningitides can be done by

(A) Glucose fermentation

(B) V . P. Reaction

(C) Maltose fermentation

(D) Indole test

Answer: (C)

43. The medium used for Mycobacterium tuberculosis is

(A) Sabouraud’s medium

(B) L J medium

(C) Pick’s medium

(D) NIH medium

Answer: (B)

44. Rapid examination of Tubercle bacilli is possible with

(A) Ziel-Nelson stain

(B) Kin young stain

(C) Auramine-Rhodamine stain

(D) Giemsa stain

Answer: (C)

45. The commonest focus of Scrofuloderma is

(A) Lung

(B) Lymph node

(C) Larynx

(D) Skin

Answer: (B)

46. For experiment work, Lepra bacilli are best cultured in

(A) Armadillos

(B) Mouse foot pad

(C) Guinea pigs

(D) Rabbit testes

Answer: (B)

47. Which of the following is a true statement regarding Enterobacteriaceae

(A) Motility is by polar flagellum

(B) Glucose is fermented by all members of the family

(C) All members are oxidase positive

(D) Nitrate reduction negative

Answer: (B)

48. Which of the following is true about Enteropathogenic E coli

(A) Causes diarrhea in infants

(B) Acts by invasion of intestinal epithelial cells

(C) Adults are mostly affected

(D) Affects immunocompromised host

Answer: (A)

49. Which of the following is true regarding Salmonella infection

(A) Urine culture is +ve in 1st week

(B) Stool culture is +ve in 1st week

(C) Blood culture is +ve in 3-7 days

(D) Widal test is +ve in 1st week

Answer: (C)

50. Varicella are classified under

(A) Enterovirus

(B) Retrovirus

(C) Poxvirus

(D) Herpesvirus

Answer: (D)

51. Negri bodies are found in

(A) Hypothalamus

(B) Hippocampus

(C) Midbrain

(D) Medulla

Answer: (B)

52. When compared to the Western blot technique, ELISA test is

(A) More sensitive, less specific

(B) More sensitive, more specific

(C) Less sensitive, less specific

(D) Less sensitive, more specific

Answer: ()A

53. Which of the following is true about malaria

(A) Size of RBC is enlarged in Vivax infection

(B) Size of RBC is enlarged in Falciparum infection

(C) Schuffner’s dots are seen in Malariae infection

(D) Relapse is seen in Falciparum infection

Answer: (A)

54. Which of the following is true about P.falciparum

(A) James dots are seen

(B) Accole forms are seen

(C) Relapses are frequent

(D) Longest incubation period

Answer: (B)

55. Which of the following is true regarding the Trophozoite of E.histolytica

(A) Has eccentric karyosomes

(B) Presence of bacteria inside

(C) Has four nuclei

(D) Shows erythrophagocytosis

Answer: (D)

56. Which of the following is true about mature cysts of E. histolytica

(A) Endoplasm and ectoplasm are clearly defined

(B) Eight chromatid bodies

(C) Shows chromatid bodies and glycogen mass

(D) Nuclear structure retains characteristics of trophozoite

Answer: (D)

57. Which one of the following does not pass through the lungs

(A) Hookworm

(B) Ascaris

(C) Strongyloides

(D) Enterobius vermicularis

Answer: (D)

58. The causative organism in Madura Mycosis Mycetoma is

(A) Nocardia

(B) Dimorphic fungus

(C) Aspergillus

(D) Dermatophytes

Answer: (A)

59. Helper cells belong to

(A) T cells

(B) Macrophages

(C) B cells

(D) Monocytes

Answer: (A)

60. The type of receptors present on T cells are

(A) IgG

(B) IgD

(C) CD₄

(D) Prostaglandins

Answer: (C)

61. What enhances multiplication of T cells in culture

(A) Phytohemagglutinin

(B) Chemotactic factor

(C) Leukotrienes

(D) Prostaglandins

Answer: (A)

62. Which of the following is an example of type IV hypersensitivity

(A) Granulomatous reaction

(B) Schwartzman reaction

(C) Schwartzman reaction

(D) Serum sickness

Answer: (A)

63. Which precipitates at 50℃-60℃ but disappears on heating

(A) Heavy chain

(B) Light chain

(C) Both

(D) None of the above

Answer: (B)

64. Prozone phenomenon is due to

(A) Excess antigen

(B) Excess antibody

(C) Hyperimmune reaction

(D) Disproportionate antigen-antibody levels

Answer: (B)

65. A substance, when added to a culture causes inhibition of multiplication but on removal causes enhanced growth. This substance is called

(A) Bacteriostatic

(B) Bactericidal

(C) Sterilization

(D) Bacteriophage

Answer: (A)

66. Bence Jones proteins are best described as

(A) μ chains

(B) γ chains

(C) Kappa & Lambda chains

(D) Fibrin split products

Answer: (C)

67. Interleukin II is secreted by

(A) B lymphocytes

(B) T lymphocytes

(C) Neutrophils

(D) Macrophages

Answer: (B)

68. All of the following are glycoproteins except :

(A) Blood antigen

(B) Albumin

(C) Immunoglobin

(D) HCG

Answer: (B)

69. Which of the following statements concerning immunoglobulins is wrong :

(A) IgM does not cross placenta

(B) IgE is ↑ed in parasitic infection

(C) IgM increased in primary response

(D) Fetal infection is characterized by increase in IgG

Answer: (D)

70. Which of the following is an example of Type IV hypersensititvity :

(A) Arthrus reaction

(B) Serum sickness

(C) Schwartzmann reaction

(D) Granulomatous reaction

Answer: (D)

71. Diagnosis of ABO incompatability can b e made from all of the following except :

(A) Sweat

(B) Saliva

(C) Semen

(D) C.S.F

Answer: (D)

72. Out of the following the true statement regarding sterilization is :

(A) Dry heat is the best method of sterilization of liquid paraffin

(B) All glass syringes are best sterilized by boiling at 100℃

(C) Bacterial vaccines are best sterilized by ethylene oxide

(D) Pasteurization of milk of flash method is done by heating at 63℃ for 30 minutes

Answer: (A)

73. True statement about Antistreptolysin ‘O’ titre is :

(A) In normal people the titre is < 200

(B) In acute glomerulonephritis the titre is low

(C) ASO titre > 200 indicate rheumatic fever

(D) Streptozyme test is an active haemagglutination test

Answer: (B)

74. True statement regarding diptheria is

(A) It can be diagnosed by demonstration of antibodies by ELISA

(B) Immunization prevents carriers state

(C) Treatment of contacts is not indicated

(D) Iron has critical value in the production of toxin

Answer: (D)

75. All of the following statements about cholera are true except :

(A) O & H antigens measure carrier state

(B) Culture medium is TCBS Agar

(C) Produces indole and reduces nitrate

(D) Synthesize neuraminidase

Answer: (A)

76. All of the following statements are true for Eltor cholera except :

(A) Infection is mild and asymptomatic

(B) They are resistant to polymyxin-B unit disc

(C) Chronic carriers are common

(D) Secondary attacks rate is high in families

Answer: (D)

77. Shigella are be divided into sub group on the basis of ability to ferment :

(A) Lactose

(B) Maltose

(C) Fructose

(D) Mennitol

Answer: (D)

78. Which bacteria acts by inhibiting protein synthesis :

(A) Pseudomonas

(B) Staphylococus

(C) Steptococcus

(D) Klebscella

Answer: (A)

79. All of the following statements about Botulism are true except :

(A) Botulism is caused by endotoxin

(B) Honey ingestion causes infant botulism

(C) Constipation is seen

(D) Detection of antitoxin in the serum can aid in diagnosis

Answer: (A)

80. Acute haemorrhagic conjunctivitis is caused by

(A) Entrovirus 70

(B) Adenovirus

(C) Poliovirus

(D) Hepadina virus

Answer: (A)

81. True statement about rabies is :

(A) Rabies infection causes life long immunity

(B) Rabies vaccine is always live attenuated

(C) Rabies has various strains

(D) Rabies is best diagnosed by immuneflorescence study

Answer: (D)

82. Most common agent causing tuberculosis in AIDS patient in tropical countries is :

(A) Mycobacterium tuberculosis

(B) Mycobacterium intracellulare

(C) Mycobacterium parvum

(D) Mycobacterium atypical

Answer: (A)

83. Epidemiological marker of Hepatitis B is

(A) HBs Ag

(B) HBe Ag

(C) Core antigen

(D) Core antibody

Answer: (D)

84. Reverse transcriptase polymerase chain reaction can aid in diagnosis of all of the following viral infections except :

(A) Adenovirus

(B) Astrovirus

(C) Rotavirus

(D) Poliovirus

Answer: (A)

85. All of the following are dimorphic fungi except :

(A) Sporotricum

(B) Blastomycetes

(C) Histoplasma

(D) Cryptococcus

Answer: (D)

86. Most common presenting symptom of thread worm infection amongst the following is :

(A) Abdominal pain

(B) Rectal prolapse

(C) Urticaria

(D) vaginitis

Answer: (A)

87. All of the following statements about toxoplasmosis are true except :

(A) Oocyst in freshly passed cat’s faeces is not infective

(B) May spread by organ transplantation

(C) Maternal infection acquired after 6 months has high risk of transmission

(D) Arthalgia, sore throat and abdominal pain are the most common manifestations

Answer: (D)

88. Plasmodium falciparum infection of man is characterized by :

(A) The erythrocytes are increased insize

(B) All stages erythrocytic schizogony are seen in peripheral blood

(C) Multiple infection of erythrocytes are seen

(D) Each erythrocytic cycle lasts 72 hours

Answer: (B)

89. Haptane is :

(A) Same as epitope

(B) Small molecular weight protein

(C) Requires carrier for specific antibody production

(D) Simple haptens are precipitate

Answer: (C)

90. All of the following forces are involved in Antigen antibody reaction except :

(A) Vander Waal’s forces

(B) Electrostatic bond

(C) Hydrogen bond

(D) Covalent bond

Answer: (D)

91. Examples of type hypersensitivity is :

(A) Lepromin test

(B) Tuberculin

(C) Casoni’s test

(D) Arthrus reaction

Answer: (C)

92. Most common cause of chronic granulomatous disease in children is :

(A) Myeloperoxidase deficiency

(B) Defective phagocytosis

(C) Defective H₂O₂production

(D) Job’s disease

Answer: (C)

93. Most common agents responsible for human bite infections are :

(A) Gram-ve bacilli

(B) Gram +ve bacilli

(C) Spirochaete

(D) Anaerobic streptococci

Answer: (D)

94. True statements about Pneumococcus are all except :

(A) Pneumolysin a thiolactivated toxin, exerts a variety of effects on ciliary & PMN’s

(B) Autolysin may contribute to the pathogenesis of pneumococcal disease by lysing the bacteria

(C) Anticapsular antibodies are seotype specific

(D) The virulence of pneumococci is dependent only on the production of the capsular polysaccharide

Answer: (D)

95. True about corynebacterium diptheria are all except :

(A) Iron is required for toxin production

(B) Toxin production is responsible of local reaction

(C) Nonsporing, noncapsular & non motile

(D) Toxin production is by lysogenic conversion

Answer: (B)

96. All of the following are true about bacillus anthrax except :

(A) Plasmid is responsible for toxin production

(B) Cutaneous anthrax generally resolve spontaneously

(C) Capsular polypeptide aids virulence by inhibiting phagocytosis

(D) Toxin is a complex of two fractions

Answer: (D)

97. Bacterial genome has been completely recognized for which one of the following agents

(A) H. pylori

(B) Yersenia enterocolitis

(C) Campylobacter jejuni

(D) Streptococcus pnumonae

Answer: (A)

98. All of the following are true about helicobacter pylori except :

(A) About 50% of world population affected

(B) 85% of population is affected in some developing countries

(C) All children’s in developing countries have immunity by five years of age

(D) Infection is common in low socioeconomic status

Answer: (C)

99. True about helicobacter pylori is :

(A) Culture and gram staining of biopsy is the gold standard investigation

(B) Controlled urea breath is negative with massive infection

(C) Anti urease antibody are produce only by invasive strains

(D) Urease activity provides protective environment to the bacilli

Answer: (D)

100. True about helicobacter pyelori are all except :

(A) Urea breath test is diagnostic

(B) Gram negative, flagellate bacilli

(C) Risk factor for development of denocarcinoma of stomach

(D) It provides life long immunity

Answer: (D)

101. Which of the following is an obligate parasite :

(A) Mycoplasma

(B) Chlamydia trachomatis

(C) Gram-Ve bacilli

(D) Gram+ve cocci

Answer: (B)

102. Plaque formation in virus is done for :

(A) Isolation and typing of viruses

(B) Cloning separation of specific viruses

(C) Determining infectivity of virus

(D) Accessing multiplication of virus

Answer: (C)

103. All of the following are true about Herpes group virus except :

(A) Ether sensitive

(B) May cause malignancy

(C) HSV II involves below diaphragm

(D) Burkitt’s lymphoma-involves-Tcells

Answer: (D)

104. Which virus become reactivate and involves the eye :

(A) Herpes-zoster

(B) CMV

(C) E.B. virus

(D) Entero-70

Answer: (A)

105. Viral enterotoxin is detected as a possible mechanism of pathogenesis in :

(A) Adeno virus

(B) Rota virus

(C) Calcivirus

(D) Astrovirus

Answer: (A)

106. Best vaccine for rota virus infection is :

(A) Asymptomatic neonatal vaccine

(B) DNA vaccine

(C) Genetic reassortment

(D) Capsular component vaccine

Answer: (C)

107. AIDS involves :

(A) T-helper cells

(B) T-suppressor cells

(C) T-cytotoxic

(D) B. cells

Answer: (A)

108. Sputum examination is not useful in diagnosis of :

(A) Trichuriasis trichura

(B) Ank. duodenale

(C) Paragomniasis

(D) Strongyloids

Answer: (A)

109. TRUE about Cryptococcus neoformis is all except :

(A) Anticapsular antigen is detected in C.S.F

(B) Common in immunocompromised patient

(C) Anticapsular antibody prevents recurrence

(D) Strongly positive mucicarmine stain of the organism in tissue is diagnostic

Answer: (C)

110. Prokaryotes are characterized by :

(A) Absence of nuclear membrane

(B) Presence of microvilli on its surface

(C) Presence of smooth endoplasmic reticulum

(D) All of the above

Answer: (A)

111. Which of the following immunoglobulins can croses placenta :

(A) IgA

(B) IgM

(C) IgG

(D) IgD

Answer: (C)

112. Type of graft, best suited for renal transplantation :

(A) Allograft

(B) Autograft

(C) Xenograft

(D) Isograft

Answer: (D)

113. True statement regarding non-coagulase staphylococci is :

(A) They are non-pathogenic

(B) They commonly infect indwelling prosthesis

(C) They may cause scarlet fever

(D) They are separated by gram staining

Answer: (B)

114. True statement regarding Pneumococcus is :

(A) Virulence is due to polysaccharide capsule

(B) Capsule is protein in nature

(C) Antibodies against capsule are not protective

(D) Resistance to antibodies has not yet been reported

Answer: (A)

115. True statement about diptheria toxin in :

(A) Toxin is phage mediated

(B) Toxin is required for local infection

(C) Endotoxaemia causes systemic manifestation

(D) Toxin Acts by inhibiting synthesis of capsule

Answer: (A)

116. Most common organism responsible for gas gangrene is :

(A) Clostridium-perfringens

(B) Clostridium-difficile

(C) Clostridium tetani

(D) Clostridium speticum

Answer: (A)

117. Most common organism responsible for pseudo membranous colitis is :

(A) Clostridium difficile

(B) Clostridium botulism

(C) Clostridium bifermentans

(D) Clostridium histolyticum

Answer: (A)

118. Which of the following statement is true about vibriocholarae :

(A) There is no natural reservoir

(B) Transported in alkaline peptone water medium

(C) Halophilic

(D) Oxidase negative

Answer: (B)

119. True statement about Widal test in typhoid is :

(A) O-antigen titre remains positive for several months & reaction to it is rapid

(B) H-antigen titre remains positive for several months & reaction to it is rapid

(C) Both remain positive for several months & reaction to both is rapid

(D) None

Answer: (B)

120. True statement about widal test in typhoid is :

(A) Widal test is confirmative in endemic areas

(B) Antibiotic treatment doesnot alter widal test results

(C) Previous infection alters widal test

(D) Widal test does not alter with prior vaccination

Answer: (C)

121. Shigella can be differenciated from E.Coli by all of the following features except :

(A) Shigella does not produce gas from glucose

(B) Shigella does not ferment lactose

(C) Shigella does not ferment mannitol

(D) Shegella has no flagella non motile

Answer: (C)

122. Which of the following statement regarding shigella dysentriae type I is true :

(A) It can lead to haemolytic uramic syndrome

(B) It produces an invasive enterotoxin

(C) It is an facultative aerobes

(D) It is MR negative

Answer: (A)

123. All of the following bacteria test “Urease positive” except :

(A) E.Coli

(B) Proteus

(C) Kleibsella

(D) Staphylococcus

Answer: (A)

124. True statement about Influenza-A is :

(A) It has a double stranded segmented RNA

(B) Pandemic are caused by antigenic drift

(C) Nucleocapsid antibody is not specific

(D) Hemaglutinin and Neuraminidase is strain specific

Answer: (D)

125. Herpes-Zoster is caused by :

(A) Herpes-simplex type I

(B) Herpes-simplex type II

(C) Epstein-barr virus

(D) Varicella

Answer: (D)

126. Cryptococcus-neoformans is a :

(A) Protozoa

(B) Fungus

(C) Parasite

(D) Mycoplasma

Answer: (B)

127. Commonest parasite of CNS in India is :

(A) Schistomiasis

(B)  Cysticercosis

(C) Trichenella-spiralis

(D) Hydatid cyst

Answer: (B)

128. Commonest helmithic infection in AIDS is :

(A) Trichuris trichura

(B) Strongyloides stercoralis

(C) Enterobius vermicularis

(D) Nector-americana

Answer: (B)

129. The most common cause of mycetoma in India:

(A) Nocardia braziliensis

(B) Actinomadura madurae

(C) Piedra

(D) Tinea cruris

Answer: (B)

130. Negri bodies are characteristic of viral infection by:

(A) Rabies

(B) Toxoplasmosis

(C) Polio

(D) Herpes simplex infection

Answer: (A)

131. True about exotoxin is:

(A) Heat stable

(B) Produced both by Gram ‘+’ ve & Gram ‘-’ ve organism

(C) No enzymatic action

(D) Produced by Gram ‘+’ ve organism only

Answer: (B)

132. Cerebral malaria is caused by plasmodium:

(A) Falciparum

(B) Ovale

(C) Malaria

(D) Vivax

Answer: (A)

133. Tinea cruris is caused by:

(A) Epidermophyton

(B)  Trichophyton

(C) Microsporum

(D) Candida

Answer: (A)

134. Rheumatoid factor is used to:

(A) Screen Rh arthritis cases

(B) Assess severity of disease

(C) Monitor of therapy of RA

(D) Sort out the resistant cases

Answer: (B)

135. A patient of immunocompromised due to transplantation is suffering from pyrexia and neutropenia. Most likely cause is:

(A ) HSV

(B) CMV

(C) Gram ‘-’ ve organism

(D) Gram ‘+’ ve organism

Answer: (B)

136. Fungi without sexual cycle are classified as:

(A) Fungi imperfecti

(B) Phycomyces

(C) Ascomycetes

(D) Basidiomycetes

Answer: (A)

137. Segmented double stranded RNA virus is seen in:

(A) Reovirus

(B) Myxovirus

(C) Rabies

(D) Parvo virus

Answer: (A)

138. Which of the following is very difficult to induce antibody?

(A) Polysaccharide

(B) Protein

(C) Antigens

(D) Repeated infections

Answer: (A)

139. Mucocutaneous leishmaniasis is caused by:

(A) L-braziliensis

(B) L.tropica

(C) L-donovani

(D) L orientalis

Answer: (A)

140. Which toxin of streptococcus causes hemolysis?

(A) Streptolysin O

(B)  Streptolysin S

(C) Streptokinase

(D) Streptodornase

Answer: (B)

141. Amoebae not found in human intestine:

(A) E.histolytica

(B) E.coli

(C) E.nana

(D) E.gingivalis

Answer: (D)

142. Once a virus is present in a cell it does not allow other viral infection is known as:

(A) Viral interference

(B) Viral inhibition

(C) Haemadsorption

(D) Cytopathic effect

Answer: (A)

143. Recently infected hepatitis B is detected by:

(A) HBs Ag

(B) Anti HBs Ag

(C) Anti HBc Ag

(D) Anti Hbe Ag

Answer: (C)

144. IgG is normal or slightly reduced in A/E

(A) Wiskott aldrich syndrome

(B) Selective IgA deficiency

(C) Di George syndrome

(D) Common variable deficiency

Answer: (D)

145. Tachy-zoites are seen in:

(A) Toxoplasma

(B) Toxocara

(C) Pulm eosinophilia

(D) Ascaris

Answer: (A)

146.In plasmodium falciparum following are seen in blood except:

(A) Schizonts

(B) Mature trophozoite

(C) Mature gametocytes

(D) None of these

Answer: (A)

147. About interferon true is :

(A) It is a synthetic antiviral agent

(B) Inhibits viral replication in cells

(C) Is specific for a particular virus

(D) None of these

Answer: (B)

148. Dark ground microscopy is used for:

(A) TPI

(B) FTA – ABS

(C) Kahn’s test

(D) VDRL

Answer: (A)

149. Electron microscopy is used for following except :

(A) To differentiate T and blymphocytes

(B) IgG deposits in kidne

(C) TPI

(D) Flagella

Answer: (C)

150. Following is true of T. pallidum, except :

(A) Can be maintained in rabbit testis

(B) Motile by peritrichate flagella

(C) To visualize, dark ground microscopy is used

(D) TPI test is very useful

Answer: (B)

151. HBV all true, except:

(A) It is a DNA virus

(B) Spreads by blood transfusions

(C) HBs Ag marker of infection

(D) Least chance of chronicity

Answer: (D)

152. CSF in meningococcal meningitis shows:

(A) Gm ‘+’ Diplococci, in pus cells

(B) Gm ‘-’ Diplococci in pus cells

(C) Gm ‘-’’ bacilli

(D) Gram ‘+’ bacilli

Answer: (B)

153. Corynebacterium diphtheria can be grown within 6-8 hours on:

(A) Potassium tellurite media with iron

(B) MaConkey’s agar

(C) Dorset egg medium

(D)  Loeffler’s serum slope

Answer: (D)

154. Pathogenicity of staphylococci is because of :

(A) Lecithinase

(B) M-protein

(C) Coagulase

(D) Hyaluronidase

Answer: (C)

155. True about tuberculin test are all except:

(A) Recent conversion in adult is an indication for ATT

(B) INH prophylazis is started if the test is positive

(C) No risk in negative

(D) May be false negative in immunocompromised patients

Answer: (C)

156. Mycobacterium leprae can be grown on :

(A) Culture media

(B) Foot pad of mouse

(C) Liver of guinea pig

(D) Kidney of rabbit

Answer: (B)

157. Sterilization of culture media containing serum is by :

(A) Autoclaving

(B) Micropore filter

(C) Gamma radiation

(D) Centrifugation

Answer: (B)

158. Maternal viraemia most commonly spreading to fetus :

(A) CMV

(B) Rubella

(C) HIV

(D) Herpes

Answer: (B)

159. Next to HBV, virus implicated in hepatocellular Ca is:

(A) HCV

(B) Herpes

(C) HAV

(D) HEV

Answer: (A)

160. Lepra bacilli can be grown in:

(A) Armadillo foot pad

(B) L-J medium

(C) SDA Agar

(D) Rabbit’s kidney

Answer: (A)

161. The difference between gram +ve and gram –ve organism is the G –ve org. contains:

(A) Teichoic acid

(B) Muramic acid

(C) N-acetyl neuraminic acid

(D) Aromatic amino acids

Answer: (A)

162. In TB, immunity is provided by:

(A) CD 4+

(B) CD 8+

(C) IgG

(D)  IgM

Answer: (A)

163. EBV does not cause:

(A) Nasopharyngeal Ca

(B) Inverted papilloma

(C) Burkitt’s lymphoma

(D) Infectious mononucleosis

Answer: (B)

164. Negative staining demonstrates, while +ve staining doesn’t demonstrate certain organism because of:

(A) LPS Capsule

(B) Polysaccharide capsule

(C) Carbohydrate capsule

(D) Fat capsule

Answer: (B)

165. Kerion is caused by:

(A) Candida

(B) Streptococcus

(C) Dermatophytes

(D) Herpes

Answer: (C)

166. Hormones are best assessed by:

(A) Flurocytometry

(B) Electrophoresis

(C) ELISA

(D) RIA

Answer: (D)

167. True about cholera vibrios is :

(A) Can tolerate wide range of alkaline pH

(B) Non motile bacilli

(C) Cann’t be grown in media

(D) NaCl stimulates growth

Answer: (A)

168. Staphyloccocal pathogenicity is indicated by:

(A) Coagulase positivity

(B) Hemolysis

(C) Lipoteichoic acid

(D) Endotoxin

Answer: (A)

169. Following transmit drug resistance except :

(A) RTF

(B) Plasmids

(C) Hfr

(D) Chromosome

Answer: (C)

170. Crytococcus has predilection for:

(A) Lungs

(B) Meninges

(C) Liver

(D)  GIT

Answer: (B)

171. Neurocysticercosis, following are true except:

(A) Not acquired by eating contamined vegetables

(B) Caused by regurgitation of larva

(C) Acquired by orofaecal route

(D) Acquired by eating pork

Answer: (A)

172. Lipid envelope is found in which virus?

(A) Reo

(B) Herpes

(C) Picorna

(D) All of the above

Answer: (B)

173. Rheumatoid factor is :

(A) IgM Ab against Fc fragment of IgG

(B) IgG Ab against Fc fragment of IgG

(C) IgM Ab against Fc fragment of IgM

(D) IgM Ab against Fc fragment of IgM

Answer: (A)

174. Best fixative for cytological studies :

(A) 95% ethanol

(B)  Rectified spirit

(C) Methanol

(D) Ethylene oxide

Answer: (A)

175. Which is not a heterophile agglutination test?

(A) Well-Felix test

(B) Widal test

(C) Paul-Bunnel test

(D) Streptoccus MG

Answer: (B)

176. Swarming growth is seen with :

(A) Clostridium welchii

(B) Proteus

(C) T. pallidum

(D) B. coli

Answer: (B)

177. The most sensitive test for H. pylori is :

(A) Breath test

(B) Rapid urease test

(C) Culture of biopsy

(D) Microscopy of biopsy

Answer: (B)

178. E. coli gives pink colour with :

(A) Chocolate agar

(B) L J medium

(C) MacConkey’s medium

(D) Saline broth

Answer: (C)

179. Neill-Mooser reaction is used to diagnose :

(A) Rickettsiae

(B) Chlamydiae

(C) Mycoplasma

(D) Herpes

Answer: (A)

180. Creutzfeld jakob disease is caused by :

(A) Prion

(B) JC virus

(C) Genetic factors

(D) Nutritional deficiency

Answer: (A)

181. Following grows in the cell free medium except :

(A) Rickettsia

(B) M. leprae

(C) Bartonella

(D) Syphilis

Answer: (A)

182. Sporicidal disinfectant is following except :

(A) Glutaralydehyde

(B) Formaldehyde

(C) Ethylene oxide

(D) Benzalkonium chloride

Answer: (D)

183. The best diagnostic test for recent hepatitis B is :

(A) HBsAg

(B) IgM anti HBc Ag

(C) Anti HBe

(D) Anti HBs

Answer: (B)

184. The HPV type leading to malignancy is

(A) 16, 18, 31

(B) 6, 11

(C) 16, 20, 32

(D) 6, 16, 22

Answer: (A)

185. BCYE medium is used to culture :

(A) Mycoplasma

(B) T. pallidium

(C) H. pylori

(D) Legionella

Answer: (D)

186. Eucalyptus camaldulensis is associated with the transmission of :

(A) Blastomyces dematitidis

(B) Histoplasma

(C) Cryptococcus

(D) Coccidiodes immitis

Answer: (C)

187. Diene’s method is used for :

(A) Mycoplasma

(B) Chlamydiae

(C) Plague

(D) Liphtheria

Answer: (A)

188. Cysticercosis is caused by larva of :

(A) Taenia solium

(B) T. saginata

(C) Echinococcus

(D) Ascaris

Answer: (A)

189. Fungi has possess a capsule is :

(A) Candida

(B) Aspergillus

(C) Cryptococcus

(D) Mucor

Answer: (C)

190. Cell wall deficient organisms are :

(A) Chlamydia

(B) Mycoplasma

(C) Streptococcus

(D) Anaerobes

Answer: (B)

191. Hepatitis C virus is:

(A) Hepadna virus

(B) Picorna virus

(C) Retro virus

(D) Flavivirus

Answer: (D)

192. All are sporonticidal except :

(A) Lysol

(B) Glutaraldehyde

(C) Ethylene dioxide

(D) Formaldehyde

Answer: (A)

193. Stage of falciparum not seen in PBS is:

(A) Schizont

(B) Gametocyte

(C) Ring form

(D) Double ring

Answer: (A)

194. Streptococcal GN is best diagnosed by :

(A) ASO titre

(B) Anti DNAse

(C) Anti hyaluronidase

(D) Culture

Answer: (B)

195. Test used in diagnosis of rheumatoid arthritis is :

(A) VDRL

(B) Rose waaler test

(C) Widal

(D) Passive hemagglutination

Answer: (B)

196. Viable but not cultivable state is seen in which of the following organism :

(A) M. Leprae

(B) M. Tuberculosis

(C) V. Cholerae

(D) Staph. aureus

Answer: (A)

197. Type-I hypersensitivity includes A/E :

(A) Auto-immune hemolytic anemia

(B) Anaphylaxis

(C) Ext. Asthma

(D) Hay Fever

Answer: (A)

198. Infection with colitis is caused by :

(A) Enterobius vermicularis

(B) Trichuris trichura

(C) Strongyloides

(D) Clonorchis

Answer: (C)

199. MC. cause of liver abscess :

(A) Streptococcus

(B) Staph aureus

(C) E. Coli

(D) Staph pyogenes

Answer: (C)

200. In Japanese Encephalitis pigs acts as :

(A) Amplifier

(B) Definitive host

(C) Intermediate host

(D) Any of the above

Answer: (A)

201. IL-1 is produced by :

(A) Macrophage

(B) Helper T lymphocytes

(C) B cells

(D) Cytotoxic T-cells

Answer: (A)

202. In India most common cause of T. B. in HIV :

(A) Myco. Tuberculosis

(B) Myco. Avium intracellulare

(C) M. bovis

(D) M. scrofulaceum

Answer: (A)

203. Cryptococcal meningitis is common is :

(A) Renal transplant recipient

(B) Agammaglobulinemia

(C) Neutropenia

(D) IgA deficiency

Answer: (A)

204. Eosinophilic meningocencephalitis is caused by :

(A) Gnathostoma spiralis

(B) Naegleria

(C) Toxocara canis

(D) Angiostrongylus cantonensis

Answer: (D)

205. Segmented virus :

(A) Influenza

(B) Polio

(C) HTLV-III

(D) Adeno virus

Answer: (A)

206. Maximum urease +ve is produced by :

(A) H. Pylori

(B) P. Mirabilis

(C) K. rhinomatis

(D) Ureaplasma

Answer: (B)

207. After Splenectomy, most common infection :

(A) Pneumococcal

(B) E. coli

(C) Klebsiella

(D) Streptococcus

Answer: (A)

208. Naeglers reaction :

(A) Cl. Tetani

(B) Cl. Botulinum

(C) Cl. Perfringenes

(D) Cl. Septicum

Answer: (C)

209. True about Mycosis fungoides :

(A) Cutaneous – T. cell lymphoma

(B) Cutaneous – B cell lymphoma

(C) A type of ALL

(D) A type of Hodgkin’s lymphoma

Answer: (B)

210. True of malaria diagnosis :

(A) Thick smear to identify parasite

(B) ABER reveals positivity by 100

(C) All have same Incubation

(D) Fluorescein Ab within 1 week

Answer: (A)

211. PML is caused by

(A) CMV

(B) Papova virus

(C) HIV

(D) Polio virus

Answer: (B)

212. Max. Hepatitis C virus transmission to foetus in pregnancy depends on :

(A) Duration of illness

(B) Time of infection

(C) Route to delivery

(D) High level of HCV RNA

Answer: (D)

213. True about H-pylori :

(A) Seen in 85 to 90% cases of gastric ulcer

(B) Seen in 20 to 25% cases of duodenal ulcer

(C) Transmitted from man to man, feco-orally & by orogastric route

(D) Common in adults of developing countries

Answer: (C)

214. Not easily culturable but well viable & used in epidemiology are/is:

(A) Staph

(B) Mycobacterium TB

(C) E. Coli

(D) Salmonella

Answer: (B)

215. True about HIV :

(A) Not transmitted through semen

(B) More chances of transmission during LSCS than normal labour

(C) More infectious than hepatitis B

(D) Male to female transmission > female to male

Answer: (D)

216. Treatment of partner is required in all infections except :

(A) Candida

(B) Herpes

(C) Trichomonas

(D) Gardnerella

Answer: (D)

217. Lyme disease caused by :

(A) Leptospira

(B) Borrelia

(C) Treponema

(D) Bordetella

Answer: (B)

218. Which of the following are AFB positive with 5% sulphuric acid?

(A) M. avium

(B) M. leprae

(C) M. tuberculosis

(D) Nocardia

Answer: (B)

219. True about Salmonella typhi infection in intestine is/are :

(A) Affects peyer’s patches

(B) Common in mesenteric border

(C) Erythrophagocytosis is characteristic

(D) Strictures are common

Answer: (A)

220. Which of the following crosses placenta?

(A) IgG

(B) IgA

(C) IgM

(D) IgD

Answer: (A)

221. Exacerbation of lesions in patients of borderline leprosy is seen in :

(A) ENL (erythema nodosum leprosum)

(B) Lepra reaction type 1

(C) Jarisch-Herxheimer reaction

(D) Resolving leprosy

Answer: (A)

222. True about mantoux is :

(A) False negative in fulminant diseases

(B) If once done, next time it is always positive

(C) Results are given in terms of positive & negative

(D) Indurations given in terms of length & breadth

Answer: (A)

223. Which of the following organisms is implicated in the causation of botryomycosis?

(A) Staphylococcus aureus

(B) Staphylococcus albus

(C) Pseudomonas aureginosa

(D) Streptococcus pneumonia

Answer: (A)

224. Staphylococcus in stool occurs in :

(A) Staphylococcal food poisoning

(B) Ischiorectal abscess

(C) Toxic shock syndrome

(D) May be a normal finding

Answer: (A)

225. The causative organism can be isolated in which among the following conditions :

(A) Serum in toxic shock syndrome

(B) Meningococcal rash

(C) Rheumatic valvulitis

(D) CSF in tetanus

Answer: (B)

226. Zoonotic diseases are all except :

(A) Typhoid

(B) Anthrax

(C) Rabies

(D) Brucella

Answer: (A)

227. The secretory component of immunoglobulin molecule is :

(A) Formed by epithelial cell of lining mucosa

(B) Formed by plasma cell

(C) Formed by epithelial cell and plasma cell

(D) Secreted by bone marrow

Answer: (A)

228. Perforins are produced by :

(A) Cytotoxic T cells

(B) Suppressor T cells

(C) Memory helper T cells

(D) Plasma cells

Answer: (A)

229. True regarding mycobacterium tuberculosis is :

(A) Produces visible colonies in 1 weeks time on Lowenstein-Jenson media

(B) Decolorized by 20% sulfuric acid

(C) Facultative aerobe

(D) Niacin positive

Answer: (D)

230. Cryptococcus can be readily demonstrated by :

(A) Albert’stain

(B) Indian’s ink stain

(C) Giema’s stain

(D) Gram’s stain

Answer: (B)

231. Apart from B cells, and T cells, there is a 3rd distinct type of lymphocyte. This is:

(A) MHC cell

(B) NK cell

(C) Macrophage

(D) Neutrophil

Answer: (B)

232. Nocardia is differentiated from Actinomyces by:

(A) Gram stain

(B) ZN stain

(C) Nacardia causes mycetoma, Actinomyces does not

(D) Nacardia is facultative anerobe

Answer: (B)

233. IgE is secreted by :

(A) Mast cell

(B) Basophils

(C) Eosinophils

(D) Plasma cells

Answer: (D)

234. Micro-organism used as weapon in biological terrorism :

(A) Small pox V

(B) Rabies V

(C) Ebda V

(D) Influenza CV

Answer: (A)

Medical PG Medicine

Medical PG Medicine

1. Increased ‘anion gap’ is not seen in

(A)  Salicylate poisoning

(B)    Renal tubular acidosis

(C)  Lactic acidosis

(D)  Ethylene glycol poisoning

Answer: (B)

2.All the following are features of Anorexia Nervosa except

(A)  Predominantly seen in females

(B)  There is Loss of weight at the time of presentation

(C)  Amenorrhea is rare

(D)  Avoidance of high calorie foods

Answer: (C)

3. All the following are used in the treatment of Pneumocystis carinii except

(A)  Pentamidine

(B)  Dapsone

(C)  Cotrimoxazole

(D)  Fluoroqinolones

Answer: (D)

4. The drug of choice in Herpes Simplex Encephalitis is

(A)  Acyclovir

(B)  Zidovudine

(C)  Amantadine

(D)  Vidarabine

Answer: (A)

5. All the following are features of Tropical Pulmonary Eosinophilia except

(A)  Eosinophilia > 3000/mmᶟ

(B)  Microfilaria in blood

(C)  Paroxysmal cough and wheezing

(D)  Bilateral chest mottling and increased broncho-vascular markings

Answer: (B)

6. osteoporosis is seen in all the following except

(A)  Thyrotoxicosis

(B)  Rheumatoid arthritis

(C)  Hypoparathyroidism

(D)  Steroid therapy

Answer: (C)

7. Osteoporosis is seen in

(A)  Thyrotoxicosis

(B)  Cushing’s disease

(C)  Menopause

(D)  All of the above

Answer: (D)

8. The most common manifestation of osteoporosis is

(A)  Compression fracture of the spine

(B)  Asymptomatic, detected incidentally by low serum Calcium

(C)  Bowing of legs

(D)  Bowing of legs

Answer: (A)

9. Which part of the spine is most commonly affected in Rheumatoid arthritis

(A)  Cervical

(B)  Lumbar

(C)  Thoracic

(D)  Sacral

Answer: (A)

10. ll the following are true about Rheumatoid arthritis except

(A)  Positive for Anti-IgG antibody

(B)  Juxta-articular osteoporosis

(C)  Morning stiffness

(D)  C Reactive protein indicates better progonosis

Answer: (D)

11. Which of the following is true regarding Rheumatoid arthritis

(A)  Typically involves small and large joints symmetrically but spares cervical spine

(B)  Causes pleural effusion with low sugar

(C)  Pulmonary nodules are absent

(D)  Enthesopathy prominent

Answer: (B)

12. Gold slats can be used in

(A)  Ankylosing spondylitis

(B)  Rheumatoid arthritis

(C)  Osteoarthritis

(D)  Behcet’s syndrome

Answer: (B)

13. All the following are features of Behcet’s syndrome except

(A)  Recurrent aphthous stomatitis

(B)  Multi-system involvement

(C)  Seen only in the tropics

(D)  Common in youngsters

Answer: (C)

14. All the following are features of Marfan’s syndrome except

(A)  Arachnodactyly

(B)  Joint hypermobility

(C)  High palate

(D)  Flat foot-pes planus

Answer: (B)

15. In Thyrotoxicosis, β-blockers do not control

(A)  Anxiety

(B)  Tremors

(C)  Tachycardia

(D)  Oxygen consumption

Answer: (D)

16. SIADH secretion is seen in all except

(A)  Lung abscess

(B)  Interstitial Nephritis

(C)  Vinka alkaloids

(D)  Bronchial adenoma

Answer: (B)

17. Henoch Schonlein purpura is characterized by all except

(A)  Thrombocytopenia

(B)  Glomerulonephritis

(C)  Arthralgia

(D)  Abdominal pain

Answer: (A)

18. All the following are true about multiple myeloma except

(A)  Osteolytic bone disease

(B)  t(8-14) translocation

(C)  Light chain proliferation

(D)  Bence-Jones proteins in urine

Answer: (B)

19. In the JVP, ‘a’ waves are absent in

(A)  Atrial fibrillation

(B)  Mitral stenosis

(C)  Tricuspid atresia

(D)  Sick sinus syndrome

Answer: (A)

20. Angina pectoris and Syncope are most likely to be associated with

(A)  Mitral stenosis

(B)  Aortic stenosis

(C)  Mitral regurgitation

(D)  Tricuspid stenosis

Answer: (B)

21. Acute Aortic regurgitation is seen in all the following except

(A)  Marfan’s syndrome

(B)  Acute myocardial infarction

(C)  Bacterial endocarditis

(D)  Ankylosing spondylitis

Answer: (B)

22. Decreased maximum mid-expiratory flow rate indicates obstruction in

(A)  Small airways

(B)  Trachea

(C)  Large airways

(D)  Trachea & Bronchi both

Answer: (A)

23. All the following are true about Chronic Obstructive lung disease except

(A)  Decreased FeV1

(B)  Decreased MEFR

(C)  Increased RV

(D)  Decreased diffusion capacity

Answer: (D)

24. Normal portal venous pressure is

(A)  3-5 cm saline

(B)  5-10 cm saline

(C)  10-15 cm saline

(D)  15-20 cm saline

Answer: (C)

25. Hepatitis B infectivity is indicated by

(A)  Anti-HBsAg

(B)  HBsAg + HBeAg

(C)  Anti-HBsAg + Anti-HBc

(D)  Anti-HBeAg + Anti-HbsAg

Answer: (B)

26. All the following are causes of Acute Pancreatitis except

(A)  Gall stones

(B)  Alcohol

(C)  Hemochromatosis

(D)  Hypercalcemia

Answer: (C)

27. The most common site of Berry aneurysm is

(A)  Junction of anterior communicating artery with anterior cerebral artery

(B)  Junction of posterior communicating artery with internal carotid artery

(C)  Bifurcation of middle cerebral artery

(D)  Vertebral artery

Answer: (A)

28. A common site for intracranial hemorrhage  is

(A)  Putamen

(B)  Cerebellum

(C)  Mid-brain

(D)  Pons

Answer: (A)

29. Hemiplegia is commonly associated with infarction of the area of distribution of the

(A)  Anterior cerebral artery

(B)  Middle cerebral artery

(C)  Posterior cerebral artery

(D)  Anterior communicating artery

Answer: (B)

30. All the following are features Pseudotumor cerebri except

(A)  Increased intracranial tension

(B)  Convulsions

(C)  Papilledema

(D)  Normal CT scan

Answer: (B)

31. The characteristic feature of a frontal lobe tumor is

(A)  Abnormal gait

(B)  Aphasia

(C)  Distractibility

(D)  Antisocial behavior

Answer: (D)

32. Neuronophagia is seen in

(A)  Amoebic encephalitis

(B)  Poliomyelitis

(C)  Tuberculous meningoencephalitis

(D)  Cerebral malaria

Answer: (B)

33. All the following are true of Craniopharyngioma except

(A)  Derived from Rathke’s pouch

(B)  Contains epithelial cells

(C)  Present in sella or infra-sellar location

(D)  Causes visual disturbances

Answer: (C)

34. Thymoma is associated with all except

(A)  Pure Red-cell aplasia

(B)  Aplastic anemia

(C)  Hypergammaglobulinemia

(D)  Myasthenia gravis

Answer: (C)

35. Thymoma is associated with

(A)  Myasthenia gravis

(B)  Scleroderma

(C)  Oesophageal atrasia

(D)  Hyper-gammaglobulinemia

Answer: (A)

36. All the following are examples of diseases causing Aplastic anemia except

(A)  PNH

(B)  Hepatitis

(C)  Pregnancy

(D)  Cold hemoglobinuria

Answer: (D)

37. All the following are seen in Multiple myeloma except

(A)  Visual Disturbance

(B)  Bleeding tendency

(C)  Proteinuria

(D)  Dystrophic calcification

Answer: (D)

38. All of the following cause Microcytic Hypochromic anemia except

(A)  Lead poisoning

(B)  Thalassemia

(C)  Iron deficiency anemial

(D)  Fanconi’s anemia

Answer: (D)

39. Treatment of choice in hairy cell leukemia is

(A)  Steroids except

(B)  Cladribine

(C)  Splenectomy

(D)  Pentostatin

Answer: (B)

40. Electric alternans is seen in

(A)  Cardiac tamponade

(B)  Restrictive candiomyopathy

(C)  Constructive pericarditis

(D)  Right Ventricular MI(RVMI)

Answer: (A)

41. Continuous murmur is seen in all the following except

(A)  Aortic sinus of valsalva rupture normal

(B)  Coarctation of Aorta

(C)  AV malformations

(D)  Peripheral pulmonary stenosis

Answer: (D)

42. Agent of first choice in an cute attack of Prinzmetal’s angina is

(A)  Diltiazem

(B)  Nitrates

(C)  Propranolol

(D)  Verapamil

Answer: (B)

43. Following are used in treatment of digitalis toxity except

(A)  Potassium

(B)  Lignocaine

(C)  Fab particles

(D)  Hemodialysis

Answer: (D)

44. Aspirin-sensitive asthma is associated with

(A)  Obesity

(B)  Urticaria

(C)  Nasal polyp

(D)  Extrinsic asthma

Answer: (C)

45. All the following are true about bronchopulmonary aspergillosis except

(A)  Central bronchiectasis

(B)  Pleural effusion

(C)  Asthma

(D)  Eosinophilia

Answer: (B)

46. Addison’s disease is characterized by all except

(A)  Hyperglycemia

(B)  Hypotension

(C)  Hyperkalemia

(D)  Hyponatremia

Answer: (A)

47. The most common cause of Cushing’s syndrome is

(A)  Pituitary adenoma

(B)  Adrenal adenoma

(C)  Ectopic ACTH

(D)  Iatrogenic steroids

Answer: (D)

48. Excess Aldosterone is associated with all the following except

(A)  Hypokalemia

(B)  Hyperkalemia

(C)  Sodium retention

(D)  Hypertension

Answer: (B)

49. A common cause of Primary hyperparathyroidism is

(A)  Multiple parathyroid adenomas

(B)  Solitary parathyroid adenoma

(C)  Adrenal hyperplasia

(D)  Ectopic PTH production

Answer: (B)

50. Tufting of the terminal phalanges is seen in

(A)  Hypoparathyroidism

(B)  Hyperparathyroidism

(C)  Hyperthyroidism

(D)  Hypothyroidism

Answer: (B)

51. The diagnosis of a patient presenting with familial Polyostosis, Precocious puberty and Pigmentation is

(A)  Tuberous sclerosis

(B)  McAlbright syndrome

(C)  Klinefelter syndrome

(D)  SLE

Answer: (B)

52. The diagnosis of a patient presenting with Seizures. Mental retardation and Sebaceous adenoma is

(A)  Hypothyroidism

(B)  Tuberous sclerosis

(C)  Toxoplasmosis

(D)  Down syndrome

Answer: (B)

53. The following are rheumatoid disease modifying drugs except

(A)  Chloroquine

(B)  Gold

(C)  Penicillamine

(D)  BAL

Answer: (D)

54. All the following are features of Scleroderma except

(A)  Dysphagia

(B)  Raynaud’s phenomenon

(C)  Skin contracture

(D)  Calcification in all the long bones

Answer: (D)

55. Which is not true of hypocalcaemia

(A)  Can occur in hypoparathyroidism

(B)  Latent tetany is seen

(C)  Prolonged QT interval

(D)  Inverse relation with Mg⁺⁺ levels

Answer: (D)

56. Hypercalcemia is not seen in

(A)  Lithium therapy

(B)  Chronic renal failure

(C)  Multiple myeloma

(D)  Vitamin deficiency

Answer: (D)

57. QT interval is shortened in

(A)  Hypocalcaemia

(B)  Hypokalemia

(C)  Hypercalcemia

(D)  Hyperkalemia

Answer: (C)

58. The CSF findings in TB meningitis are all the following except

(A)  Raised protein

(B)  Low sugar

(C)  Low chloride

(D)  High RBC count

Answer: (D)

59. Tunor associated with Polycythemia Vera is

(A)  Sarcoma

(B)  Pituitary adenoma

(C)  Cerebellar Hemangioblastoma

(D)  None of the above

Answer: (C)

60. True about CSF finding in pyogenic  meningitis is

(A)  CSF contains no organisms

(B)  ↓ Chloride, ↓Glucose

(C)  ↑ Glucose, ↑ protein

(D)  ↑ Chloride, ↓glucose

Answer: (B)

61. The drug of choice in Cryptococcal Meningitis is

(A)  Pentostatin

(B)  Amphotericin B

(C)  Clotrimazole

(D)  Zidovudine

Answer: (B)

62. The commonest cause of Intracerebral Bleed is

(A)  Thrmobocytopenia

(B)  Diabetes

(C)  Hypertension

(D)  Berry aneurysm

Answer: (C)

63. Fasciculations are seen in

(A)  Motor neuron disease

(B)  Duchenne’s muscular dystrophy

(C)  Polymyositis

(D)  None of the above

Answer: (A)

64. The differential diagnosis of Botulism includes all the following except

(A)  Myasthenia Gravis

(B)  Guillain Barre’s syndrome

(C)  Poliomyelitis

(D)  Clostridial myonecrosis

Answer: (D)

65. Vitamin B6 deficiency is associated with all of the following except

(A)  Penicillamine

(B)  INH

(C)  Cyclosporin

(D)  Cycloserine

Answer: (C)

66. Which drug is not effective against H. pylori

(A)  Colloidal Bismuth

(B)  Metronidazole

(C)  Amoxicillin

(D)  Erythromycin

Answer: (D)

67. Intestinal hypomotility is seen in the following except

(A)  Diabetes

(B)  Parkinsonism

(C)  Amyloidosis

(D)  All of the above

Answer: (D)

68. The most common route of spread in hepatitis E is

(A)  Blood transfusion

(B)  Feco-oral

(C)  Intercourse

(D)  IV injection

Answer: (B)

69. The most common site for amebiasis is

(A)  Sigmoid colon

(B)  Transverse colon

(C)  Caecum

(D)  Hepatic flexure

Answer: (C)

70. Cause of acute loss of vision in a patient of alcoholic pancreatitis is

(A)  Purtscher’s retinopathy

(B)  Sudden alcohol withdrawal

(C)  Acute congestive glaucoma

(D)  CRAO

Answer: (A)

71. Widened anionic gap is not seen in

(A)  Acute renal failure

(B)  Diarrhea

(C)  Lactic acidosis

(D)  Diabetic ketoacidosis

Answer: (B)

72. Lactic acidosis is not seen in

(A)  Methanol poisoning

(B)  Respiratory failure

(C)  Circulatory failure

(D)  Tolbutamide

Answer: (D)

73. Metabolic alkalosis is seen in all expect

(A)  Thiazide diuretic therapy

(B)  Prolonged vomiting

(C)  Uretero-sigmoidostomy

(D)  Cushing’s disease

Answer: (C)

74. Thromobocytopenia is not seen in

(A)  H.S. purpura

(B)  DIC

(C)  Leukemia

(D)  Metastasis

Answer: (A)

75. Platelet transfusion is not indicated in

(A)  Dilutional Thrombocytopenia

(B)  Immunogenic Thrombocytopenia

(C)  Aplastic Anemia

(D)  DIC

Answer: (B)

76. Bacterial endocarditis is rarely seen in

(A)  VSD

(B)  PDA

(C)  MVP

(D)  Ostium secundum ASD

Answer: (D)

77. Which the following is not seen in a chronic case of Sickle cell anemia

(A)  Hepatomegaly

(B)  Pulmonary hypertension

(C)  Cardiomegaly

(D)  Splenomegaly

Answer: (D)

78. All are seen is Hemolytic anemia except

(A)  Hemosiderinuria

(B)  Reticulocytosis

(C)  Spherocytosis

(D)  Increased haptoglobin

Answer: (D)

79. Giant ‘a’ waves in JVP occur in all expect

(A)  Junctional rhythm

(B)  Pulmonary hypertension

(C)  Tricuspid regurgitation

(D)  Complete heart block

Answer: (C)

80. Differential cyanosis is seen in:

(A)  ASD

(B)  VSD

(C)  PDA

(D)  All of the above

Answer: (C)

81. All the following are radiological features of Chronic Cor pulmonale expect

(A)  Kerley B lines

(B)  Prominent lower lobe vessels

(C)  Pleural effusion

(D)  Cardiomegaly

Answer: (B)

82. All of the following are true about Temporal Arteritis expect

(A)  Polymyalgia Rheumatica

(B)  Anemia

(C)  Low ESR

(D)  Sudden blindness

Answer: (C)

83. Kawasaki disease is associated with all of the following features expect

(A)  Erythema

(B)  Posterior cervical Lymphadenopathy

(C)  Thrombocytopenia

(D)  Conjunctivitis

Answer: (C)

84. Which of the following is not seen in ARDS

(A)  Pulmonary edema

(B)  Hpoxemia

(C)  Stiff lung

(D)  Hypercapnia

Answer: (D)

85. The commonest cause of non-gonococcal urethritis is

(A)  Chalmydia

(B)  Ureaplasma urealyticum

(C)  E. coli

(D)  Proteus

Answer: (A)

86. Which of the following statements is true of Rabies

(A)  Fluorescent antibody is the test of choice

(B)  HDCV is a live attenuated vaccine

(C)  Vaccine is given deep IM in the buttock

(D)  Local wound care is of no use

Answer: (A)

87. False-negative tuberculin test is seen in all except

(A)  After 4-6 weeks of measles attack

(B)  Immunodeficiency state

(C)  Miliary tuberculosis

(D)  Atypical mycobacterial infection

Answer: (D)

88. All the following may be used in the prophylaxis of Meningococcal infection except

(A)  Penicillin

(B)  Sulphonamides

(C)  Erythromycin

(D)  Rifampicin

Answer: (C)

89. All the following are features of Wilson’s disease except

(A)  Increased copper content in liver

(B)  Mental changes

(C)  Features of chronic active hepatitis

(D)  None of these

Answer: (B)

90. Which drug is essential in Sheehan’s syndrome

(A)  Estrogen

(B)  Cortisone

(C)  Thyroxin

(D)  Growth hormone

Answer: (B)

91. All the following are features of Guillian-Barre syndrome except

(A)  Predominant motor involvement

(B)  Residual disability

(C)  Absence of pleocytosis in CSF

(D)  Sparing of bladder function

Answer: (B)

92. Which drug is not used in SIADH

(A)  Fludrocortisone

(B)  Demeclocycline

(C)  Desmopressin

(D)  Hypertonic saline

Answer: (C)

93. All the following are features of cerebellar disease except

(A)  Intention tremors

(B)  Past pointing

(C)  Hypertonia

(D)  Ataxia

Answer: (C)

94. Which of the following is not seen in Parkinsonism

(A)  Preserved postural reflexes

(B)  Hypokinesia

(C)  Rigidity

(D)  Static tremors

Answer: (A)

95. Drug of choice in phenothhiazine-induced Dystonia is

(A)  Diphenhydramine

(B)  Metoclopramide

(C)  Trifluperamide

(D)  Benzotropine

Answer: (D)

96. Osteoporosis is caused by all except

(A)  Sarcoidosis

(B)  Old age

(C)  Hypo-parathyroidism

(D)  Steroid therapy

Answer: (C)

97. All the following are features of Pseudotumor Cerebri except

(A)  Normal-sized ventricles on CT scan

(B)  Increased protein in CSF

(C)  Papilledema

(D)  Absence of focal neurological deficit

Answer: (B)

98. Normal pressure Hydrocephalus is characterized by all except

(A)  Aphasia

(B)  Dementia

(C)  Ataxia

(D)  Urinary incontinence

Answer: (A)

99. Granulomatous hepatitis may be seen with

(A)  Carbamazepine

(B)  Allopurinol

(C)  Phenylbutazone

(D)  All of the above

Answer: (D)

100. Pancreatic cholera is characterized by all except

(A)  Hypochlorhydria

(B)  Hypokalemia

(C)  Hypokalemia

(D)  Hypocalcemia

Answer: (D)

101. All of the following statements are true about Glucose 6 phosphatase deficiency disease

(A)  Storage occurs in liver and kidney

(B)  Hyperuricemia

(C)  Gluconeogenesis is increased

(D)  None of the above

Answer: (C)

102. All of the following statements are true about Hemolytic uramic syndrome except :

(A)  Uraemia

(B)  Hypofibhrinogenemia

(C)  Thrombocytopenia

(D)  Positive coomb’s test

Answer: (D)

103. Warm autoimmune hemolytic anemia may be seen in all of the following conditions except :

(A)  Systemic lupus erythematous

(B)  α-Methyl dopa therapy

(C)  Non hodgkin’s lymphoma

(D)  Mycoplasma pneumonia

Answer: (D)

104. Megaloblastic anemia may be aused by all of the following except :

(A)  Phenytoin

(B)  Methotrexate

(C)  Pyrimethamine

(D)  Amoxycilline

Answer: (D)

105. All of the following statements about digitalis are true except :

(A)  Excretion is mainly renal

(B)  Oral absorption is good

(C)  Actively metabolized in liver

(D)  Lipid soluble

Answer: (C)

106. All of the following may precipitate digitalis toxicity except :

(A)  Hypocalcemia

(B)  Hypokalemia

(C)  Hypomagnecemia

(D)  Hypothyroidism

Answer: (A)

107. Treatment of digoxin over dose includes administration of all of the following except :

(A)  Potassium

(B)  Lignocaine

(C)  Phenyton

(D)  Hemodialysis

Answer: (D)

108. Co-arctation of aorta may be associated with all of the following except :

(A)  Bicuspid aortic value

(B)  Tuner syndrome

(C)  Renal artery stenosis

(D)  PDA

Answer: (C)

109. Mitral stenosis is associated with

(A)  Right ventricular hyper trophy

(B)  Left ventricular hypertrophy

(C)  Left axis deviation

(D)  QRS complex

Answer: (A)

110. Most common type of hepatitis responsible for epidemics in India is :

(A)  Hepatitis A

(B)  Hepatitis B

(C)  Hepatitis C

(D)  Hepatitis E

Answer: (D)

111. Commonest cause of Budd Chiary syndrome is

(A)  Commonest cause of Budd Chiary syndrome is

(B)  Hepatocellular carcinoma

(C)  Paroxysmal nocturnal hemoglobinuria

(D)  Renal cell carcinoma

Answer: (C)

112. Adult polycystic kidney is inherited as :

(A)  Autosomal-co-dominant

(B)  Autosomal dominant

(C)  Autosomal recessive

(D)  X-linked dominant

Answer: (C)

113. Most common cause of amoebic lung abscess is

(A)  Aspiration

(B)  Direct spread from liver

(C)  Hematogenous spread from liver

(D)  Hematogenous spread from gut

Answer: (B)

114. All of the following may be associated with thymoma except :

(A)  Myasthenia-gravis

(B)  Pure red cell aplasia

(C)  Superior mediastinal compression syndrome

(D)  Hypergammaglobulinemia

Answer: (D)

115. Indication of systemic steroid in rheumatoid arthritis is :

(A)  Mononeuritis multiplex

(B)  Carpul tunnel syndrome

(C)  Presence of deformities

(D)  Articular cartilage involvement

Answer: (A)

116. Least common site to be involved in osteoarthritis amongst the following is :

(A)  Hip joint

(B)  Knee joint

(C)  Carpometacarpal joint of thumb

(D)  Metacarpophalangeal joint

Answer: (D)

117. Low does  of aspirin therapy is essentially advised for all of the following conditions except :

(A)  SLE

(B)  Post myocardinal infarction

(C)  Pre ecclampsia

(D)  None

Answer: (A)

118. B/L hilar lymphadenopathy, along with non caseating granulomas is a characteristic features of :

(A)  Sarcoidosis

(B)  Scleroderma

(C)  SLE

(D)  Stein-leventhal syndrome

Answer: (A)

119. Increased amylase may be seen in all of the following except :

(A)  Pancreatic pseudocyst

(B)  Appendicitis

(C)  Perforated peptic ulcer

(D)  Ruptured ectopic pregnancy

Answer: (B)

120. Most common cause of mediastinitis is :

(A)  Tracheal rupture

(B)  Esophageal rupture

(C)  Drugs

(D)  Idiopathic

Answer: (B)

121. Features not seen in Cushing’s Syndrome is

(A)  Hypoglycemia

(B)  Hypertension

(C)  Frank psychosis

(D)  Hypokalemia

Answer: (A)

122. Most common cause of hyperparathyroidism is

(A)  Solitary adenoma

(B)  Chief cell hyperplasia

(C)  Multiple adenoma

(D)  Werner’s syndrome

Answer: (A)

123. Hypercalcemia is not a feature of one of the following conditions :

(A)  Primary hyperparathyroidism

(B)  Multiple myeloma

(C)  Tumourlysis syndrome

(D)  Sarcoidosis

Answer: (C)

124. All of the following are known to cause hyperprolactenemia except :

(A)  Methyldopa

(B)  Phenothiazines

(C)  Bromocriptine

(D)  Metoclopramide

Answer: (C)

125. Hyponatremia may be seen in all of the following conditions except :

(A)  Mucoviscidosis

(B)  SIADH

(C)  Diabetes insipidus

(D)  Adrenal hyperplasia

Answer: (C)

126. Parkinsonism is characterized by all of the following characteristic features except :

(A)  Rigidity

(B)  Akinesia

(C)  Tremors at rest

(D)  Past pointing

Answer: (D)

127. Clinical features of normal pressure hydrocephalus include all of the following except :

(A)  Dementia

(B)  Aphasia

(C)  Gait disturbance

(D)  Urinary incontinence

Answer: (B)

128. All of the following tumors are malignant except :

(A)  Glioma

(B)  Astrocytoma

(C)  Hemangioblastoma

(D)  Ependymoma

Answer: (C)

129. Spinal segment in knee jerk include all of the following except

(A)  L₁

(B)  L₂

(C)  L3

(D)  L₄

Answer: (A)

130. Following hormonal levels are increased in small cell carcinoma of lung except :

(A)  ACTH

(B)  Growth hormone

(C)  ANF

(D)  AVP

Answer: (B)

131. Best test for Small intestine malabsorption of carbohydrates is :

(A)  Lund meal test

(B)  Shilling test

(C)  D-Xylose test

(D)  Follacin test

Answer: (C)

132. Obesity is associated with all of the following except :

(A)  Osteoarthritis

(B)  Hypertensions

(C)  Gall stones

(D)  Pancreatitis

Answer: (D)

133. An X-linked recessive disease is characterized by the following inheritance :

(A)  Vertical transmission

(B)  50% female carriers if male is affected and female is normal

(C)  50% male affected if female is carrier and male is normal

(D)  50% male carriers if female is affected and male is normal

Answer: (C)

134. Tuberculous pleural effusion is characterized by all of the following features except effusion :

(A)  Harmorrhagic effusion

(B)  Pleural fluid LDH more than 60% that of serum LDH

(C)  Protein in apirated fluid is increased

(D)  ↑Mesothelial cells

Answer: (D)

135. Kala-azar is not responding to primary treatment. The treatment should now consist of :

(A)  Double dose of antimony

(B)  Amphotericin-B

(C)  Ketoconazole

(D)  Splenectomy

Answer: (B)

136. Maximum ESR is seen in :

(A)  CHF

(B)  Polycythemia vera

(C)  Multiple meyloma

(D)  Sickle cell anemia

Answer: (C)

137. Macrocytic anemia may be seen with all of the following conditions except :

(A)  Liver disease

(B)  Copper deficiency

(C)  Thiamine deficiency

(D)  Orotic aciduria

Answer: (B)

138. Features of hereditary spherocytosis include all of the following except :

(A)  ↑ Osmotic fragility

(B)  ↑ MCHC

(C)  ↑ MCV

(D)  Decrease surface area per unit volume

Answer: (C)

139. Primaquine may cause hemolysis in :

(A)  G-6-PD deficiency

(B)  NADP-deficiency

(C)  Methemoglobin reductase deficiency

(D)  Crabb’s disease

Answer: (A)

140. Splenectomy is most useful in :

(A)  Thrombocytopenia

(B)  Hereditary spherocytosis

(C)  H.S. purpura

(D)  Sickle cell anemia

Answer: (B)

141. While handling a febrile neutropenic patient all are essential except :

(A)  Repeated hand washing of hospital person

(B)  White cell infusion

(C)  Prophylactic antibiotic

(D)  Colony stimulating factor for macrophages

Answer: (B)

142. Severity of mitral stenosis is assessed by :

(A)  Loud opening snap

(B)  Length of murmur

(C)  Loud S₁

(D)  Splitting S₂

Answer: (B)

143. Drug of choice in PSVT is :

(A)  Amaidarone

(B)  Lingnocaine

(C)  Quinidine

(D)  Adenosine

Answer: (D)

144. Nosocomial pneumonia is most commonly caused by:

(A)  Gram –ve bacilli

(B)  Gram +ve bacilli

(C)  Gram –ve cocci

(D)  Mycoplasma

Answer: (A)

145. Complications of bronchiectasis include all except :

(A)  Amyloidosis

(B)  Lung abscess

(C)  Cerebral abscess

(D)  Bronchiogenic carcinoma

Answer: (D)

146. Bronchial adenoma commonly present as :

(A)  Recurrent hemoptysis

(B)  Cough

(C)  Dysponea

(D)  Chest pain

Answer: (A)

147. Bronchial hyperplasia may be caused by all except :

(A)  Smoking

(B)  Theophylline

(C)  Prematurity

(D)  Allergy

Answer: (B)

148. Weight loss may be seen in all of the following except :

(A)  Uremia

(B)  Pheochromacytoma

(C)  Adrenal insufficiency

(D)  Insulinoma

Answer: (D)

149. Osteoporosis may be seen in all except :

(A)  Hyperparathridism

(B)  Hypoparathyridism

(C)  Thyrotoxicosis

(D)  Heparin administration

Answer: (B)

150. Osteoporosis may be seen in all except :

(A)  Hypoparathyroidism

(B)  Osteogenesis imperfecta

(C)  Hypogonadism

(D)  Hypophophatasia

Answer: (A)

151. All of the following are features of primary hyperaldosteronism except :

(A)  Pedal oedema

(B)  Polyurea

(C)  Hypokalemia

(D)  Hypertension

Answer: (A)

152. Commonest cause of cerebrovascular accident is :

(A)  Arterial thrombosis

(B)  Venous thrombosis

(C)  Embolism

(D)  Tumour

Answer: (C)

153. Commonest cause of subarachnoid haemorrhage is :

(A)  Rupture of circle of wills aneurysm

(B)  Rupture or vertebral artery aneurysm

(C)  Rupture of venecomitants of corpus striatum

(D)  Rupture of dural sinusis

Answer: (A)

154. Most common nerve involved in intracranial aneurysm is :

(A)  Trochlear

(B)  VII

(C)  VIII

(D)  Occulomotor

Answer: (D)

155. True about C.S.F. finding in pyogenic meningitis is :

(A)  Protein ↑, sugar ↓

(B)  Protein ↓, sugar ↑

(C)  Chloride↑, protein

(D)  ↓Protein, ↓sugar

Answer: (A)

156. All of the following may cause metaststic tumour causing spinal cord compressions except :

(A)  Lung carcinoma

(B)  Breast carcinoma

(C)  Lymphoma

(D)  Meningioma

Answer: (D)

157. Features of SLE include all of the following except :

(A)  Recurrent abortion

(B)  Sterility

(C)  Coomb’s positive hemolytic anemia

(D)  Psychosis

Answer: (B)

158. Lupus anticoagulants may cause all of the following except :

(A)  Recurrent abortion

(B)  False +ve VDRL results

(C)  Increase prothrombin time

(D)  Arterial thrombosis

Answer: (C)

159. Indications of steroids in SLE are all except :

(A)  Myocarditis

(B)  Endocarditis

(C)  Thrombocytopenia

(D)  Neuropsychiatric symptoms

Answer: (B)

160. Bilateral parotid gland enlargement is seen in all of the following except :

(A)  Sarcoidosis

(B)  SLE

(C)  Chronic pancreatitis

(D)  Sjogern’s syndrome

Answer: (B)

161. All are seen in osteogenesis imperfect except :

(A)  Posterior dislocation of hip

(B)  Osteoporosis

(C)  Blue sclera

(D)  Ligament laxity

Answer: (A)

162. Antibodies for the following conditions may be transmitted from mother to fetus except :

(A)  Rh-incompatibility

(B)  Isoimmune thrombocytopenia

(C)  Toxic erythesma

(D)  My-gravis

Answer: (C)

163. Ranson’s criteria for prognosis in acute pancreatitis include all except

(A)  WBC > 16,000 cells/cumm

(B)  Age over 55 years

(C)  AST > 250 U/100 ml

(D)  S.amylase > 350 I.U.

Answer: (D)

164. Most common trisomy among following is :

(A)  18

(B)  21

(C)  13

(D)  5

Answer: (B)

165. Commonest lesion in middle mediastinum is :

(A)  Lipoma

(B)  Aneurysm

(C)  Congenital cyst

(D)  Germ cell tumour

Answer: (B)

166. Metabolic complication in CRF include all of the following except :

(A)  Hyperkalemia

(B)  Hypophosphatemia

(C)  Hypocalcemia

(D)  Hypokalemia

Answer: (B)

167. Treatment of Neurocysticercosis includes all of the following except :

(A)  Prazequentil

(B)  Albendazole

(C)  Niclosamide

(D)  Flubendazole

Answer: (C)

168. All of the following statements stand true for telengiectasia of colon except :

(A)  May be seen in person less than 40 years of age

(B)  May be seen in person more than 60 years of age

(C)  Common site is caecum

(D)  50% involve rectum

Answer: (D)

169. Shigella associated hemolytic uramic syndrome is associated with all of the following except :

(A)  Hyperkalemia

(B)  Thrombocytopenia

(C)  Neurological symptom

(D)  Renal microthrombi

Answer: (C)

170. LVH is commonly seen with :

(A)  Pure mitral stenosis

(B)  ASD with fossa-ovalis

(C)  Aortic incompetance

(D)  Carcinoid syndrome

Answer: (C)

171. True statement about cardiac myxoma is :

(A)  Commonest sit is left atrium

(B)  Rarely recurs after excision

(C)  Distant metastasis are seen

(D)  More common in females

Answer: (A)

172. In gram negative septicemia, early findings setting before shock of florid is :

(A)  ↑Cardiac out put, ↑Total peripheral resistance

(B)  ↑Cardiac out put, ↓ Total peripheral resistance

(C)  ↓Cardiac out put, ↑ Total peripheral resistance

(D)  ↓Cardiac out put, ↓ Total peripheral resistance

Answer: (B)

173. Hepatic-encephalopathy may be precipitated by all of the following except :

(A)  Anemia

(B)  Barbiturates

(C)  Hyper kalemia

(D)  Hypothyroidism

Answer: (C)

174. Budd-chiari syndrome is most commonly due to :

(A)  Hepatic vein obstruction

(B)  Acute portal hypertension

(C)  Congential portal hypertension

(D)  IVC-obstruction

Answer: (A)

175. Maximum increase in α-fetoprotein increased is seen with :

(A)  Hepato-cellular-carcinoma

(B)  Benign-mature teratoma

(C)  Chorio-carcinoma

(D)  Teratoma

Answer: (A)

176. Which of the following statements is not true regarding α-fetoprotein :

(A)  High levels are seen in fibrolamellar hepatic carcinoma

(B)  Pre-operative high level indicates worse prognosis

(C)  High level are seen in stomach carcinoma

(D)  Levels may be increased in Hepatitis

Answer: (A)

177. pylori is known to cause all of the following except :

(A)  Gastric ulcer

(B)  Duodenal ulcer

(C)  Gastric lymphoma

(D)  Fundal gastritis

Answer: (D)

178. Diagnostic tests for H. Pylori include all of the following except :

(A)  Urea-breath test

(B)  Rapid urease test

(C)  Gastric biopsy & Warthin-starry stain

(D)  SAFA test

Answer: (D)

179. Epidemiological studies of H. pylori are done by using :

(A)  Urea-breath test

(B)  Serological markers

(C)  Culture

(D)  Gastric-biopsy urease test

Answer: (B)

180. Acute infection with HBV is characterized by :

(A)  HBs Ag

(B)  Anti HBs Ag

(C)  IgM anti HBc Ag and HBs Ag

(D)  Anti HBc Ag

Answer: (C)

181. Most important investigation for diagnosis of Zollinger Ellison-syndrome is

(A)  Ca⁺⁺ infusion test

(B)  Secretin injection test

(C)  ACTH stimulation test

(D)  Steroid assay

Answer: (B)

182. Most common CNS manifestation of Whipple’s disease is :

(A)  Cerebellar ataxia

(B)  Supranuclear ophthalmoplegia

(C)  Seizure

(D)  Dementia

Answer: (D)

183. Megaloblastic anemia in blind loop syndrome is due to :

(A)  Vitamin B­₁₂ malabsorption

(B)  Bacterial overgrowth

(C)  Frequent diarrhoea

(D)  Decrease iron intake

Answer: (B)

184. Ankylosing spondylitis in associated with :

(A)  HLA-B27

(B)  HLA-B-8

(C)  HLA-DW4/DR4

(D)  HLA-DR3

Answer: (A)

185. Increased alkaline phosphate in seen in :

(A)  Multiple myeloma

(B)  Primary hyperparathyroidism

(C)  Chronic renal failure

(D)  Osteoporosis

Answer: (B)

186. All of the following conditions are premalignant except

(A)  Ulcerative colitis

(B)  Crohn’s disease

(C)  Brochiectasis

(D)  Paget disease of bone

Answer: (C)

187. Most common tumor of the anterior mediastinum tumour is :

(A)  Thymoma

(B)  Lymphoma

(C)  Neurogenic tumour

(D)  Vascular cyst

Answer: (A)

188. RNA fragments are sequenced by :

(A)  Eastern-blot

(B)  Western blot

(C)  Northern blot

(D)  Southern-blot

Answer: (B)

189. Most common group of diseases following Mendelian inheritance are :

(A)  Autosomal dominant

(B)  Autosomal recessive

(C)  X-linked dominant

(D)  X-linked recessive

Answer: (A)

190. Smoking predisposes to all of the following cancers except :

(A)  Carcinoma larynx

(B)  Carcinoma bladder

(C)  Carcinoma stomach

(D)  Lymphoma

Answer: (D)

191. α1-Antitrypsin deficiency is associated with :

(A)  Centriaciner emphysema

(B)  Panaciner-emphysema

(C)  Irregular emphysema

(D)  Paraseptal-emphysema

Answer: (B)

192. Most unlikely cause of acute tubular necrosis amongst the following is :

(A)  Severe-bacterial-infection

(B)  Massive burn

(C)  Severe cursh injury in the foot

(D)  Rupture of aortic aneurysm

Answer: (D)

193. Sweat chloride levels are increased in all of the following conditions except :

(A)  Ectodermal dysplasia

(B)  G-6PD-deficiency

(C)  Heriditary nephrogenic-diabetes incipidus

(D)  Obesity

Answer: (D)

194. UMN lesion is characterized by :

(A)  Weakness & spasticity

(B)  Fasciculations

(C)  Rigidity

(D)  Localized muscle atrophy

Answer: (A)

195. “Prosopagnosia” is characterized by :

(A)  Inability to read

(B)  Inability to identify faces

(C)  Inability to write

(D)  Inability to speak

Answer: (B)

196. Most common cause of sub arachroid haemorrhage is :

(A)  Hypertension

(B)  A.V. malformation

(C)  Berry aneurysm

(D)  Trauma

Answer: (C)

197. A nondiabetic, non hypertensive adult male develops sudden severe headache with altered sensorium, likely diagnosis is :

(A)  Brain tumour

(B)  Subarachnoid haemorrhage

(C)  Encephalitis

(D)  Meningitis

Answer: (B)

198. Painless burn in hand is a characteristic feature of :

(A)  Syringomgelia

(B)  Thalamic syndrome

(C)  Cord compression

(D)  Systemic-lupus-erthromatosis

Answer: (A)

199. Triad of normal-pressure hydrocephalus includes :

(A)  Tremor, aphasia dementia

(B)  Ataxia, aphasia, gait disorder

(C)  Gait disorder, urinary incontinence, dementia

(D)  Gait disorder, urinary inconitinence, lower cranial nerve palsy

Answer: (C)

200. All of the following are known predisposing factors for Alzheimer’s disease except :

(A)  Down-syndrome

(B)  Low education level

(C)  Smoking

(D)  Female sex

Answer: (C)

201. Alzheimer’s Disease is associated with :

(A)  Delerium

(B)  Delerium

(C)  Dementia

(D)  Depresssion

Answer: (C)

202. Gene for myotonic dystrophy is coded on chromosome number :

(A)  19

(B)  20

(C)  21

(D)  24

Answer: (A)

203. Fatal familial insomnia is associated with :

(A)  Prion disease

(B)  Degenerative disease

(C)  Neoplastic disease

(D)  Vascular disease

Answer: (A)

204. All are true about peripheral-neuropathy except :

(A)  Glove and stocking anaesthesia

(B)  Proximal muscle weakness

(C)  Nerve-conduction deficit

(D)  Decreased reflexes

Answer: (B)

205. All are true about subacute combined degeneration of spinal cord except :

(A)  Due to deficiency of vitamin B12

(B)  Posterior column involvement

(C)  Corticospinal involvement

(D)  Absent deep tendon reflexes

Answer: (D)

206. Oxygen content of the arterial blood is reduced in all except:

(A)  Methemoglobinemia

(B)  Fallot’s tetralogy

(C)  Carbon monoxide poisoning

(D)  Fibrosing alveolitis

Answer: (B)

207. Following are ECG features of hyperkalemia except:

(A)  High T wave

(B)  Depressed ST segment

(C)  Prolonged PR interval

(D)  Prolonged QT interval

Answer: (D)

208. Macroglossia is seen in following except:

(A)  Amyloidosis

(B)  Acromegaly

(C)  Hyperthyroidism

(D)  Down’s syndrome

Answer: (C)

209. In idiopathic polymyositis, following are seen except:

(A)  Ocular muscles commonly involved

(B)  Pharyngeal muscles involved

(C)  Cutaneous manifestations

(D)  Proximal limb muscles invariably involved

Answer: (A)

210. In chronic dialysis, most common CNS manifestation is:

(A)  Sensory loss

(B)  Paralysis

(C)  Dementia

(D)  Depression

Answer: (C)

211. PAN most commonly involves:

(A)  Large vessels

(B)  Medium and small sized vessels

(C)  Arterioles

(D)  Venules

Answer: (B)

212. In complication of falciparum malaria, which drug is not given

(A)  Phenobarbitone

(B)  Dexamethasone

(C)  Quinine

(D)  Blood transfusion

Answer: (B)

213. Right hemithorax with right side shift of mediastinum is due to:

(A)  Collapse of lung

(B)  Right side hemothorax

(C)  Hydrothorax

(D)  Bronchiectasis

Answer: (A)

214. Spontaneous hypoglycemia is caused by:

(A)  Glucagonoma

(B)  Chr. pancreatitis

(C)  Cushing’s syndrome

(D)  Hepatocellular carcinoma

Answer: (D)

215. Which does not cause malabsorption in India :

(A)  Tropical sprue

(B)  Amoebiasis

(C)  Giardiasis

(D)  Intestinal TB

Answer: (A)

216. Addison’s disease is characterized by following except:

(A)  Hyperkalemia

(B)  Hypotension

(C)  Hyponatremia

(D)  Hypocalcemia

Answer: (D)

217. Most common cause of LVH is:

(A)  MS

(B)  AR

(C)  Hypertension

(D)  Mitral valve prolapse

Answer: (C)

218. Hyperacute rejection of renal transplant is due to :

(A)  Preformed antibodies

(B)  Antigen antibody complex

(C)  T-cell sensitization

(D)  All of the above

Answer: (A)

219. In case of renal transplant, most common infection within one month is :

(A)  Pneumococcus

(B)  Gram-ve organism

(C)  Pneumocystis carinii

(D)  Cryptococcus

Answer: (C)

220. Muscular weakness due to deficiency of magnesium is enhanced by presence of:

(A)  Hyperkalemia

(B)  Metabolic alkalosis

(C)  Metabolic acidosis

(D)  Hypernatremia

Answer: (B)

221. In metabolic encephalopathy, feature is:

(A)  Tonic clonic seizures

(B)  Chorea

(C)  Asterixis

(D)  Paraplegia

Answer: (C)

222. Antibiotic associated diarrhea is most commonly caused by:

(A)  E.coli

(B)  Clostridium difficile

(C)  Proteus

(D)  Faecal streptococci

Answer: (B)

223. Metabolic alkalosis seen in following except:

(A)  Methanol poisoning

(B)  Cushing’s disease

(C)  Vomiting

(D)  Diuretic therapy

Answer: (A)

224. In hepatic cirrhosis, which of the following is increased:

(A)  Alpha 1 globulin

(B)  Alpha 2 globulin

(C)  Gamma globulin

(D)  All of the above

Answer: (C)

225. Following are features of acute GN except :

(A)  Polyuria

(B)  Haematuria

(C)  Red cell cast

(D)  Oliguia

Answer: (A)

226. Hypochromic microcytosis is seen in following except:

(A)  Lead poisoning

(B)  Sideroblastic anemia

(C)  Iron deficiency anemia

(D)  Aplastic anemia

Answer: (D)

227. Intake of exogenous steroid causes

(A)  Addison’s disease

(B)  Cushing’s syndrome

(C)  Pheochromocytoma

(D)  Conn’s syndrome

Answer: (B)

228. The following are features of primary hyperaldosteronism except :

(A)  Polyuria

(B)  Hypertension

(C)  Hypokalemia

(D)  Hyperkalemia

Answer: (D)

229. Familial periodic paralysis is caused by:

(A)  Hyponatremia

(B)  Hypertension

(C)  Hyperchloremia

(D)  Hyperkalemia

Answer: (D)

230. Most common cause of LMN facial palsy is :

(A)  Disseminated sclerosis

(B)  Bell’s palsy

(C)  DM

(D)  Leprosy

Answer: (B)

231. Carey comb murmur is seen in:

(A)  Severe mitral stenosis

(B)  Acute rheumatic carditis

(C)  Pure aortic regurgitation

(D)  Severe pulmonary HT

Answer: (B)

232. Shrinking lung is seen in:

(A)  Diffuse fibrosis

(B)  Scleroderma

(C)  SLE

(D) Rheumatoid arthritis

Answer: (C)

233. Pleural aspiration with raised LDH and amylase seen in:

(A)  Pulm. infarction

(B)  COPD

(C)  Pulm. malignancy

(D)  None

Answer: (C)

234. In MI following are used except :

(A)  Fibrinolytics

(B)  Plasminogen activator inhibitor

(C)  Anti thrombin

(D)  Platelet inhibitor

Answer: (B)

235. True about Polycythemia rubra vera is all except :

(A)  Bleeding

(B)  Thrombosis

(C)  ↓ ed ESR

(D)  Infection

Answer: (D)

236. Wilson disease all are true except :

(A)  ↑ ceruloplasmin

(B)  ↑ ceruloplasmin

(C)  ↑ Cu in cornea

(D)  Absence of sensory loss

Answer: (A)

237. Congenital erythropoietic porphyria is due to deficiency of:

(A)  Uroporphyrinogen III cosynthetase

(B)  ALA synthetase

(C)  Ferrochelatase

(D)  CP oxidase

Answer: (A)

238. In multiple myeloma following are seen except :

(A)  Anion gap raised

(B)  Lytic bone lesion

(C)  Polyarticular pains

(D)  M spike present with polyarticulopathy

Answer: (A)

239. True about criggler Najjar type II syndrome is:

(A)  Diglucuronide deficiency

(B)  Recessive trait

(C)  Kernicterus is seen

(D)  Phenobarb not useful

Answer: (A)

240. Tache noires is seen in:

(A)  Emphysema

(B)  Diphtheria

(C)  Boutoneuse fever

(D)  Bronchial asthma

Answer: (C)

241. Following are seen in mucopolysaccharidoses:

(A)  Hair on-end appearance

(B)  Beaked vertebrae

(C)  Avascular necrosis of femoral head

(D)  Osteosclerotic lesions

Answer: (B)

242. In mycoplasma pneumonia, following are seen except:

(A)  Diffuse infiltration of lungs

(B)  Cannot be cultured routinely

(C)  Best treatment by cefotaxime

(D)  Serology is useful in diagnosis

Answer: (C)

243. In addison’s disease, most diagnostic test is:

(A)  Serum, N⁺, k⁺, renin

(B)  S. Na⁺, K⁺, saline supression

(C)  Serum creatinine urea ratio

(D)  ACTH stimulation test

Answer: (D)

244. ↑ Ca+ 2 in Ca lung is seen in Ca:

(A)  Adeno

(B)  Squamous

(C)  Squamous

(D)  Large cell

Answer: (B)

245. Diarrhoea is seen in

(A)  Insulinoma

(B)  Glucagonoma

(C)  Vipoma

(D)  All

Answer: (C)

246. Adson’s test is positive in:

(A)  Cervical rib

(B)  Cervical fracture

(C)  Cervical dislocation

(D)  Cervical spondylosis

Answer: (A)

247. Serum creatinine kinase is elevated in A/E

(A)  Becker’s myopathy

(B)  Acute myoglobinic myopanthies

(C)  Acute polymyositis

(D)  Central core myopathy

Answer: (D)

248. In carcinoid syndrome, the part of heart mostly affected is

(A)  Outflow tract of RV

(B)  Inflow tract of RV

(C)  Inflow tract of LV

(D)  Outflow tract in LV

Answer: (B)

249. In steel Richardson syndrome following are seen except:

(A)  Convulsions

(B)  Ataxia

(C)  Progressive supranuclear palsy

(D)  Dementia

Answer: (A)

250. Site of lesion in endocarditis of RHD is

(A)  Along the line of closure of valves

(B)  Both sides of valves

(C)  Valve cusps

(D)  Free margin of valves

Answer: (A)

251. Free margin of valves

(A)  Myeloma

(B)  TB spine

(C)  CRF

(D)  Ankylosing spondylitis

Answer: (C)

252. Uveoparotitis is seen in :

(A)  SLE

(B)  Sjogren’s syndrome

(C)  Rheumatoid arthritis

(D)  Sarcoidosis

Answer: (D)

253. Most useful investigation in diagnosis of diabetic ketoacidosis :

(A)  Ketonemia

(B)  pH of blood

(C)  Urinary sugar

(D)  Urine ketone

Answer: (A)

254. Absent ankle jerk and extensor plantar response is found in :

(A)  Friedreich’s ataxia

(B)  Tabes dorsalis

(C)  SACD

(D)  Vit B deficiency

Answer: (A)

255. Pt. with DM. Hypertension, having CHF, the most appropriate drug is :

(A)  ACE inhibitors

(B)  Digitalis

(C)  β blockers

(D)  Diuretics

Answer: (A)

256. Pulsus alternans occurs in :

(A)  Constrictive pericarditis

(B)  Viral mycarditis

(C)  Hypokalemia

(D)  MI

Answer: (D)

257. Causative agent of infectious Mononucleosis :

(A)  EB virus

(B)  Rubella virus

(C)  Adenovirus

(D)  Herpes virus

Answer: (A)

258. pH is 7.2; PCO2 12; PO2 55 most likely cause is:

(A)  Metabolic acidosis and respiratory alkalosis

(B)  Metabolic alkalosis

(C)  Respiratory acidosis

(D)  Respiratory alkalosis

Answer: (A)

259. Not seen in pre capillary pulm. hypertension :

(A)  ↑ pressure in pulm circulation

(B)  ↑ capillary pressure

(C)  Rt-vent. hypertrophy

(D)  ↑Pulm. wedge pressure

Answer: (D)

260. ECG in young healthy person with 80 beats/min will not show:

(A)  RP interval between 120 – 200 msec

(B)  QRS interval less than 100 msec

(C)  Interval between 2 QRS is 750 msec

(D)  T. wave represents repolarization

Answer: (C)

261. Poor prognostic factor in Acute Myocardial infarction :

(A)  VPC’s in first 24 hours

(B)  Hypotension at diagnosis

(C)  Chest pain

(D)  Hypertension

Answer: (A)

262. Stimulation of proximal cut end of vagus causes:

(A)  Apnea

(B)  ↓ HR

(C)  ± BP

(D)  ↓ BP

Answer: (A)

263. Serum amylase level is raised in A/E:

(A)  Blocked salivary duct

(B)  Ruptured ectopic

(C)  Appendicitis

(D)  Pancreatitis

Answer: (C)

264. Osmotic fragility is seen in :

(A)  Sickle cell anemia

(B)  B Thalassemia

(C)  Hereditary spherocytosis

(D)  Iron deficiency anemia

Answer: (C)

265. First line of treatment in severe ascites :

(A)  Bed rest and slat restriction

(B)  Heavy peracentesis

(C)  Shunt

(D)  Large dose of diuretics

Answer: (A)

266. True about pulsus paradoxus is :

(A)  Arm – tongue circulation time is increased

(B)  ↑Stroke volume

(C)  Seen in constrictive pericarditis

(D)  ↑ HR

Answer: (C)

267. Muscle biopsy in PAN shows:

(A)  Necrotising arteritis

(B)  Atrophy

(C)  Granulomatous lesions

(D)  Ring lesions

Answer: (A)

268. During valsalva maneuver, impaired heart rate changes seen in:

(A)  Horner’s syndrome

(B)  Autonomic insufficiency

(C)  Vestibular dysfunction

(D)  Cephalic ischemia

Answer: (B)

269. Alpha chain disease is detected by:

(A)  M component in serum

(B)  Jejunal biopsy

(C)  Light chain in lumen if jejunum

(D)  All of the above

Answer: (B)

270. Cancer caused by chronic parasitic infection :

(A)  Cholangiocarcinoma

(B)  Hepatic cell Ca

(C)  Ca GB

(D)  Ca pancreas

Answer: (A)

272. Thickening of pulmonary memb. is seen in :

(A)  Asthma

(B)  Emphysema

(C)  Bronchitis

(D)  Skeletal defect

Answer: (A)

272. Dilutional hyponatremia is seen in :

(A)  Addison’s disease

(B)  DI

(C)  Diuretic therapy

(D)  None

Answer: (A)

273. At the end of ventricular diastole:

(A)  Atrial volume is more

(B)  Coronary flow is maximum

(C)  Flow in aorta drops

(D)  All of the above

Answer: (C)

274. Most dangerous dehydration is :

(A)  Hyponatremic

(B)  Hypernatremic

(C)  Isonatremic

(D)  Non-diarrhoeal cause

Answer: (B)

275. Traid of  normal pressure hydocephalus is :

(A)  Ataxia, dementia, urinary incontinence

(B)  Ataxia, aphasia, Ur. Incontinence

(C)  Ataxia, Dementia, Asphasia

(D)  Ataxia, alexia, Aphasia

Answer: (A)

276. Myoclonic seizure typically seen in:

(A)  SSPE

(B)  Cereberllar lesion

(C)  Pontine lesion

(D)  Thalamic lesion

Answer: (A)

277. Alzheimers disease, true is all except :

(A)  Anterograde amnesia

(B)  Dysdiadochokinesia

(C)  Receptive aphasia

(D)  Recogniation of familiar faces is absent

Answer: (B)

278. The characteristic and common presentation of diabetic neuropathy is :

(A)  Amyotrophy

(B)  Mononeuropathy

(C)  Symmetrical sensory neuropathy

(D)  Autonomic neuropathy

Answer: (C)

279. Diabetes control is best monitored by :

(A)  Serum glucose

(B)  Post prandial blood glucose

(C)  HbA1AC

(D)  HbA2AC

Answer: (C)

280. Double apical impulse is seen in :

(A)  MR

(B)  AR

(C)  AS

(D)  MS

Answer: (C)

281. Hand foot syndrome is due to:

(A)  Thalassemia

(B)  Sickle cell anemia

(C)  Haemochromatosis

(D)  Acromegaly

Answer: (B)

282. Most common cause of stroke in young women in India among OCP users :

(A)  Cortical vein thrombosis

(B)  Moyamoya disease

(C)  Atherosclerosis

(D)  HT

Answer: (A)

283. Spinomuscular atrophy is seen in lesion of :

(A)  Ant, horn

(B)  Peripheral nerve

(C)  NMJ

(D)  Any of the above

Answer: (A)

284. hemophilia with rheumatoid arthritis, analgesic of choice :

(A)  Ibuprofen

(B)  Aspirin

(C)  Phenylbutazone

(D)  None

Answer: (D)

285. In hyperparathyroidism all are seen except :

(A)  Osteopetrosis

(B)  Osteoporosis

(C)  Cysts

(D)  Brown tumor

Answer: (A)

286. Cutaneous marker of HIV is :

(A)  Seborrhoea

(B)  Vesicular rash

(C)  Oral candidiasis

(D)  Photosensitivity

Answer: (A)

287. In SLE, characteristic kidney lesion is:

(A)  Mesangial proliferation

(B)  Tubular fibrin deposits

(C)  Wire loops

(D)  IgG deposits

Answer: (C)

288. Hypotonia, tremor and ataxia are seen in lesion of:

(A)  Basal ganglia

(B)  Medullary pyramid

(C)  Cerebellum

(D)  Pons

Answer: (C)

289. Botulinum toxin is used in:

(A)  Focal dystonia

(B)  Myasthenia gravis

(C)  Cerebellare ataxia

(D)  Hypotonia

Answer: (A)

290. Opso myoclonus is seen in:

(A)  Neuroblastoma

(B)  Nephrobalstoma

(C)  Retino blastoma

(D)  Melanoma

Answer: (A)

291. Best Rx for CML is:

(A)  Autologous BMT

(B)  Allogenic BMT

(C)  Alpha interferon

(D)  Hydroxyurea

Answer: (B)

292. Lysis of cells cause A/E:

(A)  Hyperuricemia

(B)  Hypercalcemia

(C)  Hyper phosphatemia

(D)  Hyperkalemia

Answer: (B)

293. Giant cell (Hecht’s) pneumonia is due to:

(A)  CMV

(B)  Measles

(C)  Malaria

(D)  P. carinii

Answer: (B)

294. Pneumocystis carini pneumonia is diagnosed by:

(A)  Interstitial penumonia

(B)  Eosinophils in septa

(C)  Alveolar type I cell damaged with eosinophils

(D)  All of the above

Answer: (A)

295. Rheumatoid arthritis pericarditis is:

(A)  Serous

(B)  Fibrinous

(C)  Serofibrinous

(D)  Adhesive

Answer: (B)

296. During ventricular pressure pulses square root wave is seen in:

(A)  ASD

(B)  MVPS

(C)  Dilated cardiomyopathy

(D)  Constrictive pericarditis

Answer: (D)

297. Characteristic feature of koraskoff psychosis is:

(A)  Disorientation

(B)  Delirium

(C)  Coarse tremor

(D)  Amnesia

Answer: (D)

298. Sub cortical dementia true is:

(A)  Loss of tactile localization

(B)  Loss of memory

(C)  Total paralysis

(D)  Gait distrubances

Answer: (D)

299. Treatable cause of MR is :

(A)  Hypothyroidism

(B)  Mucopolysaccharidosis

(C)  Turner’s syndrome

(D)  Down’s syndrome

Answer: (A)

300. In CCF there is:

(A)  Oliguria

(B)  Polyuria

(C)  Olijuria during day and polyuria during night

(D)  Anuria

Answer: (A)

301. A lesion in the paracentral lobule causes :

(A)  Contralateral foot weakness

(B)  Seizures only

(C)  Migraine

(D)  Cognitive loss

Answer: (A)

302. AV fistula true is except :

(A)  Tachycardia

(B)  High stroke volume

(C)  Increased CO

(D)  None

Answer: (D)

303. Bad prognosis in multiple myeloma is indicated by :

(A)  WBC > 20000

(B)  Azotemia

(C)  Hypocalcemia

(D)  Low or normal M component production

Answer: (B)

304. Not seen in constrictive pericarditis is :

(A)  Acute pulmonary edema

(B)  Ascites

(C)  Tapping apex

(D)  Pericardial knock

Answer: (A)

305. Treatment of choice for intracranial ALL is :

(A)  Intrathecal methotrexate

(B)  Vincristine and prednisolone

(C)  Intrathecal vincristine

(D)  Prednisolone

Answer: (A)

306. Which is not a feature of cerebral malaria :

(A)  Perivascular demyelination

(B)  Durck granuloma

(C)  Prolifereated glial cells arranged radially

(D)  Lesion is not related to parasite

Answer: (D)

307. A person who has drunk the anti –freeze of his car is likely to have :

(A)  Metabolic alkalosis with normal anion gap

(B)  Metabolic acidosis with increased anion gap

(C)  Metabolic alkalosis

(D)  Respiratory alkalosis

Answer: (B)

308. Wallenberg’s syndrome does not involve :

(A)  IX

(B)  X

(C)  XI

(D)  XII

Answer: (D)

309. True about interstitial fibrosis :

(A)  FVC ↓

(B)  FEV1/FVC normal or ↑

(C)  FRC normal

(D)  FEV1/FVC ↓

Answer: (B)

310. Which among the following does not cause exaggerated jaw jerk :

(A)  Multiple sclerosis

(B)  Motor neurone disease

(C)  Pseudobulbar palsy

(D)  Syringomyelia

Answer: (D)

311. A medial temporal lesion produces :

(A)  Visual amnesia only

(B)  Auditory amnesia

(C)  Apraxia

(D)  Anterograde learning problems

Answer: (D)

312. The characteristic finding in diabetic nephropathy is :

(A)  Diffuse glomerulosclerosis

(B)  Nodular glomerulosclerosis

(C)  Armani-Ebstein reaction

(D)  Fibrin caps

Answer: (B)

313. Converging point of both pathways in coagulation is at:

(A)  Factor VIII

(B)  Stuart factor X

(C)  Factor IX

(D)  Factor VII

Answer: (B)

314. Athletic syndrome is characterized by :

(A)  Increased amplitude of QRS

(B)  Tachycardia

(C)  Decreased QT interval

(D)  U-waves

Answer: (A)

315. Opsoclonus is :

(A)  Arrythmic conjugate eye movements

(B)  Conjugate chaotic movements

(C)  Conjugate, chaotic, continuous movements

(D)  Chaotic, disconjugate movements

Answer: (A)

316. Digoxin toxicity is precipitated by :

(A)  Atenolol

(B)  Quinidine

(C)  Pulsus paradoxus

(D)  Cyanosis

Answer: (B)

317. A patient has hyperphosphatemia with short metacarpals and associated cataract. The diagnosis is :

(A)  Pseudohypoparathyroidism

(B)  Hypophosphatasia

(C)  Hyperparathyroidism

(D)  Osteomalacia

Answer: (A)

318. The chance that a health worker gets HIV from an accidental needle prick is :

(A)  1%

(B)  10%

(C)  95%

(D)  100%

Answer: (A)

319. A common finding in osteomalacia is

(A)  Low serum phosphate

(B)  Normal level of 1.25 di-hydroxy vit D3

(C)  Low serum calcium

(D)  Increased hydroxyl proline in urine

Answer: (A)

320. Decreased glucose level in pleural effusion is found in

(A)  Rheumatoid arthritis

(B)  SLE

(C)  Pneumococcal infection

(D)  P. carinii infection

Answer: (A)

321. Pressure difference of 5 mm Hg between the two upper limbs occurs in which congenital heart disease :

(A)  TOF

(B)  TGA

(C)  HOCM

(D)  Supra-valvular aortic stenosis

Answer: (D)

322. Shrinking lung is a feature of :

(A)  SLE

(B)  Rheumatoid arthritis

(C)  Scleroderma

(D)  Sarcoidosis

Answer: (A)

323. A patient had the following blood biochemical values. Calcium 6; uric acid 13; phosphorus 12; creatinin 6. Which could be the possible diagnosis :

(A)  Krait bite

(B)  Uric acid nephropathy

(C)  Hypercalcemic nephropathy

(D)  Rickets

Answer: (B)

324. Torsade de points is cause by :

(A)  Hypermagnesemia

(B)  Metabolic acidosis

(C)  Hypomagnesemia

(D)  Metabolic alkalosis

Answer: (C)

325. Which is seen in nephritic syndrome

(A)  Low serum calcium

(B)  Raised AT-III

(C)  Low lipid

(D)  Platelet activation

Answer: (A)

327. Loud S1 in mitral stenosis is seen in

(A)  Prolonged flow through mitral valve

(B)  1st degree heart block

(C)  Calcification of the valve

(D)  Immobilization of the valve

Answer: (A)

328. Not seen to tumor-lysis syndrome is :

(A)  Hyperkalemia

(B)  Hypercalcemia

(C)  Hyponatremia

(D)  Hypocalcemia

Answer: (B)

328. Which is seen in opioid poisoning :

(A)  Hyperventilation

(B)  Raised BP

(C)  Slow shallow respiration

(D)  Dyspnea

Answer: (C)

329. Drug of choice for myasthenia gravis :

(A)  Gallamine

(B)  Succinylocholine

(C)  D tubocurare

(D)  Pyridostigmine

Answer: (D)

330. Primary increase in CO2 in body is seen in:

(A)  Respiratory acidosis

(B)  Respiratory alkalosis

(C)  Metabolic acidosis

(D)  Metabolic alkalosis

Answer: (A)

331. All are true about aspirin poisoning except :

(A)  Acidosis

(B)  Hypothermia

(C)  Dehydration

(D)  Oliguria

Answer: (B)

332. Seen in Behcet’s syndrome is :

(A)  Pyoderma gangrenosum

(B)  Thrombophlebitis

(C)  Renal manifestations

(D)  Painless ulcers

Answer: (B)

333. Characteristic feature of SLE is :

(A)  Uveitis

(B)  Joint deformity

(C)  Polyserositis

(D)  Cavitating lesion in lung

Answer: (C)

334. All are complications of tubercular meningitis except :

(A)  Hydrocephalus

(B)  Infarction

(C)  Obliterative endarteritis

(D)  Sinovenous thrombosis

Answer: (D)

335. True about lyme disease is :

(A)  Arthritis is an early manifestation

(B)  Conduction block is not seen

(C)  Mood disorder is chronic neurological manifestation

(D)  Causative organism is Borrelia reccurentis

Answer: (C)

336. All are true for transplanted kidney except :

(A)  Humoral antibody responsible for rejection

(B)  CMI, responsible for rejection

(C)  Donor’s antibody on transfusion can causes rejection

(D)  HLA identity similarity is seen in 1: 100 people

Answer: (D)

337. Hypercalcemia in ECG is diagnosed by :

(A)  Increased QT interval

(B)  Decreased QT interval

(C)  Increased PR interval

(D)  Tall T waves

Answer: (B)

338. Which doesn’t cause hemolysis in G 6 PD deficiency :

(A)  Oestrogen

(B)  Salicylates

(C)  Primaquine

(D)  Nitrofurantoin

Answer: (A)

339. In Botulinism all are seen except :

(A)  Diplopia

(B)  Diarrhoea

(C)  Dysphagia

(D)  Abd. pain

Answer: (B)

340. Punch drunk state is associated with:

(A)  Alcoholism

(B)  Chorea

(C)  Parkinsonism

(D)  All

Answer: (C)

341. Earliest manifestation of hepatitis B is:

(A)  HBe Ag

(B)  HBs Ab IgM

(C)  HBc IgM

(D)  HBe AbIgM

Answer: (A)

342. Urinary 3 hydroxy 4 methyl VMA is seen with:

(A)  Neuroblastoma

(B)  Pheochromocytoma

(C)  Ganglioneuroma

(D)  Carcinoid

Answer: (B)

343. KF ring deposition is seen in:

(A)  Conjunctiva

(B)  Corneal epithelium

(C)  Descement membrane

(D)  Iris

Answer: (C)

344. Single gene autosomal recessive disease is :

(A)  Wilson’s ds

(B)  Tuberous sclerosis

(C)  Hutington’s ds

(D)  Schizophrenia

Answer: (A)

345. Feature of microscopic polyangitis is :

(A)  IgG deposits in kidney

(B)  Bronchospasm

(C)  Renal involvement in 90% cases

(D)  All of the above

Answer: (C)

346. All are HIV associated malignancies except :

(A)  Kaposi’s sarcoma

(B)  Lung carcinoma

(C)  NHLO

(D)  Anogenital CA

Answer: (B)

347. All are true about pseudopancreatic cyst except :

(A)  Not lined by epithelium

(B)  Resolves spontaneously

(C)  Raised amylase level not persistent

(D)  CT and ultrasound helps in diagnosis

Answer: (C)

348. Worst prognostic indicator of acute pancreatitis is :

(A)  Hypercalcemia

(B)  Hypocalcemia

(C)  Raised amylase level

(D)  Glycosuria

Answer: (B)

349. Ca 125 is specific marker of :

(A)  Choriocarcinoma

(B)  Teratoma

(C)  Epithelial cell carcinoma of ovary

(D)  Seminoma

Answer: (C)

350. Brain tumour is associated with A/E :

(A)  Tuberous sclerosis

(B)  Von hippel indauds

(C)  Neurofibromatosis

(D)  Sturge weber syndrome

Answer: (D)

351. Eosinophiluria is seen in :

(A)  Wegener’s granulomatosis

(B)  Interstitial nephritis

(C)  PAN

(D)  HUS

Answer: (B)

352. In tubular necrosis, ratio of urine plasma creatinine is:

(A)  20

(B)  40

(C)  20-30

(D)  30-40

Answer: (A)

353. Prerenal and renal azotemia is differentiated on the basis of:

(A)  Creatinine clearance

(B)  Serum creatinine level

(C)  Sodium fraction excretion

(D)  Urine bicarbonate level

Answer: (C)

354. CNS tumor seen in vol hippel landau syndrome is:

(A)  Meningioma

(B)  Cerebellar hemangiblastoma

(C)  CNS lymphoma

(D)  Glioma

Answer: (B)

355. All are true in chronic constrictive pericarditis except :

(A)  Kussamauls sign is present in all types

(B)  Ascites is not in proportion to edema

(C)  Commonest cause is idiopathic

(D)  Right ventricular and diastolic pressure is raised

Answer: (C)

356. Rapid x descent in unlikely in:

(A)  Constrictive pericarditis

(B)  Cardiac temponade

(C)  RV infarction

(D)  Restrictive CMP

Answer: (C)

357. MC cause of death in adults with PDA is

(A)  CCF

(B)  Infective endocarditis

(C)  Rupture

(D)  Embolism

Answer: (A)

358. Most characteristic GN in HIV is:

(A)  FSGN

(B)  MPGN

(C)  MCD

(D)  RPGN

Answer: (A)

359. Cavitating lesion in lung is seen in:

(A)  PAN

(B)  SLE

(C)  Wegeners granulomatosis

(D)  Sjorgren’s syndrome

Answer: (C)

360. Primary extraglandular sjogren’s syndrome is seen in A/E:

(A)  Rheumtoid Arthritis

(B)  Raynaud’s Phenomena

(C)  Lymphoma

(D)  Splenomegally

Answer: (A)

361. All are seen in SIRS except :

(A)  Leukopenia

(B)  Hypoxia

(C)  Hyperthermia

(D)  Tachycardia

Answer: (B)

362. True about sarcoidosis is :

(A)  Kveim tet is not helpful in diagnosis

(B)  May be associated with positive RA factor

(C)  Pleural effusion

(D)  Common is 50 years male

Answer: (B)

363. Seen in chroni inflammatory anemia is

(A)  Serum iron ↓ S. ferritin ↑ and transferrin ↓

(B)  Serum iron ↓ S. ferritin ↑ and transferrin ↑

(C)  Serum iron ↓ S. ferritin ↓ and transferrin ↑

(D)  Serum iron ↑ S. ferritin ↓ and transferrin ↓

Answer: (A)

364. Hypocalcemic hyperphosphatemia is seen in A/E:

(A)  Malabsorption

(B)  Pseudohypoparathyroidism

(C)  CRF

(D)  Hypoparathyroidism

Answer: (A)

365. Nephrocalcinosis is a feature of A/E:

(A)  Primary hyperparathyroidism

(B)  Medullary sponge kidney

(C)  Vitamin D intoxication

(D)  Pseudo hypoparathyroidism

Answer: (D)

366. In Accelerated HTN what is metabolic defect :

(A)  Normal non-ionic metabolic acidosis

(B)  Ionic gap met acidosis

(C)  Hypomagnesemia

(D)  Metabolic alkalosis

Answer: (D)

367. Amylase increased in pleural fluid is seen in following except :

(A)  Rheumatoid arthritis

(B)  Esophageal perforation

(C)  Malignancy

(D)  Gall stone pancreatitis

Answer: (A)

368. Dengue hemorrhagic fever :

(A)  Most common in previously affected patients

(B)  DHF-2 most common cause

(C)  Immunosuppresed

(D)  Incubation period 7-14 days

Answer: (A)

369. Acanthosis nigricans is caused by which drug :

(A)  Amphotericin – B

(B)  Ketoconazole

(C)  Nicotinic acid

(D)  Nalidixil acid

Answer: (C)

370. DOC in esophageal candidiasis in HIV :

(A)  Fluconazole

(B)  Miconazole

(C)  Amphotericin -B

(D)  Grisefulvin

Answer: (A)

371. Gyanecomastia is seen in following except :

(A)  Digitalis

(B)  Seminoma

(C)  Cryptorchidism

(D)  Puberty

Answer: (C)

372. Incongential erythropoitic porphyria, enzyme-defect is :

(A)  Uro synthase

(B)  Ferro ketolase

(C)  ALA synthetase

(D)  PROTO oxidase

Answer: (A)

373. True about Gardner’s syndrome is except :

(A)  CNS tumor

(B)  Osteoma

(C)  Dermoid tumour

(D)  Colonic polyps

Answer: (A)

374. 100% recurrence of disease in Down’s syndrome :

(A)  Translocation of 15 + 21 chromosome

(B)  Mosaic pattern

(C)  Trisomy –(21, 21 translocation)

(D)  Non Dysjunction

Answer: (C)

375. All following secreted by Bronchogenic Ca except :

(A)  Nor adrenaline

(B)  PTH

(C)  ACTH

(D)  ADH

Answer: (A)

376. Typical JVP finding in cardiac tamponade :

(A)  Absent ‘Y’ descent

(B)  Prominent ‘a’ wave

(C)  Absent ‘a’ wave

(D)  Prominent ‘Y’ wave

Answer: (A)

377. About cushing syndrome, true is :

(A)  Low dose dexamethasone suppress cortisone secretion

(B)  CA of adrenal is more common than adenoma

(C)  Pituitary adenoma size > 2 cm (usually)

(D)  ↑ACTH secretion is the commonest cause

Answer: (D)

378. Photosensitivity is maximum in :

(A)  Prophyria cutanea tarda

(B)  Congenital Erythropoietic pupura

(C)  Erythrogenic porphyria

(D)  Hereditary coproporphyria

Answer: (B)

379. Enzyme defect in congenital erythropoietic porphyria :

(A)  Uroporphyrinogen III synthetase

(B)  Uroporphyrinogen decarboxylase

(C)  ALA synthetase

(D)  Ferroketolase

Answer: (B)

380. All of the following causes of death is coarctation of aorta except :

(A)  Infective endocarditis

(B)  CCF

(C)  Intracranial hemorrhage

(D)  Anterior MI

Answer: (D)

381. Which is true about carotid stenosis

(A)  Ipsilateral hemiplegia by embolism of MCA

(B)  Bruit indicates severity of stenosis

(C)  Common in ECA

(D)  Aspirin reduces risk of TIA

Answer: (D)

382. Hypercalciuria is seen in :

(A)  Hyperparathyroidism

(B)  Vit D intoxication

(C)  Sarcoidosis

(D)  All

Answer: (D)

383. True about H. Pylori is all except :

(A)  It splits urea & produces ammonia to survive

(B)  Produces Gastric Ca

(C)  Gram –ve curved rods

(D)  Cag-A gene is not associated with risk – of duodenal ulcer

Answer: (D)

384. Penicillamine is commonly used in the management of following heavy metal poisoning except :

(A)  Arsenic

(B)  Copper

(C)  Lead

(D)  Mercury

Answer: (A)

385. Stress ulcers seen in burns are :

(A)  Curling’s ulcer

(B)  Cushing’s ulcer

(C)  Meleney’s ulcer

(D)  Rodent ulcer

Answer: (A)

386. Iso osmolar urine is seen in :

(A)  ATN

(B)  Severe dehydration

(C)  Diabetes insipidus

(D)  PCKD

Answer: (A)

387. ST depression & T wave inversion in V1 to V6 and aVL lead indicate :

(A)  Anterolateral wall AMI

(B)  Posterior wall AMI

(C)  Inferior wall AMI

(D)  Lateral AMI

Answer: (A)

388. Decreased osmotic fragility is seen in :

(A)  Hereditary spherocytosis

(B)  Sickle cells ds.

(C)  Autoimmune hemolytic anemia

(D)  Thalassemia

Answer: (D)

389. True about log does response curve of Vmax showing norepinephrine & propranolol is known that propranolol is a competitive antagonist of norepinephrine :

(A)  Curve paralles to each others action

(B)  Curve not parallel to each other

(C)  Cure shifted to right

(D)  Increased Vmax

Answer: (C)

390. Recurrent gross hematuria is seen in :

(A)  Alport’s syndrome

(B)  IgA nephropathy

(C)  Focal seg. GN

(D)  DM

Answer: (B)

391. Orthostatic proteinuria :

(A)  Seen in recumbent position

(B)  Is benign

(C)  Future risk of nephritic synd.

(D)  > 300 mg/day

Answer: (B)

392. Proved association of celiac sprue is with :

(A)  Dermatitis herpetiformis

(B)  Scleroderma

(C)  Pemphigus

(D)  Pemphigoid

Answer: (A)

393. True about complications of massive blood transfusion is/are a/e :

(A)  Hyperkalemia

(B)  Alkalosis

(C)  Acidosis

(D)  Hypercalcemia

Answer: (D)

394. Steps in meiosis about sister chromatid separation are :

(A)  Prophase → Metaphase → Anaphase → Telophase

(B)  Prophase → Telophase → Anaphase → Metaphase

(C)  Telephase → Anaphase → Prophase → Metaphase

(D)  Anaphase → Prophase → Metaphase → Telophase

Answer: (A)

395. Causes of large kidney in CRF are a/e :

(A)  DM

(B)  Amyloidosis

(C)  Benign nephrosclerosis

(D)  Diffuse GN

Answer: (C)

396. Trophic ulcers are seen in a/e :

(A)  Polio

(B)  Syringomyelia

(C)  Leprosy

(D)  TB. Meningitis

Answer: (A)

397. Chromosomes involved in adult polycystic kidney disease (APKD) :

(A)  6 & 11

(B)  4 & 16

(C)  4 & 16

(D)  4 & 12

Answer: (B)

398. Clubbing is seen in :

(A)  Suppurative pneumonia

(B)  L – R shunt

(C)  ABPA (Allergic bronchopulmonary aspergillosis)

(D)  Hepatorenal syndrome

Answer: (A)

399. Confirmatory investigation for acromegaly is :

(A)  Insulin induced hypoglycemia test

(B)  GH assay

(C)  ACTH infusion test

(D)  IGF levels

Answer: (B)

400. Mutation leading to sickle cell anemia :

(A)  Crossover mutation

(B)  Frameshift

(C)  Deletion

(D)  Point mutation

Answer: (D)

401. True about oxygen concentrators :

(A)  Extraction of O₂ from water

(B)  Requires electrical power

(C)  O₂ from halolite

(D)  Supplies 100 O₂

Answer: (B)

402. Extrahepatic manifestation of Hepatitis-B are :

(A)  Aplastic anemia

(B)  Cryoglobulinemia

(C)  Peptic ulcer

(D)  Gallstones

Answer: (B)

403. A patient in ICU has normal pulmonary pressure; low systemic peripheral resistance, low cardiac index; arterial Po2 = 93; diagnostic possibilities are :

(A)  Cardiogenic shock

(B)  Septic shock

(C)  Hypovolemic shock

(D)  Cardiac tamponade

Answer: (B)

404. True about HIV in pregnancy :

(A)  Perinatal transmission common

(B)  LSCS increases chances of transmission significantly

(C)  Zidovudine does not decreased risk if given to baby born to HIV positive mother

(D)  Less than 5% chance of transmission

Answer: (A)

405. ‘C’ wave in JVP corresponds to :

(A)  Atrial systole

(B)  Ventricular systole

(C)  Atrial diastole

(D)  Ventricular diastole

Answer: (B)

406. True of the following :

(A)  Interstitial nephritis is associated with salt losing nephropathy

(B)  Idiopathic hypercalciuria is most common cuase of hypercalcemia

(C)  Vitamin-D resistant rickets is a X-linked recessive disease

(D)  Calcitonin deficiency causes hyperphosphatemia

Answer: (A)

407. Which of the following HL is specific to rheumatoid arthritis :

(A)  HLADR₁

(B)  HLADR₂

(C)  HLADR₃

(D)  HLADR₄

Answer: (A)

408. True about hemoglobin degradation is/are:

(A)  Heme degradation is followed by biliverdin formation

(B)  Hemoglobin breaks into heme and protein

(C)  Heme is broken down into iron & porphyrin

(D)  Heme is broken into bilirubin directly

Answer: (A)

409. Blood for acid base gas (ABG) analysis in a bottle containing excess heparin can cause decreased value of :

(A)  pCO₂

(B)  HCO₂

(C)  pH

(D)  pO₂

Answer: (C)

410. Rheumatic heart disease can be diagnosed on the basis of :

(A)  Aschoff bodies

(B)  Vegetations along the lines of closure of valves

(C)  Endocardial involvement only

(D)  Follows skin and throat infection

Answer: (A)

411. Bronchial asthma can be diagnosed by :

(A)  Wheeze

(B)  Dyspnea

(C)  Normal FEV₁

(D)  Reversible obstruction

Answer: (D)

412. Difference between upper and lower limb blood pressures is usually :

(A)  5 mm

(B)  10 mm

(C)  20 mm

(D)  30 mm

Answer: (B)

413. Metabolic alkalosis is associated with :

(A)  Fanconi’s anemia

(B)  Acetazolamide

(C)  Spironoloactone

(D)  Hypocalcemia

Answer: (D)

414. Bartter’s syndrome is associated with :

(A)  Hyperkalemia

(B)  Decreased rennin level in plasma

(C)  Hypertension

(D)  Metabolic alkalosis

Answer: (D)

415. Best assay for deficiency of von Willebrand factor is :

(A)  Bleeding time

(B)  APTT

(C)  BT + APPTT

(D)  BT + APTT + vWF-ristocetin factor assay

Answer: (D)

416. Increased serum calcium is seen in all except :

(A)  Myxedema

(B)  Multiple myeloma

(C)  Sarcoidosis

(D)  Primary hyperparathyroidism

Answer: (A)

417. Myasthenia gravis associated with :

(A)  Decreased acetyl choline at nerve endings

(B)  Decreased myosin

(C)  Absent troponin C

(D)  Decreased myoneural junction transmission

Answer: (D)

418. The amino acid associated with atherosclerosis is :

(A)  Arginine

(B)  Homocysteine

(C)  Cysteine

(D)  Tryptophan

Answer: (B)

419. In spectroscopy, nucleotides absorb at 260 nm. This absorbence is due to:

(A)  Purines and pyrimidines

(B)  Deoxribose

(C)  Histone

(D)  Phosphate

Answer: (A)

420. Purpuric rashes are seen :

(A)  Dengue

(B)  Borrelia

(C)  Secondary syphilis

(D)  Measles

Answer: (A)

421. Causes of sterile pyuria without UTI include :

(A)  Cortical abscess

(B)  Pyelonephritis

(C)  Gonococcal infection

(D)  Analgesic nephropathy

Answer: (D)

422. HbA2 increased in:

(A)  Alfa-thalassemia

(B)  Iron deficiency anemia

(C)  Beta-thalassemia

(D)  Sickle cell trait

Answer: (C)

423. Non-irritant gas producing systemic toxicity is:

(A)  Ammonia

(B)  Hydrogen chloride

(C)  Carbon monoxide

(D)  Chlorine

Answer: (C)

424. Most common opportunistic infection in AIDS is:

(A)  Candida

(B)  Pneumocystis

(C)  Aspergillus

(D)  S. Aureus

Answer: (B)

425. Raised calcium and phosphorus are seen in :

(A)  CRF

(B)  Vitamin D intoxication

(C)  Hyperparathyroidism

(D)  Pseudohypoparathyroidism

Answer: (B)

426. Universal finding in asthma is :

(A)  Hypoxia

(B)  Hypercarbia

(C)  Hypoxemia

(D)  Respiratory acidosis

Answer: (A)

427. An obese NIDDM patient present with FBS=180 mg% and PP2BS=260 mg% Management include :

(A)  Glibenclamide

(B)  Diet therapy-exercise

(C)  Diet therapy-exercise+metformin

(D)  Insulin

Answer: (C)

428. Herpes simplex encephalitis diagnosed by :

(A)  MRI

(B)  Biopsy

(C)  Corneal scrapping and culture

(D)  CSF PCR of HSV DNA

Answer: (D)

429. Wide-split second heart sound :

(A)  ASD

(B)  LBBB

(C)  PDA

(D)  MR

Answer: (A)

430. A female taking alcohol is more prone to develop toxicity and cirrhosis compare with male when same amount of alcohol taken due to :

(A)  ↓ed alcohol dehydrogenase in stomach

(B)  ↑ed absorption

(C)  Congenital

(D)  Genetic factors

Answer: (A)

431. False positive ELISA for HIV :

(A)  Multiple mycloma

(B)  Lepromatous leprosy

(C)  SLE

(D)  None

Answer: (C)

432. Impaired oral GTT indicated by :

(A)  Fasting plasma sugar ≥ 126 mg/dl

(B)  Random blood sugar > 200 mg/dl

(C)  Fasting blood sugar < 90 mg/dl

(D)  2 hrs. after glucose load 140-200 mg/dl and fasting blood sugar < 126 mg/dl

Answer: (D)

433. PO2 level is ↓ in :

(A)  Anaemia

(B)  Co poisoning

(C)  Methaemoglobinemia

(D)  Hypoventilation

Answer: (D)

434. Celiae sprue diagnosed by :

(A)  Intestinal biopsy

(B)  Unequivocal response to gluten restriction

(C)  Finding or organism

(D)  Improvement on dapsome treatment

Answer: (B)

435. In cardiae shock :

(A)  SBP < 90 mm of Hg

(B)  DBP < 80 mm of Hg

(C)  Urine output < 20 ml/hr

(D)  Cardiae index < 3.5

Answer: (A)

436. Localized regional cerebral atrophy is seen in :

(A)  Alzheimer’s disease

(B)  Frontotemporal dementia

(C)  PML

(D)  C.J. disease

Answer: (B)

Medical PG SPM

Medical PG SPM

1. Kuppuswami’s classification is based on all except

(A) Income

(B) Occupation

(C) Education

(D) Type of house

Answer: (D)

2. Which of the following is a Live vaccine?

(A) Salk polio vaccine

(B) HDCV

(C) Hepatitis B vaccine

(D) 17-D vaccine

Answer: (D)

3. In Longitudinal studies, which of the following is true?

(A) Easy to conduct

(B) Detects only one risk factor

(C) Determines incidences of disease

(D) Have increased bias

Answer: (C)

4. All of the following are features of Cross-Over studies except

(A) All patients receive the new therapy

(B) Not suitable if drug on interest cures the disease

(C) Not suitable if drug is effective only during a certain stage of disease

(D) Not suitable if disease does not change radically during the study

Answer: (D)

5. Which of the following is true about Secular trend?

(A) Road side accident is a good example

(B) Due to environmental factors

(C) Is due to normally occurring variation in herd immunity

(D) Consistent change in a particular direction over a period of time

Answer: (D)

6. Which of the following is not true of Case-control studies?

(A) Relatively cheap

(B) Relative risk can be calculated

(C) Used for rare disease

(D) Odd’s ratio can be calculated

Answer: B(B)

7. The health status of children between 0 and 4 years in a community will be adversely affected by all except

(A) Malnutrition

(B) Birth weight of less than 2.5 Kg

(C) Maternal hemoglobin 12 gm/dL

(D) Infections

Answer: (C)

8. In a epidemic of Poliomyelitis, the best way to stop spread is by

(A) Injection of killed vaccine

(B) OPV drops to all children

(C) Isolation of cases

(D) Chlorination of all wells

Answer: (B)

9. Chandler index is used for

(A) Ancylostoma duodenale

(B) Ascariasis

(C) Guinea worm infestations

(D) Filariasis

Answer: (A)

10. All of the following are true bout Classical dengue fever except

(A) Case fatality is low

(B) Break bone fever

(C) Positive tourniquet test

(D) It is a self limiting disease

Answer: (C)

11. In India, the vector of Japanese encephalitis is

(A) Aedes, aegypti

(B) Aedes culcifaciens

(C) Culex vishnui

(D) Mansonoides

Answer: (C)

12. Under NMEP, the function of fever depot treatment is

(A) Diagnosis of cases + spraying

(B) Collection of slides + treatment of fever

(C) Treatment of fever cases only

(D) Treatment + slide collection + spraying

Answer: (B)

13. Under the NMEP, the minimum annual blood examination rate should be

(A) 10%

(B) 12%

(C) 14%

(D) 18%

Answer: (A)

14. All of the following are commonly used Filarial indices except

(A) Microfilaria rate

(B) Filarial disease rate

(C) Filarial endemicity rate

(D) Mosquito infection rate

Answer: (B)

15. The safe limit of Fluorine in drinking water is

(A) 0.2-0.5 mg/L

(B) 0.5-0.8 mg/L

(C) 0.8-1.2 mg/L

(D) 1.2-2.0 mg/L

Answer: (B)

16. Rabies can be transmitted by all the following routes except

(A) Aerosol

(B) Bites

(C) Ingestion

(D) Licks

Answer: (C)

17. The number of dose of HDCV for pre-exposure prophylaxis of Rabies is

(A) 1

(B) 3

(C) 5

(D) 6

Answer: (B)

18. The following disease require isolation to break transmission except

(A) Measles

(B) Mumps

(C) Chicken pox

(D) Tetanus

Answer: (D)

19. Still birth rate includes babies dead after

(A) 20 weeks

(B) 24 weeks

(C) 28 weeks

(D) 32 weeks

Answer: (C)

20. Protein efficiency ratio is

(A) Gain in weight per unit of protein consumed

(B) Product of digestibility coefficient and biological value divided by 100

(C) Percentage of nitrogen retained out of the nitrogen absorbed

(D) Percentage of protein absorbed after digestion

Answer: (A)

21. Highest biological value of protein is seen in

(A) Egg

(B) Fish

(C) Soyabean

(D) Gram

Answer: (A)

22. In a demographic cycle, Late expanding stage indicates

(A) High birth and death rates

(B) Decreasing DR and stationary BR

(C) Decreasing BR and decreasing DR

(D) Low DR and BR

Answer: (C)

23. Under the registration Act of 1969, Birth should be registered within

(A) 7 days after birth

(B) 20 days after birth

(C) 14 days after birth

(D) 21 days after birth

Answer: (D)

24. All the following are Standards laid down by the Factories Act except

(A) Minimum 500 cubic feet space per worker

(B) Maximum working hours including overtime per week : 60 hours

(C) Defines any factory that employs 10 more workers where power is not used

(D) Prohibition of employment of children below 14 years of age

Answer: (C)

25. Controlled Tipping is a method of disposal of

(A) Sewage

(B) Sullage

(C) Human excreta

(D) Refuse

Answer: (D)

26. Sputum examination for AFB is a type of

(A) Primary prevention

(B) Secondary prevention

(C) Tertiary prevention

(D) Primordial prevention

Answer: (B)

27. Case-control studies are used for

(A) Study of common diseases

(B) Finding multiple risk factors

(C) Finding incidence rates

(D) Finding Relative risk

Answer: (B)

28. Anthropo-zoonosis are all except

(A) Guinea worm infestation

(B) Rabies

(C) Plague

(D) Hydatid cyst

Answer: (A)

29. Which of the following is a zoonotic disease?

(A) Hydatid cyst

(B) Malaria

(C) Filariasis

(D) Dengue fever

Answer: (A)

30. Which vaccine is given earliest? 

(A) BCG

(B) Hepatitis

(C) DPT

(D) Measles

Answer: (A)

31. Which of the following is false about whooping cough?

(A) Affects children 1 years of age

(B) Contagious in the catarrhal stage

(C) Carriers are the most important source of infection

(D) Secondary attack rate is high

Answer: (C)

32. Contraindications of Pertussis DPT vaccines are all except

(A) Family history of epilepsy

(B) Any febrile illness

(C) Recent history of infectious disease

(D) Children aged below 2 years of age

Answer: (D)

33. In NMEP, the recommendation for an area with API>2 are all except

(A) Presumptive treatment

(B) DDT spraying regularly throughout the year

(C) Epidermiological investigation of all cases

(D) Follow-up of every case for one year and monthly blood smears

Answer: (D)

34. Which of the following is not true about the Tuberculin test?

(A) It is the only tool available for estimating the prevalence of TB in a community

(B) Induration of 10 mm indicates

(C) It is a specific test

(D) New cases occur more commonly in patients who are tuberculin reactors

Answer: (C)

35. Pre-exposure prophylaxis for Rabies with HDVC involves

(A) 3 doses

(B) 5 doses

(C) 6 doses

(D) 4 doses

Answer: (A)

36. Disease in which herd immunity does not protect an individual is

(A) Measles

(B) Tetanus

(C) Polio

(D) Diphtheria

Answer: (B)

37. Disease caused by mite is

(A) Scrub typhus

(B) Fever

(C) Trench fever

(D) Endemic typhus

Answer: (A)

38. Rat flea transmits all the following except

(A) Plague

(B) Salmonellosis

(C) Hymenolopsis

(D) Endemic typhus

Answer: (B)

39. India belongs to which stage of the demographic cycle

(A) Slow stationary

(B) High stationary

(C) Early expanding

(D) Late expanding

Answer: (D)

40. The growth pattern of a population with an annual growth rate of 1.5% to 2.0% is

(A) Slow growth

(B) Moderate growth

(C) Rapid growth

(D) Very rapid growth

Answer: (D)

41. An IQ between 50& 70 would be classified as what kind mental retardation

(A) Mild

(B) Moderaten

(C) Severe

(D) Borderline

Answer: (A)

42. Niacin deficiency in a maize-eating population is due to

(A) High Tryptophan

(B) High Isoleucine

(C) High leucine

(D) High Phenylalanine

Answer: (C)

43. Which of the following drugs is supposed to prevent congenital neural tube defects?

(A) Thiamine

(B) Riboflavin

(C) Folic acid

(D) Pyridoxine

Answer: (C)

44. The earliest change in iron deficiency anemia is

(A) Decreased serum Iron

(B) Decreased serum Ferritin

(C) Decreased TIBC

(D) Decreased Hemoglobin

Answer: (B)

45. The national health policy is based on

(A) Comprehensive health care

(B) Subsidized health care

(C) Socialized medicine

(D) Equitable distribution of health resources

Answer: (A)

46. Two variables can be plotted together in

(A) Pie chart

(B) Histogram

(C) Frequency polygon

(D) Scatter diagram

Answer: (D)

47. The number of degrees of freedom in a table of (4×4) is

(A) 4

(B) 8

(C) 9

(D) 16

Answer: (C)

48. The screening method of choice in an area where the prevalence of leprosy is 1/1000 is

(A) Contact survey

(B) Group survey

(C) Mass survey

(D) Any of the above

Answer: (B)

49. The longest and shortest incubation period of plague are 7 days and 3 days respectively. The time necessary to declare an area free from plague is

(A) 2 days

(B) 5 days

(C) 14 days

(D) 21 days

Answer: (C)

50. All of the following statements are true about Prevalence expect

(A) It is a rate

(B) It includes both new and old cases

(C) Prevalence = Incidence × Duration

(D) Prevalence is of more than one type

Answer: (A)

51. Berkesonian bias refers to

(A) Bias arising from different rates of admission to the hospital

(B) Bias arising from presence of confounding factors

(C) Bias arising from the cases not being representative of the general population

(D) Bias arising from improper selection of cases

Answer: ()

52. Which of the following is true of people participating in a double blind study?

(A) Participant is not aware of the study or the control group

(B) Neither the doctor nor the participants are aware of the group allocation and the treatment received

(C) The participants, the investigator and the person analyzing the data are all blinded

(D) None of the above

Answer: (B)

53. Which of the following is not a feature of a cross-sectional study?

(A) Not expensive

(B) Observational study

(C) Used for chronic diseases

(D) Well suited for establishing inter-relationships of diseases

Answer: (D)

54. In a case-control study, the most characteristic feature is

(A) Odds ratio estimation

(B) Problem bias

(C) Yield incidence rate

(D) Expensive

Answer: (A)

55. A study of lung carcinoma in non-smokers is

(A) Unifactorial

(B) Multifactorial

(C) Shows that passive smoking also increases the risk of cancer

(D) Shows that bidi-smokers have a higher incidence of lung cancer than cigarette-smokers

Answer: (B)

56. Live attenuated vaccines are all expect

(A) BCG

(B) Salk

(C) Sabin

(D) Measles

Answer: (B)

57. True about measles are all expect

(A) Secondary attack rate is 30%

(B) Fever occurs after 7-10 days of infection

(C) Immunity develops after 7 days of vaccination

(D) Single dose of vaccine gives 95% protection

Answer: (A)

58. All the following are true about Diphtheria except

(A) Incubation period-2-6 days

(B) Carriers can be prevented by immunization

(C) Schick test detects susceptibility

(D) Portal of entry is through an infective agent

Answer: (B)

59. The highest incidence of Dracuncliasis is in

(A) Rajasthan

(B) Tamil Nadu

(C) Punjab

(D) Karnataka

Answer: (A)

60. The following statement stands true for Mantoux test

(A) Test is read before 48 hrs

(B) 6-9 mm induration shows maximum chances of developing TB

(C) A positive test does not indicate that the person is suffering from disease.

(D) New cases are more likely to occur in tuberculin negative person than those who already are tuberculin reactos

Answer: (C)

61. The schedule of HDCV in rabies is

(A) 0, 3, 7, 14, 28

(B) 0, 3, 10, 30

(C) 3, 7, 14, 16, 18

(D) 0, 7, 14, 30

Answer: (A)

62. All the following are true about Japanese B Encephalitis except

(A) Man to man transmission is not reported

(B) Culex mosquito is the vector

(C) 90%-100% mortality rate

(D) Cases represent only tip of the iceberg

Answer: (C)

63. Which of the following statements about Q fever is false

(A) Highly infectious zoonotic disease

(B) The mode of transmission is by Inhalation

(C) Arthropods are involved in its transmission to man

(D) There is no rash or local lesion

Answer: (C)

64. Rickettsial disease is transmitted by all expect

(A) Epidemic typhus – Louse

(B) Endemic typhus – Flea

(C) Q-fever – C. burnettii

(D) Rickettsial pox − Tic

Answer: (D)

65. Mite causes

(A) Scrub typhus

(B) Epidermic typhus

(C) Endemic typhus

(D) None of these

Answer: (A)

66. The perinatal death rate in India is

(A) Late fetal death (still-birth) + early neonatal death (under 1 week)

(B) Late fetal death (stillbirth) + early neonatal deaths (under 4 weeks)

(C) Late fetal and early death of neonates weighing over 1000g at birth

(D) Late fetal and early death of neonates weighing over 1500g at birth

Answer: (A)

67. Total fertility rate is

(A) Completed family size

(B) Women between 15-45 years

(C) Married women between 15-45 years

(D) Unmarried women between 15-45 years

Answer: (A)

68. Denominator of maternal mortality rate is

(A) 1000 live births

(B) Mid-year population

(C) 1000 total births

(D) Total live births

Answer: (A)

69. Which of the following is true about the child-women ratio?

(A) 0-4 years of children and 15-45 years of unmarried women

(B) 0-5 years of children and 15-49 years of age of married woman

(C) 0-4 year of children and 15-44 years of all women

(D) 0-4 years of children and 20-30 years of married women

Answer: (C)

70. Calorie requirement at 1 year of age is

(A) 800

(B) 1000

(C) 1200

(D) 1500

Answer: (C)

71. BOAA is

(A) Beta-oxalyl amino alanine

(B) Beta-oxalo acetoacetate

(C) Beta-oxalyl aminoacetate

(D) Beta-oxalyl acetylacetate

Answer: (A)

72. In Niacin deficiency, all of the following are seen expect

(A) Deafness

(B) Diarrhea

(C) Dermentia

(D) Dermatitis

Answer: (A)

73. All of the following statements are true about bleaching powder expect

(A) It is a stable compound

(B) Available chlorine is 33%

(C) Has a Rapid and brief action

(D) Recommended concentration of free chlorine is 0.5 mg at the end of 1 hour

Answer: (A)

74. Confidence limit for a S.D. of 1.96 equals

(A) 63.6%

(B) 66.6%

(C) 95%

(D) 99%

Answer: (C)

75. Weight in Kg is

(A) Normal variable

(B) Discrete variable

(C) Confounding variable

(D) Continuous variable

Answer: (D)

76. All the following are examples of Nominal scale except

(A) Race

(B) Sex

(C) Iris color

(D) Blood pressure

Answer: (D)

77. The most important step in health education of a community is

(A) Contact with doctors

(B) Community discussion

(C) Announcements to the community by loud speakers

(D) Knowledge of local needs

Answer: (D)

78. Which of the following is not a features of Mass-Media education?

(A) Deals with local problems of the community

(B) Easily understandable

(C) Wide approach

(D) More effective than individual or group method

Answer: (D)

79. All of the following statements about cross sectional studies are true except :

(A) Many outcomes are possible

(B) Lesser time is required

(C) Relationship between cause and effect can be established

(D) Less expensive

Answer: (C)

80. True statement about case fatality rate is

(A) It is a ratio

(B) It is a proportion

(C) Numerator is always constant

(D) Numerator and denominator are two separate entities

Answer: (A)

81. Death rates of two countries are best compared by :

(A) Crude death rate

(B) Proportional crude death rate

(C) Standarised mortality rate

(D) Age specific death rate

Answer: (C)

82. All of the following are characteristics of case control study except :

(A) Quick results are obtained

(B) Measures incidence rate

(C) Proceeds from effect to cause

(D) Inexpensive study

Answer: (B)

83. Sensitivity is :

(A) True positive 

(B) True negative 

(C) False negative 

(D) False positive 

Answer: (A)

84. Morbidity is measured by :

(A) Active surveillance 

(B) Passive surveillance 

(C) Sentinel surveillance 

(D) Continuous surveillance 

Answer: (A)

85. Most sensitive index of recent transmission of malaria is :

(A) Spleen rate 

(B) Infant parasite rate 

(C) Annual parasite rate 

(D) Parasite density rate 

Answer: (B)

86. All of the following arboviral infections are prevalent in India except :

(A) Dengue fever 

(B) Westnile fever 

(C) Yellow fever 

(D) Chikungunya

Answer: (C)

87. Epidemiologial features true for Japanese B-encephalitis include all of the following except

(A) The virus infects extra-human hosts 

(B) Man is an incidental host 

(C) Infected pigs manifest symptoms of encephalitis 

(D) Epidemics have been reported in Karnataka 

Answer: (C)

88. True statement about plague is

(A) Vaccine is not used to control epidemic of preneumonic plague 

(B) Most important measure to control epidemic rodent control 

(C) Septicemic plague is highly infectious 

(D) Live attenuated vaccines are used 

Answer: (B)

89. Measures of primary prevention of hypertension include all of the following except :

(A) Weight reduction 

(B) Exercise promotion 

(C) Reduction of salt intake 

(D) Early diagnosis of hypertension 

Answer: (D)

90. Annual growth rate is :

(A) Crude birth rate-crude death rates 

(B) Crude death rate-crude birth rates 

(C) Crude birth rate-crude death rate x 100/crude birth rate 

(D) Crude birth rate-crude death rate X 100/Mid year population 

Answer: (A)

91. The goal of NRR = 1 can  be best achieved by use of following contraceptive methods :

(A) Condom 72/year 

(B) IUCD

(C) Vasectomy

(D) Oralpills

Answer: (C)

92. According to ICMR the most common cause of infant mortality is :

(A) Prematurity

(B) Diarrhoeal diseases

(C) Congenital anomalies 

(D) Acute respiratory infection 

Answer: (A)

93. All of the following features contribute to high risk babies except :

(A) Babies with working mothers 

(B) Spacing of less than 1 year

(C) Artificial fed babies 

(D) Weight between 70-80% of reference wt. 

Answer: (D)

94. Vitamin B12 is not found in :

(A) Soyabean

(B) Milk

(C) Meat

(D) Fish

Answer: (A)

95. First indices to change in iron deficiency anemia is :

(A) S. iron 

(B) Total iron binding capacity

(C) S. ferritin 

(D) S. haemoglobin concentration 

Answer: (C)

96. Maize are deficient in :

(A) Tryptophan

(B) Threonine

(C) Methionine

(D) Leucine

Answer: (A)

97. According to ICMR project, requirements during lactation include as extra :

(A) 300 Calories 

(B) 550 Calories 

(C) 700 Calories 

(D) 900 Calories 

Answer: (B)

98. Sanguinarine is derived from :

(A) Fusorium incamatum 

(B) Argemone oil 

(C) Jhunjhujia seeds 

(D) Khesari-dhal 

Answer: (B)

99. Mid-day meals provided in schools provide :

(A) 1/2 of total calories & 1/2 of protein 

(B) 1/3 of total calories & 1/2 of protein 

(C) 1/2 of total calories & 1/3 of protein 

(D) 1/3 of total calories & 1/4 of protein 

Answer: (B)

100. All of the following are acceptable criteria for water quality except :

(A) Dissolved O₂ of 5 mg/L 

(B) Nitrates of 1 mg/L 

(C) Nitrites of 5 mg/L 

(D) Free & saline NH₃ of 0.05 mg/L

Answer: (C)

101. Mites are known to transmit :

(A) Scabies

(B) Relapsing fever 

(C) KFD

(D) Human babesiosis 

Answer: (A)

102. Malathion is used once every :

(A) 3 months 

(B) 6 months

(C) 9 months 

(D) 12 months 

Answer: (A)

103. Area falling under 2 S.D. curve would be around

(A) 66% 

(B) 95% 

(C) 57% 

(D) 99% 

Answer: (B)

104. Square root mean of deviation is also called as :

(A) Standard deviation 

(B) Standard error 

(C) Mean deviation 

(D) Range

Answer: (A)

105. Health education :

(A) Is health promotion 

(B) Is health distortion 

(C) Is delivered through public health agencies only 

(D) Does not help in prevention of cancer 

Answer: (A)

106. All of the following are components of Epidemiological triad except :

(A) Environment

(B) Agent

(C) Host

(D) Manpower

Answer: (D)

107. All of the following statements regarding ‘prevalence’ are true except :

(A) It is Rate 

(B) It is ratio 

(C) Duration of disease affects ‘prevalence’ 

(D) Numerator and denominator are separate entities 

Answer: (A)

108. All of the following features are true about cross sectional study except :

(A) All cases are seen at one point in time 

(B) Follow up is not a necessary feature 

(C) More useful for chronic disease 

(D) Cause and effect relationship can be established 

Answer: (D)

109. Incidence rate is measured by :

(A) Case control study 

(B) Cohort study 

(C) Cross sectional study 

(D) Cross over study 

Answer: (B)

110. In a cohort study nonsmokers were found to have carcinoma lung. This indicates :

(A) Smoking does not cause lung cancer 

(B) Etiology for lung Ca is multifactorial 

(C) Smoking is the only cause of lung cancer 

(D) Passive smoking also increases lung cancer 

Answer: (B)

111. All of the following features are true about case control study except

(A) Rapid and inexpensive 

(B) No risk to subjects 

(C) Risk factor can be identified 

(D) Less prone to bias 

Answer: (D)

112. All are true about measles except :

(A) Incubation period is 10-14 days 

(B) Secondary attack rate is 30-40% 

(C) Subcutaneous vaccine is available 

(D) Vaccine is live attenuated 

Answer: (B)

113. In rabies, human diploid cell culture vaccine for post-exposure vaccination is given on the following days :

(A) 0, 7, 28 then booster dose 90 days 

(B) 0, 7, 28 then booster dose in 2 years 

(C) 0, 3, 7, 14, 30 then booster dose 90 days 

(D) 0, 3, 7, and booster dose 90 days 

Answer: (C)

114. True statement regarding ‘tuberculine test’ is :

(A) Used for diagnosis of T.B 

(B) Measures incidence of disease 

(C) More than 10 mm induration in 72 hrs indicates positive test 

(D) Measure immunity status 

Answer: (C)

115. Scrub typhus is transmitted by :

(A) Mite

(B) Louse

(C) Flea

(D) Tick

Answer: (A)

116. According to 1991 census family size is :

(A) 2.4 

(B) 3.9 

(C) 4.4 

(D) 5.6 

Answer: (B)

117. True statement about ‘total fertility rate’ is :

(A) Indicates approximate magnitude of completed family size 

(B) Numerator is ‘woman of child bearing age’ 

(C) Numerator is ‘all live births’ 

(D) Numerator is ‘married woman of child bearing age’ 

Answer: (A)

118. Denominator of maternal mortality rate is :

(A) 1000 total births 

(B) Mid year population 

(C) 1000 live births 

(D) Total live births 

Answer: (C)

119. All of the following statements hold true for perinatal mortality rate except :

(A) Deaths of neonate within one weak are included 

(B) Late still births and deaths beyond 28 weeks are included 

(C) Deaths of children weighing more than 1000 gm is are included

(D) ‘Total number of birth’ forms the denominator 

Answer: (D)

120. Most common cause of maternal mortality in India is :

(A) Anemia

(B) Haemorrhage

(C) Sepsis

(D) Abortion

Answer: (B)

121. Minimum duration of developing coal minor pneumoconiasis is :

(A) 2-4 years 

(B) 4-6 years 

(C) 8-10 years 

(D) More than 10 years 

Answer: (D)

122. Preplacement examination in dye industry includes all of the following except :

(A) Anemia

(B) Asthma

(C) Precancerous lession 

(D) Dermatitis

Answer: (A)

123. All of the following statements relating to lung carcinoma and occupation are true except :

(A) Risk is increased 

(B) Takes long time to develop

(C) It takes less time to develop as compare to general population 

(D) It takes more time develop as compare to general population 

Answer: (D)

124. S.D. is 1.96 the confidence limit is :

(A) 63.6% 

(B) 66.6% 

(C) 95% 

(D) 99% 

Answer: (C)

125. Values are arranged in ascending and descending order to calculate :

(A) Mode

(B) Mean

(C) Median

(D) Standard deviation

Answer: (C)

126. For right sided skewed deviation, true statement would be :

(A) Median is smaller than mean 

(B) More outlying observation are smaller than the rest of the values 

(C) “Tail” to the left 

(D) Not affected at all 

Answer: (A)

127. All are included in health sector policy in India except :

(A) Nutritional supplements 

(B) Medical education 

(C) Family welfare programme 

(D)

Answer: (A)

128. All are true about bleaching powder except :

(A) Has 33% available chlorine 

(B) Is a Stable compound 

(C) Has a rapid and brief action 

(D) Is used as disinfectant of wells 

Answer: (B)

129. All are gross root level workers except :

(A) Anganwadi workers 

(B) Traditional birth attendent 

(C) Village health guide 

(D) Health assistants 

Answer: (D)

130. All of the following statements are true about mass media education except :

(A) Rapid and speedy 

(B) High reach 

(C) Distorted information 

(D) Meets local community needs 

Answer: (C)

131. Indicators of ‘physical quality of life index’, includes all of the following except :

(A) Infant mortality 

(B) Life expectancy at age one 

(C) Literacy 

(D) Percapita gross national product 

Answer: (D)

132. Best comparision of health status of two population is by :

(A) Crude death rate 

(B) Proportional crude death 

(C) Standarized mortality rate 

(D) Specific death rate 

Answer: (C)

133. Study of time, place and person is called as :

(A) Experimental epidemiology 

(B) Analytical epidemiology 

(C) Descriptive epidemiology 

(D) Randomised controlled trial 

Answer: (C)

134. Bhopal gas tragedy is an example of :

(A) Point source epidemic 

(B) Propagated epidemic 

(C) Continous epidemic 

(D) Modern epidemics 

Answer: (A)

135. Reconstituted measles vaccine should be used with in :

(A) 1 hour 

(B) 3 hour 

(C) 6 hour 

(D) 12 hour 

Answer: (A)

136. The diluents used for BCG is :

(A) Distilled-water 

(B) Normal saline 

(C) Dextrose solution 

(D) Ringer-lactate 

Answer: (B)

137. Active immunization following exposure is given most commonly for :

(A) Rabies

(B) Polio

(C) Plague

(D) Measles

Answer: (A)

138. All of the following rabies vaccines are commercially available except :

(A) Killed sheep brain vaccine 

(B) Human diploid cell vaccine 

(C) Vero continous cell vaccine 

(D) Recombinant glycoprotein vaccine

Answer: (D)

139. Following facts true stand for coronary heart disease in India, except :

(A) CHD presents a decade later than in western countries 

(B) Diabetes mellitus in the commonest cause 

(C) More common in males than females 

(D) Heavy smoking is an etiological factor 

Answer: (A)

140. Commonest cause of infant mortality rate in India is

(A) Prematurity

(B) Diarrhoea

(C) Respiratory infection 

(D) Congenital malformation 

Answer: (A)

141. Extra calorie requirements for a lactating mother are :

(A) 300 Kcal/day 

(B) 400 Kcal/day 

(C) 550 Kcal/day 

(D) 600 Kcal/day 

Answer: (C)

142. Which of the following trace element cannot be completely supplemented by diet in pregnancy?

(A) Fe

(B) Ca⁺⁺ 

(C) Zn

(D) Manganese

Answer: (A)

143. Colostrum is rich in the following constituents as compared to breast milk :

(A) Minerals

(B) Proteins

(C) Fats

(D) Carbohydrates

Answer: (B)

144. The child survival and safe mother hood (CSSM) programme includes all except :

(A) Essential newborn care 

(B) Acute respiratory disease control 

(C) Nutrition supplementation 

(D) Universal immunization 

Answer: (C)

145. In reproductive and child health programme, districts are divided on the basis of

(A) Crude birth rate & infant mortality rate 

(B) Crude birth rate & female literacy rate 

(C) Crude death rate & crude birth rate 

(D) Couple protection rate & infant mortality rate 

Answer: (B)

146. ‘NALGONDA technique’ is used in :

(A) Endemic-fluorosis 

(B) Epidemic-dropsy 

(C) Endemic-ascites 

(D) Neurolathyrism 

Answer: (A)

147. Post contamination of water is indicated by :

(A) Free and saline ammonia 

(B) Dissolved H₂S 

(C) Nitrites

(D) Nitrates

Answer: (D)

148. Osmolality of WHO-ORS is :

(A) 240

(B) 280

(C) 300

(D) 330

Answer: (D)

149. Under the registration act of 1969, Death is to be registered with in :

(A) 7 days 

(B) 14 days 

(C) 21 days 

(D) 28 days 

Answer: (A)

150. Confidence limit includes :

(A) Range & standard deviation 

(B) Median and standard error

(C) Mean and standard error 

(D) Mode and standard deviation 

Answer: (C)

151. 95% of confidence limit exist between :

(A) ±1.S.D 

(B) ±2.S.D 

(C) ±3.S.D 

(D) ±4.S.D 

Answer: (B)

152. Predictive value of a positive test is defined as :

(A) True + ve/true +ve + false –ve × 100 

(B) True + ve/true +ve + false +ve × 100 

(C) False + ve/true + ve + false + ve × 100 

(D) False + ve/true +ve + false –ve × 100 

Answer: (B)

153. Population covered by a PHC in hilly region is :

(A) 20,000 

(B) 30,000 

(C) 40,000 

(D) 25,000 

Answer: (A)

154. Difference between Dispensary and PHC is that a PHC

(A) Gives integrated services 

(B) Is confined to particular area 

(C) Is managed by medical officer 

(D) Is located in rural areas 

Answer: (A)

155. Retrospective evaluation of medical performance is known as :

(A) Medical audit 

(B) Medical evaluation 

(C) Performance evaluation 

(D) Professional screening 

Answer: (A)

156. Sickness benefit under ESI is a available for a period of :

(A) 30 days 

(B) 46 days

(C) 56 days 

(D) 86 days 

Answer: (D)

157. ‘Sanitation barrier’ implies :

(A) Segregation of faeces 

(B) Personal hygiene 

(C) Elimination of flies 

(D) Water pollution 

Answer: (A)

158.  “Hospice” refers to :

(A) Group of professionals who provide counseling to families 

(B) “Colostomy bag” provision by an association of colostomy patient in USAQ 

(C) Euthansia for cancer patient 

(D) Special group of people helping the old and terminally ill patients 

Answer: (D)

Medical PG Forensic

Medical PG Forensic

1. Which organ Putrefies last in females?

(A) Prostate

(B) Uterus 

(C) Brain

(D) Breast

Answer: (B)

2. Hyoid fracture is common in

(A) Hanging

(B) Strangulation

(C) Throttling

(D) Choking

Answer: (C)

3. In which of the following conditions is a person held responsible for a crime?

(A) Alcoholic paranoia 

(B) Delirium tremens 

(C) Voluntary drunkenness 

(D) Dhatura intoxication 

Answer: (C)

4. All the following are Cardiac poisons except

(A) Aconite

(B) Opium

(C) Oleander

(D) Nicotine

Answer: (B)

5. The most poisonous Mercury salt is

(A) Chloride 

(B) Oxide

(C) Cyanide

(D) Chromate

Answer: (A)

6. The earliest manifestations of Chronic lead poisoning include

(A) Colic acid Constipation 

(B) Encephalopathy

(C) Punctate basophilia

(D) Lower limb paralysis 

Answer: (A)

7. A sea snake is

(A) Neurotoxic

(B) Hemotoxic

(C) Musculotoxic

(D) All of the above 

Answer: (C)

8. Clinical features of Organophosphorus poisoning include all expect

(A) Pinpoint pupils 

(B) Marked miosis 

(C) Tachycardia

(D) Clinical picture simulating Bronchial asthma 

Answer: (C)

9. Leading questions are permitted only in

(A) Examination in chief 

(B) Cross examination 

(C) Dying declaration 

(D) Re-examination 

Answer: (C)

10. Mummification is due to

(A) Putrefaction

(B) Dessication

(C) Saponification

(D) Necrosis

Answer: (B)

11. Diatoms are

(A) Algae

(B) Parasites

(C) Bacteria

(D) Fungi

Answer: (A)

12. Gettler’s test is positive in

(A) Hanging

(B) Drowning

(C) Strangulation

(D) Firearm injury 

Answer: (B)

13. Brush burn is due to

(A) Friction injury 

(B) Firearm injury 

(C) Electrical injury 

(D) Graze abrasion 

Answer: (D)

14. Gastric lavage is contraindicated in poisoning with

(A) Sulphuric acid 

(B) Organophosphorus compounds 

(C) Arsenic

(D) Dhatura

Answer: (A)

15. All the following are features of chronic lead poisoning except

(A) Encephalopathy

(B) Burtonian line 

(C) Cutaneous blisters

(D) Constipation

Answer: (C)

16. ‘Macewan sign’ is seen in

(A) Alcoholic intoxication 

(B) Organophosphorus poisoning 

(C) Barbiturate poisoning 

(D) Dhatura poisoning 

Answer: (A)

17. Which of the following agents causes delirium?

(A) Opium

(B) LSD

(C) Cocaine

(D) Dhatura

Answer: (D)

18. All the following are characteristic of Dhatura poisoning except

(A) Delirium

(B) Diplopia

(C) Pin-point pupils 

(D) Dysphagia

Answer: (C)

19. Poisoning with all the following causes constriction of the pupils except

(A) Dhatura

(B) Morphine

(C) Organophosphorus compounds 

(D) Pontine hemorrhage 

Answer: (A)

20. Charas is

(A) Leaves of Cannabis Indica 

(B) Flowers of Cannabis Indica 

(C) Stem of Cannabis Indica 

(D) Resin exudates of Cannabis Indica 

Answer: (D)

21. Fragmented medullary hair are found in 

(A) Negros

(B) Mongols

(C) Caucasians

(D) Europeans

Answer: (A)

22. Cutis anserina is seen in

(A) Drowning

(B) Firearm injury 

(C) Hanging

(D) Strangulation

Answer: (A)

23. Pugilistic attitude is due to

(A) Lipolysis

(B) Protein coagulation 

(C)  Carbohydrate coagulation 

(D) Lipogenesis

Answer: (B)

24. Pond’s fractures are common in which age group?

(A) Children

(B) Adolescent

(C) Adult

(D) Old age 

Answer: (A)

25. Mc Naughten’s rule is for

(A) Criminal responsibility 

(B) Civil responsibility of the insane

(C) Testamentary capacity 

(D) Capacity of the witness 

Answer: (A)

26. Transvestism is

(A) Viewing sexual acts secretly 

(B) Wearing clothes of the opposite sex 

(C) Pleasure in inflicting pain 

(D) Discussion on pornography 

Answer: (B)

27. Mercury poison acts on

(A) Proximal convoluted tubule 

(B) Distal convoluted tubule 

(C) Loop of Henle 

(D) Collecting ducts 

Answer: (A)

28. Pralidoxime is not useful is poisoning with

(A) Edrophonium

(B) Malathion

(C) Parathion

(D) DFP 

Answer: (A)

29. Best method for identification of a person is :

(A) Dactylography

(B) Anthropometry

(C) Personal appearance after death 

(D) Colour change of hair 

Answer: (A)

30. Chromosomal defect is a feature amongst all of the following except :

(A) Inter sex 

(B) Concealed sex 

(C) Pseudo hermaphrodite 

(D) True hermaphrodite 

Answer: (B)

31. In a decomposed body the first sign seen is :

(A) Greenish dislocation of skin over right iliac fossa 

(B) Greenish dislocation of skin over left iliac fossa 

(C) Purplish black dislocation on face 

(D) Purplish black dislocation over lower extremity 

Answer: (A)

32. Postmortem caloricity is seen in :

(A) Strychinine poisoning 

(B) Organophosphorus poisoning 

(C) Dhatura poisoning 

(D) Ergot poisoning 

Answer: (A)

33. The poison that can be detected in h air long after death is :

(A) Lead

(B) Mercury

(C) Arsenic

(D) Cannabis

Answer: (C)

34. Arsenic poisoning presents with symptoms mimicking :

(A) Cholera 

(B) Dhatura poisoning 

(C) Barbiturates poisoning 

(D) Morphine poisoning 

Answer: (A)

35. Barbiturates excretion in urine may be increased by :

(A) Diuretics

(B) Alkalinization

(C) Acidification

(D) Dialysis

Answer: (B)

36. Urine appears ‘Liquid Gold’ in which poisoning

(A) Heavy metals (222-M) 

(B) Barbiturates

(C) Organophosphorous

(D) Lead poisoning 

Answer: (B)

37.  Absent fragmented medulla of hair is seen in all except :

(A) Mongols

(B) Negroid

(C) Causcasions

(D) Servocrotarians

Answer: (B)

38. Goose skin or cutis anserine seen in :

(A) Drowning

(B) Lightening

(C) Strangulation

(D) Fir arm injury 

Answer: (A)

39. Pugillistic attitudes is due to

(A) Lipolysis

(B) Protein coagulation 

(C) Carbohydrate coagulation 

(D) Lipogenesis

Answer: (B)

40. Pond’s # is common in :

(A) Children

(B) Adult

(C) Elderly

(D) Menopausal woman 

Answer: (A)

41. In Nulliparous, cervical opening is :

(A) Longitudinal

(B) Circular

(C) Transverse

(D) Fimbriated

Answer: (B)

42. Necrosis of Proximal convoluted tubules is caused by :

(A) Arsenic

(B) Phenol

(C) Alcohol

(D) Amanita phalloids 

Answer: (A)

43. Smell of bitter almonds is seen in poisoning with :

(A) Phosphorus

(B) Hydrocyanic acid

(C) Nitric acid

(D) Oxalic acid 

Answer: (B)

44. Haemodialysis may be used for each of the following poisonings except :

(A) Kerosine oil 

(B) Barbiturates

(C) Alcohol

(D) Aspirin

Answer: (A)

45. Sodium fluoride may be used for preservation of :

(A) Cyanide

(B) Arsenic

(C) Alcohol

(D) Urine

Answer: (C)

46. Suspended animation may be seen with :

(A) Electrocution

(B) Strangulation/hanging

(C) Drowning

(D) Burn

Answer: (A)

47. Paltauf’s haemorrhage may be seen in :

(A) Hanging

(B) Drowning

(C) Strangulation

(D) Carbon Monoxide poisoning 

Answer: (B)

48. Contre-coup injury is a feature of injury to :

(A) Brain

(B) Stomach

(C) Spleen

(D) Heart

Answer: (A)

49. ‘Lucid Interval’ may be seen in

(A) Intracerebral haemorrhage 

(B) Insanity

(C) Subdural haemorrhage 

(D) Alcohol intake 

Answer: (B)

50. BAL is used as an antidote poisoning by :

(A) Morphine

(B) Aconite

(C) Phenol

(D) Mercury

Answer: (D)

51. Karyotyping of foetus may be done from all of the following except :

(A) Lymphocyte

(B) Monocyte

(C) Aminocyte

(D) Fibroblast

Answer: (B)

52. Transvestism is :

(A) Wearing clothes of opposite sex 

(B) Touching ones own private parts to other 

(C) Desire for sexual intercourse with dead bodies 

(D) Orgasm from visualization part of the body of a woman 

Answer: (A)

53. Postmortem finding in carbon monoxide poisoning is:

(A) Cherry red blood 

(B) Intense cyanosis 

(C) Excessive salivation 

(D) Pin point pupil 

Answer: (A)

54. Most common consistent feature after a child has been taken out from pond died from drowning is:

(A) Water in lungs alveoli 

(B) Petechial haemorrhages 

(C) Washer woman skin 

(D) Cyanosis

Answer: (A)

55. Injury that comes under sec 320 IPC

(A) Abrasion over face 

(B) Nasal bone fracture 

(C) Epistaxis

(D) Lacerated wound over scalp 

Answer: (B)

56. Atropine is used for following except:

(A) Organophosphorus poisoning 

(B) Mushroom poisoning 

(C) Physostigmine overdose 

(D) Glaucoma

Answer: (D)

57. Rigor mortis in winter season in Northern India lasts for

(A) 24-48 HR 

(B) >24 HR 

(C) 2-3 days 

(D) >3 days 

Answer: (A)

58. In organophosphorous poisoning, following are seen except:

(A) Pupillary dilatation 

(B) Salivation

(C) Bronchospasm 

(D) Sweating

Answer: (A)

59. Which is excreted typically in lead poisoning?

(A) Urobilinogen

(B) Coproporphyrin

(C) Bilirubin

(D) Bile salts

Answer: (B)

60. Maggots in rainy day are seen within

(A) 6 hours 

(B) 1-2 days 

(C) 2-4 days 

(D) 5-6 days 

Answer: (C)

61. Adipocere formation is characterized by all except:

(A) Hydrolysis and hydrogenation of fat 

(B) Foul smell 

(C) Cheesy odour 

(D) Develops in dampy clay soil in whole body

Answer: (C)

62. Burn (heat) haematoma is seen between:

(A) Skull and dura 

(B) Brain and arachnoid

(C) Arachnoid and Pia

(D) Pia and dura 

Answer: (A)

63. Putrefaction is facilitated by following except :

(A) Very high temp 

(B) Free air 

(C) Damp environment 

(D) Shallow grave 

Answer: (A)

64. Heavy metal poisoning causing PCT necrosis is:

(A) Cadmium

(B) Lead

(C) Mercury

(D) Gold

Answer: (C)

65. Antemortem and postmortem blisters can be differentiated by :

(A) Size

(B) Colour of blister fluid 

(C) Chloride & albumin content 

(D) PM blister is dry 

Answer: (C)

66. Postmortem caloricity is :

(A) ↑ed temp within 2 hours of death 

(B) Rapid fall in temp after death 

(C) ↑ed temp after 6-8 hours of death 

(D) Decreases in temperature followed by rise 

Answer: (A)

67. Lacerated wound looks like incised wound over:

(A) Scalp

(B) Abdomen

(C) Thigh

(D) Forearm

Answer: (A)

68. In aluminium phosphide poisoning, not true is:

(A) Accumulation of acetylcholine at NM junction 

(B) Cytochrome oxidase 

(C) Phosphine formation 

(D) Metabolic acidosis 

Answer: (A)

69. Diatom test is to determine death due to:

(A) Drowning

(B) Strangulation

(C) Hanging

(D) Burns

Answer: (A)

70. Suspended animation is defined as:

(A) Apparent death from which person can be aroused 

(B) True death from which person can not be aroused 

(C) May last for days or weeks 

(D) Can not be produced voluntarily 

Answer: (A)

71. First internal sign of putrefaction is found :

(A) Below the liver 

(B) In mediastinum 

(C) Below the spleen 

(D) In kidney 

Answer: (A)

72. Most common type in dactylography is :

(A) Wholrls

(B) Loops

(C) Composite

(D) Arches

Answer: (B)

73. Hanging causes large amount of injury to :

(A) Vertebral artery 

(B) Carotid A 

(C) Trachea

(D) Oesphagus

Answer: (C)

74. Most specific and sensitive for identification is:

(A) Anthropometry

(B) Dactylography

(C) Skull

(D) Pelvis

Answer: (B)

75. Rigor mortis starts in:

(A) Eyelids

(B) Heart

(C) Voluntary muscles 

(D) Limbs

Answer: (B)

76. Pugilistic attitude is due to :

(A) Coagulation of proteins 

(B) Cadaveric spasm 

(C) Rigor mortis 

(D) Coagulation of fats 

Answer: (A)

77. Testamentary capacity refers to :

(A) Ability to make a valid will 

(B) Criminal liability 

(C) Right to vote 

(D) Ability to give evidence 

Answer: (A)

78. Mc naughten’s rule refers to :

(A) Person is not responsible if he is not of a sound mind 

(B) Person is held responsible even if he is not of sound mind 

(C) Person is always held responsible 

(D) Any of the above 

Answer: (A)

79. The principal using nitrate in cyanide poisoning is to reduce :

(A) Methemoglobin production 

(B) Sufl Hb production 

(C) To correct metabolic abnormalities 

(D) Carboxy Hb production 

Answer: (A)

80. Specimens for toxicological studies is preserved in :

(A) 10% of formaldehyde 

(B) Alcohol

(C) Supersaturated solution of common salt 

(D) Normal saline 

Answer: (C)

81. The fetus can be considered viable if the following ossification center appears :

(A) Calcaneum

(B) Manubrium sterni 

(C) Lower end of femur 

(D) None of the above 

Answer: (C)

82. Marsh test is for detection of :

(A) Lead

(B) Strychnine

(C) Arsenic

(D) Opium

Answer: (C)

83. All are seen in drowning except :

(A) Miosis

(B) Wet heavy lungs 

(C) Water and weeds in stomach and lung 

(D) Froth from mouth 

Answer: (A)

84. Dhatura poisoning is characterized by :

(A) Pinpoint pupil 

(B) Dilated salivary gland 

(C) Dilated pupil with facial flush 

(D) Decrease temp 

Answer: (C)

85. Pin point pupil is seen in A/E :

(A) Pontine haemorrhage 

(B) Organo phosphorus poisoning

(C) Opium poisoning 

(D) Barbiturate poisoning 

Answer: (D)

86. Proximal tubule proteinuria and painful bone lesions are seen in overdose of :

(A) Cadmium

(B) Lead

(C) Mercury

(D) Phenol

Answer: (A)

87. Depressed # of skull results from :

(A) Heavy object with small striking surface 

(B) Heavy object with large striking surface 

(C) Fall over the ground 

(D) Light object 

Answer: (A)

88. Preservative used for toxicological specimen :

(A) 20% formalin 

(B) Saturated sodium chloride 

(C) 20% alcohol 

(D) 10% alcohol 

Answer: (B)

89. Sites notorious for incised looking wound are all except :

(A) Chest

(B) Zygoma

(C) Iliac crest 

(D) Shin

Answer: (A)

90. Use of wadding in a smooth bore gun rifle produces all except :

(A) Causes fatal injuries 

(B) Helps in lubrication 

(C) Optimum pressure 

(D) Sealing the air 

Answer: (A)

91. Best bone to assess age between 20-50 yrs. :

(A) Skull

(B) Ribs

(C) Sternum

(D) Symphysis pubis 

Answer: (A)

92. 99% sex difference can be done by all features of pelvic bones except :

(A) Subpubic angle 

(B) Greater sciatic notch 

(C) Preauricular sulcus 

(D) None of these 

Answer: (D)

93. Post mortem lividity persist till :

(A) 24 hours 

(B) 18 hours 

(C) 2-3 days 

(D) Merges with putrefactive changes 

Answer: (D)

94. Paraphilias are all except :

(A) Bisexuality

(B) Homosexuality

(C) Bestiality

(D) Frottuerism

Answer: (A)

95. The Widmark formula is used in estimation of:

(A) Cocaine

(B) Arsenic

(C) Alcohol

(D) Carbon monoxide 

Answer: (C)

96. McNaughten rule is applicable to:

(A) Calculate length of fetus 

(B) Estimate height 

(C) Insanity

(D) Dactyglography

Answer: (C)

97. Wood mark formula is used in which poisoning?

(A) Alcohol

(B) Barbiturate

(C) Carbolic acid 

(D) Clephos poisoning 

Answer: (A)

98. Amylnitrate is used in which of the following poisoning?

(A) Hydrocyanide poisoning 

(B) Red phosphorus 

(C) Yellow phosphorus 

(D) Arsenic poisoning 

Answer: (A)

99. Blue hypostasis is seen poisoning due to :

(A) H₂S 

(B) Phosphorus

(C) Organophosphorus compounds 

(D) Arsenic

Answer: (A)

Medical PG Surgery

Medical PG Surgery

1. Immediate management of a patient with Multiple fracture and Fluid loss includes the infusion of

(A) Blood

(B) Dextran

(C) Normal Saline 

(D) Ringer lactate 

Answer: (D)

2. Which of the following is the best parameter to assess fluid intake in a poly-trauma patient

(A) Urine output

(B) BP

(C) Pulse

(D) Pulse oximetry 

Answer: (A)

3. Cleft palate is best repaired at

(A) Soon after birth 

(B) At 1 month 

(C) At 6-8 months 

(D) Between 1-1.5 years 

Answer: (D)

4. The commonest site of Branchial cysts is

(A) Upper 1/3rd of the SCM 

(B) Lower 1/3rht of the SCM 

(C) Upper 2/3rd of the SCM 

(D) Lower 2/3rd of the SCM 

Answer: (A)

5. Which of the following cutaneous malignancies do not metastasize through the lymphatics

(A) Squamous cell carcinoma 

(B) Basal cell carcinoma 

(C) Melanoma

(D) Kaposi’s sarcoma 

Answer: (B)

6. Lymph node metastasis is least commonly seen with

(A) Papillary Ca Thyroid 

(B) Medullary Ca Thyroid 

(C) Follicular Ca Thyroid 

(D) Anaplastic Ca Thyroid 

Answer: (C)

7. One of the following will always present with Bilious vomiting

(A) Pyloric stenosis 

(B) Esophageal atresia 

(C) Atresia of the 3rd part of the duodenum 

(D) Malrotation of the gut 

Answer: (C)

8. A neonate with a scaphoid abdomen and respiratory distress has

(A) Congenital pyloric stenosis 

(B) Diaphragmatic hernia 

(C) Volvulus

(D) Wilm’s tumor 

Answer: (B)

9. Meckel’s diverticulum is a derivative of

(A) Allantoic diverticulum 

(B) Vitellointestinal duct 

(C) Ventral mesogastrium 

(D) Ductus arteriosus 

Answer: (B)

10. Acute mesenteric lymphadenitis is caused by

(A) E. coli 

(B) α-hemolytic streptococci 

(C) Hemophilus

(D) Yersinia

Answer: (D)

11. The commonest site of obstruction in Gall stone ileus is

(A) Proximal ileum 

(B) Distal ileum 

(C) Ileocecal junction 

(D) Transverse colon 

Answer: (B)

12. Resection of 90% of the ileum and jejunum causes all the following except

(A) Hypogastrinemia

(B) Steatorrhea

(C) Anemia

(D) Extracellular volume depletion 

Answer: (A)

13. The most unlikely clinical feature of Hepatocellular carcinoma is

(A) Hepatomegaly

(B) Raised α-fetoprotein levels 

(C) Raised alkaline phosphate 

(D) Jaundice

Answer: (D)

14. α-Fetoprotein increase in all the following except

(A) Hepatocellular carcinoma 

(B) Seminoma of the testes 

(C) GI neoplasms 

(D) Embryonal cell carcinoma 

Answer: (B)

15. Zollinger-Ellison syndrome is due to tumor of the

(A) α cells 

(B) β cells 

(C) PP cells 

(D) PP cells 

Answer: (D)

16. Zollinger-Ellison syndrome is characterized by all of the following except

(A) Post bulbar ulcer 

(B) Recurrent duodenal ulce 

(C) Severe diarrhea 

(D) Massive HCL in response to histamine injection 

Answer: (D)

17. Pancreatic pseudocyst most commonly occurs after

(A) Trauma

(B) Pancreatitis

(C) Pancreatic surgery 

(D) Pancreatic malignancy

Answer: (B)

18. Painless gross hematuria occurs in

(A) Renal cell carcinoma

(B) Polycystic kidney 

(C) Stricture of urethra 

(D) Wilm’s tumor 

Answer: (A)

19. A 75-year-old frail elderly man underwent TURP. The biopsy revealed adenocarcinoma. What is the next line of management

(A) Radiotherapy

(B) Surgery followed by Hormonal replacement therapy 

(C) Conservative treatment 

(D) Surgery followed by radiotherapy 

Answer: (C)

20. Trousseau’s sign is seen in all the following except

(A) Ca Lung 

(B) Ca Stomach 

(C) Ca Pancreas 

(D) Liposarcoma

Answer: (D)

21. All of the following produce Osteoblastic secondaries except

(A) Ca Prostate 

(B) Carcinoid tumors 

(C) Ca Breast 

(D) Multiple myeloma 

Answer: (D)

22. Bleeding from the umbilicus in an adult female during menstruation is suggestive of

(A) Bleeding diathesis 

(B) Vicarious menstruation

(C) Persistent urachus 

(D) Purpura

Answer: (B)

23. Commonest site of a rodent ulcer is

(A) Limbs

(B) Face

(C) Abdomen

(D) Trunk

Answer: (B)

24. In hand injuries the first structure to be repaired is

(A) Bone

(B) Skin

(C) Tendon

(D) Verve

Answer: (B)

25. FNAC is useful in all the following types of thyroid carcinoma except

(A) Papillary

(B) Follicular

(C) Anaplastic

(D) Medullary

Answer: (B)

26. Serum calcitonin is a marker for

(A) Anaplastic carcinoma 

(B) Papillary carcinoma 

(C) Medullary carcinoma 

(D) Follicular carcinoma 

Answer: (C)

27. Thyroid carcinoma with pulstile vascular skeletal metastasis is

(A) Papillary

(B) Follicular

(C) Medullary

(D) Anaplastic

Answer: (B)

28. ‘Hurthle cells’ are seen in

(A) Agranulomatous Thyroiditis 

(B) Hashimoto’s Thyroiditis 

(C) Papillary carcinoma of the thyroid 

(D) Thyroglossal cyst 

Answer: (B)

29. Paget’s disease of the nipple is treated by

(A) Radiotherapy 

(B) Radical mastectomy

(C) Biopsy and simple mastectomy 

(D) Chemotherapy

Answer: (C)

30. The commonest pre-malignant condition of oral cancer is

(A) Leukoplakia

(B) Aphthous ulcer

(C) Lichen planus 

(D) Erythro-leukoplakia 

Answer: (A)

31.Which of the following is TURE and Zeker’s diverticulum

(A) It is asymptomatic

(B) Occurs in the mid-esophagus 

(C) Treatment is simple excision 

(D) It occurs in children 

Answer: (C)

32. The best prognosis in carcinoma stomach is with

(A) Superficial spreading type 

(B) Ulcerative type 

(C) Linitis plastica type 

(D) Polypoidal fungating type 

Answer: (A)

33. Distended abdomen in intestinal obstruction is mainly due to

(A) Diffusion of gas from blood 

(B) Fermentation of residual food 

(C) Bacterial action 

(D) Swallowed air 

Answer: (D)

34. The following are important in maintenance of normal fecal continence except

(A) Anorectal angulation 

(B) Rectal innervation 

(C) Internal sphincter 

(D) Haustral valve 

Answer: (D)

35. Kehr sign seen in splenic injury is

(A) Pain over left shoulder 

(B) Pain over right shoulder 

(C) Pain over renal angle 

(D) Pain over pera-umbilical region 

Answer: (A)

36. ‘Carcot’s triad’s is

(A) Fever, pain, vomiting 

(B) Fever, stone, jaundice 

(C) Fever, pain, jaundice 

(D) Gall stone, vomiting-jaundice 

Answer: (C)

37. Which of the following is the preferred treatment modality for annual pancreas

(A) Whipple’s operation 

(B) Duodeno-jejunostomy 

(C) Gastro-jejunostomy 

(D) Jejuno-cystostomy 

Answer: (B)

38. Which of the following is true about renal trauma

(A) Urgent IVP is indicated 

(B) Exploration of the kidney to be done in all cases 

(C) Lumbar approach to kidney is preferred 

(D) Renal artery aneurysm is common 

Answer: (A)

39. A symptom of medullary sponge kidney disease is

(A) Nocturia

(B) Anemia

(C) Azotemia

(D) UTI

Answer: (D)

40. Most common content in ‘Hernia on glissade’ is

(A) Omentum

(B) Urinary bladder 

(C) Caecum

(D) Sigmoid colon 

Answer: (D)

41. Secondaries of all the following cause osterolytic lesions except

(A) Prostate

(B) Kidney

(C) Bronchus

(D) Thyroid

Answer: (A)

42. The most common site of enlargement of the lymph nodes in Hodgkin’s lymphoma is

(A) Mediastinal

(B) Axillary

(C) Cervical

(D) Abdominal

Answer: (C)

43. Extradural hematoma indicates rupture of

(A) Maxillary artery

(B) Middle meningeal artery

(C) Internal carotid artery 

(D) Ophthalmic artery 

Answer: (B)

44. In skull fracture, the condition in which an operation is not done immediately is

(A) Depressed fracture 

(B) Compound fracture 

(C) CSF leak 

(D) CSF leak 

Answer: (C)

45. Best treatment modality for Warthin’s tumor is

(A) Superficial parotidectomy 

(B) Radical parotidectomy 

(C) Radiotherapy

(D) Curettage

Answer: (A)

46. Which malignancy of parotid gland spreads through the neural sheath

(A) Adenoid cystic carcinoma 

(B) Adenocarcinoma

(C) Pleomorphic adenoma with malignant change 

(D) Sarcoma

Answer: (A)

47. The most common presentation of endemic goiter is

(A) Hypothyroid

(B) Diffuse goiter

(C) Hyperthyroid

(D) Solitary nodule

Answer: (B)

48. Which type of thyroid carcinoma has the best prognosis

(A) Papillary Ca 

(B) Anaplastic Ca 

(C) Follicular Ca 

(D) Medullary Ca 

Answer: (A)

49. Most probable malignancy that develops in a case of long-standing goiter is

(A) Follicular Ca 

(B) Anaplastic Ca 

(C) Papillary Ca 

(D) Medullary Ca 

Answer: (A)

50. All the following are true of DeQuervan’s Thyroiditis except

(A) Pain

(B) Increased ESR 

(C) Increased radioactive iodine uptake 

(D) Fever

Answer: (C)

51. The treatment of choice in Duct papilloma of the breast is

(A) Simple mastectomy 

(B) Microdochectomy

(C) Local wide excision 

(D) Chemotherapy

Answer: (B)

52. Mondor’s disease is

(A) Thrombophlebitis of the Superficial veins of Breast 

(B) Carcinoma of the breast 

(C) Premalignant condition of the breast 

(D) Filariasis of the breast 

Answer: (A)

53. Which histological variant of breast carcinoma is multicentric and bilateral

(A) Ductal Ca 

(B) Lobular Ca 

(C) Mucoid Ca 

(D) Colloid Ca 

Answer: (B)

54. Multiple painfull ulcers on tongue are seen in all except

(A) Aphthous ulcers 

(B) Tuberculous ulcers 

(C) Herpes ulcers 

(D) Carcinomatous ulcers 

Answer: (D)

55. Which neo-adjuvant chemotherapy is used in Esophageal carcinoma

(A) Cisplatin

(B) Cyclophosphamide

(C) Doxorubicin

(D) Methotrexate

Answer: (A)

56. Best substitute of esophagus after esophagectomy is

(A) Stomach

(B) Jejunum

(C) Left colon

(D) Right colon

Answer: (A)

57. All the following are true about Meckel’s diverticulum except

(A) Bleeding

(B) Intussusception

(C) Arises at the mesenteric border 

(D) Located 60 cm from the cecal valve 

Answer: (C)

58. The most common cause of small intestinal obstruction is

(A) Intussuception

(B) Iatrogenic adhesions 

(C) Trauma

(D) Carcinoma

Answer: (B)

59. Commonest tumor of the small intestine is

(A) Leiomyoma

(B) Lymphoma

(C) Adenocarcinoma

(D) Hemangioma

Answer: (A)

60. Meconium ileus can be caused by

(A) Fibrocystic disease of the pancreas 

(B) Liver aplasia 

(C) Cirrhosis of the liver 

(D) Malnutrition

Answer: (A)

61. All of the following are removed in Whipple’s operation except

(A) Duodenum

(B) Head of pancreas 

(C) Portal vein 

(D) Common bile duct 

Answer: (C)

62. Lithogenic bile has the following properties

(A) ↑ Bile and Cholesterol ratio 

(B) ↓ Bile and cholesterol ratio 

(C) Equal bile and cholesterol ratio 

(D) ↓ Cholesterol only 

Answer: (B)

63. Stone formation in Gall bladder is enhanced by all except

(A) Clofibrate therapy 

(B) Ilial resection

(C) Cholestyramine therapy 

(D) Vagal stimulation 

Answer: (D)

64. Which of the following does not predispose to Cholangiocarcinoma

(A) Ulcerative colitis 

(B) Clonorchis sinensis 

(C) Choledochal cyst 

(D) Chronic pancreatitis 

Answer: (D)

65. Charcot’s triad includes all of the following except

(A) Pain

(B) Fever

(C) Jaundice

(D) Vomiting

Answer: (D)

66. Triad of Renal colic, Swelling in loin which disappears after passing urine is called

(A) Kocher’s triad 

(B) Saint’s triad 

(C) Dietel’s crisis 

(D) Charcot’s triad 

Answer: (C)

67. Splenectomy is most useful in

(A) Sickle cell anemia 

(B) Thallassemia

(C) Hereditary spherocytosis 

(D) Acquired auto ammonia hemolytic anemia 

Answer: (C)

68. An absolute contraindication for IVP is

(A) Allergy to the drug 

(B) Multiple myeloma 

(C) Blood urea > 200 mg 

(D) Renal tumor 

Answer: (A)

69. True about Acute Epididymitis is

(A) Associated with urinary infection 

(B) Paintless

(C) Scrotum size is reduced

(D) Clinically does not mimic torsion of testes

Answer: (A)

70. Which type of haemangioma resolves spontaneously :

(A) Port wine stain 

(B) Strawberry angioma 

(C) Cavernous angioma 

(D) Plexiform angioma 

Answer: (B)

71. True statement for axial flap is :

(A) Carries its own vessels within it 

(B) Kept in limb 

(C) Transverse flap 

(D) Carries its own nerve in it 

Answer: (A)

72. All of the following statements about malignant melanoma are true except :

(A) Prognosis is better in female than in male 

(B) Acral lentiginous melanoma carrier a good prognosis 

(C) Stage IIA shows satellite deposits 

(D) Most common type is superficial spreading melanoma 

Answer: (B)

73. Carcinoma of buccal mucosa commonly drain to the following lymph node sites

(A) Sub mental 

(B) Sub mandibular 

(C) Supraclavicular 

(D) Cervical

Answer: (B)

74. Treatment of pleomorphic adenoma of parotid is

(A) Superfacial parodictomy 

(B) Radical removal 

(C) Deep lobe removal 

(D) Total removal 

Answer: (A)

75. Lymphnode metastasis is common feature with the following variant of soft tissue sarcoma :

(A) Fibrosarcoma

(B) Angiosarcoma

(C) Liposarcoma

(D) Neurofibrosarcoma

Answer: (B)

76. MEN-II is seen with the following type of thyroid carcinoma :

(A) Papillary

(B) Medullary

(C) anaplastic

(D) Follicular

Answer: (B)

77. Screening method for medullary carcinoma thyroid is :

(A) Serum calcitonin 

(B) S. calcium 

(C) S. alkaline phosphate 

(D) S. acid phosphatase 

Answer: (A)

78. All of the following are required for visualization of gall bladder in an oral cholecystogram except :

(A) Motor mechanism of gall bladder 

(B) Patency of cystic duct 

(C) Ability to absorb water 

(D) Functioning hepatocytes 

Answer: (A)

79. Most common site of gall stone impactation is

(A) Duedenojejunal junction 

(B) Proximal to ileocaecal junction

(C) Distal to ileocaecal junction 

(D) Colon

Answer: (B)

80. The treatment of choice for silent stones in Gall bladder is

(A) Observation

(B) Chenodeoxy cholic acid 

(C) Cholecytectomy

(D) Lithotripsy

Answer: (A)

81. All of the following are known predesposing factors for cholangiocarcinoma except :

(A) CBD stones 

(B) Clonorchis sinensis 

(C) Ulcerative colitis 

(D) Primary sclerosing cholangitis

Answer: (A)

82. All of the following statements about Pseudopancreatic cysts are true except :

(A) Percutaneous aspiration is treatment of choice

(B) Cystojejunostomy is treatment of choice 

(C) Serum amylase levels are increased 

(D) Presents as an epigastric mass 

Answer: (A)

83. True statements about congeniatal megacolon include all of the following except :

(A) Dilatation & hyper trophy of pelvic colon 

(B) Loud borborygmi 

(C) Symptoms appear with in 3 days following birth 

(D) Large stool 

Answer: (D)

84. All of the following are features of exstrophy of the bladder except :

(A) Epispadias

(B) Cloacal membrane is present 

(C) Posterior b ladder wall protrudes through the defects 

(D) Umblical and inguinal hernia 

Answer: (B)

85. Best indication for ‘Testicular biopsy in a male is

(A) Polysperamia

(B) Oligopermia

(C) Necrospermia

(D) Azoospermia

Answer: (D)

86. All of the following statements regarding bronchial cysts are true except :

(A) Seen in mediastinum 

(B) 50-70% occur in lung 

(C) Are Commonly infected

(D) Multilocular

Answer: (D)

87. Orthobaric oxygen is used in :

(A) CO poisoning 

(B) Ventilation failure 

(C) Anerobic infection 

(D) Gangrene

Answer: (A)

88. Most common cancer in India :

(A)  CA cervix

(B)  CA breast

(C)  CA lung

(D) CA oral cavity

Answer: (D)

89. Clinical features of fracture of Zygomatic bone include all of the following except :

(A) Diplopia

(B) Trismus

(C) Bleeding

(D) C.S.F. rhinorrhea 

Answer: (D)

90. β2 microglobulin is a marker for prognosis in

(A) SLE

(B) Multiple myeloma 

(C) AIDS

(D) AIDS

Answer: (B)

91. Increased acetyl cholinesterase enzyme is seen in :

(A) Neural tube defects 

(B) Diaphrgmatic hernia 

(C) Cardiac difect 

(D) Oesophgeal atresia 

Answer: (A)

92. Spontaneous regression is seen in :

(A) Portwine haemangioma 

(B) Strawberry haemangioma 

(C) Cavernous haemangioma

(D) Arterial angioma 

Answer: (B)

93. Following tumour may regress spontaneously except

(A) Retinoblastoma

(B) Malignant melanoma

(C) Osteosarcoma

(D) Choricoarcinoma

Answer: (C)

94. Treatment of choice of warthiin’s tumour is :

(A) Superficial paroidectomy 

(B) Excision 

(C) Radiotherapy

(D) Chemotherapy

Answer: (A)

95. Cleft palate is ideally repaired at :

(A) 5 month of age 

(B) 1 year of age 

(C) Before going to school 

(D) 6-8 years of age 

Answer: (B)

96. Hyoid bone is closely associated with :

(A) Bronchiogenic cyst 

(B) Cystic hygroma 

(C) Thyroglossal cyst/fistula 

(D) Branchial cyst 

Answer: (C)

97. In hand injury firs structure to be repaired should be :

(A) Skin

(B) Nerve

(C) Muscle

(D) Bone

Answer: (A)

98. All may be seen in deep burns except :

(A) Hyperthermia

(B) Increase vascular permeability

(C) Fluidloss by evoperation 

(D) Vasodilatation

Answer: (A)

99. Following drug lowers intracranial pressure except

(A) Mannitol infusion

(B) Furesemide

(C) Corticosteroids

(D) None

Answer: (D)

100. Bony metastasis is common with all of the following except :

(A) CA breast 

(B) CA lung 

(C) CA testis 

(D) CA prostate 

Answer: (C)

101. In soft tissue sarcoma the prognosis depends best upon :

(A) Site of tumour 

(B) Site of tumour 

(C) Size of tumour 

(D) Lymphadenopathy 

Answer: (B)

102. In breast carcinoma metastasis, prognosis depends best upon :

(A) Extrogen receptor status 

(B) Axillary lymphnode status 

(C) Size of tumour 

(D) Site of tumour 

Answer: (A)

103. The risk factor for increased incidence relapse in stage I, carcinoma breast includes all except :

(A) -ve estrogen / progesterone receptor status 

(B) High ‘s’ phase 

(C) Aneuploidy

(D) Decreased Her-2/neu oncogene 

Answer: (D)

104. Best prognosis amongst the following histoligical variants of breast carcinoma is seen with :

(A) Intraductal

(B) Colloid

(C) Lobular

(D) Ductal

Answer: (A)

105. Treatment of choice in duct papilloma of breast is :

(A) Simple mastectomy 

(B) Microdochectomy

(C) Local wide excision 

(D) Chemotherapy

Answer: (B)

106. Gyancomastia may be seen in all of the following conditions except :

(A) Klinefelter syndrome 

(B) Cirrhosis of liver 

(C) Cryptorchidism 

(D) Sexcordtumour of sertoli cells 

Answer: (C)

107. Adenocarcinoma of esophagus is commonly found in :

(A) Achlasia acardia 

(B) Barret’s oesophagus 

(C) Plummer Vinson syndrome 

(D) Chronic smoking 

Answer: (B)

108. Short bowel syndrome is characterized by all of the following except :

(A) Diarrhoea

(B) Weight loss 

(C) Steatorrhoea

(D) Hypogastrinemia

Answer: (D)

109. A patient was operated for colonic carcinoma and later a solitary nodule was found in the liver Treatment of choice is :

(A) Surgery

(B) Radiation

(C) Chemotherapy

(D) Conservative treatment 

Answer: (A)

110. All are ture about pseudopancreatic cyst of pancreas except :

(A) Common after acute pancreatitis 

(B) Presents as an abdominal mass 

(C) Serum amylase is increased 

(D) Most common site is in head of pancreas 

Answer: (D)

111. Meconium ileus is associated with :

(A) Fibrocystic disease of pancreas 

(B) Liver aplasia 

(C) Cirrhosis of liver 

(D) Malnutrition 

Answer: (A)

112. Commonest cause of surgical obstructive jaundice is :

(A) CBD stone 

(B) Sclerosing cholongitis 

(C) Carcinoma of liver 

(D) Chromic cholecystitis 

Answer: (A)

113. Medical treatment for gall bladder stone is amenable for :

(A) Stones less than 15 mm in size 

(B) Radioopaque stone 

(C) Calcium bilirubinate oxalate stone 

(D) Nonfunctioning stones associated with gall bladder 

Answer: (A)

114. Commonest presentation of primary biliary cirrhosis

(A) Prurtitis

(B) Pain

(C) Pain

(D) Fever

Answer: (A)

115. Gall bladder stone formation in influenced by A/E

(A) Clofibrate therapy 

(B) Clofibrate therapy 

(C) Primary biliary cirrhosis 

(D) Hypercholesterolemia 

Answer: (D)

116. Features of hypospaedias are all except :

(A) Chordae

(B) Hooded prepuce 

(C) No-treatment required with glandular veriety 

(D) Cryptorchidism

Answer: (D)

117. Ureter is diagnosed during operation by :

(A) Venous plexus 

(B) High arterial supply 

(C) Peristaltic movements 

(D) Circumferance 

Answer: (C)

118. Varicocele is common on left testis because :

(A) Left testicular vein drains into IVC which has high pressure 

(B) Left testicular vein drains into leftrenal vein which has high pressure 

(C) Left lestis is lower situated 

(D) Compression of testicular vein by rectum 

Answer: (B)

119. Commonest site of peptic ulcer is :

(A) Ist part of Duodenum 

(B) IInd part duodenum 

(C) Distal 1/3 of stomach 

(D) Pylorus of the stomach 

Answer: (A)

120. Common sites for cushing ulcers include all of the following except :

(A) Esophagus

(B) Stomach

(C) Ist part of duodenum

(D) Distal duodenum

Answer: (D)

121. Metabolic changes associated with excessive vomiting includes the following:

(A) Metabolic acidosis 

(B) Hyperchloremia

(C) Hypokalemia

(D) Decreases bicarbonates 

Answer: (C)

122. Dumping syndrome is due to :

(A) Diarrhoea

(B) Presence of hypertonic content in small intestine 

(C) Vagotomy

(D) Reduced gastric capacity 

Answer: (B)

123. Most common site for impaction of gell stones in gall stone lieus is :

(A) Ist part of duodenum 

(B) IInd part of duodenum 

(C) Terminal ileum 

(D) Colon

Answer: (C)

124. Pyoderma-gangrenosum is most commonly associated with :

(A) Ulcerative colitis 

(B) Crohns disease 

(C) Amoebic colitis 

(D) Ischemic colitis 

Answer: (A)

125. Colvesical fistula most commonly arises from :

(A) Crohn’s disease 

(B) Ulcerative colitis 

(C) Carcinoma colon 

(D) Abdomino-perineal resection 

Answer: (A)

126. Anchovy sauce pus is a feature of :

(A) Amoebic liver abscess

(B) Lung abscess 

(C) Splenic abscess 

(D) Splenic abscess 

Answer: (A)

127. Most common cause of pseudopancreatic cyst in children is:

(A) Chloledochal cyst 

(B) Annular pancreas 

(C) Drug induced pancreatitis

(D) Traumatic pancreatitis 

Answer: (D)

128. Which of the following causes least irritation of the peritoneal cavity :

(A) Bile

(B) Blood

(C) Gastric enzyme 

(D) Pancreatic enzyme 

Answer: (B)

129. Most common type of Intussusception is :

(A) Ilio-colic 

(B) Ilio-ileal 

(C) Colo-colic 

(D) None of these

Answer: (A)

130. 1 year After cholecystectomy, a patient presented with 2.5 cm size stone in CBD, Treatment of choice is :

(A) Choledocholithotomy with T-tube drainage ment 

(B) Supraduodenal choledochotomy with exploration 

(C) Trans duodenal choledochojejunostony with spincturoplasty 

(D) Endoscopic sphincterotomy with stone extraction 

Answer: (D)

131. Spigelian hernia is seen in :

(A) Lumbar triangle 

(B) Subumblical region 

(C) Paraumblical region 

(D) Supraumblical region 

Answer: (B)

132. A 9 year boy presented with abdominal pain and recurrent UTI, IVP reveales duplication of left ureter. The most likely site of ectopic opening would be :

(A) Prostatic urethra 

(B) Ejaculatory duct

(C) Seminal vesicle 

(D) Vas-deference 

Answer: (A)

133. A 6 week neonate presented with undescended testes on the right side and normal testes on left side. The treatment of choice would be :

(A) Orchipexy at 2 years 

(B) Immediately

(C) 5 year age before going to school 

(D) Puberty 

Answer: (A)

134. Treatment of extragonadal germ cell tumour is :

(A) Chemotherapy

(B) Radiotherapy

(C) Surgery

(D) Immunotherapy

Answer: (A)

135. A 60 year diabetic & hypertensive male with second grade prostatism admitted for prostatectomy developed myocardial infarction. Treatment now would be :

(A) Finasteride

(B) Radiotherapy

(C) Surgery

(D) Immunotherapy

Answer: (B)

136. A 55 year old diabetic patient presented with impotence with history of failure to get erection after papaverine intracavernous injection. Colour Doppler shows no. abnormality of arteries but shows mold venous run-off, Treatment of choice here would be :

(A) Intracavernous injection of papaverine 

(B) Penile prosthetic implants 

(C) Vacuum constriction device 

(D) Psychotherapy

Answer: (C)

137. A 25 year old married male presents with infertility. He had undergone retroperitoneal lymphnode dissection at age of 15 years for embryonal carcinoma of right testis. Semen analysis shows-quantity-0.5 ml, no. sperm, no fructose. Biopsy of testis shows normal spermato genesis. best treatment here would be :

(A) Artificial insemination of donor 

(B) Penile-prosthesis 

(C) Microtestcular aspiration and intracyto plasmic injection 

(D) None of the above 

Answer: (C)

138. Which of the following carcinomas is familial :

(A) Breast

(B) Prostate

(C) Cervix

(D) Vaginal

Answer: (A)

139. Commonest cause of pulmonary embolism is

(A) Thrombosis of leg veins 

(B) Thrombosis of prostatic veins 

(C) IVC thrombosis 

(D) Thrombosis of internal pudendal artery 

Answer: (A)

140. CABG is done for all of the following indications except :

(A) To reduces symptoms 

(B) To prevent further catastrophics 

(C) To prolong life 

(D) To prevent progress of native blood vessel disease 

Answer: (D)

141. Commonest presentation of Hodgkin’s lymphoma is

(A) Painless enlargement of lymphnode 

(B) Pruritis

(C) Fever

(D) Leuocytosis

Answer: (A)

142. A 50 year old man presented with 3 moth history of right sided cervical lymphnode enlargements. The best investigation to establish the diagnosis in this case would be :

(A) FNAC

(B) X-ray 

(C) Tuberaulin test 

(D) Lymph-node biopsy 

Answer: (D)

143. Ainhum is seen in :

(A) base of great toe 

(B) Base of fingers tips 

(C) Base of toe 

(D) Ankle

Answer: (C)

144. Renal collar which surrounds the aorta has its two limbs split by :

(A) Left renal vein 

(B) Left renal artery 

(C) Isthumus of horsechoe kidney 

(D) All of the above 

Answer: (A)

145. A lady presented with VVF due to prolong labour one year back, on examination ectopic opening is found on the interuretic bar close to the opening of right ureter. Treatment of choice is :

(A) Transvaginal repair of VVF 

(B) Transvesical VVF repair 

(C) Transperitoneal trans fistula repair with reimplantation of right ureter 

(D) All the above 

Answer: (C)

146. Commonest cancer in burn scar is:

(A) Sq. cell Ca 

(B) Fibrosarcoma 

(C) Adeno Ca 

(D) Adeno-squamous Ca 

Answer: (A)

147. In hepatocellular carcinoma, factor important is :

(A) Alcoholic hepatitis 

(B) Schistomiasis

(C) Cirrhosis

(D) Fasciolepsis buski infestation 

Answer: (C)

148. Precancerous condition of Ca stomach is:

(A) Peptic ulcer 

(B) Chronic gastric atrophy 

(C) Achalasia cardia 

(D) Curling’s ulcer

Answer: (B)

149. Definitive treatment of sigmoid volvulus is:

(A) Surgical correction 

(B) Colectomy

(C) Enema

(D) Endoscopic correction 

Answer: (A)

150. Most common malignancy affecting spleen is:

(A) Angiosarcoma

(B) Hamartoma

(C) Secondaries

(D) Lymphoma

Answer: (D)

151. What is more appropriate for diagnosis of CBD stones :

(A) Ultrasonography 

(B) ERCP

(C) OCG

(D) IV cholangiography 

Answer: (B)

152. Most common tumor of urinary bladder is:

(A) Sq. cell carcinoma 

(B) Adeno carcinoma 

(C) Transitional carcinoma 

(D) Str. squamous carcinoma 

Answer: (C)

153. Deep vein thrombosis is best diagnosed by:

(A) Plethysmography

(B) Duplex ultrasound 

(C) Radionuclide scan 

(D) CT scan 

Answer: (B)

154. Anal carcinoma is most commonly carcinoma of type

(A) Adeno carcinoma 

(B) Epidermoid

(C) Mixed

(D) None of the above 

Answer: (B)

155. Most common cause of acute intestinal obstruction:

(A) Adhesions

(B) Carcinoma

(C) Int. hernia 

(D) Lymphoma

Answer: (A)

156. Treatment of annular pancreas is:

(A) Excision of head of pancreas 

(B) Duodeno Jejunostomy 

(C) Excision of tail of pancreas 

(D) Any of the above 

Answer: (B)

157. Bilateral pulseless disease in upper limbs in caused by

(A) Aortoarteritis

(B) Coarctation of aorta 

(C) Fibromuscular dysplsia 

(D) Buerger’s disease 

Answer: (A)

158. Most favorable prognosis after radiotherapy is in:

(A) Melanoma

(B) Teratoma

(C) Seminoma

(D) Desmoid

Answer: (C)

159. In severe injury, first to be maintained is:

(A) Hypotension

(B) Dehydration

(C) Airway

(D) Cardiac status 

Answer: (C)

160. After splenectomy, most common infection is:

(A) Pneumococcus

(B) Staphylococcus

(C) Esch. coli 

(D) Pneumocystis carinii 

Answer: (A)

161. Duplication of GIT most commonly involves:

(A) Esophagus

(B) Stomach

(C) Duodenum

(D) IIeum

Answer: (D)

162. A patient comes with single thyroid nodule initial investigation of choice is:

(A) FNAC

(B) Thyroid function test 

(C) Radio nuclide scan 

(D) MRI

Answer: (A)

163. Absence of myenteric ganglion is seen in:

(A) Crohn’s disease 

(B) Ulcerative colitis 

(C) Hirschsprung’s disease 

(D) Intussuception

Answer: (C)

164. Blood stained discharge from nipple is indicative of

(A) Intra alveolar papilloma 

(B) Duct ectasia 

(C) Paget’s disease 

(D) Lobular Ca 

Answer: (B)

165. For prophylaxis of deep vein thrombosis used is:

(A) Warfarin

(B) Heparin

(C) Pneumatic shock garment 

(D) Graded stoking 

Answer: (B

166. Cervical lymph node involvement in papillary ca thyroid, best treatment is:

(A) Radio active iodine 

(B) Chemotherapy 

(C) Radical neck dissection 

(D) Steroid

Answer: (A)

167. In dysphagia lusoria true is:

(A) Esophagela web 

(B) Aberrant vessels 

(C) Oesophageal stricture 

(D) Oesophageal carcinoma 

Answer: (B)

168. In retrosternal goiter, most common presenting feature is:

(A) Dysphagia

(B) Stridor

(C) Dyspnoea

(D) Sup. vena caval syndrome 

Answer: (C)

169. Commonest artery for cannulation is:

(A) Radial

(B) Ulnar

(C) Brachial

(D) Cubital

Answer: (A)

170. Dermoid arises from:

(A) Pluripotent cell 

(B) Totipotent cell 

(C) Ectoderm

(D) Mesoderm

Answer: (B)

171. Investigation of choice in upper GI bleeding is:

(A) Ba swallow 

(B) X-ray 

(C) Endoscopy

(D) Ultrasound

Answer: (C)

172. Antenatal double bubble appearance on ultrasound is due to:

(A) Diaphragmatic hernia 

(B) Duodenal atresia 

(C) Volvulus

(D) Intussusception

Answer: (B)

173. Earliest feature of sudden increase in ICT in infant is

(A) Boggy swelling of fontanelle

(B) Papilloedema

(C) Paralysis

(D) Seizures

Answer: (B)

174. In pyloric stenosis, true is:

(A) Hypochloremic hyponatermic metabolic alkalosis with acidic urine 

(B) Hypochloremic hyponatremic metabolic alkalosis with alkaline urine

(C) Hypochloremic hyponatremic metabolic acidosis 

(D) Hyperchloremic hyponatremic metabolic acidosis 

Answer: (A)

175. Most common neoplasm of adrenal medulla is:

(A) Pheochromocytoma 

(B) Neurofibroma

(C) Neuroblastoma

(D) Adenocarcinoma

Answer: (A)

176. All are true about cancrum oris except:

(A) Associated with malnutrition and vitamin deficiency 

(B) Follows chronic infection 

(C) Involves jaw 

(D) Treatment is excision and skin grafting with tubed pedicle graft 

Answer: (C)

177. Regarding pectus excavatum all are true except:

(A) Gross CVS dysfunction

(B) Decrease in lung capacity 

(C) Cosmetic deformity 

(D) Depression in chest 

Answer: (A)

178. What is intussuscepiens :

(A) The entire complex of intussuception 

(B) The entering layer 

(C) The outer layer 

(D) The process of reducing the intussusception 

Answer: (C)

179. Which is not used in carcinoma prostate

(A) Estrogen

(B) Progesterone

(C) Cyproterone acetate 

(D) Flutamide

Answer: (B)

180. True about mesenteric cysts is:

(A) Enterogenous cyst is commonest type 

(B) Recurrence is common after enucleation 

(C) Mensentric cyst are usually multiple 

(D) Encleation is the treatment of choice of chylolymphatic cyst 

Answer: (D)

181. Cystoscopic findings in TB bladder are all except

(A) Cobblestone mucosa 

(B) Thimble bladder 

(C) Golf hole ureter 

(D) Whitish efflux form the ureteric holes 

Answer: (D)

182. Rx of congen, pyloric stenosis:

(A) Pyloromyotomy

(B) Gastroileostomy

(C) Gastrojejunostomy

(D) Vagotomy

Answer: (A)

183. Whipples triad is seen in:

(A) Islet cell tumour 

(B) Ca head pancreas 

(C) Argentaffinoma

(D) Cholangicarcinoma

Answer: (A)

184. Hypersplenism is seen in

(A) Pancytopenia

(B) Thrombocytopenia

(C) Leukopenia

(D) Polycythemia

Answer: (A)

185. Pseudolymphoma is seen in:

(A) Sjogren’s syndrome 

(B) SLE

(C) Mixed connective tissue disease 

(D) Behcet’s syndrome 

Answer: (A)

185. Pseudolymphoma is seen in:

(A) Sjogren’s syndrome 

(B) SLE

(C) Mixed connective tissue disease 

(D) Behcet’s syndrome 

Answer: (A)

186. Paget’s disease of breast following are true except:

(A) Treated by simple mastectomy 

(B) Represents underlying malignancy 

(C) Presents as eczema 

(D) Cytology diagnostic 

Answer: (D)

187. DVT, investigation of choice is:

(A) Doppler

(B) Plethysmography

(C) Venography

(D) X-ray 

Answer: (A)

188. Renal transplantation is most commonly done in:

(A) Chr. glomerulonephritis 

(B) Bilateral staghorn calculus 

(C) Horse shoe kidney 

(D) Oxalosis 

Answer: (A)

189. In pneumoperitoneum following are seen except:

(A) Hypertension

(B) Bradycardia

(C) Tachycardia

(D) Hypercapnia

Answer: (B)

190. In normal pressure hydrocephalus seen are all except:

(A) Convulsion

(B) Ataxia

(C) Dementia

(D) Incontinence

Answer: (A)

191. In gastrectomy following occurs except:

(A) Calcium deficiency

(B) Steatorrhoea

(C) Fe. deficiency 

(D) Fluid loss 

Answer: (D)

192. Lymphovenous anastomosis is done for:

(A) Filarial lymphoedema 

(B) Lymphoid cyst 

(C) Cystic hygroma 

(D) Malignant lymphoedema 

Answer: (A)

193. Radial nerve palsy causes:

(A) Claw hand 

(B) Wrist drop 

(C) Policeman’s tip deformity 

(D) Carpal tunnel syndrome 

Answer: (B)

194. Shock is clinically best assessed by:

(A) Urine output 

(B) CVP

(C) BP

(D) Hydration

Answer: (B)

195. Opsomyoclonus is seen in:

(A) Neuroblastoma

(B) Meningioma

(C) Medulloblastoma

(D) Wilms tumour 

Answer: (A)

196. In arterial injury following should be done except:

(A) Thrombectomy

(B) End-end anastomosis

(C) Ligation

(D) Venous graft 

Answer: (A)

197. Commonest site of ischemic colitis

(A) Splenic flexure 

(B) Transverse colon 

(C) Hepatic flexure 

(D) Ileocaecal junction 

Answer: (A)

198. Ulcerative colitis progressing to malignancy is characterized by following except:

(A) Risk increases with the time 

(B) Prognosis worsens 

(C) Prognosis depends on period 

(D) Forms pseudopolyps 

Answer: (D)

199. Mallory weiss syndrome in partial thickness rupture occurs at:

(A) Gastric cardia 

(B) Oesophatus mucosa 

(C) Gastro oesophageal junction 

(D) Gastroduodenal junction 

Answer: (C)

200. IVC filter is used in following except:

(A) Massive emboli 

(B) Negligible size of emboli 

(C) Repeated emboli 

(D) None

Answer: (B)

201. In pulmonary embolism, fibrinolytic therapy is responsible for

(A) Risk of haemorrhage 

(B) Prognosis good 

(C) Massive emboli 

(D) All of the above 

Answer: (A)

202. True about marjolins ulcer is:

(A) Ulcer over scar 

(B) Rapid growth 

(C) Rodent ulcer 

(D) Painful

Answer: (A)

203. Cork screw oesophagus is seen in:

(A) Diffuse oesophageal spasm 

(B) Achalasia cardia 

(C) Ca. oesophagus 

(D) Pulsion diverticulum 

Answer: (A)

204. Valvoplasty done in following except:

(A) Coarctation of aorta 

(B) PS

(C) MS

(D) AS

Answer: (A)

205. In AIDS, lymphadenopathy is most often due to:

(A) TB

(B) Lymphoma

(C) None specific enlargement of lymph node 

(D) Kaposi’s sacroma 

Answer: (C)

206. Esthesio neuroblastoma arises from

(A) Olfactory N 

(B) Maxillary N 

(C) Ophtalmic N 

(D) Nasociliary N 

Answer: (A)

207. Inverted “3” sign seen in:

(A) Ampullary Ca 

(B) Insulinoma

(C) Ca. head pancreas 

(D) Ca stomach 

Answer: (A)

208. Ca oesophagus is characterized by following except :

(A) Adenocarcinoma

(B) Middle 1/3 affected 

(C) Dysphagia is characteristic 

(D) Pernicious anemia often present 

Answer: (D)

209. Correct about diverticulosis is :

(A) Arise from diverticula of colon 

(B) Common in ileum 

(C) Protrusion of mucosa through muscular layers at point of blood vessel 

(D) All of the above 

Answer: (C)

210. Diagnostic procedure for basal cell ca :

(A) WEDGE biopsy 

(B) Shave

(C) Incisional biopsy 

(D) Punch biopsy 

Answer: (A)

211. Most haemorrhagic tumor is :

(A) Leiomyosarcoma stomach 

(B) Ca stomach 

(C) Adeno Ca GB 

(D) Ca pancreas 

Answer: (A)

212. In pancoast tumor, following is seen except:

(A) Horner’s syndrome 

(B) Rib erosion 

(C) Haemoptysis

(D) Pain in shoulder and arm 

Answer: (C)

213. Treatment of ruptured pneumo-peritoneum, as a result off colonscopy in a young patient is :

(A) Temporary colostomy 

(B) Closure + lavage 

(C) Permanent colostomy 

(D) Symptomatic

Answer: (B)

214. Opsomyoclonus is encountered as C/F of :

(A) Meningioma

(B) Neuroblastoma

(C) Neurofirbomatosis

(D) Excision

Answer: (B)

215. In pigmented basal cell carcinoma, treatment of choice is :

(A) Chemotherapy

(B) Radiotherapy

(C) Cryosurgery

(D) Cryosurgery

Answer: (C)

216. True about TIPSS :

(A) It is type of portocaval shunt

(B) It is intrahepatic shunt 

(C) Performed by passing endoscopes 

(D) Most suitable for patient going for liver transplant 

Answer: (B)

217. Liver transplant is not possible for :

(A) Liver

(B) Pancreas

(C) Small intestine

(D) Cornea

Answer: (C)

218. Gold standard for reflux esophagitis is :

(A) Manometry

(B) Barium swallow 

(C) 24 hours pH study 

(D) X-ray abdomen 

Answer: (C)

219. Metastasis in thyroid gland come most commonly from carcinoma of :

(A) Testis

(B) Prostate

(C) Breast

(D) Lungs

Answer: (C)

220. Most common presentation of intracranial aneurysm is :

(A) Coarctation of aorta 

(B) Systemic hypertension 

(C) Hypotension

(D) Intracranial haemorrhage 

Answer: (D)

221. About Ectopia vesicae, following is true except :

(A) CA bladder may occur 

(B) Ventral curvature of penis 

(C) Incontinence of urine 

(D) Visible uretero – vesical efflux 

Answer: (B)

222. Diagnosis of Hydatid disease is  by :

(A) Biopsy

(B) X-ray 

(C) Casoni test 

(D) Serum examination 

Answer: (A)

223. True about duplication of intestine is :

(A) Spherical type is MC 

(B) Tubular type is attached longitudinally with bowel 

(C) Spherical cyst communicates with lumen 

(D) All of the above 

Answer: (B)

224. Fluid levels are not visible in :

(A) Meconeum ileus 

(B) Intussusception

(C) Colon pouch 

(D) Duodenal obstruction 

Answer: (A)

225. Following are premalignant except :

(A) Tubulo villous adenoma 

(B) Hyperplastic polyp 

(C) Familial adenomatosis 

(D) Villous adenoma 

Answer: (B)

226. Following are premalignant except :

(A) Tubulo villous adenoma 

(B) Hyperplastic polyp 

(C) Familial adenomatosis 

(D) Villous adenoma 

Answer: (B)

227. During sclerotherapy (by endoscopy), following are complications except :

(A) Hepatic encephalopathy 

(B) Perforation

(C) Stenosis

(D) Fibrosis

Answer: (A)

228. In children MC type of polyp is :

(A) Juvenile poly 

(B) Solitary polyp 

(C) Familial polyposis 

(D) Multiple adenomatous polyp 

Answer: (A)

229. In ulcerative colitis, Ca arises from :

(A) Pseudopolyps 

(B) Dysplastic sites 

(C) Familial polyposis 

(D) Multiple adenomatous polyp

Answer: (B)

230. Extradural haematoma is associated with what % of severe trauma:

(A) 36% 

(B) 10% 

(C) 77% 

(D) 96% 

Answer: (B)

231. Absence of fructose in semen indicates :

(A) Obstruction of seminal vesicles 

(B) Obstruction at prostatic urethra 

(C) Vas deferens obstruction 

(D) Testicular failure 

Answer: ()

232. True about femoral hernia is:

(A) Occurs exclusively in females 

(B) Pregnancy is common cause 

(C) Doesn’t strangulate 

(D) In male associated with cryptorchidism 

Answer: (B)

233. Muscle not cut in posterolateral thoracotomy is:

(A) Serratus anterior 

(B) Lattisimus dorsi 

(C) Rhomboidus major 

(D) Pectoralis major 

Answer: (D)

234. Schatazki’s ring is:

(A) Mucosal ring at sq-columnar in 

(B) Muscular ring 

(C) Dysphagia is the symptom 

(D) Inflammatory structure 

Answer: (A)

235. What % of testicular Ca is associated with cryp torchidism:

(A) 10% 

(B) 30% 

(C) 70% 

(D) 90% 

Answer: (A)

236. Meningomyelocele patient after being operated developed hydrocephalus due to:

(A) Arnold chairi malformation 

(B) Injury to absorptive surface 

(C) Central canal injury 

(D) Arachnoidal block 

Answer: (A)

237. Hernia through foramen of bochdalek true is:

(A) Congenital hernia 

(B) In asymptomatic 

(C) Seen executive in males 

(D) Least common 

Answer: (A)

238. Left Ioin nephrectomy, Str. not cut is:

(A) Trapezius

(B) Seratus inf. post 

(C) Lat. dorsi 

(D) Internal oblique 

Answer: (A)

239. True about colonic organisms is :

(A) Distal ileum 10³ – 10⁵ org

(B) Colon 10¹⁰ – 10¹¹ org 

(C) First org in new born is coliforms and streptococcus 

(D) Chyme in jejunum contains many bacteria 

Answer: (B)

240. Peaud’ orange is due to:

(A) Skin involvement 

(B) Contraction of ligaments 

(C) Lymphatic permeation 

(D) Bacterial infection 

Answer: (C)

241. Ac. arterial ischemia, all are seen except:

(A) Pallor

(B) Rubor

(C) Pain 

(D) Paraesthesia

Answer: (B)

242. Nicoladoni branham sign is :

(A) Compression cause bradycardia 

(B) Compression cause tachycardia 

(C) Hypotension

(D) Systolic filling 

Answer: (A)

243. In Genitourinary TB, True is:

(A) Sterile pyurias consistent finding 

(B) AFB in early morning sample is always positive 

(C) MC site is pelvis 

(D) Commonest cause of pyelonephritis 

Answer: (A)

244. Investigation of choice hirschsprug’s disease is:

(A) Rectal manometry 

(B) Rectal examination 

(C) Rectal biopsy 

(D) Ba enema 

Answer: (C)

245. Major causes of distention in intestinal obstruction:

(A) Papillary Ca 

(B) Islet cell tumour 

(C) Pheochromocytoma 

(D) Glucagonoma

Answer: (C)

246. Major causes of distention in intestinal obstruction:

(A) Gas produced by coliform organism 

(B) Swallowed air 

(C) Diffuse from arterial surface 

(D) Not known 

Answer: (B)

247. Prolonged Post-op ileus is best treated by:

(A) Long tube insertion 

(B) Calcium pentothenate 

(C) Laporotomy and exploration 

(D) Peristaltic stimulants 

Answer: (C)

248. Fiberoptic endoscopy is contraindicated in:

(A) Children

(B) Aneurysm of arch of aorta 

(C) Cervical spondylosis 

(D) Hemoptysis

Answer: (B)

249. AV fistula causes:

(A) ↓Diastolic pressure 

(B) ↑enous return 

(C) ↓Venous congestion 

(D) ↑Systolic filling 

Answer: (B)

250. In testicular ca, investigation not done is:

(A) Aortography

(B) CT

(C) Biopsy

(D) Serum AFP 

Answer: (C)

251. Circumcission is included in management of Ca penis at:

(A) Glans

(B) Prepuce

(C) Glandulo prepucial 

(D) Shaft of penis 

Answer: (B)

252. The treatment of metastatic testicular carcinoma is :

(A) Bleomycin, Etoposide, Cisplatin 

(B) Vinblastine, Etoposide, Cisplatin 

(C) Doxorubicin, 5-FU mercaptopurine 

(D) Methotrexate, 5-FU, Vincristine 

Answer: (A)

253. Deficiency of the abdominal muscle is associated with :

(A) Eagle-Barrette syndrome 

(B) Christopher syndrome 

(C) Megacystitis 

(D) Megaureter 

Answer: (A)

254. The resting uretric pressure in mm :

(A) 5-7 

(B) 15 -30 

(C) 7-10 

(D) 0-5 

Answer: (B)

255. Which among the following is a feature of fracture of zygoma :

(A) Diplopia

(B) Epistaxis

(C) Bell’s palsy 

(D) Frey’s syndrome 

Answer: (A)

256. The commonest type of trachea-esophageal fistula is :

(A) Proximal end blind; distal end communicating with trachea 

(B) istal end; blind proximal and communicating with trachea 

(C) Both ends blind 

(D) Both ends open 

Answer: (A)

257. Which is not a feature of Dupuytren’s contracture:

(A) Recurrent trauma 

(B) Genetic predilection 

(C) Bilateral lesions rare 

(D) Seen equally in males and females 

Answer: (D)

258. Split skin graft can be applied over :

(A) Muscle

(B) Bone 

(C) Cartilage

(D) Eyelid

Answer: (A)

259. True about sliding esophageal hernia in all cases :

(A) Esophagus always short 

(B) Cardia goes through hiatus

(C) Cardia and fundus goes through hiatus 

(D) Peritoneal sac goes with paraesophageal hernia 

Answer: (B)

260. Burns with vesiculation, destruction of the epidermis and upper dermis is :

(A) 1st degree 

(B) 2nd degree 

(C) 3rd degree 

(D) 4th degree 

Answer: (B)

261. The most malignant form of malignant melanoma is :

(A) Nodular

(B) Hutchinson’s melanotic freckle 

(C) Lentigomaligna 

(D) Superficial spreading 

Answer: (A)

262. Uinlateral cleft lip is associated with :

(A) Posterior displacement of alar cartilage

(B) Columella elongated 

(C) Always cleft palate 

(D) Defective sucking 

Answer: (A)

263. Esophageal carcinoma is not predisposed by :

(A) Achalasia

(B) Scleroderma

(C) Corrosive intake

(D) Barret’s esophagus 

Answer: (B)

264. Meconium peritonitis occurs :

(A) Just before birth 

(B) Just after birth 

(C) Before and after birth 

(D) Due to birth trauma 

Answer: (C)

265. Not true about malignancy arising from Ulcerative colitis is :

(A) Takes atleast 10 years to develop

(B) Left sided is more common 

(C) Associated with dysplasia of the rest of the colon 

(D) Younger age of onset is associated with increased chance of carcinoma 

Answer: (B)

266. Blood loss during major surgery is best estimated by :

(A) Visual assessment 

(B) Suction bottles 

(C) Transesophageal USG Doppler 

(D) Cardiac output by thermodilution 

Answer: (B)

267. A foreign body completely obstructing the right main bronchus causes :

(A) Decreased ventilation perfusion ratio (VRF) 

(B) Increased ventilation in left lung 

(C) Perfusion doubles in right lung 

(D) Increased VF ratio in right lung

Answer: (A)

268. Least common site of lung abscess is :

(A) Left upper lobe 

(B) Left lower lobe 

(C) Right upper lobe 

(D) Right low lobe 

Answer: (A)

269. True regarding epispadias is :

(A) Always associated with dystrophy of bladder 

(B) Penile torsion 

(C) Common congenital anomaly 

(D) None of these 

Answer: (A)

270. Which is not a feature of pancreatic ascites :

(A) Low protein 

(B) Somatostatin is the drug of choice 

(C) Communication with pancreatic duct in 80%

(D) Raised amylase levels

Answer: (A)

271. Which is most malignant :

(A) Villous adenoma 

(B) Tubulovillous adenoma 

(C) Tubular adenoma 

(D) Familial polyposis 

Answer: (D)

272. In prostatic metastasis, the site most commonly involved is :

(A) Obturator nodes 

(B) Perivesical nodes 

(C) Pre-sacral nodes 

(D) Paraaortic nodes

Answer: (A)

273. Mucoepidermoid carcinoma of parotid arises from :

(A) Secretory cells 

(B) Excretory cells

(C) Myoepithelial cells 

(D) Myofibril

Answer: (A)

274. Erythma chronicum migrans is seen in :

(A) Lyme’s disease 

(B) Glucagonoma 

(C) Gastrinoma

(D) Phaeochromocytoma 

Answer: (A)

275. Which among the following is not pre-malignant :

(A) Ulcerative colitis 

(B) Peutz-jeghers syndrome 

(C) Villous adenoma 

(D) Familial adenomatoses polyopsis 

Answer: (B)

276. In extensive small bowel resection, all are seen except :

(A) Gastric hyposecretion 

(B) Decreased protein absorption 

(C) Decreased protein absorption 

(D) Oxalate stone increased 

Answer: (A)

277. The treatment of gall stone ileus is :

(A) Cholecystectomy alone 

(B) Removal of obstruction 

(C) Cholecystectomy, closure of fistula and removal of stone by enterotomy

(D) Cholecystectomy with closure of fistula 

Answer: (B)

278. Fecal soiling in children is most commonly due to :

(A) Hirschsprung’s disease 

(B) Chronic constipation 

(C) Rectal atresia 

(D) None of the above 

Answer: (B)

279. Tear-drop bladder is seen in :

(A) Tuberculosis

(B) Hunner’s ulcer

(C) Perivescial hemorrhage with rupture 

(D) Perivescial hemorrhage without rupture 

Answer: (C)

280. True about melanoma of the anal canal is :

(A) Presents usually as anal bleeding 

(B) AP resection gives better result than local excision 

(C) Local recurrence at the same site after resection 

(D) Radiosensitive

Answer: (A)

281. The CNS tumor present with calcification :

(A) Oligodendroglioma

(B) Astrocytoma

(C) Medulloblastoma

(D) Phaeochromocytoma 

Answer: (A)

282. The ideal treatment for hemothorax of  blood loss greater than 500 ml/hour:

(A) Wait and watch 

(B) Needle aspiration 

(C) Intercostal tube 

(D) Open thoracotomy with ligation of vessel 

Answer: (D)

283. Which among the following does not lead to pigment gallstones:

(A) TPN

(B) Clonorchis sinensis 

(C) Hemolytic anemia 

(D) Intercostal tube 

Answer: (A)

284. All are indicators of malingnancy in mammography except :

(A) Nodular calcification 

(B) Speckled margin 

(C) Attenuated architecture 

(D) Irregular mass 

Answer: (A)

285. Seminoma correspond to :

(A) Choriocarcinoma

(B) Dysgerminoma

(C) Granulosa tumor 

(D) Leutal cyst 

Answer: (B)

286. Placental alkaline phosphatase is marker of:

(A) Theca cell tumour 

(B) Teratoma

(C) Choriocarcinoma

(D) Seminoma

Answer: (D)

287. In hypospadias all are seen except:

(A) Hooded penis 

(B) Dorsal chordee 

(C) Spatulated glans 

(D) Meatal stenosis 

Answer: (B)

288. Epulis arises from:

(A) Epulis arises from:

(B) Root of teeth 

(C) Gingiva

(D) Pulp

Answer: (C)

289. All predisposes to oral cancer except :

(A) Erythroplakia

(B) Leukoplakia

(C) Submucosal fibrosis

(D) Lichen planus

Answer: (D)

290. About mallory weiss syndrome true is

(A) Gastrointestinal reflux 

(B) Obestiy

(C) Tumour at gastroesophageal junction 

(D) Seen in alcoholics 

Answer: (D)

291. Not included in charcot’s triad is:

(A) Fever with chills 

(B) Pain abdomen 

(C) Shock

(D) Jaundice

Answer: (C)

292. In non hemolytic jaundice, urobillinogen is seen in:

(A) Obstructive jaundice 

(B) Hepatic fibrosis 

(C) Fatty liver 

(D) Infective hepatitis 

Answer: (D)

293. Specific marker for prostatic cancer is

(A) Alkaline phosphatase 

(B) Prostate specific antigen 

(C) Acid phosphatase 

(D) CA 125 

Answer: (B)

294. Which is true about intestinal tuberculosis:

(A) Common site is appendix 

(B) Causes intestinal perforation 

(C) Commonly associated with pulmonary TB 

(D) Caused by mycobacterium TB 

Answer: (D)

295. Curling ulcer is seen in :

(A) Burns

(B) Head injury 

(C) Trauma

(D) Surgical operations 

Answer: (A)

296. Late deaths in burns is due to :

(A) Sepsis

(B) Hypovolemia

(C) Contractures

(D) Neurogenic shock 

Answer: (A)

297. All are recongnised tumour markers except :

(A) Beta HCG 

(B) Beta 2 microglobulin 

(C) Alpha fetoprotein 

(D) Acid phosphatase 

Answer: (B)

298. Connective tissue disorder which is associated with gastroesophageal reflux is :

(A) SLE

(B) Sceleroderma

(C) Behcet’s syndrome

(D) Dermatomyositis

Answer: (B)

299. Mucoepidermoid carcinoma arises form:

(A) Myothelium

(B) Epithelium

(C) Acinus

(D) Mucin secreting and epidermal cells

Answer: (D)

300. Tt. for pleomorphic adenoma is:

(A) Total parotidectomy 

(B) Superficial parotidectomy

(C) Enucleation

(D) Conservative managment 

Answer: (B)

301. True about papillary carcinoma thyroid is:

(A) Comprises 10-15% of all thyroid cancers 

(B) Cells have intranuclear vacuolation 

(C) Amyloid deposition seen 

(D) Encapsulated

Answer: (D)

302. All are true associations of ulcerative colitis except :

(A) Ertythema nodosum 

(B) Circinate balanitis 

(C) Scelrosing cholangitis 

(D) Aphtohous stomatitis 

Answer: (B)

303. All are seen in hemobilia except :

(A) Shock

(B) Colicky pain 

(C) Melena

(D) Jaundice

Answer: (A)

304. All are true about cystic hygroma except :

(A) Aspiration is diagnostic 

(B) 50% present at birth 

(C) Presents as posterior cervical swelling 

(D) Sequstration of lymphatic tissue 

Answer: (B)

305. Most common artery involved in perforation of duodenal ulcer is :

(A) Gastroduodenal artery

(B) Gastroepiploic artery 

(C) Rt gastric artery 

(D) Pancreaticoduodenal A 

Answer: (B)

306. True about appendicular rupture is A/E:

(A) Common in extremes of age 

(B) Common in people with Fecolith obstruction

(C) Early antibiotics prevent rupture 

(D) Appendicectomy is done always in presence of rupture 

Answer: (D)

307. MC abdominal organ injured in blunt trauma abdomen is:

(A) Spleen

(B) Liver

(C) Pancreas

(D) Stomach

Answer: (A)

308. Spastic ileus is seen in :

(A) Porphyria

(B) Retroperitoneal abscess

(C) Hypokalemia

(D) MI 

Answer: (A)

309. Recurrent pain abdomen with intestinal obstruction and mass passes per rectum goes in favour of :

(A) Internal herniation 

(B) Stricutre

(C) Strangulated hernia

(D) Intussusception

Answer: (D)

310. Pyloric stenosis with severe vomiting presents as:

(A) Hypokalemia hypochloremia with alkaline urine 

(B) Hyperkalemia hypochloremia with alkaline urine 

(C) Hypokalemia hypochloremia with acidic urine 

(D) Hyperkalemia hypochloremia with acidic urine 

Answer: (C)

311. Gleasons’ classification is applied for:

(A) Prostate carcinoma 

(B) Lung carcinoma 

(C) Ewing’s sarcoma 

(D) CA rectum 

Answer: (A)

312. Pancost tumour is seen with cancer of:

(A) Apical lobe of lung 

(B) Lingual lobe 

(C) Thyroid

(D) Pyriform fossa 

Answer: (A)

313. 4 cm breast tumour with 2 ipsilateral involved lymphonodes come under:

(A) T₃ N­₁ 

(B) T₃ N₂ 

(C) T₂ N₁ 

(D) T₄ N­₁ 

Answer: (C)

314. Spigelian hernia :

(A) Lateral to rectus abdominis 

(B) Obturator internus 

(C) Lies above int. oblique 

(D) Common in men 

Answer: (A)

315. Pt with recurrent diarrhea, pseudopolyp, lead pipe appearance on Ba enema has :

(A) Ulcerative colitis 

(B) Crohn’s disease

(C) Irritable bowel syndrome 

(D) Short bowel syndrome 

Answer: (A)

316. True about Berry-aneurysm is following except :

(A) Associated with familial syndrome 

(B) Most common site of rupture is apex which causes SAH 

(C) Wall contains smooth muscle fibroblasts 

(D) 90% occurs at ant. part of circulation at branching point 

Answer: (C)

317. Risk factor for carcinoma stomach are all except :

(A) Blood group A 

(B) Post gastrectomy 

(C) Old peptic ulcer

(D) Atrophic gastritis 

Answer: (C)

318. Head & neck involvement in burns in infant is :

(A) 9% 

(B) 18% 

(C) 27%

(D) 32% 

Answer: (B)

319. Gold standard test in insulinoma :

(A) CT scan

(B) Ultrasound

(C) MRI

(D) Arteriography

Answer: (D)

320. Dysphagia lusoria is due to :

(A) Oesophageal webs 

(B) Abnormalities of Arch of Aorta 

(C) Aortic aneurysm 

(D) Enlarged left atrium 

Answer: (B)

321. Commonest complication of pancreatic pseudocyst :

(A) Rupture

(B) Infection

(C) Pressure on viscera

(D) Hemorrhage

Answer: (B)

322. Mucinous ascites is seen in :

(A) Stomach Ca

(B) TB

(C) Nephrotic syndrome

(D) Cirrhosis

Answer: (A)

323. In achalasia cardia, true is :

(A) Pressure at distal end ↑ with no peristalsis

(B) Low pr. at LES with no periotalins 

(C) Pressure > 50 mm Hg with peristalsis 

(D) None

Answer: (A)

324. Recurrent obstruction, mass per rectum and diarrhea in child :

(A) Intussusception

(B) Rectal prolapse 

(C) Internal hernia 

(D) Haemorrhoids

Answer: (A)

325. 4 cm breast nodule with ipsilatral mobile LN in axilla staging :

(A) T₂N₁M₀ 

(B) T₂N2M₀ 

(C) T₁N₁M₀ 

(D) T₁N₁M₀ 

Answer: (A)

326. Patterson Brown Kelly syndrome is characterized by all except :

(A) Lower oesophageal web 

(B) Iron deficiency andema 

(C) Common in adult female 

(D) premalignant

Answer: (A)

327. Gleason’s staging is done in :

(A) Ca prostate 

(B) Ca Pancreas 

(C) CA kidney 

(D) Ca Cx 

Answer: (A)

328. 5 H. indole acetic acid in urine is seen in :

(A) Carcinoid

(B) Pheochromocytoma

(C) Hirschsprung’s disease

(D) Wilm’s tumor 

Answer: (A)

329. Dissection of which artery is seen in pregnancy :

(A) Carotid artery 

(B) Aorta

(C) Coronary A 

(D) Femoral artery 

Answer: (B)

330. Peripheral arterial occlusion (sudden onset) is characterized all expect :

(A) Paresthesia

(B) Rubor

(C) Pallor

(D) Pain

Answer: (B)

331. Most diagnostic of pulmonary emboli is :

(A) Segmental perfusion defects corresponding to abnormal chest X-ray 

(B) Multiple segmental perfusion defect with normal chest X-ray and ventilation scan 

(C) Normal chest X-ray 

(D) Tenting of diaphragm 

Answer: (A)

332. A patient presents with dysphagia of 4 weeks duration. Now he is able to swallow liquid food only. Which of the following is the one investigation to be done :

(A) Barium studies are the best in this case

(B) Upper GI endoscopy is to be done

(C) Upper GI endoscopy is to be done

(D) Esophageal manometry 

Answer: (B)

333. Which of the following is the most common site for the carcinoma of pancreas :

(A) Head

(B) Ampulla

(C) Body

(D) Tail

Answer: (A)

334. Neuroblastomas – good prognostic factor is :

(A) N-myc amplification 

(B) RAS oncogene 

(C) Hyperdiploidy

(D) Translocations

Answer: (C)

335. All are elaborated by small cell carcinoma lung, except :

(A) ADH

(B) ACTH

(C) 5-HT 

(D) Noradrenaline

Answer: (D)

336. Mucinous peritoneal fluid is found in “

(A) Carcinoma stomach

(B) Leimyosarcoma 

(C) Teratoma ovary 

(D) Mucocele of appendix 

Answer: (D)

337. Post splenectomy sepsis is common in :

(A) ITP

(B) Thalassemia

(C) Hereditary spherocytosis 

(D) Trauma

Answer: (A)

338. Treatment of choice in mixed tumor of parotid gland :

(A) Superficial parotidectomy 

(B) Total parotidectomy 

(C) Radiotherapy

(D) Chemotherapy

Answer: (A)

339. Mismatched blood transfusion in anaesthesia presents as :

(A) Hyperthermia & hypertension 

(B) Hypotension & bleeding from site of wound 

(C) Bradycardia & hypotension 

(D) Tachyacardia and hypertension 

Answer: (B)

340. True about acute pancreatitis a/e :

(A) Gallstones & alcohol are common causes

(B) Sr. amylase levels increase after 72 hr. of attack

(C) Can cause secondary pleural effusion 

(D) Can present with shock 

Answer: (B)

341. Artery to bleed in duodenal ulcer h’ge :

(A) Spleenic

(B) Gastroduodenal

(C) Left gastric

(D) Sup. mesenteric

Answer: (B)

342. Treatment of chronic cholecystitis :

(A) Cholecystectomy 

(B) Choledochocystectomy 

(C) Feculant

(D) Conservative

Answer: (A)

343. True about mesenteric venous thrombosis :

(A) Peritoneal signs are always present 

(B) Invariably involves long length of bowel

(C) I.V. Heparin is the treatment of choice 

(D) Surgery can lead to short-bowel syndrome 

Answer: (D)

344. True about enterocutaneous fistula :

(A) High output fistula drains 500 ml/day 

(B) Malignancy is most common cause 

(C) Fluid & electrolyte loss can occur

(D) No skin damage 

Answer: (C)

345. True regarding cancer colon :

(A) Obstructive features are more common with right colon 

(B) 40% cases at presentation show liver metastasis 

(C) Resection possible in 25% cases 

(D) More common in AIDS patients 

Answer: (B)

346. True about stomach carcinoma :

(A) Weightloss is commonest feature 

(B) Secondaries are most commonly seen in peritoneum & omentum 

(C) Lymphatic & hematogenous spread are rare 

(D) Radioresistant

Answer: (D)

347. Predisposition for carcinoma ethmoid sinus is by :

(A) Wood dust 

(B) Thorotrast

(C) Vinyl chloride 

(D) Nitrosamines

Answer: (A)

348. True about burns :

(A) Hyperglycemia is seen in early burns 

(B) Child with burns should have damp dressing 

(C) Chemical powder burns should be kept dry 

(D) 3rd degree burns are painfull 

Answer: (A)

349. Investigation of choice for blunt trauma abdomen in unstable patient :

(A) X-ray abdomen 

(B) USG

(C) Diagnostic Peritoneal lavage 

(D) MRI

Answer: (C)

350. Which of the following is used in the treatment of thyroid malignancy :

(A) 131I

(B) 125I

(C) 99Tc

(D) 32p

Answer: (A)

351. True about left sided colon carcinoma :

(A) Anemia

(B) Obstruction

(C) Melena

(D) Feculant

Answer: (B)

352. True about pseudocyst of pancreas :

(A) Is a true cyst 

(B) Common with penetrating trauma abdomen 

(C) Treated with cystogastrostomy if size > 2 cm 

(D) Contains a wall of firbrous granulation tissue

Answer: (D)

353. Which is true regarding Trendelenburg operations :

(A) Stripping of the superficial varicose vein 

(B) Flush ligation of the superficial varicose vein

(C) Ligation of the perforators 

(D) Ligation of small tributaries at the distal end of Superficial varicose vein

Answer: (B)

354. A 30 year old female comes with hypovolumic shock after blunt trauma of the abdomen. An emergency USG of abdomen shows splenic tear. Which of the following is to be done :

(A) CECT of the abdomen

(B) Diagnostic lavage of peritoneal cavity before proceeding

(C) Monitor the patient to assess for progression 

(D) Immediate surgery 

Answer: (D)

355. Sphincter saving surgery for rectal malignancy is not done in :

(A) Age over 50 years 

(B) Lymph node involvement 

(C) Infiltration of lamina propria 

(D) More than 4 cm from anal verge

Answer: (D)

356. Breast carcinoma is seen in women who :

(A) Consume fatty food 

(B) Have early menopause 

(C) Smoke

(D) Have nultiple sex-parterns 

Answer: (A)

357. Migratory thrombophlebitis is seen most commonly with :

(A) Pancreatic ca 

(B) Testicular ca 

(C) Gastric ca 

(D) Breast ca 

Answer: (A)

358. Abdomino-perineal resection is done in colorectal Ca on the basis of :

(A) Age of patient 

(B) Distance from anal-verge 

(C) Fixity of tomor 

(D) Hepatic metastasis 

Answer: (B)

359. True of autosomal dominant polycystic kidney disease I and II respectively :

(A) Chromosomes 16 and 5 

(B) 16 and 4 

(C) 11 and 5 

(D) 11 and 4 

Answer: (B)

360. CA-125 is associated with :

(A) Colon ca 

(B) Breast ca 

(C) Ovarian ca 

(D) Bronchogenic ca 

Answer: (B)

361. In the management of thyroglossal cyst :

(A) Central portion of hyoid excised 

(B) Sternothyroid muscle dissected 

(C) Isthmusectomy e subtotal thyroidectomy 

(D) Strap muscle of neck 

Answer: (A)

362. Posterior urethral valve is diagnosed by :

(A) MCU

(B) Cystograph

(C) Retrograde cystography 

(D) Nephrogram

Answer: (A)

363. True about gall stones :

(A) More common in female 

(B) Gall stones, hiatus hernia, CBD stones form Saints triad 

(C) Limely bile precipitated 

(D) Lithotripsy always done 

Answer: (A)

364. A patient having multiple Gall stones and CBD shows 8 mm dilation and 4 stones in CBD, the best treatment modalities are :

(A) Cholecystectomy with choledecholithotomy done at same setting at same setting 

(B) ESWL

(C) Cholecystectomy and wait for ERCP 

(D) Sphicterotomy and then cholecystectomy 

Answer: (A)

365. Suprasellar calcification with polyuria seen in :

(A) Langerhan cell histocytosis 

(B) Medulloblastoma

(C) Pinealoma

(D) Craniopharyngioma

Answer: (D)

© Copyright Entrance India - Engineering and Medical Entrance Exams in India | Website Maintained by Firewall Firm - IT Monteur